71 JEE Main Mathematics Online - Disha Experts - Unlocked

You might also like

Download as pdf or txt
Download as pdf or txt
You are on page 1of 177

1

Sets
6. A relation on the set A = {x : |x| < 3, x ÎZ },
Sets, Types of Sets, Disjoint Sets,
Complementary Sets, Subsets, where Z is the set of integers is defined by
TOPIC Ć Power Set, Cardinal Number of R = {(x, y) : y = |x|, x ¹ – 1}. Then the number of elements in
Sets, Operations on Sets the power set of R is: [Online April 12, 2014]
(a) 32 (b) 16 (c) 8 (d) 64
1. Set A has m elements and set B has n elements. If the total 7. Let X ={1,2,3,4,5}. The number of different ordered pairs
number of subsets of A is 112 more than the total number (Y, Z) that can formed such that Y Í X , Z Í X and Y Ç Z
of subsets of B, then the value of m×n is ______. is empty is : [2012]
[Sep. 06, 2020 (I)] (a) 52 (b) 35 (c) 25 (d) 53
2. Let S = {1, 2, 3, ... , 100}. The number of non-empty subsets 8. If A, B and C are three sets such that A Ç B = A Ç C and
A of S such that the product of elements in A is even is :
A È B = A È C , then [2009]
[Jan. 12, 2019 (I)]
(a) A = C (b) B = C
(a) 2100 - 1
50 50
(b) 2 2 - 1 ( ) (c) A Ç B = f (d) A = B
(c) 250 – 1 (d) 250 + 1
3. Let S = {x Î R : x ³ 0 and Venn Diagrams, Algebraic
Operations on Sets, De Morgan’s
TOPIC n
2| x - 3 | + x ( x - 6) + 6 = 0 . Then S : [2018] Law, Number of Elements in
Different Sets
(a) contains exactly one element.
(b) contains exactly two elements. 9. A survey shows that 73% of the persons working in an
(c) contains exactly four elements. office like coffee, whereas 65% like tea. If x denotes the
(d) is an empty set. percentage of them, who like both coffee and tea, then x
æ1ö cannot be : [Sep. 05, 2020 (I)]
4. If f(x) + 2f ç ÷ = 3x , x ¹ 0 and
èxø (a) 63 (b) 36 (c) 54 (d) 38
10. A survey shows that 63% of the people in a city read
S = {x Î R : f(x) = f(–x)}; then S: [2016]
newspaper A whereas 76% read newspaper B. If x% of the
(a) contains exactly two elements.
people read both the newspapers, then a possible value of
(b) contains more than two elements.
x can be : [Sep. 04, 2020 (I)]
(c) is an empty set.
(d) contains exactly one element. (a) 29 (b) 37 (c) 65 (d) 55
Let P = {q : sinq – cosq = 50 n
5. 2 cosq} and Q = {q : sinq +
cosq =
11. Let U X i = U Yi = T , where each Xi contains 10 elements
2 sinq} be two sets. Then: i =1 i =1

[Online April 10, 2016] and each Yi contains 5 elements. If each element of the set
T is an element of exactly 20 of sets Xi's and exactly 6 of
(a) P Ì Q and Q - P ¹ f
sets Yi's, then n is equal to [Sep. 04, 2020 (II)]
(b) Q Ë P (a) 15 (b) 50 (c) 45 (d) 30
(c) P = Q
(d) P Ë Q
EBD_8344
M-2 Mathematics

12. Let X = {n Î N: l £ n £ 50}. If 16. Two newspapers A and B are published in a city. It is
A = {n Î X: n is a multiple of 2} and known that 25% of the city population reads A and 20%
B = {n Î X: n is a multiple of 7}, then the number of reads B while 8% reads both A and B. Further, 30% of
elements in the smallest subset of X containing both A those who read A but not B look into advertisements and
and B is __________. [Jan. 7, 2020 (II)] 40% of those who read B but not A also look into
2 advertisements, while 50% of those who read both A and
13. Let Z be the set of integers. If A = {xÎZ : 2(x + 2) ( x – 5x + 6) = 1} B look into advertisements. Then the percentage of the
and B = {x Î Z : –3 < 2x – 1< 9}, then the number of subsets population who look into advertisements is:
of the set A × B, is : [Jan. 12, 2019 (II)] [April. 09, 2019 (II)]
(a) 215 (b) 218 (c) 212 (d) 210 (a) 13.9 (b) 12.8 (c) 13 (d) 13.5
14. In a class of 140 students numbered 1 to 140, all even 17. In a certain town, 25% of the families own a phone and
numbered students opted Mathematics course, those 15% own a car; 65% families own neither a phone nor a
whose number is divisible by 3 opted Physics course and car and 2,000 families own both a car and a phone. Consider
those whose number is divisible by 5 opted Chemistry the following three statements : [Online April 10, 2015]
course. Then the number of students who did not opt for (A) 5% families own both a car and a phone
any of the three courses is: [Jan. 10, 2019 (II)] (B) 35% families own either a car or a phone
(a) 102 (b) 42 (c) 1 (d) 38 (C) 40,000 families live in the town
15. Let A, B and C be sets such that f ¹ A Ç B Í C . Then Then,
which of the following statements is not true ? (a) Only (A) and (C) are correct.
[April 12, 2019 (II)] (b) Only (B) and (C) are correct.
(c) All (A), (B) and (C) are correct.
(a) B Ç C ¹ f
(d) Only (A) and (B) are correct.
(b) If ( A - B) Í C , then A Í C
(c) (C È A) Ç (C È B) = C
(d) If (A - C) Í B , then A Í B
2
Relations and
Functions
5. The domain of the definition of the function
Relations, Domain, Codomain and
1
Range of a Relation, Functions, f ( x) = + log10 ( x3 - x ) is: [April. 09, 2019 (II)]
TOPIC Ć Domain, Codomain and Range of a 4 - x2
Function (a) (–1, 0) È (1, 2) È (3, ¥)
(b) (–2, –1) È (–1, 0) È (2, ¥)
1. Let R1 and R2 be two relations defined as follows : (c) (–1, 0) È (1, 2) È (2, ¥)
(d) (1, 2) È (2, ¥)
R1 = {(a, b) Î R 2 : a 2 + b 2 Î Q} and
6. The range of the function
R2 = {(a, b) Î R : a + b Ï Q} , where Q is the set of all
2 2 2
x
f ( x) = , x Î R , is [Online May 7, 2012]
rational numbers. Then : [Sep. 03, 2020 (II)] 1+ x
(a) Neither R1 nor R2 is transitive.
(a) R (b) (– 1, 1) (c) R – {0} (d) [– 1, 1]
(b) R2 is transitive but R1 is not transitive.
(c) R1 is transitive but R2 is not transitive. 1
7. The domain of the function f ( x) = is [2011]
(d) R1 and R2 are both transitive. x -x
| x | +5 ö (a) (0, ¥ ) (b) (– ¥ , 0)
2. The domain of the function f ( x) = sin -1 æç ÷ is (c) (– ¥ , ¥ ) – {0} (d) (– ¥ , ¥ )
è x2 +1 ø
8. Domain of definition of the function
(-¥, - a] È [a, ¥]. Then a is equal to : 3
f ( x) = + log10 ( x 3 - x) , is [2003]
[Sep. 02, 2020 (I)] 4 - x2

17 17 - 1 1 + 17 17 (a) ( -1,0) È (1,2) È ( 2, ¥) (b) (a, 2)


(a) (b) (c) (d) +1
2 2 2 2 (c) ( -1,0) È ( a,2) (d) (1,2) È (2, ¥)
3. If R = {(x, y) : x, y Î Z, x2 + 3 y 2 £ 8} is a relation on the
Even and Odd Functions, Explicit
set of integers Z, then the domain of R–1 is : and Implicit Functions, Greatest
[Sep. 02, 2020 (I)] Integer Function, Periodic
(a) {–2, –1, 1, 2} (b) {0, 1} TOPIC n Functions, Value of a Function,
(c) {–2, –1, 0, 1, 2} (d) {–1, 0, 1} Equal Functions, Algebraic
Operations on Functions
x
4. Let f : R ® R be defined by f ( x ) = , x Î R. Then
1 + x2 9. Let [t] denote the greatest integer £ t. Then the equation
the range of f is : [Jan. 11, 2019 (I)] in x, [x]2 + 2[x + 2] – 7 = 0 has : [Sep. 04, 2020 (I)]
(a) exactly two solutions
é 1 1ù
(a) ê - , ú (b) R – [–1, 1] (b) exactly four integral solutions
ë 2 2û (c) no integral solution
é 1 1ù (d) infinitely many solutions
(c) R - ê - , ú (d) (–1, 1) – {0}
ë 2 2û
EBD_8344
M-6 Mathematics

10. Let f (x) be a quadratic polynomial such that f (–1) + f (2) = 15. Let f be an odd function defined on the set of real numbers
0. If one of the roots of f (x) = 0 is 3, then its other root lies such that for x ³ 0, f(x) = 3 sin x + 4 cos x.
in : [Sep. 02, 2020 (II)] 11p
(a) (–1, 0) (b) (1, 3) (c) (–3, –1) (d) (0, 1) Then f(x) at x = - is equal to: [Online April 11, 2014]
6
x[ x]
11. Let f (1, 3) ® R be a function defined by f (x) = , (a)
3
+2 3
3
(b) - + 2 3
1 + x2 2 2
where [x] denotes the greatest integer £ x. Then the range
of f is: [Jan. 8, 2020 (II)] 3 3
(c) -2 3 (d) - - 2 3
2 2
æ 2 3ö æ 3 4 ö æ 2 1 ö æ 3 4ö
(a) çè , ÷ø È çè , ø÷ (b) çè , ÷ø È çè , ø÷ 16. A real valued function f (x) satisfies the functional equation
5 5 4 5 5 2 5 5
f (x – y) = f (x) f (y) – f (a – x) f (a + y)
æ 2 4ö æ 3 4ö where a is a given constant and f (0) = 0, f (2a – x) is equal
(c) çè , ÷ø (d) çè , ÷ø to [2005]
5 5 5 5
(a) – f (x) (b) f (x)
1- x ö æ 2x ö (c) f (a) + f (a – x) (d) f (– x)
12. If f(x) = loge æç ÷ , |x| < 1, then f ç ÷ is equal to :
è 1+ x ø è 1+ x2 ø 17. The graph of the function y = f(x) is symmetrical about the
line x = 2, then [2004]
[April 8, 2019 (I)]
(a) 2f(x) (b) 2f(x2) (c) (f(x)) 2 (d) –2f(x) (a) f ( x ) = - f (- x) (b) f (2 + x) = f (2 - x)
x
13. Let f(x) = a (a > 0) be written as f(x) = f1(x) + f2(x), where (c) f ( x ) = f ( - x) (d) f ( x + 2) = f ( x - 2)
f1(x) is an even function and f2(x) is an odd function. Then
f1(x + y) + f1(x – y) equals : [April. 08, 2019 (II)] 18. If f : R ® R satisfies f ( x + y ) = f ( x) + f ( y ) , for all x,
(a) 2f1(x) f1(y) (b) 2f1(x + y) f1(x – y) n
(c) 2f1(x)f2(y) (d) 2f1(x + y) f2(x – y) y Î R and f(1) = 7, then S f (r ) is [2003]
r=1
é 1 3n ù
14. Let f ( n ) = ê + n , where [n] denotes the greatest 7 n (n + 1) 7n
ë 3 100 úû (a)
2
(b)
2
56 7 (n + 1)
integer less than or equal to n. Then å f ( n) is equal to: (c)
2
(d) 7 n + ( n + 1)
n=1
[Online April 19, 2014]
(a) 56 (b) 689 (c) 1287 (d) 1399
EBD_8344
M-10 Mathematics

3
Trigonometric Functions

tan A cot A
Circular System, Trigonometric 4. The expression +
Ratios, Domain and Range of 1 - cot A 1 - tan A
TOPIC Ć Trigonometric Functions,
Trigonometric Ratios of Allied can be written as : [2013]
Angles (a) sinA cosA + 1 (b) secA cosecA + 1
(c) tanA + cotA (d) secA + cosecA
5. The value of cos 255 + sin 195 is[Online May 26, 2012]
æ p pö
1. For any q Î ç , ÷ the expression
è4 2ø 3 -1 3 -1
(a) (b)
2 2 2
3(sinq – cosq)4 + 6(sinq + cosq)2 + 4sin6q equals:
æ 3 -1ö 3 +1
[Jan. 9, 2019 (I)] (c) - çç ÷÷ (d)
(a) 13 – 4cos2q + 6sin2qcos2q è 2 ø 2
(b) 13 – 4cos6q 6. Let f(x) = sin x, g(x) = x.
(c) 13 – 4cos2q + 6cos4q Statement 1: f ( x ) £ g ( x ) for x in ( 0, ¥ )
(d) 13 – 4cos4q + 2sin2qcos2q Statement 2: f(x) £ 1 for x in (0, ¥) but g(x) ® ¥ as x ® ¥.

( )
1 [Online May 7, 2012]
2. Let f k ( x ) = sin k x + cos k x where x Î R and k ³ 1. (a) Statement 1 is true, Statement 2 is false.
k
(b) Statement 1 is true, Statement 2 is true, Statement 2 is
Then f 4 ( x ) - f 6 ( x ) equals [2014]
a correct explanation for Statement 1.
1 1 (c) Statement 1 is true, Statement 2 is true, Statement 2 is
(a) (b) not a correct explanation for Statement 1.
4 12
(d) Statement 1 is false, Statement 2 is true.
1 1
(c) (d) 7. A triangular park is enclosed on two sides by a fence
6 3
and on the third side by a straight river bank. The two
æ pö sides having fence are of same length x. The maximum
3. If 2cos q + sin q = 1 ç q ¹ ÷ ,
è 2ø area enclosed by the park is [2006]
then 7 cos q + 6 sin q is equal to:[Online April 11, 2014]
x3
1 (a) 3 x 2 (b)
(a) (b) 2 2 8
2

(c)
11
(d)
46 (c) 1 x 2 (d) px 2
2 5 2
Trigonometric Functions M-11

13. The value of [April 9, 2019 (II)]


Trigonometric Identities,
Conditional Trigonometric cos210 – cos10 cos50 + cos250 is :
TOPIC n Identities, Greatest and Least 3
Value of Trigonometric Expressions (a) + cos20 (b) 3/4
4
3
3æ pö æ 3p ö 3æ pö æ 3p ö (c) (1 + cos20 ) (d) 3/2
8. The value of cos ç ÷ .cos ç ÷ + sin ç ÷ .sin ç ÷ is 2
è ø
8 è ø8 è ø
8 è 8 ø
14. Two poles standing on a hori ontal ground are of heights
[Jan. 9, 2020 (I)]
5m and 10m respectively. The line oining their tops makes
1 1 an angle of 15 with the ground. Then the distance (in m)
(a) (b)
2 2 2 between the poles, is: [April. 09, 2019 (II)]
1 1 (a) 5(2 + 3) (b) 5( 3 + 1)
(c) (d)
2 4
5
(c) (2 + 3) (d) 10( 3 - 1)
2 sin a 1 1 - cos 2b 1 2
9. If = and = ,
1 + cos 2a 7 2 10 15. The value of sin 10 sin 30 sin 50 sin 70 is:
æ pö [April. 09, 2019 (II)]
a , b Îç 0, ÷ , then tan (a + 2b) is equal to _____.
è 2ø 1 1
(a) (b)
[Jan. 8, 2020 (II)] 16 32
1 1
æ pö æpö (c) (d)
10. If L = sin 2 ç ÷ - sin 2 ç ÷ and 18 36
è 16 ø è8ø
3 5 p
16. If cos (a + b) = , sin(a – b) = and 0 < a, b < , then
æ pö æ pö 5 13 4
M = cos ç ÷ - sin 2 ç ÷ , then : [Sep. 05, 2020 (II)]
2
è 16 ø è 8ø tan(2a) is equal to : [ April 8, 2019 (I)]
63 63
1 1 p 1 1 p (a) (b)
(a) L = - + cos (b) L = - cos 52 16
2 2 2 8 4 2 4 8
21 33
1 1 p 1 1 p (c) (d)
(c) M = + cos (d) M = + cos 16 52
4 2 4 8 2 2 2 8
17. If sin 4a + 4 cos4b + 2 = 4 2 sin a cos b ; a, bÎ[0, p], then
11. The set of all possible values of q in the interval (0, p) for cos(a + b) – cos(a – b) is equal to : [Jan. 12, 2019 (II)]
which the points (1, 2) and (sin q, cos q) lie on the same (a) 0 (b) – 1
side of the line x + y = 1 is : [Sep. 02, 2020 (II)] (c) 2 (d) – 2

æ pö
(a) ç 0, ÷
æ p 3p ö
(b) ç , ÷
18. Let f k ( x ) =
1
k
( )
sin k x + cosk x for k = 1, 2, 3, ... Then for
è 2ø è4 4 ø
all x Î R, the value of f 4 ( x ) – f 6 ( x ) is equal to :
æ 3p ö æ pö [Jan. 11, 2019 (I)]
(c) ç 0, ÷ (d) ç 0, ÷ 1 1
è 4 ø è 4ø (a) (b)
12 4
12. The angle of elevation of the top of a vertical tower standing
–1 5
on a hori ontal plane is observed to be 45o from a point A (c) (d)
12 12
on the plane. Let B be the point 30 m vertically above the 19. The value of [Jan. 10, 2019 (II)]
point A. If the angle of elevation of the top of the tower p p p p
cos 2 × cos 3 × .... × cos 10 × sin 10
from B be 30o, then the distance (in m) of the foot of the 2 2 2 2
tower from the point A is: [April 12, 2019 (II)] 1 1
(a) (b)
(a) 15(3 + 3) (b) 15(5 - 3) 512 1024
1 1
(c) 15(3 - 3) (d) 15(1 + 3) (c) (d)
256 2
EBD_8344
M-12 Mathematics

20. If 5(tan2x – cos2x) = 2cos 2x + 9, then the value of cos 4x is 26. Let A and B denote the statements
: A : cos a + cos b + cos g = 0
[2017] B : sin a + sin b + sin g = 0
7 3
(a) - (b) - 3
9 5 If cos (b – g) + cos (g – a) + cos (a – b) = - , then :[2009]
2
1 2
(c) (d) (a) A is false and B is true
3 9
21. If m and M are the minimum and the maximum values of (b) both A and B are true
1 (c) both A and B are false
4 + sin 2 2x - 2 cos 4 x, x Î R, then M – m is equal to : (d) A is true and B is false
2
[Online April 9, 2016] 27. If p and q are positive real numbers such that p2 + q2 = 1,
then the maximum value of (p + q) is [2007]
9 15
(a) (b)
4 4 1 1
(a) (b)
7 1 2 2
(c) (d)
4 4
(c) 2 (d) 2.
3 1
22. If cos a + cos b = and sin a + sinb and q is the 1
2 2 28. If 0 < x < p and cos x + sin x = , then tan x is[2006]
the arithmetic mean of a and b , then sin 2q + cos 2q 2
is equal to : [Online April 11, 2015]
3 7 (a) (1 - 7 ) (b) (4 - 7 )
(a) (b) 4 3
5 5
4 8 (4 + 7 ) (1 + 7 )
(c) (d) (c) – (d)
5 5 3 4

p+q æp qö 29. If u = a 2 cos 2 q + b 2 sin 2 q + a 2 sin 2 q + b 2 cos 2 q


23. If cosec q = ( p ¹ q ¹ 0 ) , then cot çè 4 + 2 ÷ø is then the difference between the maximum and minimum
p-q
values of u2 is given by [2004]
equal to: [Online April 9, 2014]
(a) (a - b)2 (b) 2 a 2 + b2
p q (c) (a + b)2 (d) 2(a 2 + b2 )
(a) (b)
q p
30. Let a, b be such that p < a - b < 3p .

(c) (d) pq 21 27
pq If sin a + sin b = - and cos a + cos b = - , then the
65 65
24. If A = sin2 x + cos4x, then for all real x : [2011]
a -b
13 value of cos [2004]
(a) £ A £1 (b) 1 £ A £ 2 2
16
-6 3
(a) (b)
3 13 3 65 130
(c) £ A£ (d) £ A £1
4 16 4 6 3
(c) (d) -
4 5 65 130
Let cos (α + β) = and sin (a - b) = ,
( )
25.
5 13
31. The function f ( x) = log x + x 2 + 1 , is [2003]
π
where 0 £ α, β £ . Then tan 2a = [2010] (a) neither an even nor an odd function
4
(b) an even function
56 19 (c) an odd function
(a) (b) (d) a periodic function.
33 12
32. The period of sin 2 q is [2002]
20 25 (a) p 2 (b) p
(c) (d)
7 16 (c) 2 p (d) p /2
Trigonometric Functions M-13

33. Which one is not periodic? [2002] 39. Let S = {qÎ[–2 p, 2p] : 2 cos2q + 3 sinq = 0}.
(a) | sin3x | +sin 2x (b) cos x + cos2x Then the sum of the elements of S is: [April 9, 2019 (I)]
(c) cos 4x + tan2x (d) cos2x + sinx
13p
(a) (b) 5p
Solutions of Trigonometric 6 3
TOPIC Đ Equations
(c) 2p (d) p
34. If the equation cos q + sin q + l = 0 has real solutions
4 4

for q, then l lies in the interval : p


[Sep. 02, 2020 (II)] 40. If 0 £ x < , then the number of values of x for which
2
æ 5 ö é 1ù
(a) ç - , - 1÷ (b) ê -1, - ú sin x – sin 2x + sin 3x = 0, is: [Jan. 09, 2019 (II)]
è 4 ø ë 2û
(a) 3 (b) 1
æ 1 1ù é 3 5ù
(c) ç - , - ú (d) ê - , - ú (c) 4 (d) 2
è 2 4û ë 2 4û 41. The number of solutions of sin 3x = cos 2x, in the interval
35. The number of distinct solutions of the equation, æp ö
ç 2 , p ÷ is [Online April 15, 2018]
log1/2|sin x| = 2 – log1/2|cos x| in the interval [0, 2p], is _____. è ø
[Jan. 9, 2020 (I)] (a) 3 (b) 4
36. The number of solutions of the equation (c) 2 (d) 1
42. If sum of all th e solution s of th e equation
é 5p 5p ù
1 + sin 4x = cos2 3x, x Î ê - , ú is : [April 12, 2019 (I)]
ë 2 2û æ æp ö æp ö 1ö
8cos x × ç cos ç + x ÷ × cos ç - x ÷ - ÷ - 1 in [0, p] is kp,
(a) 3 (b) 5 è è6 ø è6 ø 2ø
(c) 7 (d) 4 then k is equal to : [2018]
37. Let S be the set of all a Î R such that the equation, cos 2x 13 8
+ a sin x = 2a –7 has a solution. Then S is equal to : (a) (b)
9 9
[April 12, 2019 (II)]
20 2
(a) R (b) [1, 4] (c) (d)
9 3
(c) [3,7] (d) [2, 6]
43. If 0 £ x < 2p , then the number of real values of x, which
38. If [x] denotes the greatest integer < x, then the system of
satisfy the equation
linear equations
cos x + cos 2x + cos 3x + cos 4x = 0 is: [2016]
[sin q] x + [–cos q] y = 0 (a) 7 (b) 9
[cot q] x + y = 0 [April 12, 2019 (II)] (c) 3 (d) 5
p 2p ö 44. The number of x Î [0, 2p] for which
(a) have infinitely many solutions if qÎ æç , ÷ and
è2 3 ø
2 sin 4 x + 18cos 2 x - 2 cos 4 x + 18sin 2 x = 1 is
æ 7p ö
has a unique solution if qÎ ç p, ÷. [Online April 9, 2016]
è 6 ø
(a) 2 (b) 6
p 2p ö æ 7 p ö
(b) has a unique solution if q Î æç ,
(c) 4 (d) 8
÷ È ç p, ÷. The number of values of a in [0, 2p] for which
è2 3 ø è 6 ø 45.
2sin3a – 7 sin2 a + 7 sin a = 2, is: [Online April 9, 2014]
æ p 2p ö (a) 6 (b) 4
(c) has a unique solution if qÎ ç , ÷ and have
è2 3 ø (c) 3 (d) 1
46. Let A = {q : sin(q) = tan(q)} and B = {q : cos(q) = 1} be two
æ 7p ö
infinitely many solutions if qÎ ç p, ÷. sets. Then : [Online April 25, 2013]
è 6 ø (a) A = B
(d) have infinitely many solutions if (b) A Ë B
æ p 2p ö æ 7 p ö (c) B Ë A
qÎ ç , ÷ È ç p, ÷
è2 3 ø è 6 ø (d) A Ì B and B - A ¹ f
EBD_8344
M-14 Mathematics

The possible values of q Î( 0, p) such that


47. The number of solutions of the equation
50.
sin 2x – 2 cos x + 4 sin x = 4 in the interval [0, 5p] is :
[Online April 23, 2013] sin ( q) + sin ( 4q) + sin ( 7q) = 0 are [2011RS]
(a) 3 (b) 5
(c) 4 (d) 6 π 5π π 2π 3π 8π
(a) , , , , ,
48. Statement-1: The number of common solutions of the 4 12 2 3 4 9
trigonometric equations 2 sin 2 q – cos 2q = 0 and
2 cos2 q – 3 sin q = 0 in the interval [0, 2p] is two. 2p p p 2p 3p 35p
(b) , , , , ,
Statement-2: The number of solutions of the equation, 2 9 4 2 3 4 36
cos2 q – 3 sin q = 0 in the interval [0, p] is two.
2p p p 2p 3p 8p
[Online April 22, 2013] (c) , , , , ,
9 4 2 3 4 9
(a) Statement-1 is true; Statement-2 is true; Statement-2
is a correct explanation for statement-1. 2π π 4π π 3π 8π
(b) Statement-1 is true; Statement-2 is true; Statement-2 (d) , , , , ,
9 4 9 2 4 9
is not a correct explanation for statement-1.
51. The number of values of x in the interval [0, 3p] satisfying
(c) Statement-1 is false; Statement-2 is true.
(d) Statement-1 is true; Statement-2 is false. the equation 2sin 2 x + 5 sin x - 3 = 0 is [2006]
49. The equation esinx – e–sinx – 4 = 0 has : [2012] (a) 4 (b) 6
(a) infinite number of real roots (c) 1 (d) 2
(b) no real roots 52. The number of solution of tan x + sec x = 2cos x in [0, 2 p )
(c) exactly one real root is [2002]
(a) 2 (b) 3
(d) exactly four real roots
(c) 0 (d) 1
Principle of Mathematical Induction M-23

4
Principle of
Mathematical Induction
(a) Principle of mathematical induction can be used to
Problems Based on Sum of Series, prove the formula
TOPIC Ć Problems Based on Inequality and
Divisibility (b) S ( K ) Þ S ( K + 1)

1. Consider the statement: “P(n) : n2 – n + 41 is prime.” Then (c) S ( K ) Þ


/ S ( K + 1)
which one of the following is true? [Jan. 10, 2019 (II)]
(d) S (1) is correct
(a) Both P(3) and P(5) are true.
(b) P(3) is false but P(5) is true.
(c) Both P(3) and P(5) are false. 3. If an = 7 + 7 + 7 + ... ... having n radical signs then by
(d) P(5) is false but P(3) is true.
methods of mathematical induction which is true [2002]
2. Let S ( K ) = 1 + 3 + 5... + (2 K - 1) = 3 + K 2 . Then which of (a) an > 7 " n ³ 1 (b) an < 7 " n ³ 1
the following is true [2004]
(c) an < 4 " n ³ 1 (d) an < 3 " n ³ 1
Complex Numbers and Quadratic Equations M-25

5
Complex Numbers and
Quadratic Equations
z -i
Integral Powers of lota, Algebraic 6. Let z be a complex number such that =1
Operations of Complex Numbers, z + 2i
TOPIC Ć Conjugate, Modulus and Argument 5
or Amplitude of a Complex Number and z = . Then the value of |z + 3i| is :
2
[Jan. 9, 2020 (I)]
3 + i sin q
1. If , q Î [0, 2p], is a real number, then an argument 7 15
4 - i cos q (a) 10 (b) (c) (d) 2 3
2 4
of sinq + icosq is: [Jan. 7, 2020 (II)]
7. If z be a complex number satisfying |Re(z)| + |Im(z)| = 4,
-1 æ 4 ö -1 æ 3 ö
(a) p - tan çè ÷ø (b) p - tan çè ÷ø then |z| cannot be: [Jan. 9, 2020 (II)]
3 4
17
(a) (b) (c) (d)
-1 æ 3 ö -1 æ
4ö 10 7 8
(c) - tan çè ÷ø
2
(d) tan çè ÷ø
4 3 2z - n
8. Let z Î C with Im(z) = 10 and it satisfies = 2i - 1 for
2. If the four complex numbers z, z , z - 2Re( z ) and 2z + n
z - 2Re( z ) represent the vertices of a square of side some natural number n. Then : [April 12, 2019 (II)]
4 units in the Argand plane, then |z| is equal to : (a) n = 20 and Re(z) = –10
[Sep. 05, 2020 (I)] (b) n = 40 and Re(z) = 10
(a) 4 2 (b) 4 (c) 2 2 (d) 2 (c) n = 40 and Re(z) = –10
30 (d) n = 20 and Re(z) = 10
æ -1 + i 3 ö
3. The value of çç ÷÷ is : [Sep. 05, 2020 (II)] 9. The equation z - i = z - 1 , i = -1 , represents:
è 1- i ø
[April 12, 2019 (I)]
(a) – 215 (b) 215 i (c) – 215 i (d) 65
m/2 n/3
1
æ1+ iö æ1+ iö (a) a circle of radius .
If ç =ç = 1, (m, n Î N ) , then the 2
è 1 - i ÷ø è i - 1÷ø
4.
(b) the line through the origin with slope 1.
greatest common divisor of the least values of m and n is (c) a circle of radius 1.
_________. [Sep. 03, 2020 (I)] (d) the line through the origin with slope – 1.
5. If z1, z2 are complex numbers such that Re(z1) = |z1 – 1|,
(1 + i )2 2
p 10. If a > 0 and = , has magnitude , then is
Re(z2) = |z2 – 1| and arg( z1 - z2 ) = , then Im( z1 + z2 ) is a -i 5
6
equal to : [April 10, 2019 (I)]
equal to : [Sep. 03, 2020 (II)]
1 3 3 1
2 3 1 (a) - - i (b) - - i
(a) (b) 2 3 (c) (d) 5 5 5 5
3 2 3
1 3 1 3
(c) - i (d) - + i
5 5 5 5
EBD_8344
M-26 Mathematics

11. If z and w are two complex numbers such that zw = 1 and 1 + (1 – 8a) z
18. The set of all a Î R, for which w = is a purely
1– z
p
arg(z) – arg(w) =
2
, then: [April 10, 2019 (II)] imaginary number, for all z Î C satisfying |z| = 1 and
Re z ¹ 1, is [Online April 15, 2018]
-1 + i (a) {0} (b) an empty set
(a) zw = i (b) z w =
2
ì 1 1ü
(c) í0, , – ý (d) equal to R
1- i î 4 4þ
(c) zw = -i (d) z w =
2
2 + 3isin q
5 + 3z 19. A value ofqfor which is purely imaginary, is:
1 - 2isin q
12. Let z Î C be such that |z| < 1. If w = , then :
5(1 - z ) [2016]
[April 09, 2019 (II)]
-1 æ 3 ö -1 æ 1 ö
(a) 5 Re (w) > 4 (b) 4 Im (w) > 5 (a) sin çç 4 ÷÷ (b) sin ç ÷
(c) 5 Re (w) > 1 (d) 5 Im (w) < 1 è ø è 3ø

z -a p p
13. If (a Î R ) is a purely imaginary number and | | = 2, (c) (d)
z +a 3 6
then a value of a is : [Jan. 12, 2019 (I)] 20. If is a non-real complex number, then the minimum
1 lmz 5
(a) 2 (b) 1 (c)
(d) 2 value of is : [Online April 11, 2015]
2 (lmz )5
14. Let 1 and 2 be two complex numbers satisfying | 1| = 9
(a) –1 (b) –4 (c) –2 (d) –5
and | 2 | – |3|–|4i||=|4. Then the minimum value of
| 1 – 2| is : [Jan. 12, 2019 (II)] 21. If z is a complex number such that z ³ 2, then the minimum
(a) 0 (b) 2 (c) 1 (d) 2
1
value of z + : [2014]
15. Let be a complex number such that + =3+i 2

( where i = -1 . ) (a) is strictly greater than


5
2
Then | | is equal to : [Jan. 11, 2019 (II)]
3 5
34 5 41 5 (b) is strictly greater than but less than
(a) (b) (c) (d) 2 2
3 3 4 4
Let 1 and 5
16. 2 be any two non- ero complex numbers such (c) is equal to
2
3 2
that 3 | = 1
+ 2 (d) lie in the interval (1, 2)
1|=4| 2 |. If 2 3 then:
2 1 22. For all complex numbers of the form 1 + ia, a Î R , if
[Jan. 10 2019 (II)] 2 = x + iy, then [Online April 19, 2014]
(a) y2 – 4x + 2 = 0 (b) y2 + 4x – 4 = 0
5
(a) Re( ) = 0 (b) | | = (c) y2 – 4x – 4 = 0 (d) y2 + 4x + 2 = 0
2
-i
1 17 23. Let ¹ – i be any complex number such that is a
(c) | | = (d) Im( ) = 0 +i
2 2 purely imaginary number.
ì æ p ö 3 + 2isin q ü +
1
17. Let A= íqÎ ç - , p ÷ : is purely imaginary ý . Then is: [Online April 12, 2014]
î è 2 ø 1 - 2isin q þ
(a) ero
Then the sum of the elements in A is: [Jan. 9 2019 (I)]
(b) any non- ero real number other than 1.
5p 3p 2p (c) any non- ero real number.
(a) (b) p (c) (d)
6 4 3 (d) a purely imaginary number.
Complex Numbers and Quadratic Equations M-27

24. If 1, 2 and 3, 4 are 2 pairs of complex conugate numbers, 2 2


then
30. z1 + z2 + z1 - z2 is equal to [Online May 26, 2012]

æ ö æ ö
arg ç 1 ÷ + arg ç 2 ÷ equals: [Online April 11, 2014]
(a) 2 ( z1 + z2 ) (
(b) 2 z1 + z2
2 2
)
è 4ø è 3ø
(c) z1 z2 (d) z1 2 + z2 2
p 3p 31. Let Z and W be complex numbers such that |Z| = |W|, and
(a) 0 (b) (c) (d) p
2 2 arg Z denotes the principal argument of Z.
25. Let w (Im w ¹ 0) be a complex number. Then the set of all [Online May 19, 2012]
complex number satisfying the equation Statement 1:If arg Z + arg W = p, then Z = -W .
w - w = k (1 - ) , for some real number k, is Statement 2: |Z| = |W|, implies arg Z – arg W = p.
(a) Statement 1 is true, Statement 2 is false.
[Online April 9, 2014] (b) Statement 1 is true, Statement 2 is true, Statement 2 is
(a) { : = 1} (b) { : = } a correct explanation for Statement 1.
(c) Statement 1 is true, Statement 2 is true, Statement 2 is
(c) { : ¹ 1} (d) { : = 1, ¹ 1}
not a correct explanation for Statement 1.
(d) Statement 1 is false, Statement 2 is true.
26. If z is a complex n umber of unit modulus and 32. Let Z1 and Z2 be any two complex number.
æ 1+ z ö Statement 1: Z1 - Z 2 ³ Z1 - Z 2
argument q, then arg ç
è 1 + z ÷ø
equals: [2013]
Statement 2: Z1 + Z 2 £ Z1 + Z 2 [Online May 7, 2012]
p (a) Statement 1 is true, Statement 2 is true, Statement 2 is
(a) –q (b) –q (c) q (d) p – q a correct explanation of Statement 1.
2 (b) Statement 1 is true, Statement 2 is true, Statement 2 is
27. Let z satisfy| z | = 1 and = 1– z . not a correct explanation of Statement 1.
(c) Statement 1 is true, Statement 2 is false.
Statement 1 : z is a real number. (d) Statement 1 is false, Statement 2 is true.
p 33. The number of complex numbers such that
Statement 2 : Principal argument of is |z – 1| = |z + 1| = |z – i| equals [2010]
3
(a) 1 (b) 2 (c) ¥ (d) 0
[Online April 25, 2013]
1
(a) Statement 1 is true Statement 2 is true; Statement 2 is 34. The conugate of a complex number is then that
a correct explanation for Statement 1. i –1
(b) Statement 1 is false; Statement 2 is true complex number is [2008]
(c) Statement 1 is true, Statement 2 is false. –1 1 –1 1
(a) (b) (c) (d)
(d) Statement 1 is true; Statement 2 is true; Statement 2 is i –1 i +1 i +1 i –1
not a correct explanation for Statement 1. 1
æx yö
= p + iq, then çè p + q ÷ø ( p + q )
2 2
35. If z = x - i y and z3
æ 1+ z2 ö
28. Let a = Im ç ÷ , where z is any non- ero complex
ç 2iz ÷ is equal to [2004]
è ø (a) –2 (b) –1 (c) 2 (d) 1
number. [Online April 23, 2013] 36. Let and w be complex numbers such that z + i w = 0
The set A = {a : | z | = 1 and z ¹ ±1 } is equal to: and arg zw = p. Then arg equals [2004]
(a) (– 1, 1) (b) [– 1, 1] (c) [0, 1) (d) (– 1, 0] 5p p 3p p
(a) (b) (c) (d)
Z2 4 2 4 4
29. If Z1 ¹ 0 and Z2 be two complex numbers such that
Z1 æ1+ i ö
x
37. If ç ÷ = 1 then [2003]
è1- i ø
2Z1 + 3Z 2
is a purely imaginary number, then 2Z - 3Z is equal to: (a) x = 2n + 1 , where n is any positive integer
1 2 (b) x = 4n , where n is any positive integer
[Online April 9, 2013] (c) x = 2n , where n is any positive integer
(a) 2 (b) 5 (c) 3 (d) 1 (d) x = 4n + 1 , where n is any positive integer..
EBD_8344
M-28 Mathematics

38. If z and w are two non- ero complex numbers such that
æ z -1 ö
p 46. If Re çè ÷ = 1, where z = x + iy, then the point (x, y) lies
zw = 1 and Arg ( z ) - Arg (w ) = , then zw is equal to 2z + i ø
2 on a: [Jan. 7, 2020 (I)]
[2003]
(a) – 1 (b) 1 (c) – i (d) i æ 1 3ö
(a) circle whose centre is at çè - , - ÷ø .
39. If | – 4 | < | – 2 |, its solution is given by [2002] 2 2
(a) Re(z) > 0 (b) Re(z) < 0
2
(c) Re(z) > 3 (d) Re(z) > 2 (b) straight line whose slope is - .
40. z and w are two non ero complex numbers such that 3
| z | = | w| and Arg + Arg w = p then equals [2002] 3
(a) w (b) – w (c) w (d) – w (c) straight line whose slope is
2
.

Rotational Theorem, Square Root 5


of a Complex Number, Cube Roots (d) circle whose diameter is .
2
TOPIC n of Unity, Geometry of Complex
Numbers, De-moiver’s Theorem,
Powers of Complex Numbers 47. If z = 3 + i ( i = -1 ) , then (1 + iz + z5 + iz8)9 is equal
2 2
41. Let z = x + iy be a non- ero complex number such that to: [April 08, 2019 (II)]
(a) 0 (b) 1
z2 = i |z|2, where i = -1 , then z lies on the:
(c) (– 1 + 2i)9 (d) – 1
[Sep. 06, 2020 (II)]
(a) line, y = –x (b) imaginary axis 3
x + iy
(c) line, y = x (d) real axis 48.
æ
è
1 ö
Let ç -2 - i ÷ =
3 ø 27
( )
i = -1 , where x and y are real
42. If a and b are real numbers such that (2 + a )4 = a + ba , numbers then y – x equals : [Jan. 11, 2019 (I)]
(a) 91 (b) – 85 (c) 85 (d) – 91
-1 + i 3
where a = , then a + b is equal to : 5 5
2 æ 3 iö æ 3 iö
[Sep. 04, 2020 (II)] 49. Let z = çç + ÷÷ + çç - ÷÷ . If R(z) and I(z)
(a) 9 (b) 24 (c) 33 (d) 57 è 2 2ø è 2 2ø
respectively denote the real and imaginary parts of z,
3
æ 2p 2p ö then: [Jan. 10, 2019 (II)]
1 + sin + i cos
ç 9 9 ÷ is : (a) I(z) = 0 (b) R(z) > 0 and I(z) > 0
43. The value of ç
2p 2p ÷ (c) R(z) < 0 and I(z) > 0 (d) R(z) = – (c)
ç 1 + sin - i cos ÷
è 9 9 ø n
æ1+ i 3 ö
[Sep. 02, 2020 (I)] 50. The least positive integer n for which çç ÷÷ = 1, is
è1 – i 3 ø
1 1
(a) (1 - i 3) (b) ( 3 - i) [Online April 16, 2018]
2 2
(a) 2 (b) 6 (c) 5 (d) 3
1 1 51. The point represented by 2 + i in the Argand plane moves
(c) - ( 3 - i ) (d) - (1 - i 3)
2 2 1 unit eastwards, then 2 units northwards and finally from
44. The imaginary part of (3 + 2 -54)1/ 2 - (3 - 2 -54)1/ 2 there 2 2 units in the south–westwards direction. Then
its new position in the Argand plane is at the point
can be : [Sep. 02, 2020 (II)]
represented by : [Online April 9, 2016]
(a) - 6 (b) -2 6 (c) 6 (d) 6 (a) 1 + i (b) 2 + 2i (c) –2 – 2i (d) –1 – i
100 100 52. A complex number is said to be unimodular if | | = 1.
-1 + i 3
. If a = (1 + a) å a and b = å a ,
2k 3k
45. Let a =
2 k =0 k =0 1 -2 2
Suppose 1 and 2 are complex numbers such that 2- 1
then a and b are the roots of the quadratic equation: 2

[Jan. 8, 2020 (II)] is unimodular and 2 is not unimodular. Then the point 1
(a) x2 + 101x + 100 = 0 (b) x2 – 102 x + 101 = 0 lies on a: [2015]
(c) x2 – 101x + 100 = 0 (d) x2 + 102x +101 = 0
Complex Numbers and Quadratic Equations M-29

(a) circle of radius 2.


Solutions of Quadratic Equations,
(b) circle of radius 2.
Sum and Product of Roots, Nature
(c) straight line parallel to x-axis TOPIC Đ of Roots, Relation Between Roots
(d) straight line parallel to y-axis. and Co-efficients, Formation of an
2 Equation with Given Roots.
53. If z ¹ 1 and z is real, then the point represented by the
z -1 61. If a and b be two roots of the equation x2 – 64x + 256 = 0.
complex number z lies : [2012]
(a) either on the real axis or on a circle passing through 1 1
the origin. æ a 3 ö 8 æ b3 ö 8
Then the value of ç ÷ + ç ÷ is: [Sep. 06, 2020 (I)]
(b) on a circle with centre at the origin è b5 ø è a5 ø
(c) either on the real axis or on a circle not passing through
the origin. (a) 2 (b) 3 (c) 1 (d) 4
(d) on the imaginary axis. 62. If a and b are the roots of the equation 2x(2x + 1) = 1, then
b is equal to: [Sep. 06, 2020 (II)]
54. If w( ¹ 1) is a cube root of unity, and (1 + w )7 = A + Bw. (a) 2a(a + 1) (b) –2a(a + 1)
Then (A, B) equals [2011] (c) 2a(a – 1) (d) 2a2
(a) (1, 1) (b) (1, 0) (c) (–1, 1) (d) (0, 1) 63. The product of the roots of the equation 9x2 – 18| x | + 5 = 0,
55. If | z + 4 | £ 3, then th e maximum value of is : [Sep. 05, 2020 (I)]
| z + 1 | is [2007] 5 25 5 25
(a) 6 (b) 0 (c) 4 (d) 10 (a) (b) (c) (d)
9 81 27 9
z If a and b are the roots of the equation, 7 x 2 - 3 x - 2 = 0,
56. If w = and | w | = 1, then lies on [2005] 64.
1
z- i a b
3 the the value of + is equal to :
(a) an ellipse (b) a circle 1- a 2
1 - b2
(c) a straight line (d) a parabola [Sep. 05, 2020 (II)]
57. If z1 and z2 are two non- ero complex numbers such that 27 1 3 27
(a) (b) (c) (d)
| z1 + z2 | = | z1 | + | z2 | , then arg z1 – arg z2 is equal to 32 24 8 16
[2005] 2z + i
65. Let u = , z = x + iy and k > 0. If the curve
p -p z - ki
(a) (b) – p (c) 0 (d)
2 2 represented by Re(u) + Im(u) = 1 intersects the y-axis at the
points P and Q where PQ = 5, then the value of k is :
58. If the cube roots of unity are 1, w , w 2 then the roots of [Sep. 04, 2020 (I)]
the equation ( x –1)3 + 8 = 0, are [2005] (a) 3/2 (b) 1/2 (c) 4 (d) 2
66. Let l ¹ 0 be in R. If a and b are roots of the equation,
(a) –1, –1 + 2 w , – 1 – 2 w2
(b) –1, – 1, – 1 x 2 - x + 2l = 0 and a and g are the roots of the equation,
bg
(c) – 1, 1 – 2 w , 1 – 2 w2 3 x 2 - 10 x + 27 l = 0, then is equal to :
l
(d) – 1, 1 + 2 w , 1 + 2 w
2
[Sep. 04, 2020 (II)]
59. If | z 2 - 1|=| z |2 +1, then lies on [2004] (a) 27 (b) 18 (c) 9 (d) 36
67. If a and b are the roots of the equation x2 + px + 2 = 0 and
(a) an ellipse (b) the imaginary axis
(c) a circle (d) the real axis 1 1
and are the roots of the equation 2x2 + 2qx + 1 = 0,
60. The locus of the centre of a circle which touches the circle a b
| z – z1 | = a and | – 2 | = b externally (z, z1 & z2 are complex
numbers) will be [2002] æ 1 öæ 1 öæ 1 öæ 1ö
then ç a - ÷ ç b - ÷ ç a + ÷ ç b + ÷ is equal to :
(a) an ellipse (b) a hyperbola è a øè b øè b øè aø
(c) a circle (d) none of these [Sep. 03, 2020 (I)]
EBD_8344
M-30 Mathematics

9 9 77. Let p, q Î R. If 2 – 3 is a root of the quadratic equation,


(a) (9 + q 2 ) (b) (9 - q 2 )
4 4 x2 + px + q = 0, then: [April 9, 2019 (I)]
9 9 (a) p2 – 4q + 12 = 0 (b) q2 – 4p – 16 = 0
(c) (9 + p 2 ) (d) (9 - p 2 ) (c) q2 + 4p + 14 = 0 (d) p2 – 4q – 12 = 0
4 4
78. If m is chosen in the quadratic equation
68. The set of all real values of l for which the quadratic
(m2 + 1) x2 – 3x + (m2 + 1)2 = 0
equations, (l 2 + 1) x 2 - 4lx + 2 = 0 always have exactly such that the sum of its roots is greatest, then the absolute
one root in the interval (0, 1) is : [Sep. 03, 2020 (II)] difference of the cubes of its roots is: [April 09, 2019 (II)]
(a) (0, 2) (b) (2, 4] (c) (1, 3] (d) (–3, –1)
(a) 10 5 (b) 8 3 (c) 8 5 (d) 4 3
69. Let a and b be the roots of the equation, 5 x 2 + 6 x - 2 = 0. 79. The sum of the solutions of the equation
If Sn = a + b , n = 1, 2,3, ..., then : [Sep. 02, 2020 (I)]
( )
n n
x -2 + x x - 4 + 2 = 0, (x > 0) is equal to:
(a) 6S6 + 5S5 = 2S4 (b) 6S6 + 5S5 + 2S4 = 0
[April 8, 2019 (I)]
(c) 5S6 + 6S5 = 2S4 (d) 5S6 + 6S5 + 2S4 = 0 (a) 9 (b) 12 (c) 4 (d) 10
70. The number of real roots of the equation, 80. If a and b be the roots of the equation x2 – 2x + 2 = 0, then
e4x + e3x – 4e2x + ex + 1 = 0 is: [Jan. 9, 2020 (I)]
n
(a) 1 (b) 3 (c) 2 (d) 4 æaö
the least value of n for which ç ÷ = 1 is :
71. The least positive value of ‘a’ for which the equation, èbø
33 [April 8, 2019 (I)]
2x2 + (a – 10)x + = 2a has real roots is _______.
2 (a) 2 (b) 5 (c) 4 (d) 3
[Jan. 8, 2020 (I)] 81. If l be the ratio of the roots of the quadratic equation in x,
72. If the equation, x2 + bx + 45 = 0 (b Î R) has conugate 3m2x2 + m(m – 4)x + 2 = 0, then the least value of m for
complex roots and they satisfy |z + l| = 2 10 , then: 1
[Jan. 8, 2020 (I)] which l + = 1, is : [Jan. 12, 2019 (I)]
l
(a) b2 – b = 30 (b) b2 + b = 72
(c) b2–b = 42 (d) b2 + b = 12 (a) 2 - 3 (b) 4 - 3 2
73. Let a and b be the roots of the equation x2 – x – l = 0. If
pk = (a)k + (b)k, k ³ l, then which one of the following (c) –2 + 2 (d) 4 - 2 3
statements is not true ? [Jan. 7, 2020 (II)] 82. If one real root of the quadratic equation 81x2 + kx + 256 = 0
(a) p3 = p5 – p4 is cube of the other root, then a value of k is :
(b) P5 = 11 [Jan. 11, 2019 (I)]
(c) (p1 + p2 + p3 + p4 + p5) = 26 (a) –81 (b) 100 (c) 144 (d) – 300
(d) p5 = p2 × p3 83. Consider the quadratic equation (c – 5)x2 – 2cx + (c – 4) = 0,
74. Let a and b be two real roots of the equation (k +1) tan2x
c ¹ 5. Let S be the set of all integral values of c for which
– 2 . ltan x = (1 – k), where k(¹ –1) and l are real numbers. one root of the equation lies in the interval (0, 2) and its
If tan2(a + b) = 50, then a value of l is: [Jan. 7, 2020 (I)] other root lies in the interval (2, 3). Then the number of
(a) 10 2 (b) 10 (c) 5 (d) 5 2 elements in S is: [Jan. 10, 2019 (I)]
75. If a and b are the roots of the quadratic equation, x2 + x sin q (a) 18 (b) 12 (c) 10 (d) 11
84. The value of l such that sum of the squares of the roots
æ pö a12 + b12 of the quadratic equation, x2 + (3 – l)x + 2 = l has the
– 2sinq = 0 0, qÎ ç 0, ÷ , then is
è 2ø (a -12 + b-12 )(a - b)24 least value is: [Jan. 10, 2019 (II)]
equal to : [April 10, 2019 (I)] 15 4
(a) (b) 1 (c) (d) 2
212
212 8 9
(a)
(sin q - 4)12
(b)
(sin q + 8)12 85. Let a and b be two roots of the equation x2 + 2x + 2 = 0, then
a15 + b15 is equal to: [Jan. 9, 2019 (I)]
212 26 (a) – 256 (b) 512 (c) – 512 (d) 256
(c)
(sin q - 8)6
(d)
(sin q + 8)12 86. The number of all possible positive integral values of a
for which the roots of the quadratic equation,
76. The number of real roots of the equation
6x2 – 11x + a = 0 are rational numbers is:
5 + 2 x - 1 = 2 x (2 x - 2) is: [April 10, 2019 (II)] [Jan. 09, 2019 (II)]
(a) 3 (b) 2 (c) 4 (d) 5
(a) 3 (b) 2 (c) 4 (d) 1
Complex Numbers and Quadratic Equations M-31

87. If both the roots of the quadratic equation x2 – mx + 4 = 0 (a) p(b) = 11 (b) p(b) = 19
are real and distinct and they lie in the interval [1, 5], (c) p(–2) = 19 (d) p(–2) = 11
then m lies in the interval: [Jan. 09, 2019 (II)] 97. The sum of all real values of x satisfying the equation
(a) (– 5, – 4) (b) (4, 5) 2
+ 4x -60
(c) (5, 6) (d) (3, 4) (x 2 - 5x + 5) x = 1 is : [2016]
88. Let z0 be a root of the quadratic equation, (a) 6 (b) 5 (c) 3 (d) – 4
x2 + x + 1 = 0. If z = 3 + 6i z081 – 3i z093, then arg z is 98. If x is a solution of the equation,
equal to: [Jan. 09, 2019 (II)]
æ 1ö
p p p 2x + 1 - 2x - 1 = 1, ç x ³ ÷ , then
(a) (b) (c) (d) 0 è 2ø 4x 2 - 1 is equal
4 6 3 to : [Online April 10, 2016]
89. Let p, q and r be real numbers (p ¹ q, r ¹ 0), such that the
3 1
1 1 1 (a) (b) (c) 2 2 (d) 2
roots of the equation + = are equal in 4 2
x+ p x+q r 99. Let a and b be the roots of equation x2 – 6x – 2 = 0. If an =
magnitude but opposite in sign, then the sum of squares
a10 - 2a 8
of these roots is equal to. [Online April 16, 2018] an – bn, for n ³ 1, then the value of 2a 9 is equal to:
(a) p2 + q2 + r2 (b) p2 + q2
[2015]
p2 + q2 (a) 3 (b) – 3 (c) 6 (d) – 6
(c) 2 (p2 + q2) (d)
2 100. If the two roots of the equation, (a – 1)(x4 + x2 + 1) +
90. If an angle A of a D ABC satisfies 5 cos A + 3 = 0, then the (a + 1)(x2 + x + 1)2 = 0 are real and distinct, then the set
roots of the quadratic equation, 9x2 + 27x + 20 = 0 are. of all values of ‘a’ is : [Online April 11, 2015]
[Online April 16, 2018]
æ 1ö æ 1 ö æ 1ö
(a) sin A, sec A (b) sec A, tan A (a) ç 0, ÷ (b) ç - , 0 ÷ È ç 0, ÷
(c) tan A, cos A (d) sec A, cot A è 2ø è 2 ø è 2ø
91. If tan A and tan B are the roots of the quadratic equation,
æ 1 ö
3x2 – 10x – 25 = 0 then the value of (c) ç - , 0 ÷ (d) (– ¥, –2) È (2, ¥)
3 sin2 (A + B) – 10 sin (A + B). cos (A + B) – 25 cos2 (A + B) è 2 ø
is [Online April 15, 2018] 101. If 2 + 3i is one of the roots of the equation 2x3 – 9x2 + kx – 13
(a) 25 (b) – 25 (c) – 10 (d) 10 = 0, k Î R, then the real root of this equation :
92. If f (x) is a quadratic expression such that f (a) + f (b) = 0, [Online April 10, 2015]
and – 1 is a root of f (x) = 0, then the other root of f (x) = 0 1
is [Online April 15, 2018] (a) exists and is equal to – .
2
5 8 5 8
(a) - (b) - (c) (d) 1
8 5 8 5 (b) exists and is equal to .
2
93. If a, b Î C are the distinct roots, of the equation (c) exists and is equal to 1.
x 2 - x + 1 = 0 , then a101 + b107 is equal to : [2018] (d) does not exist.
102. If a Î R and the equation
(a) 0 (b) 1 (c) 2 (d) – 1
- 3 ( x - [ x ]) + 2 ( x - [ x ]) + a 2 = 0
94. If, for a positive integer n, the quadratic equation, 2

x(x + 1) + (x + 1) (x + 2) + ..... + (x + n - 1 ) (x + n) = 10n (where [x] denotes the greatest integer £ x ) has no integral
has two consecutive integral solutions, then n is equal to: solution, then all possible values of a lie in the interval:
[2017] [2014]
(a) 11 (b) 12 (c) 9 (d) 10
95. The sum of all the real values of x satisfying the equation (a) ( -2, -1) (b) ( -¥, -2 ) È ( 2, ¥ )

2
( )
( x -1) x 2 +5x -50
= 1 is : [Online April 9, 2017]
(c) ( -1, 0 ) È ( 0,1) (d) (1, 2 )
(a) 16 (b) 14 (c) – 4 (d) – 5 103. The equation 3x 2 + x + 5 = x - 3 , where x is real, has;
96. Let p(x) be a quadratic polynomial such that p(0)=1. If p(x)
[Online April 19, 2014]
leaves remainder 4 when divided by x–1 and it leaves
(a) no solution (b) exactly one solution
remainder 6 when divided by x+1; then :
(c) exactly two solution (d) exactly four solution
[Online April 8, 2017]
EBD_8344
M-32 Mathematics

104. The sum of the roots of the equation, 112. The value of k for which the equation
x2 + |2x – 3| – 4 = 0, is: [Online April 12, 2014] (k – 2)x2 + 8x + k + 4 = 0 has both roots real, distinct and
(a) 2 (b) – 2 (c) (d) - 2 negative is [Online May 7, 2012]
2
(a) 6 (b) 3 (c) 4 (d) 1
105. If a and b are roots of the equation,
113. Let for a ¹ a1 ¹ 0,
x 2 - 4 2 kx + 2e 4 lnk - 1 = 0 for some k, and
a2 + b2 = 66, then a3 + b3 is equal to: f ( x ) = ax 2 + bx + c, g ( x) = a1 x 2 + b1 x + c1an = -

+ c1and p ( x ) = f ( x ) - g ( x ) .
[Online April 11, 2014]
= + + ( )= + 1
(a) 248 2 (b) 280 2 (c) -32 2 (d) -280 2
If p ( x ) = 0 only for x = -1 and p (– 2) = 2, then the value
1 1
106. If and are the roots of the equation, of p (b) is : [2011 RS]
a b (a) 3 (b) 9 (c) 6 (d) 18
ax2 + bx + 1 = 0 (a ¹ 0, a, b, Î R), then the equation, 114. Sachin and Rahul attempted to solve a quadratic equation.
Sachin made a mistake in writing down the constant term
( ) ( )
x x + b3 + a 3 - 3abx = 0 as roots : and ended up in roots (4,3). Rahul made a mistake in writing
down coefficient of x to get roots (3,2). The correct roots
[Online April 9, 2014] of equation are : [2011 RS]
3 3 1 1 (a) 6, 1 (b) 4, 3 (c) – 6, – 1 (d) – 4, – 3
(a) a 2 and b 2 (b) ab 2 and a 2b
115. Let a, b be real and z be a complex number. If
3 3 z2 + az + b = 0 has two distinct roots on the line Re z =1,
(c) ab and ab (d) - and b- 2 then it is necessary that : [2011]
a 2

107. If p an d q are non- ero real numbers and (a) b Î (-1, 0) (b) b = 1

a3 + b3 = - p, ab = q, then a quadratic equation whose (c) b Î (1, ¥) (d) b Î (0,1)


116. If a and b are the roots of the equation
a 2 b2 x2 – x + 1 = 0, then a2009 + b2009 = [2010]
roots are , is : [Online April 25, 2013]
b a (a) –1 (b) 1 (c) 2 (d) –2
(a) px2 – qx + p2 = 0 (b) qx2 + px + q2 = 0 117. If the roots of the equation bx2 + cx + a = 0 be imaginary,
(c) px2 + qx + p2 = 0 (d) qx2 – px + q2 = 0 then for all real values of x, the expression 3b2x2 + 6bcx +
2c2 is : [2009]
3p (a) less than 4ab (b) greater than – 4ab
108. If a and b are roots of the equation x 2 + px + = 0,
4 (c) 1ess than – 4ab (d) greater than 4ab
118. If the difference between the roots of the equation
such that | a - b |= 10, then p belongs to the set :
[Online April 22, 2013] x2 + ax + 1 = 0 is less than 5 , then the set of possible
(a) {2, – 5} (b) {– 3, 2} (c) {– 2, 5} (d) {3, – 5} values of a is [2007]
109. If a complex number z statisfies the equation (a) (3, ¥) (b) (- ¥, - 3) (c) (– 3, 3) (d) (-3, ¥)
z + 2 | z + 1| +i = 0 , then | z | is equal to : 119. All the values of m for which both roots of the equation
[Online April 22, 2013] x 2 - 2mx + m 2 - 1 = 0 are greater than – 2 but less than 4,
lie in the interval [2006]
(a) 2 (b) 3 (c) 5 (d) 1
(a) -2 < m < 0 (b) m > 3
110. Let p, q, r Î R and r > p > 0. If the quadratic equation
px2 + qx + r = 0 has two complex roots a and b, then |a| + (c) -1 < m < 3 (d) 1 < m < 4
|b| is [Online May 19, 2012] 120. If the roots of the quadratic equation
(a) equal to1 x 2 + px + q = 0 are tan30 and tan15 ,
(b) less than 2 but not equal to 1
respectively, then the value of 2 + q – p is [2006]
(c) greater than 2 (a) 2 (b) 3 (c) 0 (d) 1
(d) equal to 2
111. If the sum of the square of the roots of the equation 121. If z 2 + z + 1 = 0 , where is complex number, then the value
x2 – (sina – 2) x – (1 + sina) = 0 is least, then a is equal to 2
æ 1ö
2
æ 1ö
2 2
æ 1ö
[Online May 12, 2012] of èç
z + ÷ + ç z 2 + ÷ + ç z3 + ÷ + ......... + æç z 6 + 1ö
÷ is
zø è z2 ø è z3 ø è z6 ø
p p p p [2006]
(a) (b) (c) (d) (a) 18 (b) 54 (c) 6 (d) 12
6 4 3 2
Complex Numbers and Quadratic Equations M-33

131. Difference between the corresponding roots of x2+ax+b=0


p æ Pö
122. In a triangle PQR, Ð R = . If tan ç ÷ and and x2+bx+a=0 is same and a ¹ b, then [2002]
2 è 2ø
(a) a + b + 4 = 0 (b) a + b – 4 = 0
æ Qö (c) a – b – 4 = 0 (d) a – b + 4 = 0
– tan ç ÷ are the roots of ax 2 + bx + c = 0, a ¹ 0 then 132. If a ¹ b but a2 = 5a – 3 and b2 = 5b – 3 then the equation
è 2ø
having a/b and b/a as its roots is [2002]
[2005] 2
(a) 3x – 19x + 3 = 0 2
(b) 3x + 19x – 3 = 0
(a) a = b + c (b) c = a + b (c) 3x2 – 19x – 3 = 0 (d) x2 – 5x + 3 = 0.
(c) b = c (d) b = a + c
123. If the roots of the equation x 2 – bx + c = 0 be two Condition for Common Roots,
Maximum and Minimum value of
consecutive integers, then b 2 – 4c equals [2005] TOPIC Ė Quadratic Equation, Quadratic
(a) – 2 (b) 3 (c) 2 (d) 1 Expression in two Variables,
Solution of Quadratic Inequalities.
124. If one root of the equation x 2 + px + 12 = 0 is 4, while the
133. If 5, 5r, 5r2 are the lengths of the sides of a triangle, then r
2
equation x + px + q = 0 has equal roots , then the value cannot be equal to: [Jan. 10, 2019 (I)]
of ‘q’ is [2004] 3 5 7 3
(a) (b) (c) (d)
49 4 4 4 2
(a) 4 (b) 12 (c) 3 (d)
4 134. Let a, b Î R, a ¹ 0 be such that the equation,
ax2 – 2bx + 5 = 0 has a repeated root a, which is also a root
125. If (1 - p) is a root of quadratic equation
of the equation, x2 – 2bx – 10 = 0. If b is the other root of
x 2 + px + (1 - p) = 0 then its root are [2004] this equation, then a2 + b2 is equal to :[Jan. 9, 2020 (II)]
(a) 25 (b) 26 (c) 28 (d) 24
(a) –1, 2 (b) –1, 1 (c) 0, –1 (d) 0, 1
135. If lÎ R is such that the sum of the cubes of the roots of the
126. The number of real solutions of the equation
equation, x2 + (2 – l) x + (10 – l) = 0 is minimum, then the
x 2 - 3 x + 2 = 0 is [2003] magnitude of the difference of the roots of this equation is
(a) 3 (b) 2 (c) 4 (d) 1 [Online April 15, 2018]
127. The value of 'a' for which one root of the quadratic equation (a) 20 (b) 2 5 (c) 2 7 (d) 4 2
136. If | z – 3 + 2i | £ 4 then the difference between the greatest
(a 2 - 5a + 3) x 2 + (3a - 1) x + 2 = 0 is twice as large as the
value and the least value of | z | is [Online April 15, 2018]
other is [2003]
(a) 13 (b) 2 13 (c) 8 (d) 4 + 13
1 2 2 1
(a) - (b) (c) - (d) x2 x2
137. If the equations + bx – 1 = 0 and + x + b = 0 have a
3 3 3 3
common root different from –1, then |b| is equal to :
128. Let Z1 and Z 2 be two roots of the equation [Online April 9, 2016]
Z 2 + aZ + b = 0 , Z being complex. Further , assume that (a) 2 (b) 3 (c) 3 (d) 2
the origin, Z1 and Z 2 form an equilateral triangle. Then 138. If non- ero real numbers b and c are such that
min f(x) > max g(x), where f(x) = x2 + 2bx + 2c2 and
[2003]
g(x) = – x2 – 2cx + b2 (x Î R);
(a) a 2 = 4b (b) a 2 = b
c
(c) a 2 = 2b (d) a 2 = 3b then lies in the interval: [Online April 19, 2014]
b
129. If p and q are the roots of the equation
x2 + px + q = 0, then [2002] æ 1ö é1 1 ö
(a) ç 0, ÷ (b) ê , ÷
(a) p = 1, q = –2 (b) p = 0, q = 1 è 2ø ë2 2 ø
(c) p = –2, q = 0 (d) p = – 2, q = 1
é 1 ù
130. Product of real roots of the equation
t2 x2 + | x | + 9 = 0 [2002]
(c) ê 2ú
ë 2, û
(d) ( 2, ¥ )
(a) is always positive (b) is always negative
(c) does not exist (d) none of these
EBD_8344
M-34 Mathematics

139. If equations ax2 + bx + c = 0 ( a, b, c Î R, a ¹ 0 ) and 144. The quadratic equations x 2 – 6x + a = 0 and


x2 – cx + 6 = 0 have one root in common. The other roots
2x2 + 3x + 4 = 0 have a common root, then a : b : c equals:
of the first and second equations are integers in the ratio
[Online April 9, 2014]
4 : 3. Then the common root is [2009]
(a) 1 : 2 : 3 (b) 2 : 3 : 4 (c) 4 : 3 : 2 (d) 3 : 2 : 1
140. If the equations x2 + 2x + 3 = 0 and ax2 + bx + c = 0, (a) 1 (b) 4 (c) 3 (d) 2
a,b,c Î R, have a common root, then a : b : c is [2013]
3x 2 + 9 x + 17
(a) 1 : 2 : 3 (b) 3 : 2 : 1 (c) 1 : 3 : 2 (d) 3 : 1 : 2 145. If x is real, the maximum value of is [2006]
3x 2 + 9 x + 7
x-5
141. The least integral value a of x such that
2
>0,
x + 5x - 14 (a)
1
(b) 41 (c) 1 (d)
17
satisfies : [Online April 23, 2013] 4 7
(a) a2 + 3a – 4 = 0 (b) a2 – 5a + 4 = 0 2 2
(c) a2 – 7a + 6 = 0 (d) a2 + 5a – 6 = 0 146. If both the roots of the quadratic equation x - 2 kx + k
142. The values of ‘a’ for which one root of the equation + k – 5 = 0 are less than 5, then k lies in the interval
x2 – (a + 1 ) x + a2 + a – 8 = 0 exceeds 2 and the other is [2005]
lesser than 2, are given by : [Online April 9, 2013] (a) (5, 6] (b) (6, ¥ ) (c) (– ¥ , 4) (d) [4, 5]
(a) 3 < a < 10 (b) a ³ 10 147. The value of a for which the sum of the squares of the
(c) -2 < a < 3 (d) a £ -2 roots of the equation x2 – (a – 2) x – a – 1 = 0 assume the
least value is [2005]
4 (a) 1 (b) 0 (c) 3 (d) 2
143. If z - = 2 , then the maximum value of | | is equal to :
z
[2009]
(a) 5 + 1 (b) 2 (c) 2 + 2 (d) 3 +1
EBD_8344
M-56 Mathematics

6
Linear Inequalities

(a) contains exactly two elements.


Solution of Linear Inequality and (b) is a singleton.
System of Linear Inequalities, (c) is an empty set.
Representation of Solution of (d) contains at least four elements.
Linear Inequality in One Variable
on a Number Line, Representation 5. All the pairs (x,y) that satisfy the inequality
TOPIC Ć of Solution of a Linear Inequality 1
and System of Linear Inequalities 2 sin 2 x - 2sin x + 5. £ 1 also satisfy the equation:
in a Cartesian Plane, Equations and 4sin 2 y
Inequations Involving Absolute
[April 10, 2019 (I)]
Value Functions, Greatest Integer
Functions, Logarithmic Functions (a) 2|sin x| = 3sin y (b) 2 sin x = sin y
(c) sin x = 2 sin y (d) sin x = | sin y |
1. The region represented by {z = x + iy Î C : |z| – Re(z) £ 1} is 6. The number of integral values of m for which the equation
also given by the inequality: [Sep. 06, 2020 (I)] (1 + m2)x2 – 2(1 + 3m)x + (1 + 8m) = 0 has no real root is :

æ 1ö [April 08, 2019 (II)]


(a) y 2 ³ 2( x + 1) (b) y 2 £ 2 ç x + ÷
è 2ø (a) 1 (b) 2
(c) infinitely many (d) 3
1 2
(c) y 2 £ x + (d) y ³ x + 1 7. The number of integral values of m for which the quadratic
2 expression, (1 + 2m)x2 – 2(1 + 3m)x + 4(1 + m), xÎR, is
2. Consider the two sets : always positive, is : [Jan. 12, 2019 (II)]
A = {m Î R : both the roots of x2 – (m + 1)x + m + 4 = 0 (a) 3 (b) 8
are real} and B = [– 3, 5). (c) 7 (d) 6
Which of the following is not true? [Sep. 03, 2020 (I)] x x
æ 3ö æ 4 ö
8. If f ( x ) = ç ÷ + ç ÷ - 1 , x Î R , then the equation
(a) A - B = (-¥, - 3) È (5, ¥) è5ø è5ø
(b) A Ç B = {-3} f(x) = 0 has : [Online April 9, 2014]
(a) no solution
(c) B – A = (–3, 5)
(b) one solution
(d) A È B = R (c) two solutions
3. If A = {x Î R : |x| < 2} and (d) more than two solutions
B = {x Î R : |x – 2| ³ 3}; then : [Jan. 9, 2020 (II)] 9. If a, b, c are distinct +ve real numbers and a2 + b2 + c2 = 1
(a) A Ç B = (–2, –1) (b) B – A = R – (–2, 5) then ab + bc + ca is [2002]
(a) less than 1 (b) equal to 1
(c) A È B = R – (2, 5) (d) A – B = [–1, 2)
(c) greater than 1 (d) any real no.
4. Let S be the set of all real roots of the equation,
3x(3x – 1) + 2 = |3x – 1| + | 3x – 2|. Then S:[Jan. 8, 2020 (II)]
Permutations and Combinations M-59

7
Permutations and
Combinations
6. The number of 6 digit numbers that can be formed using
Fundamental Principle of Counting, the digits 0, 1, 2, 5, 7 and 9 which are divisible by 11 and no
Factorials, Permutations, Counting digit is repeated, is: [April 10, 2019 (I)]
Formula for Permutations, (a) 72 (b) 60
Permutations in Which Things may (c) 48 (d) 36
TOPIC Ć be Repeated, Permutations in
Which all Things are Different, 7. The number of four-digit numbers strictly greater than
Number of Permutations Under 4321 that can be formed using the digits 0, 1, 2, 3, 4, 5
Certain Restricted Conditions, (repetition of digits is allowed) is: [April 08, 2019 (II)]
Circular Permutations
(a) 288 (b) 360
1. Two families with three members each and one family with (c) 306 (d) 310
four members are to be seated in a row. In how many ways 8. Consider three boxes, each containing 10 balls labelled
can they be seated so that the same family members are 1, 2,..., 10. Suppose one ball is randomly drawn from each
not separated? [Sep. 06, 2020 (I)] of the boxes. Denote by n i, the label of the ball drawn from
(a) 2! 3! 4! (b) (3!)3 × (4!) the ith box, (i = 1, 2, 3). Then, the number of ways in which
(c) (3!)2 × (4!) (d) 3! (4!)3 the balls can be chosen such that n1 < n2 < n3 is :
2. The value of (2 × 1P0 - 3 × 2 P1 + 4 × 3 P2 - ... up to 51th term) [Jan. 12, 2019 (I)]
(a) 120 (b) 82
+ (1! – 2! + 3! – ... up to 51th term) is equal to :
(c) 240 (d) 164
[Sep. 03, 2020 (I)]
9. The number of natural numbers less than 7,000 which
(a) 1 – 51(51)! (b) 1 + (51)!
can be formed by using the digits 0,1,3,7,9 (repetition of
(c) 1 + (52)! (d) 1
digits allowed) is equal to: [Jan. 09, 2019 (II)]
3. If the letters of the word 'MOTHER' be permuted and all
(a) 374 (b) 372
the words so formed (with or without meaning) be listed as
in a dictionary, then the position of the word 'MOTHER' is (c) 375 (d) 250
________. [NA Sep. 02, 2020 (I)] 10. Let S be the set of all triangles in the xy-plane, each
4. If the number of five digit numbers with distinct digits having one vertex at the origin and the other two
and 2 at the 10th place is 336 k, then k is equal to: vertices lie on coordinate axes with integral coordinates.
[Jan. 9, 2020 (I)] If each triangle in S has area 50 sq. units, then the
(a) 4 (b) 6
number of elements in the set S is:[Jan. 09, 2019 (II)]
(c) 7 (d) 8
(a) 9 (b) 18
5. Total number of 6-digit numbers in which only and all the
(c) 36 (d) 32
five digits 1, 3, 5, 7 and 9 appear, is: [Jan. 7, 2020 (I)]
11. The number of numbers between 2,000 and 5,000 that can
1 be formed with the digits 0, 1, 2, 3, 4, (repetition of digits is
(a) (6!) (b) 6!
2 not allowed) and are multiple of 3 is?
5 [Online April 16, 2018]
(c) 56 (d) (6!)
2 (a) 30 (b) 48
(c) 24 (d) 36
EBD_8344
M-60 Mathematics

12. n – digit numbers are formed using only three digits 2, 5 (a) 1120 (b) 1880
and 7. The smallest value of n for which 900 such distinct (c) 1960 (d) 1240
numbers can be formed, is [Online April 15, 2018] 21. Two women and some men participated in a chess
(a) 6 (b) 8 tournament in which every participant played two games
with each of the other participants. If the number of games
(c) 9 (d) 7 that the men played between themselves exceeds the
13. The number of ways in which 5 boys and 3 girls can be number of games that the men played with the women by
seated on a round table if a particular boy B1 and a particular 66, then the number of men who participated in the
girl G1 never sit adacent to each other, is : tournament lies in the interval: [Online April 19, 2014]
[Online April 9, 2017] (a) [8, 9] (b) [10, 12)
(a) 5 × 6! (b) 6 × 6! (c) (11, 13] (d) (14, 17)
(c) 7! (d) 5 × 7! 22. 8-digit numbers are formed using the digits 1, 1, 2, 2, 2, 3, 4,
14. If all the words, with or without meaning, are written using 4. The number of such numbers in which the odd digits do
the letters of the word QUEEN and are arranged as in no occupy odd places, is: [Online April 12, 2014]
English dictionary, then the position of the word QUEEN (a) 160 (b) 120
is : [Online April 8, 2017] (c) 60 (d) 48
(a) 44th (b) 45th 23. An eight digit number divisible by 9 is to be formed using
(c) 46th (d) 47th digits from 0 to 9 without repeating the digits. The number
of ways in which this can be done is:
15. If all the words (with or without meaning) having five
[Online April 11, 2014]
letters, formed using the letters of the word SMALL and
(a) 72 (7!) (b) 18 (7!)
arranged as in a dictionary; then the position of the word
(c) 40 (7!) (d) 36 (7!)
SMALL is : [2016]
24. The sum of the digits in the unit’s place of all the 4-digit
(a) 52nd (b) 58th
numbers formed by using the numbers 3, 4, 5 and 6, without
(c) 46th (d) 59th
repetition, is: [Online April 9, 2014]
10
(a) 432 (b) 108
16. The sum å (r 2 + 1) × (r!) is equal to : (c) 36 (d) 18
r =1
25. 5 - digit numbers are to be formed using 2, 3, 5, 7, 9 without
[Online April 10, 2016]
repeating the digits. If p be the number of such numbers
(a) 11 × (11!) (b) 10 × (11!)
that exceed 20000 and q be the number of those that lie
(c) (11!) (d) 101 × (10!)
between 30000 and 90000, then p : q is :
17. If the four letter words (need not be meaningful) are to be
formed using the letters from the word [Online April 25, 2013]
"MEDITERRANEAN" such that the first letter is R and (a) 6 : 5 (b) 3 : 2
the fourth letter is E, then the total number of all such (c) 4 : 3 (d) 5 : 3
words is : [Online April 9, 2016] 26. Assuming the balls to be identical except for difference in
(a) 110 (b) 59 colours, the number of ways in which one or more balls
can be selected from 10 white, 9 green and 7 black balls is:
11!
(c) (d) 56 [2012]
(2!)3
(a) 880 (b) 629
18. The number of points, having both co-ordinates as (c) 630 (d) 879
integers, that lie in the interior of the triangle with vertices
27. If seven women and seven men are to be seated around a
(0, 0), (0, 41) and (41, 0) is : [2015] circular table such that there is a man on either side of
(a) 820 (b) 780 every woman, then the number of seating arrangements is
(c) 901 (d) 861 [Online May 26, 2012]
19. The number of integers greater than 6,000 that can be
(a) 6! 7! (b) (6!)2
formed, using the digits 3, 5, 6, 7 and 8, without repetition, 2
(c) (7!) (d) 7!
is : [2015]
(a) 120 (b) 72 28. If the letters of the word SACHIN are arranged in all possible
ways and these words are written out as in dictionary,
(c) 216 (d) 192 then the word SACHIN appears at serial number [2005]
20. The number of ways of selecting 15 teams from 15 men (a) 601 (b) 600
and 15 women, such that each team consists of a man and
a woman, is: [Online April 10, 2015] (c) 603 (d) 602
Permutations and Combinations M-61

29. How many ways are there to arrange the letters in the 39. The total number of 3-digit numbers, whose sum of digits
word GARDEN with vowels in alphabetical order[2004] is 10, is _________. [NA Sep. 03, 2020 (II)]
(a) 480 (b) 240 40. Let n > 2 be an integer. Suppose that there are n Metro
(c) 360 (d) 120 stations in a city located along a circular path. Each pair of
stations is connected by a straight track only. Further, each
30. The range of the function f ( x) =7 - x Px -3 is [2004]
pair of nearest stations is connected by blue line, whereas
(a) {1, 2, 3, 4, 5} (b) {1, 2, 3, 4, 5, 6} all remaining pairs of stations are connected by red line. If
(c) {1, 2, 3, 4,} (d) {1, 2, 3,} the number of red lines is 99 times the number of blue lines,
31. The number of ways in which 6 men and 5 women can dine then the value of n is : [Sep. 02, 2020 (II)]
at a round table if no two women are to sit together is
given by [2003] (a) 201 (b) 200
(a) 6! × 5! (b) 6 × 5 (c) 101 (d) 199
(c) 30 (d) 5 × 4 41. If Cr º 25Cr and C0 + 5×C1 + 9×C2 + ... + (101)×C25 = 225×k, then
32. The sum of integers from 1 to 100 that are divisible by 2 or k is equal to ________. [NA Jan. 9, 2020 (II)]
42. An urn contains 5 red marbles, 4 black marbles and 3 white
5 is [2002]
marbles. Then the number of ways in which 4 marbles can
(a) 3000 (b) 3050 be drawn so that at the most three of them are red is
(c) 3600 (d) 3250 _________. [NA Jan. 8, 2020 (I)]
33. Number greater than 1000 but less than 4000 is formed 43. If a, b and c are the greatest values of 19Cp, 20Cq and 21Cr
using the digits 0, 1, 2, 3, 4 (repetition allowed). Their respectively, then: [Jan. 8, 2020 (I)]
number is [2002]
a b c a b c
(a) 125 (b) 105 (a) = = (b) = =
(c) 374 (d) 625 11 22 21 10 11 21
34. Total number of four digit odd numbers that can be formed a b c a b c
(c) = = (d) = =
using 0, 1, 2, 3, 5, 7 (using repetition allowed) are [2002] 11 22 42 10 11 42
(a) 216 (b) 375 44. The number of 4 letter words (with or without meaning)
(c) 400 (d) 720 that can be formed from the eleven letters of the word
‘EXAMINATION’ is _________. [NA Jan. 8, 2020 (II)]
Combinations, Counting Formula 45. The number of ordered pairs (r, k) for which 6. 35Cr
for Combinations, Division and = (k2 – 3).36Cr + 1, where k is an integer, is:
Distribution of Objects, [Jan. 7, 2020 (II)]
TOPIC n Dearrangement Theorem, Sum (a) 3 (b) 2
of Numbers, Important Result (c) 6 (d) 4
About Point 46. The number of ways of choosing 10 obects out of 31
obects of which 10 are identical and the remaining 21 are
35. The number of words (with or without meaning) that can
distinct is: [April 12, 2019 (I)]
be formed from all the letters of the word “LETTER” in
which vowels never come together is ______. (a) 220 – 1 (b) 221
[NA Sep. 06, 2020 (II)] (c) 220 (d) 220+1
36. The number of words, with or without meaning, that can 47. A group of students comprises of 5 boys and n girls. If
be formed by taking 4 letters at a time from the letters of the the number of ways, in which a team of 3 students can
word ‘SYLLABUS’ such that two letters are distinct and randomly be selected from this group such that there is at
two letters are alike, is ______. [NA Sep. 05, 2020 (I)]
least one boy and at least one girl in each team, is 1750,
37. There are 3 sections in a question paper and each section
contains 5 questions. A candidate has to answer a total of then n is equal to : [April 12, 2019 (II)]
5 questions, choosing at least one question from each (a) 28 (b) 27
section. Then the number of ways, in which the candidate (c) 25 (d) 24
can choose the questions, is : [Sep. 05, 2020 (II)]
48. Suppose that 20 pillars of the same height have been
(a) 3000 (b) 1500
erected along the boundary of a circular stadium. If the
(c) 2255 (d) 2250
top of each pillar has been connected by beams with the
38. A test consists of 6 multiple choice questions, each having
4 alternative answers of which only one is correct. The top of all its non-adacent pillars, then the total number of
number of ways, in which a candidate answers all six beams is : [April 10, 2019 (II)]
questions such that exactly four of the answers are correct, (a) 170 (b) 180
is __________. [NA Sep. 04, 2020 (II)] (c) 210 (d) 190
EBD_8344
M-62 Mathematics

49. A committee of 11 members is to be formed from 8 males Y together can throw a party inviting 3 ladies and 3 men,
and 5 females. If m is the number of ways the committee is so that 3 friends of each of X and Y are in this party, is :
formed with at least 6 males and n is the number of ways [2017]
the committee is formed with at least 3 females, then: (a) 484 (b) 485
(c) 468 (d) 469
[April 9, 2019 (I)]
n+2
(a) m + n = 68 (b) m = n = 78 C6
(c) n = m – 8 (d) m = n = 68 57. If n-2 = 11, then n satisfies the equation :
P2
50. All possible numbers are formed using the digits 1, 1, 2, 2, [Online April 10, 2016]
2, 2, 3, 4, 4 taken all at a time. The number of such numbers (a) n2 + n – 110 = 0 (b) n2 + 2n – 80 = 0
in which the odd digits occupy even places is : (c) n2 + 3n – 108 = 0 (d) n2 + 5n – 84 = 0
[April 8, 2019 (I)] 15 æ 15 C r ö
The value of å r2 ç ÷
(a) 180 (b) 175
58. ç 15 C ÷ is equal to :
(c) 160 (d) 162 r =1 è r -1 ø
51. There are m men and two women participating in a chess [Online April 9, 2016]
tournament. Each participant plays two games with every (a) 1240 (b) 560
other participant. If the number of games played by the (c) 1085 (d) 680
men between themselves exceeds the number of games 59. Let A and B be two sets containing four and two elements
played between the men and the women by 84, then the respectively. Then the number of subsets of the set A × B,
value of m is [Jan. 12, 2019 (II)] each having at least three elements is : [2015]
(a) 12 (b) 11 (a) 275 (b) 510
(c) 9 (d) 7 (c) 219 (d) 256
60. If in a regular polygon the number of diagonals is 54,
3 then the number of sides of this polygon is
20 æ 20
Ci - 1 ö k
52. If å çç 20 C + 20
÷ = , then k equals:
Ci - 1 ÷ø 21
[Online April 11, 2015]
i =1 è i (a) 12 (b) 6
(c) 10 (d) 9
[Jan. 10, 2019 (I)]
61. Let A and B two sets containing 2 elements and 4 elements
(a) 400 (b) 50
respectively. The number of subsets of A × B having 3 or
(c) 200 (d) 100 more elements is [2013]
53. Consider a class of 5 girls and 7 boys. The number of (a) 256 (b) 220
different teams consisting of 2 girls and 3 boys that can (c) 219 (d) 211
be formed from this class, if there are two specific boys A
62. Let Tn be the number of all possible triangles formed by
and B, who refuse to be the members of the same team, is: oining vertices of an n-sided regular polygon. If
[Jan. 9, 2019 (I)] Tn+1 – Tn = 10, then the value of n is : [2013]
(a) 500 (b) 200 (a) 7 (b) 5
(c) 300 (d) 350 (c) 10 (d) 8
54. The number of four letter words that can be formed using 63. On the sides AB, BC, CA of a DABC, 3, 4, 5 distinct points
the letters of the word BARRACK is (excluding vertices A, B, C) are respectively chosen. The
[Online April 15, 2018] number of triangles that can be constructed using these
(a) 144 (b) 120 chosen points as vertices are : [Online April 23, 2013]
(a) 210 (b) 205
(c) 264 (d) 270 (c) 215 (d) 220
55. From 6 different novels and 3 different dictionaries, 4 64. The number of ways in which an examiner can assign 30
novels and 1 dictionary are to be selected and arranged in marks to 8 questions, giving not less than 2 marks to any
a row on a shelf so that the dictionary is always in the question, is : [Online April 22, 2013]
middle. The number of such arrangements is : [2018] (a) 30 (b) 21
C7 C8
(a) less than 500
(c) 21 (d) 30 C8
(b) at least 500 but less than 750 C7
(c) at least 750 but less than 1000 65. A committee of 4 per sons is to be formed from
(d) at least 1000 2 ladies, 2 old men and 4 young men such that it includes
56. A man X has 7 friends, 4 of them are ladies and 3 are at least 1 lady, at least 1 old man and at most 2 young men.
men. His wife Y also has 7 friends, 3 of them are ladies Then the total number of ways in which this committee
can be formed is : [Online April 9, 2013]
and 4 are men. Assume X and Y have no common
friends. Then the total number of ways in which X and (a) 40 (b) 41
(c) 16 (d) 32
Permutations and Combinations M-63

66. The number of arrangements that can be formed from the (a) at least 500 but less than 750
letters a, b, c, d, e, f taken 3 at a time without repetition and (b) at least 750 but less than 1000
each arrangement containing at least one vowel, is (c) at least 1000
[Online May 19, 2012] (d) less than 500
(a) 96 (b) 128 74. How many different words can be formed by umbling the
(c) 24 (d) 72 letters in the word MISSISSIPPI in which no two S are
67. If n = mC2, then the value of nC2 is given by adacent? [2008]
[Online May 19, 2012] (a) 8. 6C4. 7C4 (b) 6.7. 8C4
(a) 3(m + 1C4) (b) m – 1 C4 (c) 6. 8. 7C4. (d) 7. 6C4. 8C4
(c) m + 1C4 (d) 2(m + 2C4) 75. The set S = {1, 2, 3, ......., 12} is to be partitioned into three
68. Statement 1: If A and B be two sets having p and q elements sets A, B, C of equal si e.
respectively, where q > p. Then the total number of functions Thus A È B È C = S, A Ç B = B Ç C = A Ç C = f. The
from set A to set B is qp. [Online May 12, 2012] number of ways to partition S is [2007]
Statement 2: The total number of selections of p different
obects out of q obects is qCp. 12! 12!
(a) (b)
(a) Statement 1 is true, Statement 2 is false. (4!) 3
(4!) 4
(b) Statement 1 is true, Statement 2 is true, Statement 2 is
not a correct explanation of Statement 1. 12! 12!
(c) (d)
(c) Statement 1 is false, Statement 2 is true 3!(4!) 3
3!(4!) 4
(d) Statement 1 is true, Statement 2 is true, Statement 2 is
a correct explanation of Statement 1. 76. At an election, a voter may vote for any number of
candidates, not greater than the number to be elected.
69. If the number of 5-elemen t subsets of the set There are 10 candidates and 4 are of be selected, if a voter
A= {a1, a2, ...., a20} of 20 distinct elements is k times the votes for at least one candidate, then the number of ways
number of 5-element subsets containing a4, then k is in which he can vote is [2006]
[Online May 7, 2012] (a) 5040 (b) 6210
20 (c) 385 (d) 1110
(a) 5 (b)
7 6
10 77. The value of 50
C4 + å 56 - r C3 is [2005]
(c) 4 (d) r =1
3
70. There are 10 points in a plane, out of these 6 are collinear. 55 55
(a) C4 (b) C3
If N is the number of triangles formed by oining these
points. Then : [2011RS] 56 56
(c) C3 (d) C4
(a) N £ 100 (b) 100 < N £ 140
78. The number of ways of distributing 8 identical balls in 3
(c) 140 < N £ 190 (d) N > 190 distinct boxes so that none of the boxes is empty is [2004]
71. Statement-1: The number of ways of distributing 10
(a) 8 (b) 21
identical balls in 4 distinct boxes such that no box is empty C3
is 9C3 .
(c) 38 (d) 5
Statement-2: The number of ways of choosing any 3 places
from 9 different places is 9C3. [2011] 79. A student is to answer 10 out of 13 questions in an
(a) Statement-1 is true, Statement-2 is true; Statement-2 is examination such that he must choose at least 4 from the
first five questions. The number of choices available to
not a correct explanation for Statement-1. him is [2003]
(b) Statement-1 is true, Statement-2 is false. (a) 346 (b) 140
(c) Statement-1 is false, Statement-2 is true. (c) 196 (d) 280
(d) Statement-1 is true, Statement-2 is true; Statement-2 is
80. If nCr denotes the number of combination of n things
a correct explanation for Statement-1.
72. There are two urns. Urn A has 3 distinct red balls and urn taken r at a time, then the expression
n
B has 9 distinct blue balls. From each urn two balls are Cr +1 + nC r -1 + 2´n Cr equals [2003]
taken out at random and then transferred to the other. The n +1 n+ 2
number of ways in which this can be done is [2010] (a) Cr +1 (b) Cr
(a) 36 (b) 66 (c) n + 2 Cr +1 (d) n +1Cr
(c) 108 (d) 3 81. Five digit number divisible by 3 is formed using 0, 1, 2, 3, 4,
73. From 6 different novels and 3 different dictionaries,4 novels 6 and 7 without repetition. Total number of such numbers
and 1 dictionary are to be selected and arranged in a row are [2002]
on a shelf so that the dictionary is always in the middle. (a) 312 (b) 3125
Then the number of such arrangement is: [2009] (c) 120 (d) 216
EBD_8344
M-72 Mathematics

8
Binomial Theorem
7. If the fourth term in the Binomial expansion of
Binomial Theorem for a Positive 6
Integral Index ‘x’, Expansion of æ2 log x ö
TOPIC Ć ç + x 8 ÷ (x > 0) is 20 × 87, then a value of x is:
Binomial, General 1 èx ø
Term, Coefficient of any Power of ‘x’ [April 9, 2019 (I)]
1. If {p} denotes the fractional part of the number p, then (a) 83 (b) 82
ìï 3200 üï (c) 8 (d) 8–2
í ý , is equal to : [Sep. 06, 2020 (I)] 8. If some three consecutive coefficients in the binomial
îï 8 ïþ expansion of (x + 1)n in powers of x are in the ratio 2:15:70,
5 7 then the average of these three coefficients is:
(a) (b)
8 8 [April 09, 2019 (II)]
3 1 (a) 964 (b) 232
(c) (d)
8 8 (c) 227 (d) 625
2. The natural number m, for which the coefficient of x in the 9. The sum of the co-efficients of all even degree terms in x
22 6 6
æ 1ö æ
binomial expansion of ç x m + 2 ÷ is 1540, is ______. in the expansion of ç x + x 3 - 1 ö÷ + æç x - x3 - 1 ö÷ , (x >
è x ø è ø è ø
[NA Sep. 05, 2020 (I)] 1) is equal to : [April 8, 2019 (I)]
3. The coefficient of x4 in
the expansion of (1 + x + x2 + x3)6 (a) 29 (b) 32
in powers of x, is ____________.[NA Sep. 05, 2020 (II)] (c) 26 (d) 24
20
a7 10. If the fourth term in the binomial expansion of
4. Let (2 x 2 + 3 x + 4)10 = å ar x r . Then is equal to
a13 6
r =0 æ 1
1 ö
___________. [NA Sep. 04, 2020 (I)] ç 1 + log x + x 12 ÷ is equal to 200, and x > 1, then the
If a and b be the coefficients of x4 and x2 respectively in ç x 10 ÷
5. è ø
the expansion of
value of x is: [April 08, 2019 (II)]

(x + ) ( )
6 6
x 2 - 1 + x - x 2 - 1 , then: [Jan. 8, 2020 (II)] (a) 100 (b) 10
(c) 103 (d) 104
(a) a + b = 60 (b) a + b = –30 11. Let (x + 10) + (x – 10) = a0 + a1x + a2x2 + .... + a50x50,
50 50
(c) a – b = 60 (d) a – b = –132
a2
6. The smallest natural number n, such that the coefficient for all x Î R; then is equal to : [Jan. 11, 2019 (II)]
a0
n
æ 2 1 ö
of x in the expansion of ç x + 3 ÷ is nC23 , is : (a) 12.50 (b) 12.00
è x ø (c) 12.25 (d) 12.75

( )
[April 10, 2019 (II)] 5
12. If the third term in the binomial expansion of 1 + x log 2 x
(a) 38 (b) 58
(c) 23 (d) 35 equals 2560, then a possible value of x is:
[Jan. 10, 2019 (I)]
Binomial Theorem M-73

1 21. If the coefficients of x–2 and x–4 in the expansion of


(a) (b) 4 2 18
4 æ 1 ö
ç 3 1 ÷
ç x + 1 ÷ , (x > 0), are m and n respectively, then
1 m
(c) (d) 2 2
8 ç ÷ n
è 2x 3 ø
13. The positive value of l for which the co-efficient of x2
is equal to : [Online April 10, 2016]
10
æ l ö (a) 27 (b) 182
in the expression x 2 ç x + 2 ÷ is 720, is:
è x ø 5 4
[Jan. 10, 2019 (II)] (c) (d)
4 5
(a) 4 (b) 2 2 22. If the coefficients of the three successive terms in the
binomial expansion of (1 + x)n are in the ratio 1 : 7 : 42, then
(c) 5 (d) 3
the first of these terms in the expansion is:
2403 k [Online April 10, 2015]
14. If the fractional part of the number is , then k th th
15 15 (a) 8 (b) 6
is equal to: [Jan. 9, 2019 (I)] (c) 7th (d) 9th
(a) 6 (b) 8 23. If the coefficents of x 3 and x 4 in the expansion of
(c) 4 (d) 14 (1 + ax + bx ) (1 - 2 x )
2 18
in powers of x are both ero, then
15. The coefficient of x10 in the expansion of (1 + x)2 (1 + x2)3 (a, b) is equal to: [2014]
(1 + x3)4 is equal to [Online April 15, 2018]
(a) 52 (b) 44 æ 272 ö æ 272 ö
(a) ç14, ÷ (b) ç16, ÷
(c) 50 (d) 56 è 3 ø è 3 ø
16. If n is the degree of the polynomial,
æ 251 ö æ 251 ö
8 8 (c) ç 16, ÷ (d) ç 14, ÷
é 1 ù é 1 ù è 3 ø è 3 ø
ê ú +ê ú and
ê 5 x 3 + 1 – 5 x3 – 1 ú
ë û
ê 5 x 3 + 1 + 5 x3 – 1 ú
ë û 24. {
If X = 4n - 3n - 1 : n Î N and }
n
Y = {9 ( n - 1) : n Î N } , where N is the set of natural
m is the coefficient of x in it, then the ordered pair (n, m) is
equal to [Online April 15, 2018]
(a) (12 , (20)4) (b) (8, 5 (10)4) numbers, then X È Y is equal to: [2014]
(c) (24 , (10)8) (d) (12, 8 (10)4) (a) X (b) Y
17. The coefficient of x2 in the expansion of the product (c) N (d) Y – X
(2 – x2). ((1 + 2x + 3x2)6 + (1 – 4x2)6) is
[Online April 16, 2018] 5
å a i (1 + x )
i
(a) 106 (b) 107 25. If 1 + x4 + x5 = , for all x in R, then a2 is:
(c) 155 (d) 108 i =0
18. The sum of the co-efficients of all odd degree terms in the [Online April 12, 2014]
expansion of [2018]
(a) – 4 (b) 6
(x + x - 1) + (x - x - 1) ,(x > 1) is :
3 5 3 5
(c) – 8 (d) 10
(a) 0 (b) 1 55
æ xö
(c) 2 (d) – 1 26. If ç 2 + ÷ is expanded in the ascending powers of x and
è 3ø
19. The coefficient of x –5 in the binomial expansion of
10 the coefficients of powers of x in two consecutive terms of
æ x +1 x -1 ö the expansion are equal, then these terms are:
ç - ÷ where x ¹ 0, 1, is :
ç 23 1 1 ÷ [Online April 12, 2014]
è x - x3 +1 x - x 2 ø (a) 7th and 8th (b) 8th and 9th
[Online April 9, 2017] (c) 28th and 29th (d) 27th and 28th
(a) 1 (b) 4 27. The number of terms in the expansion of
(c) – 4 (d) – 1
(1 + x)101 (1 + x2 – x)100 in powers of x is:
20. If (27)999 is divided by 7, then the remainder is :
[Online April 9, 2014]
[Online April 8, 2017]
(a) 302 (b) 301
(a) 1 (b) 2
(c) 202 (d) 101
(c) 3 (d) 6
EBD_8344
M-74 Mathematics

28. If for positive integers r > 1, n > 2, the coefficients of the 35. The coefficient of x7 in the expansion of (1– x – x2 + x3 )6
(3r)th and (r + 2)th powers of x in the expansion of (1 + x)2n is [2011]
are equal, then n is equal to : [Online April 25, 2013] (a) –132 (b) –144
(a) 2r + 1 (b) 2r –1 (c) 132 (d) 144
(c) 3r (d) r + 1 36. The remainder left out when 82n – (62)2n+1 is divided by 9
29. The sum of the rational terms in the binomial expansion of is: [2009]
10
æ 1 1ö (a) 2 (b) 7
ç 2 2 + 35 ÷ is : [Online April 23, 2013] (c) 8 (d) 0
ç ÷
è ø n
(a) 25 (b) 32 37. Statement -1 : å (r + 1) nCr = (n + 2)2n –1.
(c) 9 (d) 41 r =0
30. If the 7th term in the binomial expansion of n
9 Statement-2: å (r + 1) nCr x r = (1 + x)n + nx(1 + x )n –1.
æ 3 ö r =0
ç3 + 3 ln x ÷ , x > 0, is equal to 729, then x can be :
è 84 ø [2008]
[Online April 22, 2013] (a) Statement -1 is false, Statement-2 is true
(a) e 2 (b) e (b) Statement -1 is true, Statement-2 is true; Statement -2
is a correct explanation for Statement-1
e
(c) (d) 2e (c) Statement -1 is true, Statement-2 is true; Statement -2
2 is not a correct explanation for Statement-1
( ) ( )
2n 2n
31. If n is a positive integer, then 3 +1 - 3 -1 is : (d) Statement -1 is true, Statement-2 is false
[2012] 38. In the binomial expansion of (a – b)n, n ³ 5, the sum of 5th
(a) an irrational number and 6th terms is ero, then a/b equals [2007]
(b) an odd positive integer n -5 n-4
(a) (b)
(c) an even positive integer 6 5
(d) a rational number other than positive integers 6
5
(c) (d) .
32. The number of terms in the expansion of y1/5 + x ( 1/10 55
) ,
n-4 n -5

39. For natural numbers m, n if (1 - y ) m (1 + y ) n


in which powers of x and y are free from radical signs are
[Online May 12, 2012] = 1 + a1 y + a2 y 2 + ....... and a1 = a2 = 10, then (m, n) is
(a) six (b) twelve
(c) seven (d) five (a) (20, 45) (b) (35, 20) [2006]
33. If f(y) = 1 – (y – 1) + (y – 1)2 – (y – 1)3 (c) (45, 35) (d) (35, 45)
+ ... – (y – 1)17, 11
If the coefficient of x in éê ax 2 + æç 1 ö÷ ùú
7
then the coefficient of y2 in it is [Online May 7, 2012] 40. equals the
ë è bx ø û
(a) 17 C2 (b) 17 C3
(c) 18 C2 (d) 18 C3 11
é 1 öù
coefficient of x in ê ax - æç
-7
Statement - 1 : For each natural number n, (n + 1)7–1 is ú , then a and b satisfy
è bx 2 ÷ø û
34.
divisible by 7. ë
the relation [2005]
Statement - 2 : For each natural number n, n 7 - n is (a) a – b = 1 (b) a + b = 1
divisible by 7. [2011 RS] a
(c) =1 (d) ab = 1
(a) Statement-1 is true, Statement-2 is true; Statement-2 is b
a correct explanation for Statement-1.
41. The coefficient of xn in expansion of (1 + x )(1 - x )n is
(b) Statement-1 is true, Statement-2 is true; Statement-2
[2004]
is NOT a correct explanation for Statement-1
(c) Statement-1 is true, Statement-2 is false (a) ( -1)n -1 n (b) ( -1)n (1 - n)
(d) Statement-1 is false, Statement-2 is true
(c) ( -1)n -1 (n - 1)2 (d) (n - 1)
Binomial Theorem M-75

42. The number of integral terms in the expansion of


n
For a positive integer n, æç1 + ö÷ is expanded in increasing
1
( 3 + 8 5)256 is [2003] 50.
è xø
(a) 35 (b) 32
(c) 33 (d) 34 powers of x. If three consecutive coefficients in this
43. r and n are positive integers r > 1, n > 2 and coefficient of expansion are in the ratio, 2 : 5 : 12, then n is equal to
(r+2)th term and 3rth term in the expansion of (1 + x)2n are __________. [NA Sep. 02, 2020 (II)]
equal, then n equals [2002]
16
(a) 3r (b) 3r + 1 æ x 1 ö
51. In the expansion of ç + ÷ , if l1 is the least
(c) 2r (d) 2r + 1 è cos q x sin q ø
44. The coefficients of xp and xq in the expansion of (1+ x )p+q
p p
are [2002] value of the term independent of x when £ q £ and
(a) equal 8 4
(b) equal with opposite signs l2 is the least value of the term independent of x when
(c) reciprocals of each other p p
£ q £ , then the ratio l2 : l1 is equal to :
(d) none of these 16 8
[Jan. 9, 2020 (II)]
Middle Term, Greatest Term, (a) 1 : 8 (b) 16 : 1
Independent Term, Particular Term
TOPIC n from end in Binomial Expansion, (c) 8 : 1 (d) 1 : 16
Greatest Binomial Coefficients 52. The total number is irrational terms in the binomial
60
45. If the constant term in the binomial expansion of æ 1 1 ö
ç 7 5 - 310 ÷
10 expansion of ç ÷ is : [Jan. 12, 2019 (II)]
æ k ö è ø
ç x 2÷ is 405, then |k| equals: [Sep. 06, 2020 (II)]
è x ø (a) 55 (b) 49
(c) 48 (d) 54
(a) 9 (b) 1
53. A ratio of the 5th term from the begining to the 5th term
(c) 3 (d) 2
46. If for some positive integer n, the coefficients of three 10
æ 1 ö
consecutive terms in the binomial expansion of (1 + x)n +5 ç 1 ÷
are in the ratio 5 : 10 : 14, then the largest coefficient in this ç 23 + 1 ÷
from the end in the binomial expansion of
çç ÷
expansion is : [Sep. 04, 2020 (II)]
è 2(3) 3 ÷ø
(a) 462 (b) 330
(c) 792 (d) 252 is: [Jan. 12, 2019 (I)]
47. If the number of integral terms in the expansion of 1 1
(a) 1 : 2(6) 3 (b) 1: 4(16) 3
(312
+ 5 ) is exactly 33, then the least value of n is :
18 n

1 1
[Sep. 03, 2020 (I)]
(c) 4(36) 3 :1 (d) 2(36) 3 :1
(a) 264 (b) 128
(c) 256 (d) 248 54. The term independent of x in the binomial expansion of
48. If the term independent of x in the expansion of 8
æ 1 5 öæ 2 1 ö
ç 1 - + 3 x ÷ç 2 x - ÷ is : [Online April 11, 2015]
9
æ3 2 1 ö
ç x - ÷ is k, then 18k is equal to : è x øè xø
è2 3x ø
[Sep. 03, 2020 (II)] (a) 496 (b) –496
(a) 5 (b) 9 (c) 400 (d) –400
(c) 7 (d) 11 55. The term independent of x in expansion of
49. Let a > 0, b > 0 be such that a3 + b2 = 4. If the maximum 10
æ x +1 x -1 ö
value of the term independent of x in the binomial çè 2/ 3 1/ 3 - ÷ is [2013]
1 1
x - x + 1 x - x1/ 2 ø
-
expansion of (a x 9 + b x 6 )10is 10k, then k is equal to : (a) 4 (b) 120
[Sep. 02, 2020 (I)] (c) 210 (d) 310
(a) 336 (b) 352
(c) 84 (d) 176
EBD_8344
M-76 Mathematics

56. The ratio of the coefficient of x15 to the term independent 63. If 20C1 + (22) 20C2 +(32) 20C3+ ………. + (202) 20C20 = A(2b), then
15 the ordered pair (A, b) is equal to : [April 12, 2019 (II)]
æ 2ö
of x in the expansion of ç x 2 + ÷ is : (a) (420, 19) (b) (420, 18)
è xø
(c) (380, 18) (d) (380, 19)
[Online April 9, 2013] 64. The coefficient of x in the product (1+x)(1–x) 10
18

(a) 7 : 16 (b) 7 : 64 (1+x+x2)9 is : [April 12, 2019 (I)]


(c) 1 : 4 (d) 1 : 32 (a) 84 (b) –126
n (c) –84 (d) 126
æ 1ö
57. The middle term in the expansion of ç1 - ÷
è
(1 - x )n in 65. 2 3
If the coefficients of x and x are both ero, in the expansion

of the expression (1 + ax + bx2) (1–3x)15 in powers of x, then
powers of x is [Online May 26, 2012] the ordered pair (a, b) is equal to: [April 10, 2019 (I)]
(a) – 2nCn–1 (b) – 2nCn (a) (28, 861) (b) (–54, 315)
(c) 2nCn – 1 (d) 2n Cn (c) (28, 315) (d) (–21, 714)
58. The coefficient of the middle term in the binomial expansion 66. The sum of the series
4 6
in powers of x of (1 + ax ) and of (1 - ax ) is the same if 2·20C0 + 5·20C1 + 8·20C2 + 11·20C3 + … + 62·20C20
�矄 ⼄盐 ᨨ  [April 8, 2019 (I)]
a equals [2004] 26 25
(a) 2 (b) 2
3 10 (c) 223 (d) 224
(a) (b) 67. The sum of the real values of x for which the middle term in
5 3
8
æ x3 3 ö
-3 -5 the binomial expansion of çç + ÷÷ equals 5670 is :
(c)
10
(d)
3 è 3 xø
[Jan. 11, 2019 (I)]
(a) 0 (b) 6
Properties of Binomial Coefficients, (c) 4 (d) 8
Number of Terms in the Expansion 68. The value of r for which
TOPIC Đ of (x+y+z)n, Binomial theorem for 20
any Index, Multinomial theorem, Cr 20C0 + 20 Cr -120C1 + 20 Cr - 220C2 + ... + 20 C020 Cr
Infinite Series is maximum, is : [Jan. 11, 2019 (I)]
(a) 15 (b) 20
20 (c) 11 (d) 10
59. The value of å 50-r C6 is equal to : [Sep. 04, 2020 (I)]
å { 50 Cr × 50 - r C25 - r } = K ( 50 C25 ),
25
r=0
51C 30C 50C 30C
69. If then K is equal
(a) 7 – 7 (b) 7 – 7 r=0
(c) 50C – 30C (d) 51C + 30C
6 6 7 7 to: [Jan. 10, 2019 (II)]
60. The coefficient of x4 in the expansion of (1 + x + x2)10 is (a) (25)2 (b) 225 – 1
_______. [NA Jan. 9, 2020 (I)] (c) 224 (d) 225
3
61. If the sum of the coefficients of all even powers of x in the
4 æ 1 - t6 ö
70. The coefficient of t in the expansion of ç
ç 1 - t ÷÷
product è ø
(1 + x + x2 + ... + x2n) (1 – x + x2 – x3 + ... + x2n) is 61, then n [Jan. 09, 2019 (II)]
(a) 14 (b) 15
is equal to ¾¾¾. [NA Jan. 7, 2020 (I)] (c) 10 (d) 12
62. The term independent of x in the expansion of 71. The value of
(21C1 – 10C1) + (21C2 – 10C2) + (21C3 – 10C3) + (21C4 – 10C4)
æ 1 x8 ö æ 2 3 ö 6
çç - ÷÷ . ç 2 x - 2 ÷ is equal to : + .... + (21C10 – 10C10) is : [2017]
è 60 81 ø è x ø
(a) 220 – 210 (b) 221 – 211
[NA April 12, 2019 (II)]
(c) 221 – 210 (d) 220 – 29
(a) –72 (b) 36
(c) –36 (d) –108
Binomial Theorem M-77

n (a) Statement -1 is false, Statement-2 is true


æ 2 4ö
72. If the number of terms in the expansion of ç1 - + 2 ÷ , (b) Statement -1 is true, Statement-2 is true; Statement -2
è x x ø
is a correct explanation for Statement-1
x ¹ 0, is 28, then the sum of the coefficients of all the terms
(c) Statement -1 is true, Statement-2 is true; Statement -
in this expansion, is : [2016]
2 is not a correct explanation for Statement-1
(a) 243 (b) 729
(c) 64 (d) 2187 (d) Statement -1 is true, Statement-2 is false
73. The sum of coefficients of integral power of x in the 77. The sum of the series [2007]

( )
50
binomial expansion 1 - 2 x is : [2015] 20
C0 - 20
C1 + 20
C2 - 20
C3 + ..... -..... + 20
C10 is

(a)
2
(
1 50
3 -1 ) (b)
2
(
1 50
2 +1 ) (a) 0 (b) 20
C10

(c)
2
(
1 50
3 +1 ) (d)
2
( )
1 50
3 (c) - 20 C10 (d)
1
2
20
C10

74. The coefficient of x 1012 in the expansion of


3
(1 + xn + x253)10, (where n £ 22 is any positive integer), is 78. If x is so small that x and higher powers of x may be
[Online April 19, 2014]
3 3
(a) 1 (b) 10 C4 æ 1 ö
(1 + x) 2 - ç 1 + x÷
(c) 4n (d) 253 C 4 è 2 ø
neglected, then may be
1
10 10
75. Let S1 = å j ( j - 1)10C J , S2 = å j10C j (1 - x ) 2
j =1 j =1
approximated as [2005]
10
and S3 = å j 2 10
Cj. [2010] 3 3
j =1
(a) 1 - x 2 (b) 3 x + x2
8 8
Statement -1 : S3 = 55 × 29.
3 x 3 2
Statement - 2: S1 = 90 × 28 and S2 = 10 × 28 . (c) - x2 (d) - x
8 2 8
(a) Statement -1 is true, Statement -2 is true ; Statement - 79. If x is positive, the first negative term in the expansion of
2 is not a correct explanation for Statement -1.
(1 + x)27 5 is [2003]
(b) Statement -1 is true, Statement -2 is false.
(a) 6th term (b) 7th term
(c) Statement -1 is false, Statement -2 is true .
(c) 5th term (d) 8th term
(d) Statement - 1 is true, Statement 2 is true ; Statement -2 80. The positive integer ust greater than (1 + 0.0001)10000 is
is a correct explanation for Statement -1. [2002]
76. In a shop there are five types of ice-creams available. A (a) 4 (b) 5
child buys six ice-creams. (c) 2 (d) 3
Statement-1 : The number of different ways the child can 81. If the sum of the coefficients in the expansion of (a + b)n is
buy the six ice-creams is 10C5. 4096, then the greatest coefficient in the expansion is
Statement -2 : The number of different ways the child can [2002]
buy the six ice-creams is equal to the number of different (a) 1594 (b) 792
ways of arranging 6 A’s and 4 B’s in a row. [2008] (c) 924 (d) 2924
EBD_8344
M-90 Mathematics

9
Sequences and Series

7. If the sum of first 11 terms of an A.P., a1 , a2 , a3 , ... is 0


TOPIC Ć Arithmetic Progression
(a1 ¹ 0), then the sum of the A.P., a1 , a3 , a5 , ..., a23 is
1. The common difference of the A.P. b1, b2, ..., bm is 2 more ka1, where k is equal to : [Sep. 02, 2020 (II)]
than the common difference of A.P. a 1, a 2, ..., a n. If
121 121 72 72
a40 = – 159, a100 = – 399 and b100 = a70, then b1 is equal to: (a) - (b) (c) (d) -
[Sep. 06, 2020 (II)] 10 10 5 5
(a) 81 (b) – 127 (c) – 81 (d) 127 8. The number of terms common to the two A.P.’s 3, 7, 11, ...,
2. If 32sin2a–1, 14 and 34–2sin2a are the first three terms of an 407 and 2, 9, 16, ..., 709 is _____. [NA Jan. 9, 2020 (II)]
A.P. for some a, then the sixth term of this A.P is:
1 1
[Sep. 05, 2020 (I)] 9. If the 10th term of an A.P. is and its 20th term is ,
20 10
(a) 66 (b) 81 (c) 65 (d) 78
then the sum of its first 200 terms is: [Jan. 8, 2020 (II)]
3. If the sum of the first 20 terms of the series
log (71/ 2 ) x + log (71/ 3 ) x + log (71/ 4 ) x + ... is 460, then x is 1 1
(a) 50 (b) 50 (c) 100 (d) 100
4 2
equal to : [Sep. 05, 2020 (II)]
10. Let f : R ® R be such that for all x Î R, (21+x + 21–x), f (x)
(a) 72 (b) 71/2 (c) e 2 (d) 746/21
and (3x + 3–x) are in A.P., then the minimum value of f (x) is:
4. Let a1 , a2 ,....., an be a given A.P. whose common [Jan. 8, 2020 (I)]
difference is an integer and Sn = a1 + a2 + .... + an . If (a) 2 (b) 3 (c) 0 (d) 4
11. Five numbers are in A.P., whose sum is 25 and product is
a1 = 1, an = 300 and 15 £ n £ 50, then the ordered pair
1
( Sn - 4 , an - 4 ) is equal to : [Sep. 04, 2020 (II)] 2520. If one of these five numbers is - , then the greatest
2
(a) (2490, 249) (b) (2480, 249) number amongst them is: [Jan. 7, 2020 (I)]
(c) (2480, 248) (d) (2490, 248)
5. If the first term of an A.P. is 3 and the sum of its first 25 21
(a) 27 (b) 7 (c) (d) 16
terms is equal to the sum of its next 15 terms, then the 2
common difference of this A.P. is : [Sep. 03, 2020 (I)] 12. Let Sn denote the sum of the first n terms of an A.P. If S4 = 16
1 1 1 1 and S6 = –48, then S10 is equal to : [April 12, 2019 (I)]
(a) (b) (c) (d) (a) –260 (b) –410 (c) –320 (d) –380
6 5 4 7
6. In the sum of the series 13. If a1, a2, a3, …… are in A.P. such that a1 + a7 + a16 = 40,
then the sum of the first 15 terms of this A.P. is :
3 1 4
20 + 19 + 19 + 18 + ... upto nth term is 488 and then [April 12, 2019 (II)]
5 5 5
(a) 200 (b) 280 (c) 120 (d) 150
nth term is negative, then : [Sep. 03, 2020 (II)]
(a) n = 60 th
(b) n term is –4 14. If a1, a2, a3, ..... an are in A.P. and a1 + a4 + a7 + ….+ a16 = 114,
then a1 + a6 + a11 + a16 is equal to : [April 10, 2019 (I)]
2
(c) n = 41 (d) nth term is -4 (a) 98 (b) 76 (c) 38 (d) 64
5

Downloaded from @Freebooksforjeeneet


Sequences and Series M-91

15. Let the sum of the first n terms of a non-constant A.P.,


1 1 1
24. If x1 , x2, ....., xn and , 2 ,..... are two A.P's such that
n(n - 7) h1 h hn
a1, a2, a3, ………….. be 50n + A, where A is a
2 x3 = h2 = 8 and x8 = h7 = 20, then x5. h10 equals.
constant. If d is the common difference of this A.P., then [Online April 15, 2018]
the ordered pair (d, a50) is equal to: [April 09, 2019 (I)]
(a) 2560 (b) 2650 (c) 3200 (d) 1600
(a) (50, 50 + 46A) (b) (50, 50 + 45A)
(c) (A, 50 + 45A) (d) (A, 50 + 46A) 25. Let a1,a 2 ,a 3 ,..., a 49 be in A.P. such that

10 12
16. Let å f (a + k) = 16(2 10
- 1) , where the function f satisfies å a 4k +1 = 416 and a 9 + a 43 = 66. If
k =1 k =0
f(x + y) = f(x) f(y) for all natural numbers x, y and f(a) = 2. a12 + a 22 + ... + a17
2
= 140m , then m is equal to : [2018]
Then the natural number ‘a’ is:
[ April 09, 2019 (I)] (a) 68 (b) 34 (c) 33 (d) 66
(a) 2 (b) 16 (c) 4 (d) 3 26. For any three positive real numbers a, b and c,
17. If the sum and product of the first three terms in an A.P. 9(25a2 + b2) + 25(c2 – 3ac) = 15b(3a + c). Then :
are 33 and 1155, respectively, then a value of its 11th term [2017]
is: [April 09, 2019 (II)] (a) a, b and c are in G.P.
(a) –35 (b) 25 (c) –36 (d) –25 (b) b, c and a are in G.P.
18. The sum of all natural numbers ‘n’ such that 100 < n < 200 (c) b, c and a are in A.P.
and H.C.F. (91, n) > 1 is : [April 08, 2019 (I)] (d) a, b and c are in A.P.
(a) 3203 (b) 3303 (c) 3221 (d) 3121 27. If three positive numbers a, b and c are in A.P. such that
19. If nC4, nC5 and nC6 are in A.P., then n can be : abc = 8, then the minimum possible value of b is :
[Jan. 12, 2019 (II)] [Online April 9, 2017]
(a) 9 (b) 14 (c) 11 (d) 12 (a) 2 (b) 4 3
1
(c) 4 3
2
(d) 4
20. If 19th term of a non- ero A.P. is ero, then its (49th term) :
(29th term) is : [Jan. 11, 2019 (II)] 28. Let a1, a2, a3, ...., an, be in A.P. If a3 + a7 + a11 + a15 = 72,
(a) 4 : 1 (b) 1 : 3 (c) 3 : 1 (d) 2 : 1 then the sum of its first 17 terms is equal to :
21. The sum of all two digit positive numbers which when [Online April 10, 2016]
divided by 7 yield 2 or 5 as remainder is: (a) 306 (b) 204 (c) 153 (d) 612
[Jan. 10, 2019 (I)] 29. Let a and b be the roots of equation px + qx + r = 0, p ¹ 0.
2

(a) 1256 (b) 1465 (c) 1365 (d) 1356 1 1


If p, q, r are in A.P and + = 4, then the value of | a – b|
30 a b
22. Let a 1 , a 2 , ....., a 30 be an A.P., S = å a i and is: [2014]
i=1
34 2 13 61 2 17
15 (a) (b) (c) (d)
T= å a (2i - 1) . 9 9 9 9
i= 1 30. The sum of the first 20 terms common between the series 3
If a5 = 27 and S – 2T = 75, then a10 is equal to: + 7 + 11 + 15 + ......... and 1 + 6 + 11 + 16 + ......, is
[Jan. 09, 2019 (I)] [Online April 11, 2014]
(a) 52 (b) 57 (c) 47 (d) 42 (a) 4000 (b) 4020 (c) 4200 (d) 4220
1 1 1 31. Given an A.P. whose terms are all positive integers. The
23. Let , , ,....., (xi ¹ 0 for i = 1, 2, ...., n) be in A.P.. sum of its first nine terms is greater than 200 and less than
x1 x2 x3
220. If the second term in it is 12, then its 4th term is:
such that x1 = 4 and x21 = 20. If n is the least positive
[Online April 9, 2014]
æ1ön
integer for which xn > 50, then å ç ÷ is equal to. (a) 8 (b) 16 (c) 20 (d) 24
i = 1 è xi ø 32. If a1, a2, a3,...., an, .... are in A.P. such that a4 – a7 + a10 = m,
[Online April 16, 2018] then the sum of first 13 terms of this A.P., is :
[Online April 23, 2013]
13 13 1
(a) 3 (b) (c) (d) (a) 10 m (b) 12 m (c) 13 m (d) 15 m
8 4 8

Downloaded from @Freebooksforjeeneet


EBD_8344
M-92 Mathematics

33. Given sum of the first n terms of an A.P. is 2n + 3n2. Another (a) 34 minutes (b) 125 minutes
A.P. is formed with the same first term and double of the (c) 135 minutes (d) 24 minutes
common difference, the sum of n terms of the new A.P. is :
41. Let a1 , a2 , a3 ............ be terms on A.P. If
[Online April 22, 2013]
(a) n + 4n (b) 6n – n (c) n + 4n (d) 3n + 2n2
2 2 2
a1 + a2 + ...........a p p2 a
= 2 , p ¹ q , then 6 equals
34. Let a1, a2, a3,... be an A.P, such that a1 + a2 + ........... + aq q a21
a1 + a2 + ... + a p p3 [2006]
= ; p ¹ q . Then a6 is equal to:
a1 + a2 + a3 + ... + aq q3
a21 41 7 2 11
(a) (b) (c) (d)
[Online April 9, 2013] 11 2 7 41
42. If the coefficients of rth, (r + 1)th, and (r + 2)th terms in the
41 31 11 121
(a) (b) (c) (d)
11 121 41 1861 the binomial expansion of (1 + y )m are in A.P., then m and
35. If 100 times the 100th term of an AP with non ero common r satisfy the equation [2005]
difference equals the 50 times its 50th term, then the 150th 2
(a) m 2 – m (4r – 1) + 4 r – 2 = 0
term of this AP is : [2012]
2
(a) – 150 th
(b) 150 times its 50 term (b) m 2 – m (4r + 1) + 4 r + 2 = 0
(c) 150 (d) Zero
(c) m2 – m (4r + 1) + 4 r 2 – 2 = 0
36. If the A.M. between pth and qth terms of an A.P. is equal
2 2
to the A.M. between rth and sth terms of the same A.P., (d) m – m (4r – 1) + 4 r + 2 = 0
then p + q is equal to [Online May 26, 2012] 43. Let Tr be the rth term of an A.P. whose first term is a and
(a) r + s – 1 (b) r + s – 2 (c) r + s + 1 (d) r + s common difference is d. If for some positive integers
37. Suppose q and f (¹ 0) are such that sec (q + f), sec q and 1 1
m, n, m ¹ n, Tm = and Tn = , then a – d equals
æ fö n m
sec (q – f) are in A.P. If cos q = k cos ç ÷ for some k, then
è 2ø [2004]
k is equal to [Online May 19, 2012] 1 1 1
(a) + (b) 1 (c) (d) 0
m n mn
1
(a) ± 2 (b) ± 1 (c) ± (d) ± 2 44. If 1, log9 (31–x + 2), log3 (4.3x – 1) are in A.P. then x equals
2
[2002]
100 (a) log3 4 (b) 1 – log3 4
38. Let an be the n th term of an A.P. If å a2r = a and (c) 1 – log4 3 (d) log4 3
r=1

100
TOPIC n Geometric Progression
å a2r –1 = β, then the common difference of the A.P. is
r=1

[2011] ¥

a -b α–β
45. If f (x + y) = f (x) f (y) and å f ( x) = 2 , x, y Î N, where N is
(a) a - b (b) (c) b - a (d) x =1
100 200
f (4)
39. A man saves ` 200 in each of the first three months of his the set of all natural numbers, then the value of is :
f (2)
service. In each of the subsequent months his saving
increases by ` 40 more than the saving of immediately [Sep. 06, 2020 (I)]
previous month. His total saving from the start of service 2 1 1 4
will be ` 11040 after [2011] (a) (b) (c) (d)
3 9 3 9
(a) 19 months (b) 20 months
46. Let a, b, c, d and p be any non ero distinct real numbers
(c) 21 months (d) 18 months such that (a2 + b2 + c2)p2 – 2 (ab + bc + cd)p + (b2 + c2 +
40. A person is to count 4500 currency notes. Let an denote d2) = 0. Then : [Sep. 06, 2020 (I)]
the number of notes he counts in the nth minute. If a1 = a2
(a) a, c, p are in A.P. (b) a, c, p are in G.P.
= ... = a10 = 150 and a10, a11, ... are in an AP with common
difference –2, then the time taken by him to count all notes (c) a, b, c, d are in G.P. (d) a, b, c, d are in A.P.
is [2010]

Downloaded from @Freebooksforjeeneet


Sequences and Series M-93

47. Suppose that a function f : R®R satisfies f (x + y) = f(x)f(y)


x10 - x + 45a( x - 1)
n S= , then k is equal to :
x -1
for all x, y Î R and f (a) = 3. If å f (i) = 363 , then n is
i =1 [Sep. 02, 2020 (II)]
equal to ______. [NA Sep. 06, 2020 (II)] (a) – 5 (b) 1 (c) – 3 (d) 3
48. If 210 + 29×31 + 28×32 + . . . . + 2×39 + 310 = S – 211 then S is 1 1 1 1
equal to: [Sep. 05, 2020 (I)] 55. The product 24 . 416 . 8 48 . 16128 ... to ¥ is equal to:
(a) 311 – 212 (b) 311 [Jan. 9, 2020 (I)]
1 1
311 10
(c) +2 (d) 2×311 (a) 2 2 (b) 2 4 (c) 1 (d) 2
2 56. Let an be the nth term of a G.P. of positive terms.
49. If the sum of the second, third and fourth terms of a positive
100 100 200
term G.P. is 3 and the sum of its sixth, seventh and eighth
terms is 243, then the sum of the first 50 terms of this G.P. is:
If å a2n+1 = 200 and å a2n = 100, then å an is equal
n =1 n =1 n =1
[Sep. 05, 2020 (II)] to : [Jan. 9, 2020 (II)]
1 49 1 50 (a) 300 (b) 225 (c) 175 (d) 150
(a) (3 - 1) (b) (3 - 1)
26 26 ¥ ¥
p
If x = å (-1) tan2n q and y = å cos q, for 0 < q <
n 2n
57. ,
2 50 1 50 n= 0 n=0 4
(c) (3 - 1) (d) (3 - 1)
13 13 then : [Jan. 9, 2020 (II)]
(a) x(1 + y) = 1 (b) y(1 – x) = 1
50. Let a and b be the roots of x2 - 3x + p = 0 and g and d be
(c) y(1 + x) = 1 (d) x(1 – y) = 1
the roots of x2 - 6 x + q = 0. If a, b, g, d form a geometric 58. The greatest positive integer k, for which 49k + 1 is a
progression. Then ratio (2q + p) : (2q – p) is : factor of the sum 49125 + 49124 + ... + 492 + 49 + l, is:
[Sep. 04, 2020 (I)] [Jan. 7, 2020 (I)]
(a) 3 : 1 (b) 9 : 7 (c) 5 : 3 (d) 33 : 31 (a) 32 (b) 63 (c) 60 (d) 65
59. Let a1, a2, a3, ... be a G. P. such that a1 < 0, a1 + a2 = 4 and
æ1 1 1 ö 9
Th e value of (0.16) log 2.5 ç + 2 + 3 + ... to ¥ ÷ is
51.
è3 3 3 ø a3 + a4 = 16. If å ai = 4l, then l is equal to:
i =1
equal to _________ . [NA Sep. 03, 2020 (I)]
[Jan. 7, 2020 (II)]
52. The sum of the first three terms of a G.P. is S and their
product is 27. Then all such S lie in : [Sep. 02, 2020 (I)] 511
(a) –513 (b) –171 (c) 171 (d)
(a) (-¥, - 9] È [3, ¥) (b) [-3, ¥)
3
60. The coefficient of x7 in the expression
(c) (-¥, - 3] È [9, ¥) (d) (-¥, 9]
(1 + x)10 + x(l + x)9 + x2(l + x)8 + ... + x10 is:
53. If | x |< 1, | y |< 1 and x ¹ y, then the sum to infinity of the [Jan. 7, 2020 (II)]
following series [Sep. 02, 2020 (I)] (a) 210 (b) 330 (c) 120 (d) 420
( x + y ) + ( x + xy + y ) + ( x + x y + xy + y ) + .... is :
2 2 3 2 2 3 61. If a, b and g are three consecutive terms of a non-
constant G.P. such that the equations ax 2 + 2bx + g = 0
x + y - xy x + y + xy and x 2 + x – 1 = 0 have a common root, then a (b+ g) is
(a) (b) equal to : [April 12, 2019 (II)]
(1 + x)(1 + y ) (1 + x)(1 + y )
(a) 0 (b) ab (c) ag (d) bg
x + y - xy x + y + xy 62. Let a, b and c be in G.P. with common ratio r, where a ¹ 0
(c) (d)
(1 - x )(1 - y ) (1 - x )(1 - y ) 1
and 0 < r £ . If 3a, 7b and 15c are the first three terms of
54. Let S be the sum of the first 9 terms of the series : 2
an A.P., then the 4th term of this A.P. is :
{x + ka} + {x 2 + (k + 2)a} + {x 3 + (k + 4)a} [April 10, 2019 (II)]

+{x4 + (k + 6)a} + ... where a¹0 and x ¹ 1. If 2


a
7
a
(a) (b) 5 a (c) (d) a
3 3

Downloaded from @Freebooksforjeeneet


EBD_8344
M-94 Mathematics

63. If three distinct numbers a, b, c are in G.P. and the equations


a
ax2 + 2bx + c = 0 and dx2 + 2ex + f = 0 have a common root, consecutive terms of a G.P., then is equal to:
then which one of the following statements is correct? c
[April 08, 2019 (II)] [Jan. 09, 2019 (II)]

d e f 1 7
, , are in A.P.. (a) 2 (b) (c) (d) 4
(a) (b) d, e, f are in A.P. 2 13
a b c
70. If a, b and c be three distinct real numbers in G.P. and
d e f
(c) d, e, f are in G.P. (d) , , are in G.P.. a + b + c = xb, then x cannot be: [Jan. 09, 2019 (I)]
a b c
(a) – 2 (b) – 3 (c) 4 (d) 2
64. The product of three consecutive terms of a G.P. is 512. If 71. If b is the first term of an infinite G. P whose sum is five,
4 is added to each of the first and the second of these then b lies in the interval. [Online April 15, 2018]
terms, the three terms now form an A.P. Then the sum of
(a) (– ¥, – 10) (b) (10, ¥)
the original three terms of the given G.P. is :
(c) (0, 10) (d) (– 10, 0)
[Jan. 12, 2019 (I)]
(a) 36 (b) 32 (c) 24 (d) 28 2 3 n
æ3ö æ3ö æ 3ö æ 3ö
65. Let a and b be the roots of the quadratic equation 72. Let A n = ç ÷ - ç ÷ + ç ÷ - ... + (-1)n - 1 ç ÷ and
è4ø è4ø è 4ø è 4ø
x2 sinq – x (sinq cosq + 1) + cosq = 0 (0 < q < 45 ), and
Bn = 1 – An. Then, the least odd natural number p, so that
æ ö Bn > An, for all n ³ p is
n ( -1)
¥ n [Online April 15, 2018]
a < b. Then å ç a + n
ç ÷ (a) 5 (b) 7 (c) 11 (d) 9
n =0 è b ÷ is equal to :
ø 73. If a, b, c are in A.P. and a2, b2, c2 are in G.P. such that
[Jan. 11, 2019 (II)] 3
a < b < c and a + b + c = , then the value of a is
4
1 1 1 1
(a) - (b) + [Online April 15, 2018]
1 - cos q 1 + sin q 1 + cos q 1 - sin q
1 1 1 1
1 1 1 1 (a) - (b) -
(c) + (d) - 4 3 2 4 4 2
1 - cos q 1 + sin q 1 + cos q 1 - sin q
1 1 1 1
a3 a9 (c) - (d) -
4 4 2 2
Let a1, a2, ..., a10 be a G.P. If a = 25, then a equals :
2
66.
1 5 74. If the 2nd, 5th and 9th terms of a non-constant A.P. are in
[Jan. 11, 2019 (I)] G.P., then the common ratio of this G.P. is : [2016]
(a) 54 (b) 4(52) (c) 53 (d) 2(52)
7 8 4
67. The sum of an infinite geometric series with positive terms (a) 1 (b) (c) (d)
4 5 3
27
. Then the 75. Let = 1 + ai be a complex number, a > 0, such that 3 is
is 3 and the sum of the cubes of its terms is
19 areal number. Then the sum 1 + + 2 + .... + 11 is equal to:
common ratio of this series is : [Jan. 11, 2019 (I)] (Online April 10, 2016)

1 2 2 4 (a) 1365 3i (b) -1365 3i


(a) (b) (c) (d)
3 3 9 9 (c) -1250 3i (d) 1250 3i
68. Let Sn = 1 + q + q2 + .... + qn and
76. If m is the A.M. of two distinct real numbers l and n(l, n >
2 n 1) and G1, G2 and G3 are three geometric means between l
æ q + 1ö æ q + 1ö æ q + 1ö
Tn = 1 + ç ÷ +ç ÷ + ... + ç ÷
è 2 ø è 2 ø è 2 ø and n, then G14 + 2G 42 + G 34 equals. [2015]

where q is a real number and q ¹ 1. If (a) 4 lmn2 (b) 4 l2m2n2 (c) 4 l2 mn (d) 4 lm2n
101 S100 = aT100, then a is
101C + 101C .S + .... + 101C . 77. The sum of the 3rd and the 4th terms of a G.P. is 60 and
1 2 1
equal to : [Jan. 11, 2019 (II)] the product of its first three terms is 1000. If the first term
(a) 299 (b) 202 (c) 200 (d) 2100 of this G.P. is positive, then its 7th term is :
69. Let a, b and c be the 7th, 11th and 13th terms respectively [Online April 11, 2015]
of a non-constant A.P. If these are also the three (a) 7290 (b) 640 (c) 2430 (d) 320

Downloaded from @Freebooksforjeeneet


Sequences and Series M-95

78. Three positive numbers form an increasing G. P. If the middle


term in this G.P. is doubled, the new numbers are in A.P. (a) 5 (b)
1
2
( 5 -1 )
then the common ratio of the G.P. is: [2014]

(a) 2 - 3 (b) 2 + 3 (c)


1
2
(
1- 5 ) (d)
1
2
5

(c) 2+ 3 (d) 3 + 2
10
æ 2k p 2k p ö
79. The least positive integer n such that 87. The value of å çè sin 11
+ i cos
11 ø
÷ is [2006]
k =1
2 2 2 1
1 - - 2 - .... - n -1 < , is: [Online April 12, 2014] (a) i (b) 1 (c) – 1 (d) – i
3 3 3 100
88. If the expan sion in powers of x of the fun ction
(a) 4 (b) 5 (c) 6 (d) 7
1
80. In a geometric progression, if the ratio of the sum of first 5 is a0 + a1 x + a2 x 2 + a3 x 3 ...... then an is
(1 - ax)(1 - bx)
terms to the sum of their reciprocals is 49, and the sum of
the first and the third term is 35. Then the first term of this [2006]
geometric progression is: [Online April 11, 2014]
n n
(a) b - a
n n
(a) 7 (b) 21 (c) 28 (d) 42 (b) a - b
81. The coefficient of x50 in the binomial expansion of b-a b-a
(1 + x) 1000 + x (1 + x) 999 + x 2 (1 + x) 99 8 + ....
+ x1000 is:
n +1
- bn +1 bn +1 - a n +1
[Online April 11, 2014] (c) a (d)
b-a b-a
(1000 )! (1000 )!
(a)
(50 )!( 950 )! (b)
( 49 )!( 951)! 89. Let two numbers have arithmetic mean 9 and geometric
mean 4. Then these numbers are the roots of the quadratic
equation [2004]
(1001)! (1001)!
(c)
( 51)!( 950 )! (d)
(50 )!( 951)! (a) x 2 - 18 x - 16 = 0 (b) x 2 - 18 x + 16 = 0

82. Given a sequence of 4 numbers, first three of which are in (c) x 2 + 18 x - 16 = 0 (d) x 2 + 18 x + 16 = 0
G.P. and the last three are in A.P. with common difference
90. Sum of infinite number of terms of GP is 20 and sum of their
six. If first and last terms of this sequence are equal, then
square is 100. The common ratio of GP is [2002]
the last term is : [Online April 25, 2013]
(a) 5 (b) 3/5 (c) 8/5 (d) 1/5
(a) 16 (b) 8 (c) 4 (d) 2
91. Fifth term of a GP is 2, then the product of its 9 terms is
83. If a, b, c, d and p are distinct real numbers such that
[2002]
(a2 + b2 + c2) p2 – 2p (ab + bc + cd) + (b2 + c2 + d2) £ 0,
(a) 256 (b) 512
then
(c) 1024 (d) none of these
[Online May 12, 2012]
(a) a, b, c, d are in A.P. (b) ab = cd æ pö
92. The sum of all values of qÎ ç 0, ÷ satisfying
(c) ac = bd (d) a, b, c, d are in G.P. è 2ø
84. The difference between the fourth term and the first term
of a Geometrical Progresssion is 52. If the sum of its first 3
sin22q + cos4 2q = is: [Jan. 10, 2019 (I)]
three terms is 26, then the sum of the first six terms of the 4
progression is [Online May 7, 2012]
5p p 3p
(a) 63 (b) 189 (c) 728 (d) 364 (a) p (b) (c) (d)
4 2 8
85. The first two terms of a geometric progression add up to
12. the sum of the third and the fourth terms is 48. If the Harmonic Progression, Relation
terms of the geometric progression are alternately positive TOPIC Đ Between A. M., G. M. and H.M. of
and negative, then the first term is [2008] two Positive Numbers
(a) –4 (b) –12 (c) 12 (d) 4
86. In a geometric progression consisting of positive terms, 93. If m arithmetic means (A.Ms) and three geometric means
each term equals the sum of the next two terms. Then the (G.Ms) are inserted between 3 and 243 such that 4th A.M.
common ratio of its progression is equals [2007] is equal to 2nd G.M., then m is equal to ___________.
[Sep. 03, 2020 (II)]

Downloaded from @Freebooksforjeeneet


EBD_8344
M-96 Mathematics

94. If the arithmetic mean of two numbers a and b, a > b > 0, is


Arithmetic-Geometric Sequence
a+b TOPIC Ė (A.G.S.), Some Special Sequences
five times their geometric mean, then is equal to :
a-b
[Online April 8, 2017] 101. If 1 + (1 – 22 × 1) + (1 – 42 × 3) + (1 – 62 × 5) + . . . . . . . + (1 – 202 ×
6 3 2 7 3 5 6 19) = a - 220b, then an ordered pair (a, b) is equal to :
(a) (b) (c) (d)
2 4 12 12 [Sep. 04, 2020 (I)]
(a) (10, 97) (b) (11, 103)
p (c) (10, 103) (d) (11, 97)
95. If A > 0, B > 0 and A + B = , then the minimum value of
6
102. Let f :R ® R be a function which satisfies
tanA + tanB is : [Online April 10, 2016]
f ( x + y ) = f ( x) + f ( y ), " x, y ÎR. If f (a) = 2 and
(a) 3- 2 (b) 4 - 2 3
( n -1)

(c)
2
(d) 2 - 3
g (n) = å f (k ), n Î N , then the value of n, for which
k =1
3
g(n) = 20, is : [Sep. 02, 2020 (II)]
96. Let x, y, be positive real numbers such that x + y + = 12
(a) 5 (b) 20 (c) 4 (d) 9
and x3y4 5 = (0.1) (600)3. Then x3 + y3 + 3 is equal to :
[Online April 9, 2016] 7
n(n + 1)(2 n + 1)
103. The sum, å 4
is equal to __________.
(a) 342 (b) 216 (c) 258 (d) 270 n =1

97. Let G be the geometric mean of two positive numbers a [Jan. 8, 2020 (II)]
1 1 1 20
and b, and M be the arithmetic mean of
a
and . If
b M
:G 104. The sum å (1 + 2 + 3 + ... + k ) is _______.
k =1
is 4 : 5, then a : b can be: [Online April 12, 2014]
[Jan. 8, 2020 (I)]
(a) 1 : 4 (b) 1 : 2 (c) 2 : 3 (d) 3 : 4 105. If the sum of the first 40 terms of the series, 3 + 4 + 8 + 9
98. If a1, a2, .........., an are in H.P., then the expression + 13 + 14 + 18 +19 + ... is (102)m, then m is equal to:
a1a2 + a2a3 + .......... + an–1an is equal to [2006] [Jan. 7, 2020 (II)]
(a) n(a1 - an ) (b) (n - 1)(a1 - an ) (a) 20 (b) 25 (c) 5 (d) 10
106. For x Î R, let [x] denote the greatest integer < x, then the
(c) na1an (d) (n - 1)a1an
sum of the series
¥ ¥ ¥
é 1ù é 1 1 ù é 1 2 ù é 1 99 ù
99. If x = å a n , y = å bn , z = å c n where a, b, c are in êë - 3 úû + êë - 3 - 100 úû + êë- 3 - 100 úû + L + êë - 3 - 100 úû is
n =0 n=0 n=0
A.P and |a | < 1, | b | < 1, | c | < 1 then x, y, z are in [2005] [April 12, 2019 (I)]
(a) G. P. (a) –153 (b) –133 (c) –131 (d) –135
(b) A.P.
3 ´ 13 5 ´ (13 + 23 ) 7 ´ (13 + 23 + 33 )
(c) Arithmetic - Geometric Progression 107. The sum + + ) + …...
(d) H.P. 12 12 + 22 12 + 22 + 32
100. If the sum of the roots of the quadratic equation upto 10th term, is : [April 10, 2019 (I)]
(a) 680 (b) 600 (c) 660 (d) 620
ax 2 + bx + c = 0 is equal to the sum of the squares of
13 + 23 13 + 23 + 33
a b c 108. The sum 1 + + + ¼¼ +
their reciprocals, then , and are in [2003] 1+ 2 1+ 2 + 3
c a b
(a) Arithmetic - Geometric Progression 13 + 23 + 33 + ... + 153 1
(b) Arithmetic Progression - (1 + 2 + 3 + ... + 15 is equal to :
1 + 2 + 3 + ... + 15 2
(c) Geometric Progression
(d) Harmonic Progression. [April 10, 2019 (II)]
(a) 620 (b) 1240 (c) 1860 (d) 660

Downloaded from @Freebooksforjeeneet


Sequences and Series M-97

109. The sum of the series 1 + 2 × 3 + 3 × 5 + 4 × 7 + ….. upto


12 + 2 × 22 + 32 + 2.4 2 + 52 + 2.62 + ...
11th term is: [April 09, 2019 (II)]
(a) 915 (b) 946 (c) 945 (d) 916 If B - 2A = 100 l , then l is equal to : [2018]
110. Some identical balls are arranged in rows to form an (a) 248 (b) 464 (c) 496 (d) 232
equilateral triangle. The first row consists of one ball, the 117. Let a, b, c Î R. If f(x) = ax2 + bx + c is such that a + b + c = 3
second row consists of two balls and so on. If 99 more 10
identical balls are added to the total number of balls used and f(x + y) = f(x) + f(y) + xy, " x, y Î R, then å f ( n ) is equal
in forming the equilateral triangle, then all these balls can n =1
be arranged in a square whose each side contains exactly to : [2017]
2 balls less than the number of balls each side of the triangle (a) 255 (b) 330 (c) 165 (d) 190
contains. Then the number of balls used to form the
equilateral triangle is: [April 09, 2019 (II)] 1 1+ 2 1+ 2 + 3
118. Let Sn = 3 + 3 3 + 3 3 3 + .....
(a) 157 2) 262 (c) 225 (d) 190 1 1 +2 1 +2 +3
20
1 1 + 2 + ...... + n
111. The sum å k 2k is equal to : [April 08, 2019 (II)] +
1 + 23 + ....... + n3
3
, If 100 Sn = n, then n is equal to :
k =1
[Online April 9, 2017]
3 11 11 21
(a) 2 – 17 (b) 1 – 20 (c) 2 – 19 (d) 2 – 20 (a) 199 (b) 99 (c) 200 (d) 19
2 2 2 2 119. If the sum of the first n terms of the series
1 + 2 + 3 + ..... + k 3 + 75 + 243 + 507 + ...... is 435 3 , then n equals :
112. Let Sk = .
2 [Online April 8, 2017]
5 (a) 18 (b) 15 (c) 13 (d) 29
2
If S12 + S22 + ..... + S10 = A. Then A is equal to 120. If the sum of the first ten terms of the series
12
2 2 2 2
[Jan. 12, 2019 (I)] æ 3ö æ 2ö æ 1ö 2 æ 4ö 16
ç 1 5 ÷ + ç 2 5 ÷ + ç 3 5 ÷ + 4 + ç 4 5 ÷ + ......., is 5 m,
(a) 283 (b) 301 (c) 303 (d) 156 è ø è ø è ø è ø
113. If the sum of the first 15 terms of the series then m is equal to : [2016]
3 3 3 3 (a) 100 (b) 99 (c) 102 (d) 101
æ3ö æ 1ö æ 1ö 3 æ 3ö
ç ÷ + ç1 ÷ + ç 2 ÷ + 3 + ç 3 ÷ + ....... is equal to 121. For x Î R, x ¹ -1 , if (1 + x)2016 + x (1 + x)2015 + x2
è ø è ø è
4 2 4 ø è 4ø
2016
225 k then k is equal to :
å ai xi , then a17 is equal to :
[Jan. 12, 2019 (II)]
(1 + x)2014 + .... + x2016 =
(a) 108 (b) 27 (c) 54 (d) 9 i =0
114. The sum of the following series [Jan. 09, 2019 (II)] [Online April 9, 2016]
9 (12 + 22 + 32 ) 12 (12 + 22 + 32 + 42 ) 2017! 2016!
1+ 6 + + (a) (b) 17! 1999!
7 9 17!2000!

15 (12 + 22 + .... + 52 ) 2016! 2017!


+ +... up to 15 terms, is: (c) (d)
11 16! 2000!
(a) 7520 (b) 7510 (c) 7830 (d) 7820 122. The sum of first 9 terms of the series. [2015]
115. The sum of the first 20 terms of the series 13 13 + 23 13 + 23 + 33
+ + + ....
3 7 15 31 1 1+ 3 1+ 3 + 5
1 + + + + + ... is? [Online April 16, 2018]
2 4 8 16 (a) 142 (b) 192 (c) 71 (d) 96
1 1 5 k
(a) 38 + (b) 39 + 1
220 219 123. If å n(n + 1)(n + 2)( n + 3) = 3
, then k is equal to
n =1
1 1 [Online April 11, 2015]
(c) 39 + (d) 38 + 19
220 2
1 17 55 19
116. Let A be the sum of the first 20 terms and B be the sum of (a) (b) (c) (d)
6 105 336 112
the first 40 terms of the series

Downloaded from @Freebooksforjeeneet


EBD_8344
M-98 Mathematics

30 18 22 20 16
124. The value of å ( r + 2) ( r - 3) is equal to : (a)
11
(b)
13
(c)
11
(d)
9
r =16
133. Statement-1: The sum of the series 1 + (1 + 2 + 4) +
[Online April 10, 2015] (4 + 6 + 9) + (9 + 12 + 16) + .... + (361 + 380 + 400) is 8000.
(a) 7770 (b) 7785 (c) 7775 (d) 7780

å ( k 3 - ( k - 1)3 ) = n3, for any natural


n

125. If (10 ) + 2 (11) 10


9 1
( ) + 3(11)
8 2
(10 ) 7
+ .....
Statement-2:
k =1
number n. [2012]
+10 (11) = k (10 ) , then k is equal to:
9 9
[2014]
(a) Statement-1 is false, Statement-2 is true.
121 441 (b) Statement-1 is true, statement-2 is true; statement-2 is
(a) 100 (b) 110 (c) (d) a correct explanation for Statement-1.
10 100
(c) Statement-1 is true, statement-2 is true; statement-2 is
126. The number of terms in an A.P. is even; the sum of the odd not a correct explanation for Statement-1.
terms in it is 24 and that the even terms is 30. If the last term (d) Statement-1 is true, statement-2 is false.
exceeds the first term by 10
1
, then the number of terms in 134. If the sum of the series 12 + 2.22 + 32 + 2.42 + 52 + ... 2.62 +...
n ( n + 1)
2 2
the A.P. is: [Online April 19, 2014] upto n terms, when n is even, is , then the sum
2
(a) 4 (b) 8 (c) 12 (d) 16
of the series, when n is odd, is [Online May 26, 2012]
127. If the sum
n 2 ( n - 1)
3 5 7 (a) n2(n + 1) (b)
+ + + ...... + up to 20 terms is equal 2
12 12 + 22 12 + 22 + 32
n2 ( n + 1)
k (c) (d) n2(n – 1)
to , then k is equal to: [Online April 9, 2014] 2
21
(a) 120 (b) 180 (c) 240 (d) 60 4 10 28
135. The sum of the series 1 + + + + ... upto n terms is
128. The sum of first 20 terms of the sequence 0.7, 0.77, 0.777,....., 3 9 27
is [2013] [Online May 19, 2012]
7 7
(a) (179 - 10-20 ) (b) (99 - 10-20 ) 7 1 2 5 7 1
81 9 (a) n + - n -1 (b) n - + n -1
6 6 3.2 3 6 2.3
7 7
(c) (179 + 10 -20 ) (d) (99 + 10 -20 ) 1 1 1 1
81 9 (c) n + - (d) n - - n -1
2 2.3 n 3 3.2
129. The value of l2 + 32 + 52 + .......................+ 252 is :
[Online April 25, 2013] 136. The sum of the series
(a) 2925 (b) 1469 (c) 1728 (d) 1456
1 1 1
130. The sum of the series : + + + ...
1+ 2 2+ 3 3+ 4
(b)2 + 2(d)2 + 3(6)2 + ... upto 10 terms is :
[Online April 23, 2013] upto 15 terms is [Online May 12, 2012]
(a) 11300 (b) 11200 (c) 12100 (d) 12300 (a) 1 (b) 2 (c) 3 (d) 4
137. The sum of the series
3 5 7
131. The sum
2
+ 2 2
+ + .... upto 11-terms 12 + 2.22 + 32 + 2.42 + 52 + 2.62 + .... + 2(2m)2 is
1 1 +2 1 + 22 + 32
2
[Online May 7, 2012]
is: [Online April 22, 2013]
(a) m(2m + 1) 2 2
(b) m (m + 2)
7 11 11 60
(a) (b) (c) (d) (c) m2(2m + 1) (d) m(m + 2)2
2 4 2 11
138. The sum to infinite term of the series
132. The sum of the series : [Online April 9, 2013]
2 6 10 14 ... is [2009]
1 1 1+ + + + +
1+ + + ....... upto 10 terms, is : 3 32 33 34
1+ 2 1+ 2 + 3
(a) 3 (b) 4 (c) 6 (d) 2

Downloaded from @Freebooksforjeeneet


Sequences and Series M-99

1 1 1 2
139. The sum of series - + - ....... upto infinity is é n(n + 1) ù n2 (n + 1)
(a) ê
ë 2 úû
2! 3! 4! (b)
2
[2007]
1 1 n(n + 1)2 3n( n + 1)
(a) - (b) + (c) e –2 (d) e –1 (c) (d)
e 2 e 2 4 2
140. The sum of the series n n
1 r t
1 1 1 143. If Sn = å nC and t n = å nC , then n is equal to
1+ + + + ....................ad inf. is [2005] r=0 r r =0 r
Sn
4.2! 16.4! 64.6 !
[2004]
e -1 e +1 e -1 e +1 2n – 1 1 1
(a) (b) (c) (d) n -1 n
e e 2 e 2 e (a) (b) (c) n – 1 (d)
2 2 2
1 1 1 144. The sum of the series [2003]
141. The sum of series + + + ..... is [2004]
2! 4! 6! 1 1 1
- + .......... .. up to ¥ is equal to
1.2 2.3 3.4
(e2 - 2) (e - 1)2
(a) (b)
e 2e æ 4ö
(a) log e ç ÷ (b) 2 log e 2
èeø
2 2
(e - 1) (e - 1)
(c) (d) (c) log e 2 - 1 (d) log e 2
2e 2
3 3 3 3
145. 1 – 2 + 3 – 4 +...+9 = 3 [2002]
142. The sum of the first n terms of the series
(a) 425 (b) –425 (c) 475 (d) –475
12 + 2.2 2 + 3 2 + 2.4 2 + 5 2 + 2.6 2 + ... 146. The value of 21/4. 41/8. 81/16 ... ¥ is [2002]
(a) 1 (b) 2 (c) 3/2 (d) 4
n(n + 1)2
is when n is even. When n is odd the sum is
2
[2004]

Downloaded from @Freebooksforjeeneet


EBD_8344
M-122 Mathematics

10
Straight Lines and
Pair of Straight Lines
5. A triangle has a vertex at (1, 2) and the mid points of the
Distance Formula, Section two sides through it are (–1, 1) and (2, 3). Then the centroid
Formula, Results of Triangle, of this triangle is : [April 12, 2019 (II)]
Locus, Equation of Locus, Slope of
TOPIC Ć a Straight Line, Slope of a line æ 7ö æ1 ö æ1 ö æ1 5ö
joining two points, Parallel and (a) ç1, ÷ (b) ç , 2 ÷ (c) ç ,1÷ (d) ç , ÷
Perpendicular Lines è 3 ø è 3 ø è 3 ø è 3 3ø
6. Let O(0, 0) and A(0, 1) be two fixed points. Then the locus
1. A triangle ABC lying in the first quadrant has two vertices as of a point P such that the perimeter of DAOP is 4, is :
[April 8, 2019 (I)]
A(1, 2) and B(3, 1). If ÐBAC = 90°, and ar (DABC ) = 5 5 sq.
(a) 8x2 – 9y2 + 9y = 18 (b) 9x2 – 8y2 + 8y = 16
units, then the abscissa of the vertex C is:
(c) 9x2 + 8y2 – 8y = 16 (d) 8x2 + 9y2 – 9y = 18
[Sep. 04, 2020 (I)]
7. Two vertices of a triangle are (0, 2) and (4, 3). If its
(a) 1 + 5 (b) 1 + 2 5 (c) 2 + 5 (d) 2 5 - 1 orthocentre is at the origin, then its third vertex lies in
2. If the perpendicular bisector of the line segment oining which quadrant? [Jan. 10, 2019 (II)]
the points P (1, 4) and Q (k, 3) has y-intercept equal to – 4, (a) third (b) second
then a value of k is : [Sep. 04, 2020 (II)] (c) first (d) fourth
8. Let the orthocentre and centroid of a triangle be A(–3, 5)
(a) – 2 (b) – 4 (c) 14 (d) 15 and B(3, 3) respectively. If C is the circumcentre of this
3. If a DABC has vertices A(–1, 7), B(–7, 1) and C(5, –5), then triangle, then the radius of the circle having line segment
its orthocentre has coordinates : [Sep. 03, 2020 (II)] AC as diameter, is : [2018]

æ 3 3ö 5 3 5
(a) ç - , ÷ (b) (–3, 3) (a) 2 10 (b) 3 (c) (d) 10
è 5 5ø 2 2
9. A square, of each side 2, lies above the x-axis and has one
æ3 3ö
(c) ç , - ÷ (d) (3, –3) vertex at the origin. If one of the sides passing through the
è5 5ø origin makes an angle 30 with the positive direction of the
x-axis, then the sum of the x-coordinates of the vertices of
æ3 ö the square is : [Online April 9, 2017]
4. Let A(l, 0), B(6, 2) and C ç , 6÷ be the vertices of a triangle
è2 ø
(a) 2 3 - 1 (b) 2 3 - 2 (c) 3 - 2 (d) 3 - 1
ABC. If P is a point inside the triangle ABC such that the
10. A ray of light is incident along a line which meets another
triangles APC, APB and BPC have equal areas, then the
line, 7x – y + 1 = 0, at the point (0, 1). The ray is then
length of the line segment reflected from this point along the line, y + 2x = 1. Then the
æ 7 1ö equation of the line of incidence of the ray of light is :
PQ, where Q is the point çè - , - ÷ø , is ¾¾¾. [Online April 10, 2016]
6 3
(a) 41x – 25y + 25 = 0 (b) 41x + 25y – 25 = 0
[NA Jan. 7, 2020 (I)] (c) 41x – 38y + 38 = 0 (d) 41x + 38y – 38 = 0

Downloaded from @Freebooksforjeeneet


Straight Lines and Pair of Straight Lines M-123

11. Let L be the line passing through the point P(1, 2) such
(a) (3x + 1) 2 + (3 y ) 2 = a 2 - b 2
that its intercepted segment between the co-ordinate axes
is bisected at P. If L1 is the line perpendicular to L and (b) (3x - 1) 2 + (3 y ) 2 = a 2 - b 2
passing through the point (–2, 1), then the point of
intersection of L and L1 is : [Online April 10, 2015] (c) (3x - 1) 2 + (3 y ) 2 = a 2 + b 2
æ 4 12 ö æ 3 23ö
(a) çè , ÷ø (b) çè , ÷ø (d) (3x + 1) 2 + (3 y ) 2 = a 2 + b 2
5 5 5 10
19. A triangle with vertices (4, 0), (–1, –1), (3, 5) is [2002]
æ 11 29 ö æ 3 17 ö (a) isosceles and right angled
(c) çè , ÷ø (d) çè , ÷ø
20 10 10 5 (b) isosceles but not right angled
(c) right angled but not isosceles
æ 8ö
12. The points çè 0, ÷ø , (1, 3) and (82, 30) : (d) neither right angled nor isosceles
3
[Online April 10, 2015] Various Forms of Equation of a
TOPIC n Line
(a) form an acute angled triangle.
(b) form a right angled triangle.
20. Let 墰: R ® R be defined as
(c) lie on a straight line.
(d) form an obtuse angled triangle. ì 5 æ 1ö
ï x sin çè x ÷ø + 5 x , x < 0
2
13. The x-coordinate of the incentre of the triangle that has
the coordinates of mid points of its sides as (0, 1) (1, 1) and ïï
(1, 0) is : [2013] f ( x) = í 0, x=0
ï
ï x 5 cos æç ö÷ + lx 2 , x > 0
(a) 2 + 2 (b) 2 - 2 (c) 1 + 2 (d) 1 - 2 1
ïî è xø
14. A light ray emerging from the point source placed at P( l, 3)
is reflected at a point Q in the axis of x. If the reflected ray The value of l for which 墰" (0) exists, is ______.
passes through the point R (6, 7), then the abscissa of Q is: [NA Sep. 06, 2020 (I)]
[Online April 9, 2013] 21. Let C be the centroid of the triangle with vertices (3, –1),
7 5 (1, 3) and (2, 4). Let P be the point of intersection of the
(a) 1 (b) 3 (c) (d) lines x + 3y – 1 = 0 and 3x – y + 1 = 0. Then the line passing
2 2
through the points C and P also passes through the point:
15. Let A (h, k), B(1, 1) and C (2, 1) be the vertices of a right
[Jan. 9, 2020 (I)]
angled triangle with AC as its hypotenuse. If the area of
the triangle is 1square unit, then the set of values which 'k' (a) (–9, –6) (b) (9, 7) (c) (7, 6) (d) (–9, –7)
can take is given by [2007] 22. Slope of a line passing through P(2, 3) and intersecting
(a) {–1, 3} (b) {–3, –2} (c) {1, 3} (d) {0, 2} the line x + y = 7 at a distance of 4 units from P, is:
16. If a vertex of a triangle is (1, 1) and the mid points of two [April 9, 2019 (I)]
sides through this vertex are (–1, 2) and (3, 2) then the
1- 5 7 -1 5 -1
centroid of the triangle is [2005] (a) (b) 1 - 7 (c) (d)
1+ 5 1+ 7 7 +1 5 +1
æ 7ö æ -1 7 ö æ 7ö
(a) ç - 1, ÷ (b) ç , ÷ (c) ç1, ÷ (d) æç , ö÷
1 7 23. A point on the straight line, 3x + 5y = 15 which is
è 3ø è 3 3ø è 3ø è3 3ø equidistant from the coordinate axes will lie only in :
17. If the equation of the locus of a point equidistant from the [April 8, 2019 (I)]
th st
(a) 4 quadrant (b) 1 quadrant
point (a1, b1 ) and (a2, b2 ) is
(c) 1st and 2nd quadrants (d) 1st, 2nd and 4th quadrants
(a1 - b2 ) x + (a1 - b2 ) y + c = 0 , then the value of c is 24. Two vertical poles of heights, 20 m and 80 m stand apart
on a hori ontal plane. The height (in meters) of the point
[2003]
of intersection of the lines oining the top of each pole to
1 2 the foot of the other, from this hori ontal plane is :
(a) a12 + b12 - a 2 2 - b2 2 (b) a2 + b2 2 - a12 - b12
2 [April 08, 2019 (II)]
1 2 (a) 15 (b) 18 (c) 12 (d) 16
(c) a12 - a2 2 + b12 - b2 2 (d) ( a1 + a2 2 + b12 + b2 2 ) 25. If a straight line passing through the point P(–3, 4) is such
2
that its intercepted portion between the coordinate axes is
18. Locus of centroid of the triangle whose vertices are
bisected at P, then its equation is : [Jan. 12, 2019 (II)]
(a cos t , a sin t ), (b sin t , - b cos t ) and (1, 0), where t is a (a) 3x – 4y + 25 = 0 (b) 4x – 3y + 24 = 0
parameter, is [2003] (c) x – y + 7 = 0 (d) 4x + 3y = 0

Downloaded from @Freebooksforjeeneet


EBD_8344
M-124 Mathematics

26. If in a parallelogram ABDC, the coordinates of A, B and C 34. The point (2, 1) is translated parallel to the line L : x – y = 4
are respectively (1, 2), (3, 4) and (2, 5), then the equation of by 2 3 units. If the new points Q lies in the third quadrant,
the diagonal AD is : [Jan. 11, 2019 (II)]
then the equation of the line passing through Q and
(a) 5x – 3y + 1 = 0 (b) 5x + 3y – 11 = 0
perpendicular to L is : [Online April 9, 2016]
(c) 3x – 5y + 7 = 0 (d) 3x + 5y – 13 = 0
27. A point P moves on the line 2x – 3y + 4 = 0. If Q(1, 4) and (a) x + y = 2 - 6 (b) 2x + 2y = 1 - 6
R(3, –2) are fixed points, then the locus of the centroid of
DPQR is a line: [Jan. 10, 2019 (I)] (c) x + y = 3 - 3 6 (d) x + y = 3 - 2 6
3 35. A straight line L through the point (3, – 2) is inclined at
(a) with slope (b) parallel to x-axis
2 an angle of 60 to the line 3 x + y = 1. If L also
2 intersects the x-axis, then the equation of L is :
(c) with slope (d) parallel to y-axis [Online April 11, 2015]
3
28. If the line 3x + 4y – 24 = 0 intersects the x-axis at the point (a) y + 3x+2–3 3 = 0
A and the y-axis at the point B, then the incentre of the
triangle OAB, where O is the origin, is: [Jan. 10, 2019 (I)] (b) 3y+x–3+2 3 =0
(a) (3, 4) (b) (2, 2) (c) (4, 3) (d) (4, 4) (c) y – 3x+2+ 3 3 = 0
29. A straight line through a fixed point (2, 3) intersects the
coordinate axes at distinct points P and Q. If O is the origin (d) 3 y – x + 3 + 2 3 = 0
and the rectangle OPRQ is completed, then the locus of R 36. The circumcentre of a triangle lies at the origin and its
is : [2018] centroid is the mid point of the line segment oining the
(a) 2x + 3y = xy (b) 3x + 2y = xy points (a2 + 1, a2 + 1) and (2a, – 2a), a ¹ 0. Then for any a,
the orthocentre of this triangle lies on the line:
(c) 3x + 2y = 6xy (d) 3x + 2y = 6
[Online April 19, 2014]
30. In a triangle ABC, coordianates of A are (1, 2) and the (a) y – 2ax = 0
equations of the medians through B and C are x + y = 5 and
x = 4 respectively. Then area of D ABC (in sq. units) is (b) y – (a2 + 1)x = 0
[Online April 15, 2018] (c) y + x = 0
(a) 5 (b) 9 (c) 12 (d) 4 (d) (a – 1)2x – (a + 1)2y = 0
31. Two sides of a rhombus are along the lines, x – y + 1 = 0 37. If a line intercepted between the coordinate axes is trisected
and 7x – y – 5 = 0. If its diagonals intersect at (–1, –2), then at a point A(4, 3), which is nearer to x-axis, then its equation
which one of the following is a vertex of this rhombus? is: [Online April 12, 2014]
[2016] (a) 4x – 3y = 7 (b) 3x + 2y = 18
(c) 3x + 8y = 36 (d) x + 3y = 13
æ1 8ö æ 10 7 ö
(a) ç , - ÷ (b) ç - , - ÷ 38. Given three points P, Q, R with P(5, 3) and R lies on the
è 3 3ø è 3 3ø x-axis. If equation of RQ is x – 2y = 2 and PQ is parallel to
(c) (–3, –9) (d) (–3, –8) the x-axis, then the centroid of DPQR lies on the line:
32. A straight line through origin O meets the lines 3y = 10 – 4x [Online April 9, 2014]
and 8x + 6y + 5 = 0 at points A and B respectively. Then O (a) 2x + y – 9 = 0 (b) x – 2y + 1 = 0
divides the segment AB in the ratio : (c) 5x – 2y = 0 (d) 2x – 5y = 0
[Online April 10, 2016]
(a) 2 : 3 (b) 1 : 2 (c) 4 : 1 (d) 3 : 4 39. A ray of light along x + 3 y = 3 gets reflected upon
33. If a variable line drawn through the intersection of the reaching x-axis, the equation of the reflected ray is [2013]
x y x y (a) y = x + (b) 3y = x –
lines + = 1 and + = 1 , meets the coordinate axes 3 3
3 4 4 3
at A and B, (A ¹ B) , then the locus of the midpoint of AB
(c) y = 3x - 3 (d) 3y = x -1
is : [Online April 9, 2016] 40. Let A (–3, 2) and B (–2, 1) be the vertices of a triangle ABC.
(a) 7xy = 6 (x + y) If the centroid of this triangle lies on the line 3x + 4y + 2 = 0,
(b) 4 (x + y)2 – 28 (x + y) + 49 = 0 then the vertex C lies on the line :
(c) 6xy = 7 (x + y) [Online April 25, 2013]
(d) 14 (x + y)2 – 97 (x + y) + 168 = 0 (a) 4x + 3y + 5 = 0 (b) 3x + 4y + 3 = 0
(c) 4x + 3y + 3 = 0 (d) 3x + 4y + 5 = 0

Downloaded from @Freebooksforjeeneet


Straight Lines and Pair of Straight Lines M-125

41. If the extremities of the base of an isosceles triangle are the 49. A straight line through the point A (3, 4) is such that
points (2a, 0) and (0, a) and the equation of one of the its intercept between the axes is bisected at A. Its equation
sides is x = 2a, then the area of the triangle, in square units, is [2006]
is : [Online April 23, 2013] (a) x + y = 7 (b) 3x - 4 y + 7 = 0
5 2 5 2 25a 2 (c) 4 x + 3 y = 24 (d) 3 x + 4 y = 25
(a) a (b) a (c) (d) 5a2
4 2 4 50. The line parallel to the x- axis and passing through the
42. If the x-intercept of some line L is double as that of the line, intersection of the lines ax + 2by + 3b = 0 and
3x + 4y = 12 and the y-intercept of L is half as that of the bx – 2ay – 3a = 0, where (a, b) ¹ (0, 0) is [2005]
same line, then the slope of L is : [Online April 22, 2013]
3
(a) – 3 (b) – 3/8 (c) – 3/2 (d) – 3/16 (a) below the x - axis at a distance of from it
43. If the line 2x + y = k passes through the point which divides 2
the line segment oining the points (1,1) and (2,4) in the 2
ratio 3 :2, then k equals : [2012] (b) below the x - axis at a distance of from it
3
29 11
(a) (b) 5 (c) 6 (d) 3
5 5 (c) above the x - axis at a distance of from it
44. The line parallel to x-axis and passing through the point of 2
intersection of lines ax + 2by + 3b = 0 and bx – 2ay – 3a = 0, 2
where (a, b) ¹ (0, 0) is [Online May 26, 2012] (d) above the x - axis at a distance of from it
3
(a) above x-axis at a distance 2/3 from it
51. The equation of the straight line passing through the point
(b) above x-axis at a distance 3/2 from it
(4, 3) and making intercepts on the co-ordinate axes whose
(c) below x-axis at a distance 3/2 from it
sum is –1 is [2004]
(d) below x-axis at a distance 2/3 from it
45. If the point (1, a) lies between the straight lines x y x y
(a) - = 1 and + =1
x + y = 1 and 2(x + y) = 3 then a lies in interval 2 3 -2 1
[Online May 12, 2012]
x y x y
(b) - = -1 and + = -1
æ3 ö æ 3ö æ 1ö 2 3 -2 1
(a) ç , ¥ ÷ (b) ç1, ÷
è 2 ø è 2ø
(c) ( -¥, 0 ) (d) ç 0, ÷
è 2ø
x y x y
46. If the straight lines x + 3y = 4, 3x + y = 4 and x + y = 0 form (c) + = 1 and + = 1
2 3 2 1
a triangle, then the triangle is [Online May 7, 2012]
(a) scalene x y x y
(d) + = -1 and + = -1
(b) equilateral triangle 2 3 -2 1
(c) isosceles 52. Let A(2, - 3) and B ( -2, 3) be vertices of a triangle ABC.
(d) right angled isosceles If the centroid of this triangle moves on the line
47. If A (2, – 3) and B (– 2, 1) are two vertices of a triangle and
2 x + 3 y = 1, then the locus of the vertex C is the line
third vertex moves on the line 2 x + 3 y = 9, then the locus
[2004]
of the centroid of the triangle is : [2011RS]
(a) 3x - 2 y = 3 (b) 2 x - 3 y = 7
(a) x - y = 1 (b) 2 x + 3 y = 1
(c) 3x + 2 y = 5 (d) 2 x + 3 y = 9
(c) 2 x + 3 y = 3 (d) 2 x - 3 y = 1
53. Locus of mid point of the portion between the axes of
2 x x cos a + y sina = p whre p is constant is [2002]
48. If (a, a ) falls inside the angle made by the lines y = ,
2 4
(a) x2 + y2 = (b) x2 + y2 = 4p2
x > 0 and y = 3x , x > 0 , then a belong to [2006] p2

æ 1ö
(a) æç 0, ö÷ (b) (3, ¥) (c) æç , 3ö÷
1 1 1 1 2 1 1 4
(d) ç - 3, - ÷ (c) + 2 = 2 (d) 2
+ 2 = 2
è 2ø è 2 ø è 2ø x 2
y p x y p

Downloaded from @Freebooksforjeeneet


EBD_8344
M-126 Mathematics

59. If the two lines x + (a–1) y = 1 and 2x + a2y =1 (a Î R – {0,1})


Distance Between two Lines, Angle are perpendicular, then the distance of their point of
Between two Lines and Bisector of intersection from the origin is: [April 09, 2019 (II)]
the Angle Between the two Lines, 2 2 2 2
Perpendicular Distance of a Point (a) (b) (c) (d)
5 5 5 5
TOPIC Đ from a Line, Foot of the
Perpendicular, Position of a Point 60. A rectangle is inscribed in a circle with a diameter lying
with Respect to a Line, Pedal along the line 3y = x + 7. If the two adacent vertices of the
Points, Condition for Concurrency rectangle are (–8, 5) and (6, 5), then the area of the rectangle
of Three Lines (in sq. units) is: [April 09, 2019 (II)]
(a) 84 (b) 98 (c) 72 (d) 56
54. Let L denote the line in the xy-plane with x and y intercepts 61. Suppose that the points (h, k), (1, 2) and (– 3, 4) lie on the
as 3 and 1 respectively. Then the image of the point (–1, – 4) line L1. If a line L2 passing through the points (h, k) and
in this line is: [Sep. 06, 2020 (II)] (4, 3) is perpendicular on L1, then equals :
[April 08, 2019 (II)]
æ 11 28 ö æ 29 8 ö 1
(a) ç , ÷ (b) ç , ÷ 1
è5 5 ø è 5 5ø (a) 3 (b) 0 (c) 3 (d) –
7
æ 8 29 ö æ 29 11 ö 62. If the straight line, 2x – 3y + 17 = 0 is perpendicular to the
(c) ç , ÷ (d) ç , ÷ line passing through the points (7, 17) and (15, b), then b
è5 5 ø è 5 5ø
equals : [Jan. 12, 2019 (I)]
1 2 35 35
55. If the line, 2x – y + 3 = 0 is at a distance and from (a) (b) –5 (c) - (d) 5
5 5 3 3
the lines 4x – 2y + a = 0 and 6x – 3y + b = 0, respectively, 63. Two sides of a parallelogram are along the lines, x + y = 3
then the sum of all possible value of a and b is ______. and x – y + 3 = 0. If its diagonals intersect at (2, 4), then
[NA Sep. 05, 2020 (I)] one of its vertex is: [Jan. 10, 2019 (II)]
56. The locus of the mid-points of the perpendiculars drawn (a) (3, 5) (b) (2, 1) (c) (2, 6) (d) (3, 6)
from points on the line, x = 2y to the line x = y is: 64. Consider the set of all lines px + qy + r = 0 such that
[Jan. 7, 2020 (II)] 3p + 2q + 4r = 0. Which one of the following statements is
(a) 2x – 3y = 0 (b) 5x – 7y = 0 true? [Jan. 9, 2019 (I)]
(c) 3x – 2y = 0 (d) 7x – 5y = 0 æ3 1ö
(a) The lines are concurrent at the point ç , ÷ .
57. A straight line L at a distance of 4 units from the origin è4 2ø
makes positive intercepts on the coordinate axes and the
(b) Each line passes through the origin.
perpendicular from the origin to this line makes an angle
(c) The lines are all parallel.
of 60o with the line x + y = 0. Then an equation of the line
L is: [April 12, 2019 (II)] (d) The lines are not concurrent.
65. Let the equations of two sides of a triangle be
(a) x + 3 y = 8 3x – 2y + 6 = 0 and 4x + 5y – 20 = 0. If the orthocentre
of this triangle is at (1, 1), then the equation of its third
(b) ( 3 + 1) x + ( 3 - 1) y = 8 2 side is: [Jan. 09, 2019 (II)]
(c) 3x + y = 8 (a) 122y – 26x – 1675 = 0
(b) 122y + 26x + 1675 = 0
(d) None of these
(c) 26x + 61y + 1675 = 0
58. Lines are drawn parallel to the line 4x – 3y + 2 = 0, at a
(d) 26x – 122y – 1675 = 0
3 66. The foot of the perpendicular drawn from the origin, on
distance from the origin. Then which one of the
5 the line, 3x + y = l(l ¹ 0) is P. If the line meets x-axis at A and
following points lies on any of these lines ? y-axis at B, then the ratio BP : PA is
[April 10, 2019 (II)] [Online April 15, 2018]
(a) 9 : 1 (b) 1 : 3 (c) 1 : 9 (d) 3 : 1
æ 1 2ö æ 1 1ö
(a) ç - , ÷ (b) ç , - ÷ 67. The sides of a rhombus ABCD are parallel to the lines,
è 4 3ø è 4 3ø x – y + 2 = 0 and 7x – y + 3 = 0. If the diagonals of the
rhombus intersect at P(1, 2) and the vertex A (different
æ1 1ö æ 1 2ö from the origin) is on the y-axis, then the ordinate of A is
(c) ç , ÷ (d) ç - , - ÷
è 4 3ø è 4 3ø [Online April 15, 2018]
7 7 5
(a) 2 (b) (c) (d)
4 2 2

Downloaded from @Freebooksforjeeneet


Straight Lines and Pair of Straight Lines M-127

68. Let a, b, c and d be non- ero numbers. If the point of 76. Consider the straight lines
intersection of the lines 4ax + 2ay + c = 0 and 5bx + 2by + d L1 : x – y = 1
=0 lies in the fourth quadrant and is equidistant from the L2 : x + y = 1
two axes then [2014] L3 : 2x + 2y = 5
(a) 3bc – 2ad = 0 (b) 3bc + 2ad = 0 L4 : 2x – 2y = 7
(c) 2bc – 3ad = 0 (d) 2bc + 3ad = 0 The correct statement is [Online May 26, 2012]
69. Let PS be the median of the triangle vertices
P(2, 2), Q(6, –1) and R(7, 3). The equation of the line (a) L1 P L4 , L2 P L3 , L1 intersect L4.
passing through (1, –1) and parallel to PS is: [2014] (b) L1 ^ L2 , L1 P L3 , L1 intersect L2.
(a) 4x + 7y + 3 = 0 (b) 2x – 9y – 11 = 0
(c) 4x – 7y – 11 = 0 (d) 2x + 9y + 7 = 0 (c) L1 ^ L2 , L2 P L3 ,L1 intersect L4.
70. If a line L is perpendicular to the line 5x – y = 1, and the area (d) L1 ^ L2 , L1 ^ L3 , L2 intersect L4.
of the triangle formed by the line L and the coordinate axes
77. If a, b, c Î R and 1 is a root of equation ax2 + bx + c = 0,
is 5, then the distance of line L from the line x + 5y = 0 is:
then the curve y = 4ax2 + 3bx + 2c, a ¹ 0 intersect x-axis at
[Online April 19, 2014]
[Online May 26, 2012]
7 5 7 5 (a) two distinct points whose coordinates are always
(a) (b) (c) (d) rational numbers
5 13 13 7
71. If the three distinct lines x + 2ay + a = 0, x + 3by + b = 0 and (b) no point
x + 4ay + a = 0 are concurrent, then the point (a, b) lies on (c) exactly two distinct points
a: [Online April 12, 2014] (d) exactly one point
(a) circle (b) hyperbola 78. Let L be the line y = 2x, in the two dimensional plane.
(c) straight line (d) parabola [Online May 19, 2012]
72. The base of an equilateral triangle is along the line given Statement 1: The image of the point (0, 1) in L is the point
by 3x + 4y = 9. If a vertex of the triangle is (1, 2), then the æ 4 3ö.
length of a side of the triangle is: [Online April 11, 2014] ç , ÷
è5 5ø
2 3 4 3 4 3 2 3
(a) (b) (c) (d) æ 4 3ö
15 15 5 5 Statement 2: The points (0, 1) and ç , ÷ lie on opposite
è5 5ø
73. If the image of point P(2, 3) in a line L is Q(4, 5), then the
sides of the line L and are at equal distance from it.
image of point R(0, 0) in the same line is:
(a) Statement 1 is true, Statement 2 is false.
[Online April 25, 2013]
(b) Statement 1 is true, Statement 2 is true, Statement 2 is
(a) (2, 2) (b) (4, 5) (c) (3, 4) (d) (7, 7) not a correct explanation for Statement 1.
74. Let q1 be the angle between two lines 2x + 3y + c1 = 0 and (c) Statement 1 is true, Statement 2 is true, Statement 2 is
– x + 5y + c2 = 0 and q2 be the angle between two lines a correct explanation for Statement 1.
2x + 3y + c1 = 0 and – x + 5y + c3 = 0, where c1, c2, c3 are any (d) Statement 1 is false, Statement 2 is true.
real numbers :
79. If two vertices of a triangle are (5, –1) and (–2, 3) and its
Statement-1: If c2 and c3 are proportional, then q1 = q2. orthocentre is at (0, 0), then the third vertex is
Statement-2: q1 = q2 for all c2 and c3. [Online May 12, 2012]
[Online April 23, 2013] (a) (4, – 7) (b) (– 4, – 7) (c) (– 4, 7) (d) (4, 7)
(a) Statement-1 is true, Statement-2 is true; Statement-2 is 80. If two vertical poles 20 m and 80 m high stand apart on a
a correct explanation of Statement-1. hori ontal plane, then the height (in m) of the point of
(b) Statement-1 is true, Statement-2 is true; Statement-2 is intersection of the lines oining the top of each pole to the
not a correct explanation of Statement-1. foot of other is [Online May 7, 2012]
(c) Statement-1 is false; Statement-2 is true. (a) 16 (b) 18 (c) 50 (d) 15
(d) Statement-1 is true; Statement-2 is false. 81. The point of intersection of the lines
75. If the three lines x – 3y = p, ax + 2y = q and (a3 + 3)x + ay + a – 3 = 0 and
ax + y = r form a right-angled triangle then : (a5 + 2)x + (a + 2)y + 2a + 3 = 0 (a real) lies on the y-axis for
[Online April 9, 2013] [Online May 7, 2012]
(a) a2 – 9a + 18 = 0 (b) a2 – 6a – 12 = 0 (a) no value of a (b) more than two values of a
(c) a2 – 6a – 18 = 0 (d) a2 – 9a + 12 = 0 (c) exactly one value of a (d) exactly two values of a

Downloaded from @Freebooksforjeeneet


EBD_8344
M-128 Mathematics

89. A square of side a lies above the x-axis and has one vertex
82. The lines x + y = a and ax – y = 1 intersect each other in
at the origin. The side passing through the origin makes an
the first quadrant. Then the set of all possible values of a
in the interval : [2011RS] æ pö
angle aç 0 < a < ÷ with the positive direction of x-axis. The
è 4ø
(a) ( 0, ¥ ) (b) [1, ¥) (c) ( -1, ¥) (d) ( -1,1) equation of its diagonal not passing through the origin is
83. The lines L1 : y – x = 0 and L2 : 2x + y = 0 intersect the line [2003]
L3 : y + 2 = 0 at P and Q respectively. The bisector of the
(a) y (cos a + sin a ) + x(cos a - sin a ) = a
acute angle between L1 and L2 intersects L3 at R. [2011]
Statement-1: The ratio PR : RQ equals 2 2 : 5 (b) y (cos a - sin a ) - x(sin a - cos a ) = a
Statement-2: In any triangle, bisector of an angle divides (c) y (cos a + sin a ) + x(sin a - cos a ) = a
the triangle into two similar triangles. (d) y (cos a + sin a ) + x (sin a + cos a ) = a
(a) Statement-1 is true, Statement-2 is true; Statement-2 is
not a correct explanation for Statement-1.
(b) Statement-1 is true, Statement-2 is false.
TOPIC Ė Pair of Straight Lines
(c) Statement-1 is false, Statement-2 is true.
90. The equation y = sin x sin (x + 2) – sin 2 (x + 1) represents
(d) Statement-1 is true, Statement-2 is true; Statement-2 is
a straight line lying in : [April 12, 2019 (I)]
a correct explanation for Statement-1.
(a) second and third quadrants only
84. The lines p(p2 +1)x – y + q = 0 and (b) first, second and fourth quadrant
(p2 + 1)2x + (p2 + 1)y + 2q = 0 are perpendicular to a common (c) first, third and fourth quadrants
line for : [2009] (d) third and fourth quadrants only
(a) exactly one values of p 91. If one of the lines of my2 + (1– m2) xy – mx2= 0 is a bisector
(b) exactly two values of p of the angle between the lines xy = 0, then m is [2007]
(c) more than two values of p (a) 1 (b) 2 (c) –1/2 (d) –2
(d) no value of p 92. If one of the lines given by
85. The shortest distance between the line y – x = 1 and the
6 x 2 - xy + 4cy 2 = 0 is 3x + 4y = 0, then c equals [2004]
curve x = y2 is : [2009]
(a) –3 (b) 1 (c) 3 (d) 1
2 3 3 2 3 3 2 93. If the sum of the slopes of the lin es given by
(a) (b) (c) (d)
8 5 4 8
x 2 - 2cxy - 7 y 2 = 0 is four times their product c has the
86. The perpendicular bisector of the line segment oining
P (1, 4) and Q(k, 3) has y-intercept –4. Then a possible value [2004]
value of k is [2008] (a) –2 (b) –1 (c) 2 (d) 1
(a) 1 (b) 2 (c) –2 (d) – 4 94. If the pair of straight lines x 2 - 2 pxy - y 2 = 0 and
87. Let P = (–1, 0), Q = (0, 0) and R = (3, 3 3 ) be three point.
x 2 - 2qxy - y 2 = 0 be such that each pair bisects the angle
The equation of the bisector of the angle PQR is [2007]
between the other pair, then [2003]
3 (a) pq = –1 (b) p = q (c) p = –q (d) pq = 1.
(a) x+ y = 0 (b) x + 3 y = 0
2 95. The pair of lines represented by
3ax2 + 5xy + (a2 – 2)y2 = 0
3
(c) 3x + y = 0 (d) x + y=0 are perpendicular to each other for [2002]
2 (a) two values of a (b) " a
88. If x1, x2 , x3 and y1, y2 , y3 are both in G.P. with the same (c) for one value of a (d) for no values of a
common ratio, then the points ( x1, y1 ), ( x 2 , y 2 ) and 96. If the pair of lines ax + 2hxy + by2 + 2gx + 2fy + c = 0
2

intersect on the y-axis then [2002]


( x3 , y3 ) [2003] 2
(a) 2fgh = bg + ch 2 (b) bg ¹ ch
2 2

(a) are vertices of a triangle (c) abc = 2fgh (d) none of these
(b) lie on a straight line
(c) lie on an ellipse
(d) lie on a circle.

Downloaded from @Freebooksforjeeneet


11
Conic Sections

8. If a line, y = mx + c is a tangent to the circle, (x – 3)2 + y2 = 1


TOPIC Ć Circles and it is perpendicular to a line L1, where L1 is the tangent
æ 1 1 ö
1. If the length of the chord of the circle, x2 + y 2 = r 2 (r > 0) to the circle, x2 + y2 = 1 at the point ç , ÷ ; then:
è 2 2ø
along the line, y – 2x = 3 is r, then r2 is equal to :
[Jan. 8, 2020 (II)]
[Sep. 05, 2020 (II)] (a) c2 – 7c + 6 = 0 (b) c2 + 7c + 6 = 0
9 24 12 (c) c2 + 6c + 7 = 0 (d) c2 – 6c + 7 = 0
(a) (b) 12 (c) (d) 9. Let the tangents drawn from the origin to the circle,
5 5 5
x2 + y2 – 8x – 4y + 16 = 0 touch it at the points A and B. The
2. The circle passing through the intersection of the circles,
(AB)2 is equal to: [Jan. 7, 2020 (II)]
x2 + y 2 - 6 x = 0 and x2 + y 2 - 4 y = 0, having its centre 52 56 64 32
(a) (b) (c) (d)
on the line, 2 x - 3 y + 12 = 0, also passes through the 5 5 5 5
point: [Sep. 04, 2020 (II)] 10. If the angle of intersection at a point where the two circles
(a) (–1, 3) (b) (–3, 6) (c) (–3, 1) (d) (1, –3) with radii 5 cm and 12 cm intersect is 90o, then the length
3. Let PQ be a diameter of the circle x2 + y2 = 9. If a and b are (in cm) of their common chord is : [April 12, 2019 (I)]
the lengths of the perpendiculars from P and Q on the 13 120 60 13
straight line, x + y = 2 respectively, then the maximum (a) (b) (c) (d)
5 13 13 2
value of ab is ______________. [NA Sep. 04, 2020 (II)]
4. The diameter of the circle, whose centre lies on the line 11. A circle touching the x-axis at (3, 0) and making an intercept
x + y = 2 in the first quadrant and which touches both the of length 8 on the y-axis passes through the point :
lines x = 3 and y = 2, is __________. [April 12, 2019 (II)]
[NA Sep. 03, 2020 (I)] (a) (3, 10) (b) (3, 5)
5. The number of integral values of k for which the line, (c) (2, 3) (d) (1, 5)
12. If the circles x2 + y2 + 5Kx + 2y + K = 0 and 2 (x2 + y2) +
3x + 4y = k intersects the circle, x2 + y 2 - 2 x - 4 y + 4 = 0 2Kx + 3y – 1= 0, (KÎR), intersect at the points P and Q,
at two distinct points is ________. [NA Sep. 02, 2020 (I)] then the line 4x + 5y – K = 0 passes through P and Q, for:
[April 10, 2019 (I)]
6. A circle touches the y-axis at the point (0, 4) and passes
(a) infinitely many values of K
through the point (2, 0). Which of the following lines is
(b) no value of K.
not a tangent to this circle? [Jan. 9, 2020 (I)]
(c) exactly two values of K
(a) 4x – 3y + 17 = 0 (b) 3x – 4y – 24 = 0 (d) exactly one value of K
(c) 3x + 4y – 6 = 0 (d) 4x + 3y – 8 = 0 13. The line x = y touches a circle at the point (1, 1). If the
7. If the curves, x2 – 6x + y2 + 8 = 0 and x2 – 8y + y2 + 16 – k = 0, circle also passes through the point (1, –3), then its radius
(k > 0) touch each other at a point, then the largest value is: [April 10, 2019 (I)]
of k is ______. [NA Jan. 9, 2020 (II)]
(a) 3 (b) 2 2 (c) 2 (d) 3 2

Downloaded from @Freebooksforjeeneet


EBD_8344
M-148 Mathematics

14. The locus of the centres of the circles, which touch the 21. If a variable line, 3x + 4y – l = 0 is such that the two circles
circle, x2 + y2 = 1 externally, also touch the y-axis and lie in x2 + y2 – 2x – 2y + 1 = 0 and x2 + y2 – 18x – 2y + 78 = 0 are
the first quadrant, is: [April 10, 2019 (II)] on its opposite sides, then the set of all values of l is the
interval : [Jan. 12, 2019 (I)]
(a) x = 1 + 4 y , y ³ 0 (b) y = 1 + 2 x , x ³ 0 (a) (2, 17) (b) [13, 23]
(c) [12, 21] (d) (23, 31)
(c) y = 1 + 4 x , x ³ 0 (d) x = 1 + 2 y , y ³ 0
22. A square is inscribed in the circle x 2 + y 2 - 6 x + 8 y - 103 = 0
ìa + i ü with its sides parallel to the coordinate axes. Then the
15. All the points in the set S = í : µ Î R ý (i = -1)
îa -1 þ distance of the vertex of this square which is nearest to the
origin is : [Jan. 11, 2019 (I)]
lie on a: [April 09, 2019 (I)]
(a) 6 (b) 137 (c) 41 (d) 13
(a) straight line whose slope is 1.
23. Two circles with equal radii are intersecting at the points
(b) circle whose radius is 1.
(0, 1) and (0, –1). The tangent at the point (0, 1) to one of
(c) circle whose radius is 2. the circles passes through the centre of the other circle.
(d) straight line whose slope is –1. Then the distance between the centres of these circles is :
16. If a tangent to the circle x2 + y2 = 1intersects the coordinate [Jan. 11, 2019 (I)]
axes at distinct points P and Q, then the locus of the mid- (a) 1 (b) 2 (c) 2 2 (d) 2
point of PQ is: [April 09, 2019 (I)] 24. A circle cuts a chord of length 4a on the x-axis and passes
2 2 2 2
(a) x + y – 4x y = 0 (b) x + y2 – 2xy = 0
2 through a point on the y-axis, distant 2b from the origin.
Then the locus of the centre of this circle, is :
(c) x2 + y2 – 16x2y2 = 0 (d) x2 + y2 – 2x2y2 = 0
[Jan. 11, 2019 (II)]
17. The common tangent to the circles x2 + y2 = 4 and x2 + y2 + (a) a hyperbola (b) an ellipse
6x + 8y – 24 = 0 also passes through the point: (c) a straight line (d) a parabola
[April 09, 2019 (II)] 25. If a circle C passing through the point (4, 0) touches the
circle x2 + y2 + 4x – 6y = 12 externally at the point
(a) (4, –2) (b) (– 6, 4) (c) (6, –2) (d) (– 4, 6) (1, – 1), then the radius of C is: [Jan 10, 2019 (I)]
18. The sum of the squares of the lengths of the chords (a) 2 5 (b) 4 (c) 5 (d) 57
intercepted on the circle, x2 + y2 = 16, by the lines, x + y = n,
26. If the area of an equilateral triangle inscribed in the
n Î N, where N is the set of all natural numbers, is :
[April 08, 2019 (I)] circle, x2 + y2 + 10x + 12y + c = 0 is 27 3 sq. units then
(a) 320 (b) 105 (c) 160 (d) 210 c is equal to: [Jan. 10, 2019 (II)]
19. If a circle of radius R passes through the origin O and (a) 13 (b) 20 (c) – 25 (d) 25
intersects the coordinate axes at A and B, then the locus of 27. Three circles of radii a, b, c (a < b < c) touch each other
the foot of perpendicular from O on AB is : externally. If they have x-axis as a common tangent, then:
[Jan 09, 2019 (I)]
[Jan. 12, 2019 (II)]
1 1 1 1 1 1
(a) (x 2
+y )
2 2 2 2 2
= 4R x y (a)
a
=
b
+
c
(b)
b
=
a
+
c

( x2 + y 2 )
3
(b) = 4 R2 x 2 y2 (c) a, b, c are in A.P (d) a , b, c are in A.P..
28. If the circles x + y – 16x – 20y + 164 = r2 and
2 2

(x – 4)2 + (y – 7)2 = 36 intersect at two distinct points,


( x2 + y 2 )
2
(c) = 4 Rx 2 y 2 then: [Jan. 09, 2019 (II)]
(a) r > 11 (b) 0 < r < 1
(d) ( x2 + y2 ) ( x + y ) = R2 xy (c) r = 11 (d) 1 < r < 11
29. The straight line x + 2y = 1 meets the coordinate axes at A
20. Let C1 and C2 be the centres of the circles x2 + y2 – 2x –2y – 2 = 0 and B. A circle is drawn through A, B and the origin. Then
and x2 + y2 – 6x –6y + 14 = 0 respectively. If P and Q are the sum of perpendicular distances from A and B on the
the points of intersection of these circles then, the area tangent to the circle at the origin is : [Jan. 11, 2019 (I)]
(in sq. units) of the quadrilateral PC1QC2 is : 5 5
[Jan. 12, 2019 (I)] (a) (b) 2 5 (c) (d) 4 5
2 4
(a) 8 (b) 6 (c) 9 (d) 4

Downloaded from @Freebooksforjeeneet


Conic Sections M-149

30. If the tangent at (1, 7) to the curve x 2 = y - 6 touches the 8


(a) a circle with radius .
3
circle x 2 + y2 + 16x + 12y + c = 0 then the value of c is :
10
[2018] (b) a circle with diameter .
3
(a) 185 (b) 85 (c) 95 (d) 195
31. If a circle C, whose radius is 3, touches externally the circle, 16
(c) an ellipse with length of maor axis .
x2 + y2 + 2x – 4y – 4 = 0 at the point (2, 2), then the length 3
of the intercept cut by this circle c, on the x-axis is equal to 16
[Online April 16, 2018] (d) an ellipse with length of minor axis
9
(a) 5 (b) 2 3 (c) 3 2 (d) 2 5 40. If a point P has co–ordinates (0, –2) and Q is any point on
32. A circle passes through the points (2, 3) and (4, 5). If its the circle, x2 + y2 – 5x – y + 5 = 0, then the maximum value
centre lies on the line, y – 4x + 3 = 0, then its radius is equal of (PQ)2 is : [Online April 8, 2017]
to [Online April 15, 2018]
25 + 6
(a) 5 (b) 1 (c) 2 (d) 2 (a) (b) 14 + 5 3
2
33. Two parabolas with a common vertex and with axes along
x-axis and y-axis, respectively, intersect each other in the 47 + 10 6
(c) (d) 8 + 5 3
first quadrant. if the length of the latus rectum of each 2
parabola is 3, then the equation of the common tangent to 41. If two parallel chords of a circle, having diameter 4 units,
the two parabolas is? [Online April 15, 2018] lie on the opposite sides of the centre and subtend angles
(a) 3 (x + y) + 4 = 0 (b) 8 (2x + y) + 3 = 0
æ1ö
(c) 4 (x + y) + 3 = 0 (d) x + 2y + 3 = 0 cos -1 ç ÷ and sec–1 (7) at the centre respectively, then
è7ø
34. The tangent to the circle C1 : x2 + y2 – 2x – 1 = 0 at the point
(2, 1) cuts off a chord of length 4 from a circle C2 whose the distance between these chords, is :
centre is (3, – 2). The radius of C2 is [Online April 8, 2017]
[Online April 15, 2018] 4 8 8 16
(a) (b) (c) (d)
(a) 6 (b) 2 (c) 2 (d) 3 7 7 7 7
35. The radius of a circle, having minimum area, which touches 42. If one of the diameters of the circle, given by the equation,
the curve y = 4 – x2 and the lines, y = |x| is : [2017] x2 + y2 – 4x + 6y – 12 = 0, is a chord of a circle S, whose
(a) 4 ( 2 +1 ) (b) 2 ( 2 +1 ) centre is at (–3, 2), then the radius of S is:
(a) 5 (b) 10 (c) 5 2 (d) 5 3
[2016]

(c) 2( 2 - 1) (d) 4( 2 - 1) 43. Equation of the tangent to the circle, at the point (1, –1)
whose centre is the point of intersection of the straight
36. The equation lines x – y = 1 and 2x + y = 3 is : [Online April 10, 2016]
æ iz - 2 ö (a) x + 4y + 3 = 0 (b) 3x – y – 4 = 0
Im ç ÷ + 1 = 0, z Î C, z ¹ i represents a part of a circle (c) x – 3y – 4 = 0 (d) 4x + y – 3 = 0
è z -i ø
44. A circle passes through (–2, 4) and touches the y-axis at
having radius equal to : [Online April 9, 2017]
(0, 2). Which one of the following equations can represent
3 1 a diameter of this circle ? [Online April 9, 2016]
(a) 2 (b) 1 (c) (d)
4 2 (a) 2x – 3y + 10 = 0 (b) 3x + 4y – 3 = 0
37. A line drawn through the point P(4, 7) cuts the circle (c) 4x + 5y – 6 = 0 (d) 5x + 2y + 4 = 0
x2 + y2 = 9 at the points A and B. Then PA·PB is equal to : 45. Locus of the image of the point (2, 3) in the line
[Online April 9, 2017] (2x – 3y + 4) + k (x – 2y + 3) = 0, k Î R, is a : [2015]
(a) 53 (b) 56 (c) 74 (d) 65 (a) circle of radius 2.
38. The two adacent sides of a cyclic quadrilateral are 2 and
5 and the angle between them is 60 . If the area of the (b) circle of radius 3.
(c) straight line parallel to x-axis
quadrilateral is 4 3 , then the perimeter of the quadrilat-
(d) straight line parallel to y-axis
eral is : [Online April 9, 2017] 46. The n umber of common tangents to the cir cles
(a) 12.5 (b) 13.2 (c) 12 (d) 13 x2 + y2 – 4x – 6x – 12 = 0 and x2 + y2 + 6x + 18y + 26 = 0, is:
39. Let Î C, the set of complex numbers. Then the equation, [2015]
2| +3i| – | – i| = 0 represents : [Online April 8, 2017] (a) 3 (b) 4 (c) 1 (d) 2

Downloaded from @Freebooksforjeeneet


EBD_8344
M-150 Mathematics

47. If the incentre of an equilateral triangle is (1, 1) and the 55. The set of all real values of l for which exactly two common
equation of its one side is 3x + 4y + 3 = 0, then the tangents can be drawn to the circles
equation of the circumcircle of this triangle is : x2 + y2 – 4x – 4y + 6 = 0 and
[Online April 11, 2015] x2 + y2 – 10x – 10y + l = 0 is the interval:
(a) x2 + y2 – 2x – 2y – 14 = 0 [Online April 11, 2014]
(b) x2 + y2 – 2x – 2y – 2 = 0 (a) (12, 32) (b) (18, 42)
(c) x2 + y2 – 2x – 2y + 2 = 0 (c) (12, 24) (d) (18, 48)
(d) x2 + y2 – 2x – 2y – 7 = 0 56. If the point (1, 4) lies inside the circle
x2 + y2 – 6x – 10y + P = 0 and the circle does not touch or
48. If a circle passing through the point (–1, 0) touches y-
intersect the coordinate axes, then the set of all possible
axis at (0, 2), then the length of the chord of the circle
values of P is the interval: [Online April 9, 2014]
along the x-axis is : [Online April 11, 2015]
(a) (0, 25) (b) (25, 39)
3 5 (c) (9, 25) (d) (25, 29)
(a) (b) 3 (c) (d) 5
2 2 1 1 1
57. Let a and b be any two numbers satisfying + = .
49. Let the tangents drawn to the circle, x2 + y2 = 16 from the a b 2 2 4
point P(0, h) meet the x-axis at point A and B. If the area of Then, the foot of perpendicular from the origin on the
DAPB is minimum, then h is equal to :
x y
[Online April 10, 2015] variable line, + = 1 , lies on: [Online April 9, 2014]
a b
(a) 4 2 (b) 3 3 (c) 3 2 (d) 4 3
50. If y + 3x = 0 is the equation of a chord of the circle, (a) a hyperbola with each semi-axis = 2
x2 + y2 – 30x = 0, then the equation of the circle with this (b) a hyperbola with each semi-axis = 2
chord as diameter is : [Online April 10, 2015] (c) a circle of radius = 2
(a) x2 + y2 + 3x + 9y = 0 (b) x2 + y2 + 3x – 9y = 0 (d) a circle of radius = 2
(c) x2 + y2 – 3x – 9y = 0 (d) x2 + y2 – 3x + 9y = 0 58. The circle passing through (1, –2) and touching the axis of
51. The largest value of r for which the region represented by x at (3, 0) also passes through the point [2013]
the set {w Î C|w – 4 – i| £r} is contained in the region (a) (–5, 2) (b) (2, –5) (c) (5, –2) (d) (–2, 5)
59. If a circle of unit radius is divided into two parts by an arc
represented by the set ( z Î c / | z - 1 |£| z + i | ) , is equal of another circle subtending an angle 60 on the
to: [Online April 10, 2015] circumference of the first circle, then the radius of the arc
is: [Online April 25, 2013]
5 3
(a) 2 (b) 2 2 (c) 2 (d) 17
2 2 1
(a) 3 (b) (c) 1 (d) 2
52. Let C be the circle with centre at (1, 1) and radius = 1. If T is 2
the circle centred at (0, y), passing through origin and 60. Statement 1: The only circle having radius 10 and a
touching the circle C externally, then the radius of T is diameter along line 2x + y = 5 is x2 + y2 – 6x + 2y = 0.
equal to [2014] Statement 2 : 2x + y = 5 is a normal to the circle
1 1 3 x2 + y2 – 6x + 2y = 0. [Online April 25, 2013]
3
(a) (b) (c) (d) (a) Statement 1 is false; Statement 2 is true.
2 4 2 2
(b) Statement 1 is true; Statement 2 is true, Statement 2 is
53. The equation of circle described on the chord a correct explanation for Statement 1.
3x + y + 5 = 0 of the circle x2 + y2 = 16 as diameter is: (c) Statement 1 is true; Statement 2 is false.
[Online April 19, 2014]
(d) Statement 1 is true; Statement 2 is true; Statement 2 is
(a) x2 + y2 + 3x + y – 11 = 0
not a correct explanation for Statement 1.
(b) x2 + y2 + 3x + y + 1 = 0
61. If the circle x2 + y2 – 6x – 8y + (25 – a2) = 0 touches the axis
(c) x2 + y2 + 3x + y – 2 = 0
of x, then a equals. [Online April 23, 2013]
(d) x2 + y2 + 3x + y – 22 = 0
54. For the two circles x2 + y2 = 16 and (a) 0 (b) ± 4 (c) ± 2 (d) ± 3
x2 + y2 – 2y = 0, there is/are [Online April 12, 2014] 62. If a circle C passing through (4, 0) touches the circle
(a) one pair of common tangents x2 + y2 + 4x – 6y – 12 = 0 externally at a point (1, –1), then
(b) two pair of common tangents the radius of the circle C is : [Online April 22, 2013]
(c) three pair of common tangents (a) 5 (b) 2 5 (c) 4 (d) 57
(d) no common tangent

Downloaded from @Freebooksforjeeneet


Conic Sections M-151

63. If two vertices of an equilateral triangle are 70. The equation of the circle passing through the point (1, 0)
A (– a, 0) and B (a, 0), a > 0, and the third vertex C lies and (0, 1) and having the smallest radius is - [2011 RS]
above x-axis then the equation of the circumcircle of DABC
is : [Online April 22, 2013] (a) x2 + y 2 - 2 x - 2 y + 1 = 0
(b) x2 + y2 – x – y = 0
(a) 3x2 + 3y2 – 2 3ay = 3a 2
(c) x2 + y2 + 2x + 2y – 7= 0
(b) 3 x 2 + 3 y 2 - 2ay = 3a 2 (d) x2 + y2 + x + y – 2 = 0
71. The two circles x2 + y2 = ax and x2 + y2 = c2 (c > 0) touch
(c) x 2 + y 2 - 2ay = a 2 each other if [2011]
(a) | a | = c (b) a = 2c
(d) x 2 + y 2 - 3ay = a 2 (c) | a | = 2c (d) 2 | a | = c
64. If each of the lines 5x + 8y = 13 and 4x – y = 3 contains a 72. The circle x2 + y2 = 4x + 8y + 5 intersects the line 3x – 4y = m
diameter of the circle at two distinct points if [2010]
x2 + y2 – 2(a2 – 7a + 11) x – 2 (a2 – 6a + 6) y + b3 + 1 = 0, (a) – 35 < m < 15 (b) 15 < m < 65
then [Online April 9, 2013] (c) 35 < m < 85 (d) – 85 < m < – 35
(a) a = 5 and b Ï (-1,1) (b) a = 1 and b Ï (-1,1) 73. If P and Q are the points of intersection of the circles

(c) a = 2 and b Ï (-¥,1) (d) a = 5 and b Î (-¥,1) x 2 + y2 + 3 x + 7 y + 2 p - 5 = 0 and

65. The length of the diameter of the circle which touches the x2 + y2 + 2x + 2y – p2=0 then there is a circle passing
x-axis at the point (1,0) and passes through the point (2,3) is: through P, Q and (1, 1) for: [2009]
[2012] (a) all except one value of p
(b) all except two values of p
10 3 6 5 (c) exactly one value of p
(a) (b) (c) (d)
3 5 5 3 (d) all values of p
66. The number of common tangents of the circles given by 74. Three distinct points A, B and C are given in the
x2 + y2 – 8x – 2y + 1 = 0 and x2 + y2 + 6x + 8y = 0 is 2-dimensional coordinates plane such that the ratio of the
[Online May 26, 2012] distance of any one of them from the point (1, 0) to the
(a) one (b) four (c) two (d) three
67. If the line y = mx + 1 meets the circle x2 + y2 + 3x = 0 in two 1
distance from the point (–1, 0) is equal to . Then the
points equidistant from and on opposite sides of x-axis, 3
then [Online May 19, 2012] circumcentre of the triangle ABC is at the point: [2009]
(a) 3m + 2 = 0 (b) 3m – 2 = 0 æ5 ö æ5 ö
(c) 2m + 3 = 0 (d) 2m – 3 = 0 (a) çè , 0÷ø (b) çè , 0÷ø
4 2
68. If three distinct points A, B, C are given in the 2-dimensional
coordinate plane such that the ratio of the distance of æ5 ö
each one of them from the point (1, 0) to the distance from (c) çè , 0÷ø (d) (0, 0)
3
1
(– 1, 0) is equal to , then the circumcentre of the triangle 75. The point diametrically opposite to the point
2 P(1, 0) on the circle x2 + y2 + 2x + 4y – 3 = 0 is [2008]
ABC is at the point [Online May 19, 2012] (a) (3, – 4) (b) (–3, 4)
æ5 ö (c) (–3, –4) (d) (3, 4)
(a) ç , 0 ÷ (b) (0, 0)
è3 ø 76. Consider a family of circles which are passing through the
point (– 1, 1) and are tangent to x-axis. If (h, k) are the
æ1 ö
(c) ç , 0 ÷ (d) (3, 0) coordinate of the centre of the circles, then the set of values
è3 ø of k is given by the interval [2007]
69. The equation of the circle passing through the point (1, 2)
1
and through the points of intersection of (a) - 1 £ k £ 1 (b) k £
x2 + y2 – 4x – 6y – 21 = 0 and 3x + 4y + 5 = 0 is given by 2 2 2
[Online May 7, 2012]
(a) x2 + y2 + 2x + 2y + 11 = 0 1 1
(c) 0 £ k £ (d) k ³
(b) x2 + y2 – 2x + 2y – 7 = 0 2 2
(c) x2 + y2 + 2x – 2y – 3 = 0
(d) x2 + y2 + 2x + 2y – 11 = 0

Downloaded from @Freebooksforjeeneet


EBD_8344
M-152 Mathematics

77. Let C be the circle with centre (0, 0) and radius 3 units. The 82. If a circle passes through the point (a, b) and cuts the
equation of the locus of the mid points of the chords of the
circle x 2 + y 2 = 4 orthogonally, then the locus of its centre
circle C that subtend an angle of 2p at its center is is [2004]
3
(a) 2ax - 2by - (a 2 + b 2 + 4) = 0
(a) x 2 + y 2 = 3 (b) x 2 + y 2 = 1 [2006]
2 (b) 2ax + 2by - (a 2 + b 2 + 4) = 0

9 (c) 2ax - 2by + (a 2 + b 2 + 4) = 0


(d) x 2 + y 2 =
27
(c) x 2 + y 2 =
4 4
(d) 2ax + 2by + (a 2 + b 2 + 4) = 0
78. If the lines 3x - 4 y - 7 = 0 and 2 x - 3 y - 5 = 0 are two
diameters of a circle of area 49p square units, the equa- 83. A variable circle passes through the fixed point A( p, q ) and
tion of the circle is [2006] touches x-axis . The locus of the other end of the diameter
through A is [2004]
(a) x 2 + y 2 + 2 x - 2 y - 47 = 0
(a) ( y - q)2 = 4 px (b) ( x - q)2 = 4 py
2 2
(b) x + y + 2 x - 2 y - 62 = 0
(c) ( y - p )2 = 4qx (d) ( x - p)2 = 4qy
2 2
(c) x + y - 2 x + 2 y - 62 = 0
84. If the lines 2 x + 3 y + 1 = 0 and 3x - y - 4 = 0 lie along
2 2
(d) x + y - 2 x + 2 y - 47 = 0 diameter of a circle of circumference 10p, then the equation
79. If a circle passes through the point (a, b) and cuts the of the circle is [2004]

circle x 2 + y 2 = p 2 orthogonally, then the equation of (a) x 2 + y 2 + 2 x - 2 y - 23 = 0


the locus of its centre is [2005]
(b) x 2 + y 2 - 2 x - 2 y - 23 = 0
2 2 2 2 2
(a) x + y – 3ax – 4by + ( a + b - p ) = 0
(c) x 2 + y 2 + 2 x + 2 y - 23 = 0
2 2 2
(b) 2ax + 2by – ( a - b + p ) = 0
(d) x 2 + y 2 - 2 x + 2 y - 23 = 0
2 2
(c) x + y – 2ax – 3by + ( a - b - p ) = 0
2 2 2
85. Intercept on the line y = x by the circle x 2 + y 2 - 2 x = 0 is
2 2 2
(d) 2ax + 2by – ( a + b + p ) = 0 AB. Equation of the circle on AB as a diameter is [2004]
80. If the pair of lines ax 2 + 2 (a + b)xy + by 2 = 0 lie along (a) x 2 + y 2 + x - y = 0 (b) x 2 + y 2 - x + y = 0
diameters of a circle and divide the circle into four sectors
such that the area of one of the sectors is thrice the area of (c) x 2 + y 2 + x + y = 0 (d) x 2 + y 2 - x - y = 0
another sector then [2005]
86. If the two circles ( x - 1) 2 + ( y - 3) 2 = r 2 and
2 2
(a) 3a - 10ab + 3b = 0
x 2 + y 2 - 8 x + 2 y + 8 = 0 intersect in two distinct point,
(b) 3a - 2ab + 3b = 0
2 2
then [2003]
(c) 3a 2 + 10ab + 3b 2 = 0 (a) r > 2 (b) 2 < r < 8
(c) r < 2 (d) r = 2.
(d) 3a 2 + 2ab + 3b 2 = 0
87. The lines 2 x - 3 y = 5 and 3x - 4 y = 7 are diameters of a
81. If the circles x 2 + y 2 + 2ax + cy + a = 0 and circle having area as 154 sq.units.Then the equation of
the circle is [2003]
x 2 + y 2 – 3ax + dy – 1 = 0 intersect in two distinct points
P and Q then the line 5x + by – a = 0 passes through P and (a) x 2 + y 2 - 2 x + 2 y = 62
Q for [2005]
(a) exactly one value of a (b) x 2 + y 2 + 2 x - 2 y = 62
(b) no value of a
(c) infinitely many values of a (c) x 2 + y 2 + 2 x - 2 y = 47
(d) exactly two values of a
(d) x 2 + y 2 - 2 x + 2 y = 47 .

Downloaded from @Freebooksforjeeneet


Conic Sections M-153

88. If the chord y = mx + 1 of the circle x2+y2=1 subtends an 1


angle of measure 45 at the maor segment of the circle (a) PN = 4 (b) MQ =
3
then value of m is [2002]
1
(a) 2 ± 2 (b) –2 ± 2 (c) MQ = (d) PN = 3
4
(c) –1 ± 2 (d) none of these 96. Let the latus ractum of the parabola y2 = 4x be the common
chord to the circles C1 and C2 each of them having radius
89. The centres of a set of circles, each of radius 3, lie on the
circle x2+y2=25. The locus of any point in the set is[2002] 2 5. Then, the distance between the centres of the circles
(a) 4 £ x2 + y2 £ 64 (b) x2 + y2 £ 25 C1 and C2 is : [Sep. 03, 2020 (II)]

(c) ³ 25
x2 + y2 (d) 3 £ £9x2 + y2 (a) 8 5 (b) 8 (c) 12 (d) 4 5
90. The centre of the circle passing through (0, 0) and (1, 0) 97. The area (in sq. units) of an equilateral triangle inscribed in
and touching the circle x2 + y2 = 9 is [2002] the parabola y2 = 8x, with one of its vertices on the vertex
of this parabola, is : [Sep. 02, 2020 (II)]
æ1 1ö æ1 ö æ3 1ö æ1 3ö
(a) ç 2 , 2 ÷ (b) ç 2 , - 2 ÷ (c) ç 2 , 2 ÷ (d) ç 2 , 2 ÷ (a) 64 3 (b) 256 3
è ø è ø è ø è ø
91. The equation of a circle with origin as a centre and passing (c) 192 3 (d) 128 3
through equilateral triangle whose median is of length 3a 98. If one end of a focal chord AB of the parabola y2 = 8x is at
is [2002]
(a) x2 + y2 = 9a2 (b) x2 + y2 = 16a2 æ 1 ö
Aç , - 2 ÷ , then the equation of the tangent to it at B
(c) x2 + y2 = 4a2 (d) x2 + y2 = a2 è 2 ø
is: [Jan. 9, 2020 (II)]
TOPIC n Parabola (a) 2x + y – 24 = 0 (b) x – 2y + 8 = 0
(c) x + 2y + 8 = 0 (d) 2x – y – 24 = 0
92. Let L1 be a tangent to the parabola y2 = 4(x + 1) and L2 be 99. The locus of a point which divides the line segment oining
a tangent to the parabola y2 = 8(x + 2) such that L1 and L2 the point (0, –1) and a point on the parabola, x2 = 4y,
intersect at right angles. Then L1 and L2 meet on the internally in the ratio 1 : 2, is: [Jan. 8, 2020 (I)]
straight line : [Sep. 06, 2020 (I)] 2
(a) 9x – 12y = 8 2
(b) 9x – 3y = 2
(a) x + 3 = 0 (b) 2x + 1 = 0 (c) x2 – 3y = 2 (d) 4x2 – 3y = 2
(c) x + 2 = 0 (d) x + 2y = 0 100. Let a line y = mx (m > 0) intersect the parabola, y2 = x at a
93. The centre of the circle passing through the point (0, 1) point P, other than the origin. Let the tangent to it at P
and touching the parabola y = x2 at the point (2,4) is: meet the x-axis at the point Q, If area (DOPQ) = 4 sq. units,
[Sep. 06, 2020 (II)] then m is equal to _______. [NA Jan. 8, 2020 (II)]
101. If y = mx + 4 is a tangent to both the parabolas, y2 =4x and
æ -53 16 ö æ 6 53 ö x2 = 2by, then b is equal to: [Jan. 7, 2020 (I)]
(a) ç , ÷ (b) ç , ÷
è 10 5 ø è 5 10 ø (a) –32 (b) –64 (c) –128 (d) 128
2
102. The tangents to the curve y = (x – 2) –1 at its points of
æ 3 16 ö æ -16 53 ö
(c) ç , ÷ (d) ç , ÷ intersection with the line x – y = 3, intersect at the point :
è 10 5 ø è 5 10 ø [April 12, 2019 (II)]
94. If the common tangent to the parabolas, y2 = 4x and x2 = 4y
æ5 ö æ 5 ö æ5 ö æ 5 ö
also touches the circle, x2 + y2 = c2, then c is equal to: (a) ç ,1÷ (b) ç - , -1÷ (c) ç , -1÷ (d) ç - ,1÷
[Sep. 05, 2020 (I)] è 2 ø è 2 ø è 2 ø è 2 ø
103. If the line ax + y = c, touches both the curves x2 + y2 = 1
1 1 1 1
(a) (b) (c) (d) and y2 = 4 2x , then |c| is equal to [April 10, 2019 (II)]
2 2 2 4 2
95. Let P be a point on the parabola, y2 = 12x and N be the foot 1 1
of the perpendicular drawn from P on the axis of the (a) 2 (b) (c) (d) 2
2 2
parabola. A line is now drawn through the mid-point M of
104. The area (in sq. units) of the smaller of the two circles that
PN, parallel to its axis which meets the parabola at Q. If the
touch the parabola, y2 = 4x at the point (1, 2) and the x-axis
4 is: [April 09, 2019 (II)]
y-intercept of the line NQ is , then : [Sep. 03, 2020 (I)]
3 (a) 8p (2 – 2) (b) 4p (2 – 2)
(c) 4p (3 + 2 ) (d) 8p (3 – 2 2)

Downloaded from @Freebooksforjeeneet


EBD_8344
M-154 Mathematics

105. If one end of a focal chord of the parabola, y2 = 16x is at 114. Axis of a parabola lies along x-axis. If its vertex and focus
(1, 4), then the length of this focal chord is: are at distance 2 and 4 respectively from the origin, on the
[April 09, 2019 (I)] positive x-axis then which of the following points does
(a) 25 (b) 22 (c) 24 (d) 20 not lie on it? [Jan 09, 2019 (I)]
106. The shortest distance between the line y = x and the curve (a) (5, 2 6 ) (b) (8, 6)
y2 = x – 2 is : [April 08, 2019 (I)]
(c) (6, 4 2 ) (d) (4, – 4)
7 7 11
(a) 2 (b) (c) (d) 115. Equation of a common tangent to the circle, x2 + y2 – 6x = 0
8 4 2 4 2
and the parabola, y2 = 4x, is : [Jan 09, 2019 (I)]
107. If the tangents on the ellipse 4x2 + y2 = 8 at the points (1, 2)
(a) 2 3 y = 12x + 1 (b) 3 y=x+3
and (a, b) are perpendicular to each other, then a2 is equal
to : [April 08, 2019 (I)] (c) 2 3 y = – x – 12 (d) 3 y = 3x + 1
128 64 4 2 116. Let A (4, – 4) and B (9, 6) be points on the parabola,
(a) (b) (c) (d) y2 = 4x. Let C be chosen on the arc AOB of the parabola,
17 17 17 17
where O is the origin, such that the area of DACB is
108. The tangent to the parabola y2 = 4x at the point where it
maximum. Then, the area (in sq. units) of DACB, is:
intersects the circle x2 + y2 = 5 in the first quadrant, passes
[Jan. 09, 2019 (II)]
through the point : [April 08, 2019 (II)]
1 1 3
(a) 31 (b) 30 (c) 32 (d) 31
æ 1 4ö æ1 3ö
(a) ç - , ÷ (b) ç , ÷ 4 2 4
è 3 3ø è4 4ø 117. Tangent and normal are drawn at P(16, 16) on the parabola
æ3 7ö æ 1 1ö y2 = 16x , which intersect the axis of the parabola at A and
(c) ç , ÷ (d) ç - , ÷
è4 4ø è 4 2ø B, respectively. If C is the centre of the circle through the
109. The equation of a tangent to the parabola, x2 = 8y, which points P, A and B and ÐCPB = q , then a value of tan q is:
makes an angle q with the positive direction of x-axis, is : [2018]
[Jan. 12, 2019 (II)]
(a) y = x tanq + 2 cotq (b) y = x tanq – 2 cotq 4 1
(a) 2 (b) 3 (c) (d)
(c) x = y cotq + 2 tanq (d) x = y cotq – 2 tanq 3 2
118. Tangents drawn from the point (– 8, 0) to the parabola
110. Equation of a common tangent to the parabola y2 = 4x and y2 = 8x touch the parabola at P and Q. If F is the focus of
the hyperbola xy = 2 is : [Jan. 11, 2019 (I)] the parabola, then the area of the triangle PFQ (in sq. units)
(a) x + y + 1 = 0 (b) x – 2y + 4 = 0 is equal to [Online April 15, 2018]
(c) x + 2y + 4 = 0 (d) 4x + 2y + 1 = 0 (a) 48 (b) 32 (c) 24 (d) 64
111. If the area of the triangle whose one vertex is at the vertex 119. If y = mx + c is the normal at a point on the parabola y2 = 8x
of the parabola, y2 + 4 (x – a2) = 0 and the other two whose focal distance is 8 units, then |c| is equal to :
vertices are the points of intersection of the parabola and [Online April 9, 2017]
y-axis, is 250 sq. units, then a value of ‘a’ is :
(a) 2 3 (b) 8 3 (c) 10 3 (d) 16 3
[Jan. 11, 2019 (II)]
120. If the common tangents to the parabola, x2 = 4y and the
(a) 5 5 (b) 5(21/3) (c) (10)2/3 (d) 5 circle, x2 + y2 = 4 intersect at the point P, then the distance
112. If the parabolas y2 = 4b(x – c) and y2 = 8ax have a common of P from the origin, is : [Online April 8, 2017]
normal, then which one of the following is a valid choice
(a) 2 +1 (b) 2(3 + 2 2)
for the ordered triad (a, b, c)? [Jan 10, 2019 (I)]
æ1 ö (c) 2( 2 + 1) (d) 3 + 2 2
(a) ç , 2, 3 ÷ (b) (1, 1, 3) 121. Let P be the point on the parabola, y2 = 8x which is at a
è2 ø
minimum distance from the centre C of the circle,
æ1 ö x2 + (y + 6)2 = 1. Then the equation of the circle, passing
(c) ç , 2, 0 ÷ (d) (1, 1, 0)
è 2 ø through C and having its centre at P is: [2016]
113. The length of the chord of the parabola x2 = 4y having 2 2 x
(a) x + y - + 2y - 24 = 0
equation x - 2 y + 4 2 = 0 is: [Jan. 10, 2019 (II)] 4
(b) x2 + y2 – 4x + 9y + 18 = 0
(a) 3 2 (b) 2 11 (c) 8 2 (d) 6 3 (c) x2 + y2 – 4x + 8y + 12 = 0
(d) x2 + y2 – x + 4y – 12 = 0

Downloaded from @Freebooksforjeeneet


Conic Sections M-155

122. P and Q are two distinct points on the parabola, y2 = 4x, (b) Statement-1 is true; Statement-2 is true; Statement-2
with parameters t and t1 respectively. If the normal at P is not a correct explanation for Statement-1.
passes through Q, then the minimum value of t12 is : (c) Statement-1 is true; Statement-2 is false.
(d) Statement-1 is false; Statement-2 is true.
[Online April 10, 2016] 130. The point of intersection of the normals to
(a) 8 (b) 4 (c) 6 (d) 2 the parabola y 2 = 4x at the ends of its latus rectum is :
123. Let O be the vertex and Q be any point on the parabola, [Online April 23, 2013]
x2 = 8y. If the point P divides the line segment OQ internally
(a) (0, 2) (b) (3, 0) (c) (0, 3) (d) (2, 0)
in the ratio 1 : 3, then locus of P is : [2015]
131. Statement-1: The line x – 2y = 2 meets the parabola,
(a) y2 = 2x (b) x2 = 2y y2 + 2x = 0 only at the point (– 2, – 2).
(c) x2 = y (d) y2 = x
124. Let PQ be a double ordinate of the parabola, y2 = – 4x, Statement-2: The line y = mx -
1
( m ¹ 0) is tangent to
where P lies in the second quadrant. If R divides PQ in 2m
the ratio 2 : 1 then the locus of R is :
æ 1 1ö
[Online April 11, 2015] the parabola, y2 = – 2x at the point ç - ,- ÷ .
(a) 3y2 = – 2x (b) 3y2 = 2x è 2m 2 m ø
(c) 9y2 = 4x (d) 9y2 = – 4x [Online April 22, 2013]
125. The slope of the line touching both the parabolas y 2 = 4 x (a) Statement-1 is true; Statement-2 is false.
(b) Statement-1 is true; Statement-2 is true; Statement-2
and x 2 = -32 y is [2014] is a correct explanation for statement-1.
1 2 1 3 (c) Statement-1 is false; Statement-2 is true.
(a) (b) (c) (d) (d) Statement-1 a true; Statement-2 is true; Statement-2 is
8 3 2 2
not a correct explanation for statement-1.
126. A chord is drawn through the focus of the parabola y2 = 6x
such that its distance from the vertex of this parabola is æ 3ö x2 y 2
132. The normal at ç 2, ÷ to the ellipse, + = 1 touches
5 è 2ø 16 3
, then its slope can be: [Online April 19, 2014]
2 a parabola, whose equation is [Online May 26, 2012]
(a) y2 = – 104 x (b) y2 = 14 x
5 3 2 2 2
(c) y = 26x (d) y2 = – 14x
(a) (b) (c) (d)
2 2 5 3 133. The chord PQ of the parabola y2 = x, where one end P of
127. Two tangents are drawn from a point (– 2, – 1) to the curve, the chord is at point (4, – 2), is perpendicular to the axis of
y2 = 4x. If a is the angle between them, then |tan a| is equal the parabola. Then the slope of the normal at Q is
to: [Online April 12, 2014] [Online May 26, 2012]
1 1
1 1 (a) – 4 (b) - (c) 4 (d)
(a) (b) (c) 3 (d) 3 4 4
3 3
128. Let L1 be the length of the common chord of the curves 1
134. Statement 1: y = mx - is always a tangent to the
x2 + y2 = 9 and y2 = 8x, and L2 be the length of the latus m
rectum of y2 = 8x, then: [Online April 11, 2014] parabola, y2 = – 4x for all non- ero values of m.
(a) L1 > L2 (b) L1 = L2 Statement 2: Every tangent to the parabola, y2 = – 4x will
meet its axis at a point whose abscissa is non-negative.
L1
(c) L1 < L2 (d) L = 2 [Online May 7, 2012]
2 (a) Statement 1 is true, Statement 2 is true; Statement 2 is
a correct explanation of Statement 1.
129. Given : A circle, 2x2 + 2y2 = 5 and a parabola, y2 = 4 5 x.
(b) Statement 1 is false, Statement 2 is true.
Statement-1 : An equation of a common tangent to these (c) Statement 1 is true, Statement 2 is false.
curves is y = x + 5 . (d) Statement 1 is true, Statement 2 is true, Statement 2 is
not a correct explanation of Statement 1.
5 135. The shortest distance between line y – x =1 and curve
Statement-2 : If the line, y = mx + (m ¹ 0) is their
m x = y2 is [2011]
common tangent, then m satisfies m4 – 3m2 + 2 = 0.[2013] 8 4
3 2 3
(a) Statement-1 is true; Statement-2 is true; Statement-2 (a) (b) (c) (d)
8 3 2 3 4
is a correct explanation for Statement-1.

Downloaded from @Freebooksforjeeneet


EBD_8344
M-156 Mathematics

136. If two tangents drawn from a point P to the parabola


y2 = 4x are at right angles, then the locus of P is [2010] TOPIC Đ Ellipse
(a) 2x + 1 = 0 (b) x = – 1
(c) 2x – 1 = 0 (d) x = 1 145. Which of the following points lies on the locus of the foot
137. A parabola has the origin as its focus and the line x = 2 as of perpendicular drawn upon any tangent to the ellipse,
the directrix. Then the vertex of the parabola is at [2008] x2 y 2
(a) (0, 2) (b) (1, 0) + = 1 from any of its foci? [Sep. 06, 2020 (I)]
4 2
(c) (0, 1) (d) (2, 0)
138. The equation of a tangent to the parabola (a) (-2, 3) (b) (-1, 2)
y2 = 8x is y = x + 2. The point on this line from which the
(c) (-1, 3) (d) (1, 2)
other tangent to the parabola is perpendicular to the given
tangent is [2007] 146. If the normal at an end of a latus rectum of an ellipse
(a) (2, 4) (b) (–2, 0) (c) (–1, 1) (d) (0, 2) passes through an extermity of the minor axis, then the
139. The locus of the vertices of the family of parabolas eccentricity e of the ellipse satisfies: [Sep. 06, 2020 (II)]
(a) e4 + 2e2 – 1 = 0 (b) e2 + e – 1 = 0
a3 x2 a2 x
y= + - 2a is [2006] (c) e4 + e2 – 1 = 0 (d) e2 + 2e – 1 = 0
3 2
105 3 147. If the co-ordinates of two points A and B are ( 7, 0) and
(a) xy = (b) xy =
64 4
(- 7, 0) respectively and P is any point on the conic,
35
(c) xy = (d) xy = 64 9x2 + 16y2 = 144, then PA + PB is equal to :
16 105 [Sep. 05, 2020 (I)]
140. Let P be the point ( 1, 0 ) and Q a point on the locus (a) 16 (b) 8 (c) 6 (d) 9
y 2 = 8 x . The locus of mid point of PQ is [2005] 148. If the point P on the curve, 4x2 + 5y2 = 20 is farthest from
the point Q(0, – 4), then PQ2 is equals to :
(a) y 2 – 4x + 2 = 0 (b) y 2 + 4x + 2 = 0 [Sep. 05, 2020 (I)]
2
(a) 36 (b) 48 (c) 21 (d) 29
(c) x + 4y + 2 = 0 (d) x 2 – 4y + 2 = 0
x2 y2
141. A circle touches the x- axis and also touches the circle with 149. Let + = 1 (a > b) be a given ellipse, length of whose
centre at (0,3 ) and radius 2. The locus of the centre of the a 2 b2
circle is [2005] latus rectum is 10. If its eccentricity is the maximum value
(a) an ellipse (b) a circle 5
(c) a hyperbola (d) a parabola of the function, f(t ) = + t - t 2 , then a2 + b2 is equal to:
12
142. If a ¹ 0 and the line 2bx + 3cy + 4d = 0 passes through [Sep. 04, 2020 (I)]
the points of intersection of the parabolas (a) 145 (b) 116 (c) 126 (d) 135
150. Let x = 4 be a directrix to an ellipse whose centre is at the
y 2 = 4ax and x 2 = 4ay, then [2004]
1
origin and its eccentricity is . If P(1, b), b > 0 is a point on
(a) d 2 + (3b - 2c ) 2 = 0 (b) d 2 + (3b + 2c)2 = 0 2
(c) d 2 + (2b - 3c ) 2 = 0 (d) d 2 + (2b + 3c ) 2 = 0 this ellipse, then the equation of the normal to it at P is :
[Sep. 04, 2020 (II)]
143. The normal at the point (bt12 , 2bt1 ) on a parabola meets (a) 4x – 3y = 2 (b) 8x – 2y = 5
(c) 7x – 4y = 1 (d) 4x – 2y = 1
the parabola again in the point (bt 2 2 , 2bt 2 ) , then [2003]
151. A hyperbola having the transverse axis of length 2 has
2 2
(a) t 2 = t1 + (b) t 2 = -t1 - the same foci as that of the ellipse 3x2 + 4y2 = 12, then this
t1 t1
hyperbola does not pass through which of the following
2 2 points? [Sep. 03, 2020 (I)]
(c) t2 = -t1 + (d) t 2 = t1 -
t1 t1
æ 1 ö æ 3 ö
144. Two common tangents to the circle x2 + y2 = 2a2 and (a) ç , 0÷ (b) çç - 2 , 1÷÷
parabola y2 = 8ax are [2002] è 2 ø è ø
(a) x = ± ( y + 2 a ) (b) y = ± ( x + 2 a ) æ 3 1 ö
æ 1 ö
(c) x = ± ( y + a ) (d) y = ± ( x + a ) (c) ç 1, - ÷ (d) çç 2 , ÷
è 2ø è 2 ÷ø

Downloaded from @Freebooksforjeeneet


Conic Sections M-157

|x| | y| 159. An ellipse, with foci at (0, 2) and (0, –2) and minor axis of
152. Area (in sq. units) of the region outside + = 1 and length 4, passes through which of the following points ?
2 3
[April 12, 2019 (II)]
x2 y 2
inside the ellipse + = 1 is : [Sep. 02, 2020 (I)] (a) ( 2, 2) (b) (2, 2)
4 9
(a) 6(p - 2) (b) 3(p - 2) (c) (2, 2 2) (d) (1, 2 2)

(c) 3(4 - p) (d) 6(4 - p) x2 y 2


160. If the line x – 2y = 12 is tangent to the ellipse + =1
a 2 b2
x2 y 2
153. If e1 and e2 are the eccentricities of the ellipse, + =1
18 4 æ -9 ö
at the point ç 3, ÷ , then the length of the latus rectum
2 2 è 2 ø
and the hyperbola, x - y = 1 respectively and (e1, e2) is
9 4 of the ellipse is : [April 10, 2019 (I)]
a point on the ellipse, 15x2 + 3y2 = k, then k is equal to (a) 9 (b) 12 2 (c) 5 (d) 8 3
[Jan. 9, 2020 (I)] 161. The tangent and normal to the ellipse 3x2 + 5y2 = 32 at the
(a) 16 (b) 17 (c) 15 (d) 14
point P(2, 2) meet the x-axis at Q and R, respectively. Then
154. The length of the minor axis (along y-axis) of an ellipse in the area (in sq. units) of the triangle PQR is :
4 [April 10, 2019 (II)]
the standard form is . If this ellipse touches the line, x
3 34 14 16 68
+ 6y = 8; then its eccentricity is: [Jan. 9, 2020 (II)] (a) (b) (c) (d)
15 3 3 15
1 11 5 1 5 1 11 162. If the tangent to the parabola y2 = x at a point (a, b), (b Ō
(a) (b) (c) (d)
2 3 6 2 3 3 3 Ā) is also a tangent to the ellipse, x2 + 2y2 = 1, then a is
155. Let the line y = mx and the ellipse 2x2 + y2 = 1 intersect at a equal to: [April 09, 2019 (II)]
point P in the first quadrant. If the normal to this ellipse at (a) 2 -1 (b) 2 2 - 1
æ 1 ö (c) 2 2 + 1
P meets the co-ordinate axes at ç - ,0 and (0, b), (d) 2 +1
è 3 2 ÷ø
163. In an ellipse, with centre at the origin, if the difference of
then b is equal to: [Jan. 8, 2020 (I)]
the lengths of maor axis and minor axis is 10 and one of
2 2 2 2 2
(a) (b) (c) (d) the foci is at (0, 5 3 ), then the length of its latus rectum
3 3 3 3 is: [April 08, 2019 (II)]
156. If the distance between the foci of an ellipse is 6 and the (a) 10 (b) 5 (c) 8 (d) 6
distance between its directrices is 12, then the length of
164. Let S and S¢ be the foci of an ellipse and B be any one of
its latus rectum is: [Jan. 7, 2020 (I)]
the extremities of its minor axis. If DS¢BS is a right angled
3 triangle with right angle at B and area (DS¢BS) = 8 sq. units,
(a) 3 (b) 3 2 (c) (d) 2 3
2 hen the length of a latus rectum of the ellipse is :
[Jan. 12, 2019 (II)]
x2 y 2
157. If 3x + 4y = 12 2 is a tangent to the ellipse + =1 (a) 4 (b) 2 2 (c) 4 2 (d) 2
a2 9
for some a Î R, then the distance between the foci of the 165. If tangents are drawn to the ellipse x 2 + 2 y 2 = 2 at all
ellipse is: [Jan. 7, 2020 (II)] points on the ellipse other than its four vertices then the
(a) 2 7 (b) 4 (c) 2 5 (d) 2 2 mid points of the tangents intercepted between the
158. If the normal to the ellipse 3x2 + 4y2=12 at a point P on it is coordinate axes lie on the curve : [Jan. 11, 2019 (I)]
parallel to the line, 2x + y = 4 and the tangent to the ellipse
at P passes through Q (4,4) then PQ is equal to :
1
+
1
=1 x2 y 2
(a) (b) + =1
[April 12, 2019 (I)] 4x2 2 y2 4 2

5 5 61 221 157 1
+
1
=1 x2 y 2
(a) (b) (c) (d) (c) (d) + =1
2 2 2 2 2x2 4 y2 2 4

Downloaded from @Freebooksforjeeneet


EBD_8344
M-158 Mathematics

166. Two sets A and B are as under : 172. If the distance between the foci of an ellipse is half the
length of its latus rectum, then the eccentricity of the
A = {(a, b) Î R ´ R :| a - 5 | < 1 and | b - 5 | < 1};
ellipse is: [Online April 11, 2015]
2 2
B = {(a,b) Î R ´ R : 4(a - 6) + 9(b - 5) £ 36}. Then : 2 2 –1
(a) (b) 2 –1
[2018] 2
(a) A Ì B
1 2 –1
(b) A Ç B = f (an empty set) (c)
2
(d)
2
(c) neither A Ì B nor B Ì A 173. The locus of the foot of perpendicular drawn from the
centre of the ellipse x2 + 3y2 = 6 on any tangent to it is
(d) B Ì A
[2014]
167. If the length of the latus rectum of an ellipse is 4 units and
( x2 + y2 )
2
the distance between a focus and its nearest vertex on the (a) = 6 x2 + 2 y 2
3
( x2 + y2 )
maor axis is units, then its eccentricity is? 2
2 (b) = 6 x2 - 2 y 2
[Online April 16, 2018]
( x2 - y2 )
2
(c) = 6x2 + 2 y2
1 2 1 1
(a) (b) (c) (d)
( x2 - y 2 )
2 3 9 3 2
(d) = 6 x2 - 2 y 2
168. The eccentricity of an ellipse having centre at the origin,
axes along the co-ordinate axes and passing through the 174. A stair-case of length l rests against a vertical wall and a
points (4, –1) and (–2, 2) is : [Online April 9, 2017] floor of a room. Let P be a point on the stair-case, nearer to
its end on the wall, that divides its length in the ratio 1 : 2.
1 2 3 3 If the stair-case begins to slide on the floor, then the locus
(a) (b) (c) (d)
2 5 2 4 of P is: [Online April 11, 2014]
169. Consider an ellipse, whose centre is at the origin and its 1
(a) an ellipse of eccentricity
3 2
maor axis is along the x–axis. If its eccentricity is and
5 3
(b) an ellipse of eccentricity
the distance between its foci is 6, then the area (in sq. 2
units) of the quadrilateral inscribed in the ellipse, with the
l
vertices as the vertices of the ellipse, is : (c) a circle of radius
2
[Online April 8, 2017]
(a) 8 (b) 32 (c) 80 (d) 40 3
(d) a circle of radius l
2
x 2 y2
170. If the tangent at a point on the ellipse + = 1 meets 175. If OB is the semi-minor axis of an ellipse, F1 and F2 are its
27 3 foci and the angle between F1B and F2B is a right angle,
the coordinate axes at A and B, and O is the origin, then the then the square of the eccentricity of the ellipse is:
minimum area (in sq. units) of the triangle OAB is : [Online April 9, 2014]
[Online April 9, 2016] 1 1 1 1
(a) (b) (c) (d)
9 9 2 2 2 2 4
(a) 3 3 (b) (c) 9 (d) 176. The equation of the circle passing through the foci of the
2 3
171. The area (in sq. units) of the quadrilateral formed by the x2 y 2
ellipse + = 1, and having centre at (0, 3) is [2013]
tangents at the end points of the latera recta to the ellipse 16 9
(a) x2 + y2 – 6y – 7 = 0
x 2 y2
+ = 1, is : [2015] (b) x2 + y2 – 6y + 7 = 0
9 5
(c) x2 + y2 – 6y – 5 = 0
27 27 (d) x2 + y2 – 6y + 5 = 0
(a) (b) 27 (c) (d) 18
2 4

Downloaded from @Freebooksforjeeneet


Conic Sections M-159

177. A point on the ellipse, 4x2 + 9y2 = 36, where the normal is
8 2 4 5
parallel to the line, 4x –2y – 5 = 0, is : (a) (b) (c) (d)
3 3 3 3
[Online April 25, 2013]
184. In an ellipse, the distance between its foci is 6 and minor
æ 9 8ö æ8 9ö axis is 8. Then its eccentricity is [2006]
(a) ç , ÷ (b) ç , - ÷
è 5 5ø è5 5ø (a) 3 (b) 1 (c) 4 (d) 1
5 2 5 5
æ 9 8ö æ8 9ö
(c) ç - , ÷ (d) ç , ÷ 185. An ellipse has OB as semi minor axis, F and F ' its focii
è 5 5ø è 5 5ø
and the angle FBF ' is a right angle. Then the eccentricity
178. Let the equations of two ellipses be
of the ellipse is [2005]
x2 y 2 x2 y 2 1 1 1
E1 : + = 1 and E2 : + =1, (a) 1 (b) (c) (d)
3 2 16 b 2 2 2 4 3
186. The eccentricity of an ellipse, with its centre at the origin,
1
If the product of their eccentricities is , then the length 1
2 is . If one of the directrices is x = 4, then the equation of
2
of the minor axis of ellipse E2 is :[Online April 22, 2013] the ellipse is: [2004]
(a) 8 (b) 9 (c) 4 (d) 2
(a) 4 x 2 + 3 y 2 = 1 (b) 3 x 2 + 4 y 2 = 12
179. Equation of the line passing through the points of
intersection of the parabola x 2 = 8y and the ellipse (c) 4 x 2 + 3 y 2 = 12 (d) 3 x 2 + 4 y 2 = 1
x2
+ y 2 = 1 is : [Online April 9, 2013]
3 TOPIC Ė Hyperbola
(a) y – 3 = 0 (b) y + 3 = 0
187. If the line y = mx + c is a common tangent to the hyperbola
(c) 3y + 1 = 0 (d) 3y – 1 = 0
x2 y 2
x 2 - = 1 and the circle x2 + y2 = 36, then which one of
180. If P1 and P2 are two points on the ellipse + y 2 = 1 at 100 64
4 the following is true? [Sep. 05, 2020 (II)]
which the tangents are parallel to the chord oining the 2
(a) c = 369 (b) 5m = 4
points (0, 1) and (2, 0), then the distance between P1 and (c) 4c2 = 369 (d) 8m + 5 = 0
P2 is [Online May 12, 2012]
(a) 2 2 (b) 5 (c) 2 3 (d) 10 x2 y2
188. Let P(3, 3) be a point on the hyperbola, - = 1. If the
181. Equation of the ellipse whose axes are the axes of a2 b2
coordinates and which passes through the point (–3, 1) normal to it at P intersects the x-axis at (9, 0) and e is its
eccentricity, then the ordered pair (a2, e2) is equal to :
2
and has eccentricity is [2011] [Sep. 04, 2020 (I)]
5
(a) 5x2 + 3y2 – 48 = 0 (b) 3x2 + 5y2 – 15 = 0 æ9 ö æ3 ö æ9 ö
(c) 5x2 + 3y2 – 32 = 0 (d) 3x2 + 5y2 – 32 = 0 (a) ç , 3 ÷ (b) ç , 2 ÷ (c) ç , 2 ÷ (d) (9, 3)
è2 ø è2 ø è2 ø
182. The ellipse x 2 + 4 y 2 = 4 is inscribed in a rectangle 189. Let e 1 and e 2 be the eccentricities of the ellipse,
aligned with the coordinate axes, which in turn is inscribed x2 y 2 x2 y 2
in another ellipse that passes through the point (4, 0). Then + 2 = 1 (b < 5) and the hyperbola, - =1
25 b 16 b2
the equation of the ellipse is : [2009]
respectively satisfying e1e2 = 1. If a and b are the distances
(a) x 2 + 12 y 2 = 16 (b) 4 x 2 + 48 y 2 = 48 between the foci of the ellipse and the foci of the hyperbola
respectively, then the ordered pair (a, b) is equal to :
(c) 4 x 2 + 64 y 2 = 48 (d) x 2 + 16 y 2 = 16 [Sep. 03, 2020 (II)]
183. A focus of an ellipse is at the origin. The directrix is the line
æ 20 ö
1 (a) (8, 12) (b) ç , 12 ÷
x = 4 and the eccentricity is . Then the length of the è 3 ø
2
semi-maor axis is [2008] æ 24 ö
(c) ç , 10 ÷ (d) (8, 10)
è 5 ø

Downloaded from @Freebooksforjeeneet


EBD_8344
M-160 Mathematics

190. A line parallel to the straight line 2x – y = 0 is tangent to the 198. If the eccentricity of the standard hyperbola passing
through the point (4, 6) is 2, then the equation of the
x2 y 2
hyperbola - = 1 at the point (x1, y1). Then x12 + 5 y12 tangent to the hyperbola at (4, 6) is : [April. 08, 2019 (II)]
4 2 (a) x – 2y + 8 = 0 (b) 2x – 3y + 10 = 0
is equal to : [Sep. 02, 2020 (I)] (c) 2x – y – 2 = 0 (d) 3x – 2y = 0
(a) 6 (b) 8 (c) 10 (d) 5 199. If the vertices of a hyperbola be at (–2, 0) and (2, 0) and one
of its foci be at (–3, 0), then which one of the following
p
191. For some q Î æç 0, ö÷ , if the eccentricity of the hyperbola, points does not lie on this hyperbola?[Jan. 12, 2019 (I)]
è 2ø
x2 - y 2 sec2 q = 10 is 5 times the eccentricity of the
(a) ( -6, 2 10 ) (b) ( 2 6, 5)
ellipse, x2 sec 2 q + y 2 = 5, then the length of the latus (c) ( 4, 15 ) (d) ( 6,5 2 )
rectum of the ellipse, is : [Sep. 02, 2020 (II)] 200. If a hyperbola has length of its conugate axis equal to 5
and the distance between its foci is 13, then the eccentricity
(a) 2 6 (b) 30 of the hyperbola is : [Jan. 11, 2019 (II)]

2 5 4 5 13 13 13
(c) (d) (a) (b) 2 (c) (d)
3 3 12 6 8
201. Let the length of the latus rectum of an ellipse with its
192. If a hyperbola passes through the point P(10,16) and it
maor axis along x-axis and centre at the origin, be 8. If the
has vertices at (± 6,0), then the equation of the normal to
distance between the foci of this ellipse is equal to the
it at P is: [Jan. 8, 2020 (II)]
length of its minor axis, then which one of the following
(a) 3x + 4y = 94 (b) 2x + 5y = 100 points lies on it? [Jan. 11, 2019 (II)]
(c) x + 2y = 42 (d) x + 3y = 58
193. Let P be the point of intersection of the common tangents (a) (4 2, 2 2 ) (b) (4 3, 2 2 )
to the parabola y2 = 12x and hyperbola 8x2 – y2 = 8. If S and
S¢ denote the foci of the hyperbola where S lies on the (c) (4 3, 2 3) (d) (4 2, 2 3)
positive x-axis then P divides SS¢ in a ratio : 202. The equation of a tangent to the hyperbola 4x2 – 5y2 = 20
[April 12, 2019 (I)] parallel to the line x – y = 2 is: [Jan 10, 2019 (I)]
(a) 13 : 11 (b) 14 : 13 (c) 5 : 4 (d) 2 : 1 (a) x – y + 1 = 0 (b) x – y + 7 = 0
194. The equation of a common tangent to the curves, y2 = 16x (c) x – y + 9 = 0 (d) x – y – 3 = 0
and xy = – 4, is : [April 12, 2019 (II)] 203. Let
(a) x – y + 4 = 0 (b) x + y + 4 = 0
ìï y2 x2 üï
(c) x – 2y + 16 = 0 (d) 2x – y + 2 = 0 S = í( x, y ) Î R 2 : - = 1ý ,
195. If a directrix of a hyperbola centred at the origin and passing ïî 1 + r 1 - r ïþ

through the point (4, -2 3) is 5x = 4 5 and its where r ¹ ± 1 Then S represents: [Jan. 10, 2019 (II)]
(a) a hyperbola whose eccentricity is
eccentricity is e, then : [April 10, 2019 (I)]
(a) 4e4 – 24e2 + 27 = 0 (b) 4e4 – 12e2 – 27 = 0 2
, when 0 < r < 1
(c) 4e4 – 24e2 + 35 = 0 (d) 4e4 + 8e2 – 35 = 0 1- r
196. If 5x + 9 = 0 is the directrix of the hyperbola (b) an ellipse whose eccentricity is
16x2 – 9y2 = 144, then its corresponding focus is :
2
[April 10, 2019 (II)] , when r > 1
r +1
æ 5 ö æ5 ö
(a) (5, 0) (b) ç - ,0 ÷ (c) ç , 0 ÷ (d) (–5, 0) (c) a hyperbola whose eccentricity is
è 3 ø è3 ø
2
197. If the line y = mx + 7 3 is normal to the hyperbola , when 0 < r < 1
r +1
x 2 y2 (d) an ellipse whose eccentricity is
- = 1, then a value of m is : [April 09, 2019 (I)]
24 18 1
, when r > 1
r +1
5 15 2 3
(a) (b) (c) (d)
2 2 5 5

Downloaded from @Freebooksforjeeneet


Conic Sections M-161

210. The eccentricity of the hyperbola whose length of the latus


p
204. Let 0 < q < . If the eccentricity of the hyperbola rectum is equal to 8 and the length of its conugate axis is
2 equal to half of the distance between its foci, is : [2016]
x2 y2 2 4 4
- = 1 is greater than 2, then the length of (a) (b) 3 (c) (d)
cos 2 q sin 2 q 3 3 3
211. A hyperbola whose transverse axis is along the maor axis
its latus rectum lies in the interval: [Jan 09, 2019 (I)] x 2 y2
(a) (3, ¥) (b) (3/2, 2] (c) (2, 3] (d) (1, 3/2] of the conic, + = 4 and has vertices at the foci of
3 4
205. A hyperbola has its centre at the origin, passes through
the point (4, 2) and has transverse axis of length 4 along 3
this conic. If the eccentricity of the hyperbola is , then
the x-axis. Then the eccentricity of the hyperbola is : 2
[Jan. 09, 2019 (II)] which of the following points does NOT lie on it ?
[Online April 10, 2016]
3 2
(a) (b) 3 (c) 2 (d) (a) ( 5, 2 2) (b) (0, 2)
2 3
(c) (5, 2 3) (d) ( 10, 2 3)
206. Tangents are drawn to the hyperbola 4x 2 - y2 = 36 at
212. Let a aand b respectively be the semitransverse and semi-
the points P and Q. If these tangents intersect at the point conugate axes of a hyperbola whose eccentricity satisfies
T(0, 3) then the area (in sq. units) of DPTQ is : [2018] the equation 9e2 – 18e + 5 = 0. If S(5, 0) is a focus and
(a) 54 3 (b) 60 3 (c) 36 5 (d) 45 5 5x = 9 is the corresponding directrix of this hyperbola, then
a2 – b2 is equal to : [Online April 9, 2016]
207. The locus of the point of intersection of the lines,
(a) –7 (b) –5 (c) 5 (d) 7
2 x – y + 4 2k = 0 and 2kx + ky – 4 2 = 0 (k is any 113. An ellipse passes through the foci of the hyperbola,
non- ero real parameter) is. [Online April 16, 2018] 9x2 – 4y2 = 36 and its maor and minor axes lie along the
tr ansverse and conugate axes of th e hyperbola
(a) A hyperbola with length of its transverse axis 8 2 respectively. If the product of eccentricities of the two
(b) An ellipse with length of its maor axis 8 2 1
conics is , then which of the following points does not
2
1 lie on the ellipse? [Online April 10, 2015]
(c) An ellipse whose eccentricity is
3
æ 13 ö æ 39 ö
(a) ç , 6÷ (b) ç , 3÷
(d) A hyperbola whose eccentricity is 3 è 2 ø è 2 ø

208. A hyperbola passes through the point P ( 2, 3 ) æ1


(c) çè 13,
2

÷
2 ø
(d) ( 13, 0 )
and has foci at (± 2, 0). Then the tangent to this hyperbola 114. The tangent at an extremity (in the first quadrant) of latus
at P also passes through the point : [2017]
x 2 y2
(a) (- 2, - 3 ) (b) (3 2, 2 3 ) rectum of the hyperbola
4
-
5
= 1 , meet x-axis and
y-axis at A and B respectively. Then (OA)2 – (OB)2, where
(c) (2 2,3 3) (d) ( 3, 2 ) O is the origin, equals:
20 16
[Online April 19, 2014]
4
209. The locus of the point of intersection of the straight lines, (a) - (b) (c) 4 (d) -
9 9 3
tx – 2y – 3t = 0
115. Let P (3 sec q, 2 tan q) and Q (3 sec f, 2 tan f) where
x – 2ty + 3 = 0 (t ÎR), is : [Online April 8, 2017]
p
2 q+f = , be two distinct points on the hyperbola
(a) an ellipse with eccentricity 2
5
x 2 y2
(b) an ellipse with the length of maor axis 6 - = 1 . Then the ordinate of the point of intersection
9 4
(c) a hyperbola with eccentricity 5 of the normals at P and Q is: [Online April 11, 2014]
(d) a hyperbola with the length of conugate axis 3 11 11 13 13
(a) (b) - (c) (d) -
3 3 2 2

Downloaded from @Freebooksforjeeneet


EBD_8344
M-162 Mathematics

216. A common tangent to the conics x 2 = 6y and 220. The equation of the hyperbola whose foci are
2x2 – 4y2 = 9 is: [Online April 25, 2013] (– 2, 0) and (2, 0) and eccentricity is 2 is given by :
[2011RS]
3
(a) x - y = (b) x + y = 1 (a) x2 – 3y2 = 3 (b) 3x2 – y2 = 3
2 (c) – x2 + 3y2 = 3 (d) – 3x2 + y2 = 3
9 x2 y2
(c) x + y = (d) x – y = 1 221. For the Hyperbola - = 1 , which of the
2 cos 2 a sin 2 a
following remains constant when a varies = ? [2007]
x2 y 2
217. A tangent to the hyperbola - = 1 meets x-axis at P (a) abscissae of vertices (b) abscissae of foci
4 2 (c) eccentricity (d) directrix.
and y-axis at Q. Lines PR and QR are drawn such that
222. The locus of a point P (a, b) moving under the condition
OPRQ is a rectangle (where O is the origin). Then R lies on:
[Online April 23, 2013] that the line y = ax + b is a tangent to the hyperbola
4 2 2 4 x2 y2
(a) + =1 (b) - =1 - = 1 is [2005]
x2 y2 x2 y2 a 2 b2
(a) an ellipse (b) a circle
2 4 4 2 (c) a parabola (d) a hyperbola
(c) + =1 (d) - =1
x2 y2 x2 y2 x2 y 2
223. The foci of the ellipse + = 1 and the hyperbola
16 b2
x2 y 2
218. If the foci of the ellipse + = 1 coincide with the foci
16 b 2 x2 y 2 1
- = coincide. Then the value of b 2 is[2003]
144 81 25
2 2 (a) 9 (b) 1 (c) 5 (d) 7
of the hyperbola x - y = 1 , then b2 is equal to
144 81 25
[Online May 19, 2012]
(a) 8 (b) 10 (c) 7 (d) 9

x2 y 2
219. If the eccentricity of a hyperbola - = 1 , which
9 b2

13
passes through (k, 2), is , then the value of k2 is
3
[Online May 7, 2012]
(a) 18 (b) 8 (c) 1 (d) 2

Downloaded from @Freebooksforjeeneet


12
Limits and Derivatives
Limit of a Function, Left Hand & 6. Let f(x) = 5 - x - 2 and g ( x) = x + 1 , xÎ R. If f(x) attains
Right Hand limits, Existance of maximum value at a and g(x) attains minimum value at b,
Limits, Sandwitch Theorem,
TOPIC Ć Evaluation of Limits when X® ¥, ( x - 1)( x 2 - 5 x + 6)
Limits by Factorisation, then lim is equal to :
x ®-ab x2 - 6 x + 8
Substitution & Rationalisation
[April 12, 2019 (II)]
1. If a is the positive root of the equation, p(x) = x2 – x – 2 = 0, (a) 1/2 (b) –3/2 (c) –1/2 (d) 3/2
1 - cos(p( x )) x + 2 sin x
then lim is equal to: [Sep. 05, 2020 (I)] lim
x ®a + x+a-4 7. x ®0 is
x 2 + 2sin x + 1 - sin 2 x - x + 1
3 3 1 1 [April 12, 2019 (II)]
(a) (b) (c) (d)
2 2 2 2 (a) 6 (b) 2 (c) 3 (d) 1

x (e ( 1+ x 2 + x 4 -1) / x
- 1) x4 - 1 x3 - k 3
2. lim [Sep. 05, 2020 (II)] 8. If lim = lim 2 , then k is:[April 10, 2019 (I)]
x ®1 x - 1 x ®k x - k 2
x®0
1 + x2 + x4 - 1

(a) is equal to e (b) is equal to 1 8 3 3 4


(a) (b) (c) (d)
3 8 2 3
(c) is equal to 0 (d) does not exist
3. Let [t] denote the greatest integer £ t. If for some
x 2 - ax + b
9. If lim = 5 , then a + b is equal to :
1 - x+ | x | x ®1 x -1
l Î R - {0, 1}, lim = L, then L is equal to :
x ®0 l - x + [ x] [April 10, 2019 (II)]
[Sep. 03, 2020 (I)] (a) –4 (b) 5 (c) –7 (d) 1
1 sin 2 x
(a) 1 (b) 2 (c) (d) 0 10. lim equals : [April 8, 2019 (I)]
2 x ®0 2 - 1 + cos x

ïì 1 æ x2 x2 x2 x 2 ö üï
4. If lim í 8 çç1 - cos - cos + cos cos ÷÷ ý = 2 - k , (a) 4 2 (b) 2 (c) 2 2 (d) 4
x ®0 ï x 2 4 2 4 ø þï
î è
then the value of k is __________. [NA Sep. 03, 2020 (I)] cot3 x - tanx
11. lim is: [Jan. 12, 2019 (I)]
p æ pö
3x + 33- x - 12 x ® cos x +
ç
5. lim - x /2 1- x is equal to ¾¾¾. 4
è 4 ÷ø
x ®2 3 -3
[NA Jan. 7, 2020 (I)] (a) 4 (b) 4 2 (c) 8 2 (d) 8

Downloaded from @Freebooksforjeeneet


EBD_8344
M-208 Mathematics

p - 2 sin -1 x cot x - cos x


12. lim is equal to: [Jan. 12, 2019 (II)] 19. lim equals : [2017]
x ®1- 1- x x®
p
( p - 2x )3
2
1 2 p
(a) (b) (c) (d) p 1 1 1 1
2p p 2 (a) (b) (c) (d)
4 24 16 8
13. Let [x] denote the greatest integer less than or equal to x.
Then :
3x - 3
lim
( ) (
20. is equal to : [Online April 8, 2017]
tan p sin 2 x + | x | - sin ( x [ x ]) ) 2x - 4 - 2
2 x ®3
lim :
x ®0 x2 1 3 1
(a) 3 (b) (c) (d)
[Jan. 11, 2019 (I)] 2 2 2 2
(a) does not exist (b) equals p
(c) equals p + 1 (d) equals 0 (1 - cos 2x)(3 + cosx)
21. lim is equal to : [2015]
x cot ( 4 x )
x ®0 x tan 4x
14. lim is equal to : [Jan. 11, 2019 (II)]
x ®0 sin 2 x cot 2 ( 2 x ) (a) 2 (b)
1
2 (c) 4 (d) 3
(a) 0 (b) 2 (c) 4 (d) 1
15. For each t Î R, let [t] be the greatest integer less than or 2
equal to t. Then, [Jan. 10, 2019 (I)] e x - cos x
22. lim is equal to : [Online April 10, 2015]
æp ö
x® 0 sin 2 x
(1 - | x | + sin |1 - x |) sin ç [1 - x] ÷
lim è2 ø 3 5
x ®1+ |1 - x | [1 - x] (a) 2 (b) 3 (c) (d)
2 4

(a) equals 1
(c) equals – 1
(b) equals 0
(d) does not exist 23. lim
(
sin p cos 2 x ) is equal to: [2014]
x®0 2
x
1 + 1 + y4 - 2 p
16. lim [Jan. 9, 2019 (I)] (a) -p (b) p (c) (d) 1
y®0 y4 2

(a) exists and equals


4 2
1
24. If lim
{
tan ( x - 2 ) x 2 + ( k - 2 ) x - 2k } = 5,
2
x ®2 x - 4x + 4
1 then k is equal to: [Online April 11, 2014]
(b) exists and equals 2 2
( 2 +1) (a) 0 (b) 1 (c) 2 (d) 3
(1 - cos 2 x)(3 + cos x)
1 25. lim is equal to [2013]
(c) exists and equals x®0 x tan 4 x
2 2
(d) does not exist 1 1
(a) - (b) (c) 1 (d) 2
17. For each xÎR, let [x] be greatest integer less than or equal 4 2
to x. Then [Jan. 09, 2019 (II)]
x ([ x ] + | x |) sin [ x] 26. lim
(
sin p cos 2 x ) equals [Online May 26, 2012]
lim is equal to: 2
x ®0 x x ®0 x
(a) – sin 1 (b) 1 (c) sin 1 (d) 0 (a) – p (b) 1 (c) – 1 (d) p
x tan 2 x - 2 x tan x æ x - sin x ö æ 1ö
18. lim equals. [Online April 15, 2018] lim ç
x ®0 (1 - cos 2 x) 2
27.
è ÷ø sin çè ÷ø [Online May 7, 2012]
x®0 x x
1 1 1 (a) equals 1 (b) equals 0
(a) 1 (b) - (c) (d)
2 4 2 (c) does not exist (d) equals – 1

Downloaded from @Freebooksforjeeneet


Limits and Derivatives M-209

28. Let f : R ® [0, ¥) be such that lim f(x) exists and 1 - cos 2 x
x®5
35. lim is [2002]
x ®0 2x
lim
( f ( x )) 2
-9
=0 (a) 1 (b) –1
[2011RS]
x®5 x -5
(c) ero (d) does not exist
Then lim f(x) equals :
x®5 Limits Using L-hospital's Rule,
(a) 0 (b) 1 (c) 2 (d) 3 TOPIC n Evaluation of Limits of the form
1¥, Limits by Expansion Method
æ 1 - cos{2( x - 2)} ö
29. lim ç ÷ [2011]
x®2 è x-2 ø 1 1
(a + 2 x) 3 - (3 x ) 3
(a) equals (b) equals – 36. lim ( a ¹ 0) is equal to :
2 2 x ®a 1 1
(3a + x) 3 - (4 x ) 3
1
(c) equals (d) does not exist [Sep. 03, 2020 (II)]
2
30. Let f : R ® R be a positive increasing function with 4 1
æ 2 ö3 æ 2 öæ 2 ö 3
f (3 x) f (2 x) (a) ç ÷ (b) ç ÷ç ÷
lim = 1 then lim = [2010] è3ø è 3 øè 9 ø
x ®¥ f ( x) x ®¥ f ( x)
4 1
2 3
(a) (b) (c) 3 (d) 1 æ 2 ö3 æ 2 öæ 2 ö 3
3 2 (c) ç ÷ (d) ç ÷ç ÷
è9ø è 9 øè 3 ø
31. Let a and b be the distinct roots of ax 2 + bx + c = 0 ,
x + x 2 + x3 + ... + x n - n
1 - cos(ax 2 + bx + c) 37. If lim = 820, (n Î N) then the
then lim is equal to [2005] x ®1 x -1
x®a ( x - a) 2
value of n is equal to ________. [NA Sep. 02, 2020 (I)]
2
a 1/ x
(a) (a - b ) 2 (b) 0 æ æp öö
2 38. lim ç tan ç + x÷ ÷ is equal to : [Sep. 02, 2020 (II)]
x®0 è è 4 øø
-a 2 1
(c) (a - b ) 2 (d) (a - b) 2 (a) e (b) 2 (c) 1 (d) e 2
2 2
1/ x 2
æ 3 x2 + 2 ö
é æ xö ù lim ç 2
ê1 - tan çè 2 ÷ø ú [1 - sin x]
39.
x ®0 è 7 x + 2 ø
÷ is equal to: [Jan. 8, 2020 (I)]
ë û
32. lim is [2003]

p é æ xö ù 3
2 ê1 + tan çè ÷ø ú [ p - 2 x ] 1 1
ë 2 û (a) (b) (c) e2 (d) e
e e2
1 1 x t sin(10t ) dt
(a) ¥ (b)
8
(c) 0 (d)
32
40. lim ò is equal to: [Jan. 8, 2020 (II)]
x® 0 0 x

log x n - [ x] 1 1 1
33. lim , n Î N , ([x] denotes greatest integer less (a) 0 (b) (c) - (d) -
x ®0 [ x] 10 5 10
than or equal to x) [2002] 41. If a and b are the roots of the equation 375x2–25x–2=0,
(a) has value -1 (b) has value 0 n n
(c) has value 1 (d) does not exist then nlim
®¥
å a r + nlim
®¥
å br is equal to :
r =1 r =1
x
æ x + 5x + 3 ö
2 [April 12, 2019 (I)]
34. lim ç 2 ÷ [2002]
x ®¥ çè x + x + 2 ÷ø 21 29 1 7
(a) (b) (c) (d)
(a) e4 (b) e2 (c) e3 (d) 1 346 358 12 116

Downloaded from @Freebooksforjeeneet


EBD_8344
M-210 Mathematics

42. Let f : R ® R be a differentiable function satisfying 2x


æ a b ö
1 48. If lim çç1+ + ÷÷ = e 2 , then the values of a and b, are
æ 1 + f ( 3 + x ) – f ( 3) öx x ®¥ è x x 2 ø
f ¢(3) + f ¢(2) = 0. Then lim ç ÷ is equal
x ®0 è 1 + f ( 2 – x ) – f ( 2 ) ø [2004]
to : [April 08, 2019 (II)] (a) a = 1 and b = 2 (b) a = 1, b Î R
(a) 1 (b) e–1 (c) e (d) e2 (c) a Î R, b = 2 (d) a Î R, b Î R
43. For each t Î R , let [t] be the greatest integer less than or
equal to t. Then [2018] xf (2) - 2 f ( x)
49. Let f (x) = 4 and f ¢ (x) = 4. Then lim
x ®2 x-2
æé 1 ù é 2ù é15 ù ö
lim x ç ê ú + ê ú + ... + ê ú ÷ is given by [2002]
+ èë x û ëx û ë x ûø
x ®0 (a) 2 (b) –2 (c) – 4 (d) 3
(a) is equal to 15. (b) is equal to 120.
(c) does not exist (in R). (d) is equal to 0. Derivatives of Polynomial &
Trigonometric Functions,
1
(27 + x ) 3 - 3
TOPIC Đ Derivative of Sum, Difference,
44. lim equals. [Online April 16, 2018] Product & Quotient of two
x®0 2
functions
9 - (27 + x ) 3
50. Let f (x) be a polynomial of degree 4 having extreme values
1 1 1 1
(a) - (b) (c) - (d) æ f ( x) ö
3 6 6 3 at x = 1 and x = 2. If lim ç 2 + 1÷ = 3 then f (– 1) is equal
x ®0 è x ø
1
( )
to [Online April 15, 2018]
45. Let p = lim 1 + tan 2 x 2x then log p is equal to :
x ®0+ (a)
1
(b)
3
(c)
5
(d)
9
[2016] 2 2 2 2
1 1 51. Let f(x) be a polynomial of degree four having extreme
(a) (b) (c) 2 (d) 1
2 4 é f (x) ù
values at x = 1 and x = 2. If xlim 1 + 2 ú = 3, then f(2) is
®0 êë x û
(1 - cos 2x) 2
46. lim is : [Online April 10, 2016] equal to : [2015]
x ®0 2x tan x - x tan 2x
(a) 0 (b) 4
1 1 (c) – 8 (d) – 4
(a) 2 (b) - (c) –2 (d) 52. Let f (1) = –2 and f ¢ (x) ³ 4.2 for 1 £ x £ 6 . The possible
2 2
value of f (6) lies in the interval : [April 25, 2013]
2x (b) (– ¥ , 12)
æ a 4ö (a) [15, 19)
47. If lim ç1 + - 2 ÷ = e3, then 'a' is equal to : (d) [19, ¥ )
x ®¥ è x x ø (c) [12, 15)
53. If f (x) = 3x10 – 7x8 + 5x6 – 21x3 + 3x2 – 7, then
[Online April 9, 2016]
f (1 - a ) - f (1)
3 1 2 lim is [Online May 19, 2012]
(a) 2 (b) (c) (d) a®0 a3 + 3a
2 2 3
53 53 55 55
(a) - (b) (c) - (d)
3 3 3 3

Downloaded from @Freebooksforjeeneet


Statistics M-229

14
Statistics
7. The mean of the data set comprising of 16 observations is
Arithmetic Mean, Geometric Mean, 16. If one of the observation valued 16 is deleted and three
TOPIC Ć Harmonic Mean, Median & Mode new observations valued 3, 4 and 5 are added to the data,
then the mean of the resultant data, is: [2015]
1. Consider the data on x taking the values 0, 2, 4, 8, ..., 2n with
frequencies nC0, nC1, nC2, ..., nCn respectively. If the mean of (a) 15.8 (b) 14.0 (c) 16.8 (d) 16.0
8. Let the sum of the first three terms of an A. P, be 39 and the
this data is 728 , then n is equal to ______. sum of its last four terms be 178. If the first term of this
2n A.P. is 10, then the median of the A.P. is :
[NA Sep. 06, 2020 (II)] [Online April 10, 2015]
2. The minimum value of 2sin x + 2cos x is : [Sep. 04, 2020 (II)] (a) 28 (b) 26.5 (c) 29.5 (d) 31
9. A factory is operating in two shifts, day and night, with 70
1 1
-1+ 1- and 30 workers respectively. If per day mean wage of the
(a) 2 (b) 2 -1+ 2 (c) 21- 2
2 (d) 2 2
day shift workers is ` 54 and per day mean wage of all the
3. If for some x Î R, the frequency distribution of the marks workers is ` 60, then per day mean wage of the night shift
obtained by 20 students in a test is : workers (in `) is : [Online April 10, 2015]
Marks 2 3 5 7 (a) 69 (b) 66 (c) 74 (d) 75
2 2 10. In a set of 2n distinct observations, each of the
Frequency (x+1) 2x – 5 x – 3x x
observations below the median of all the observations is
then the mean of the marks is : [April 10, 2019 (I)]
increased by 5 and each of the remaining observations is
(a) 3.2 (b) 3.0 (c) 2.5 (d) 2.8 decreased by 3. Then the mean of the new set of
4. The mean and the median of the following ten numbers in observations: [Online April 9, 2014]
increasing order 10, 22, 26, 29, 34, x, 42, 67, 70, y are 42 and (a) increases by 1 (b) decreases by 1
y (c) decreases by 2 (d) increases by 2
35 respectively, then is equal to: [April. 09, 2019 (II)]
x
11. If the median and the range of four numbers
(a) 9/4 (b) 7/2 (c) 8/3 (d) 7/3 {x, y, 2x + y, x – y}, where 0 < y < x < 2y, are 10
5. The mean of a set of 30 observations is 75. If each other and 28 respectively, then the mean of the numbers is :
observation is multiplied by a non- ero number l and then [Online April 23, 2013]
each of them is decreased by 25, their mean remains the
(a) 18 (b) 10 (c) 5 (d) 14
same. The l is equal to [Online April 15, 2018]
12. The mean of a data set consisting of 20 observations is 40.
10 4 1 2 If one observation 53 was wrongly recorded as 33, then
(a) (b) (c) (d)
3 3 3 3 the correct mean will be : [Online April 9, 2013]
6. The mean age of 25 teachers in a school is 40 years. A (a) 41 (b) 49 (c) 40.5 (d) 42.5
teacher retires at the age of 60 years and a new teacher is 13. The median of 100 observations grouped in classes of
appointed in his place. If now the mean age of the teachers equal width is 25. If the median class interval is 20 - 30 and
in this school is 39 years, then the age (in years) of the the number of observations less than 20 is 45, then the
newly appointed teacher is : [Online April 8, 2017] frequency of median class is [Online May 19, 2012]
(a) 25 (b) 30 (c) 35 (d) 40 (a) 10 (b) 20 (c) 15 (d) 12

Downloaded from @Freebooksforjeeneet


EBD_8344
M-230 Mathematics

14. The frequency distribution of daily working expenditure 21. The mean and variance of 7 observations are 8 and 16,
of families in a locality is as follows: respectively. If five observations are 2, 4, 10, 12, 14, then
Expenditure 0-50 50-100 100-150 150-200 200-250 the absolute difference of the remaining two observations
in `. (x ): is : [Sep. 05, 2020 (I)]
No. of 24 33 37 b 25 (a) 1 (b) 4 (c) 2 (d) 3
families (f ): 22. If the mean and the standard deviation of the data 3, 5, 7, a,
b are 5 and 2 respectively, then a and b are the roots of the
If the mode of the distribution is ` 140, then the value of b equation : [Sep. 05, 2020 (II)]
is [Online May 7, 2012]
(a) 34 (b) 31 (c) 26 (d) 36 (a) x 2 - 10 x + 18 = 0 (b) 2 x 2 - 20 x + 19 = 0
15. The average marks of boys in class is 52 and that of girls is (c) x 2 - 10 x + 19 = 0 (d) x 2 - 20 x + 18 = 0
42. The average marks of boys and girls combined is 50. 23. The mean and variance of 8 observations are 10 and 13.5,
The percentage of boys in the class is [2007] respectively. If 6 of these observations are 5, 7, 10, 12, 14,
(a) 80 (b) 60 (c) 40 (d) 20 15, then the absolute difference of the remaining two
observations is : [Sep. 04, 2020 (I)]
16. Let x1 , x 2 , .............. xn be n observations such that (a) 9 (b) 5 (c) 3 (d) 7
å xi2 = 400 and å xi = 80. Then the possible value of n 24. If a variance of the following frequency distribution :
among the following is [2005] Class 10-20 20-30 30-40
(a) 15 (b) 18 (c) 9 (d) 12 Frequency 2 x 2
17. If in a frequency distribution, the mean and median are 21 is 50, then x is equal to _____________.
and 22 respectively, then its mode is approximately [2005] [NA Sep. 04, 2020 (II)]
(a) 22.0 (b) 20.5 (c) 25.5 (d) 24.0 25. For the frequency distribution :
18. The median of a set of 9 distinct observations is 20.5. If
each of the largest 4 observations of the set is increased Variate (x ) : x1 x2 x 1 …x 15
by 2, then the median of the new set [2003] Frequency (f ) : f1 f2 f 3 …f 15
(a) remains the same as that of the original set 15

(b) is increased by 2 where 0 < x1 < x2 < x3 < ... < x15 = 10 and å fi > 0, the
i =1
(c) is decreased by 2 standard deviation cannot be : [Sep. 03, 2020 (I)]
(d) is two times the original median.
(a) 4 (b) 1 (c) 6 (d) 2
19. In a class of 100 students there are 70 boys whose average
marks in a subect are 75. If the average marks of the 26. Let xi (1 £ i £ 10) be ten observations of a random variable X.
complete class is 72, then what is the average of the girls? 10 10

[2002] If å ( xi - p) = 3 and å ( x - p)
i= 1
i
2
= 9 where
i =1
(a) 73 (b) 65 (c) 68 (d) 74
0 ¹ p Î R, then the standard deviation of these
observations is : [Sep. 03, 2020 (II)]
Quartile, Measures of Dispersion,
Quartile Deviation, Mean 3 4 9 7
TOPIC n Deviation, Variance & Standard (a) (b) (c) (d)
5 5 10 10
Deviation, Coefficient of Variation
27. Let X = {x Î N : 1 £ x £ 17} and Y =+ {ax b : x Î X and
n n a, b Î R, a > 0}. If mean and variance of elements of Y
20. If å ( xi - a) = n and å ( xi - a)2 = na , (n, a > 1), then are 17 and 216 respectively then a + b is equal to :
i =1 i =1 [Sep. 02, 2020 (I)]
the standard deviation of n observations x1, x2, ..., xn is : (a) 7 (b) –7 (c) –27 (d) 9
[Sep. 06, 2020 (I)] 28. If the variance of the terms in an increasing A.P.,
(a) a – 1 (b) n a - 1 b1 , b2 , b3 , ....., b11 is 90, then the common difference of
(c) n(a - 1) (d) a -1 this A.P. is ___________. [NA Sep. 02, 2020 (II)]

Downloaded from @Freebooksforjeeneet


Statistics M-231

29. Let the observations xi(1 £ i £ 10) satisfy the equations, 39. If the sum of the deviations of 50 observations from 30 is
10 10 50, then the mean of these observations is :
å ( xi - 5) = 10 and å ( xi - 5)2 = 40. If m and l are the [Jan. 12, 2019 (I)]
i =1 i =1
mean and the variance of the observations, x1 – 3, x2 – 3, (a) 30 (b) 51 (c) 50 (d) 31
..., x10 – 3, then the ordered pair (m, l) is equal to: 40. The mean and the variance of five observations are 4 and
[Jan. 9, 2020 (I)] 5.20, respectively. If three of the observations are 3, 4 and
(a) (3, 3) (b) (6, 3) (c) (6, 6) (d) (3, 6) 4; then the absolute value of the difference of the other
30. The mean and the standard deviation (s.d.) of 10 two observations, is : [Jan. 12, 2019 (II)]
observations are 20 and 2 respectively. Each of these 10 (a) 7 (b) 5 (c) 1 (d) 3
observations is multiplied by p and then reduced by q, 41. The outcome of each of 30 items was observed; 10 items
where p ¹ 0 and q ¹ 0. If the new mean and new s.d.
1 1
become half of their original values, then q is equal to: gave an outcome - d each, 10 items gave outcome
[Jan. 8, 2020 (I)] 2 2
(a) –5 (b) 10 (c) –20 (d) –10 1
each and the remaining 10 items gave outcome + d each.
31. The mean and variance of 20 observations are found to 2
be 10 and 4, respectively. On rechecking, it was found 4
that an observation 9 was incorrect and the correct If the variance of this outcome data is then |d| equals :
3
observation was 11. Then the correct variance is:
[Jan. 8, 2020 (II)] [Jan. 11, 2019 (I)]
(a) 3.99 (b) 4.01 (c) 4.02 (d) 3.98 2 5
(a) (b) 2 (c) (d) 2
32. If the variance of the first n natural numbers is 10 and the 3 2
variance of the first m even natural numbers is 16, then 42. A data consists of n observations:
m + n is equal to ¾¾¾. [NA Jan. 7, 2020 (I)]
n n
33. If the mean and variance of eight numbers 3, 7, 9, 12, 13,
20, x and y be 10 and 25 respectively, then x × y is equal to
x1, x2, ...., xn. If å ( xi + 1)2 = 9n and å ( xi - 1) 2 = 5n
i =1 i =1
_________. [NA Jan. 7, 2020 (II)]
then the standard deviation of this data is:
34. If the data x1, x2, ……, x10 is such that the mean of first four
[Jan. 09, 2019 (II)]
of these is 11, the mean of the remaining six is 16 and the
sum of squares of all of these is 2,000 ; then the standard (a) 2 (b) 5 (c) 5 (d) 7
deviation of this data is : [April 12, 2019 (I)]
43. The mean of five observations is 5 and their variance is
(a) 2 2 (b) 2 (c) 4 (d) 2 9.20. If three of the given five observations are 1, 3 and 8,
then a ratio of other two observations is:
35. If both the mean and the standard deviation of 50
observations x1, x2, ….. , x50 are equal to 16, then the mean [Jan. 10, 2019 (I)]
of (x1 – 4)2, (x2 – 4)2, …, (x50 – 4)2 is : [April 10, 2019 (II)] (a) 10 : 3 (b) 4 : 9 (c) 5 : 8 (d) 6 : 7
(a) 400 (b) 380 (c) 525 (d) 480 44. If mean and standard deviation of 5 observations
x1, x2, x3, x4, x5 are 10 and 3, respectively, then the variance
36. If the standard deviation of the numbers –1, 0, 1, k is 5
of 6 observations x1, x2, ..., x5 and – 50 is equal to:
where k > 0, then k is equal to: [April 09, 2019 (I)]
[Jan. 10, 2019 (II)]
5
(a) 2 6 (b) 2 10 (c) 4 (d) 6 (a) 509.5 (b) 586.5 (c) 582.5 (d) 507.5
3 3
45. 5 students of a class have an average height 150 cm and
37. The mean and variance of seven observations are 8 and
variance 18 cm2. A new student, whose height is 156 cm,
16, respectively. If 5 of the observations are 2, 4, 10, 12, 14,
then the product of the remaining two observations is : oined them. The variance (in cm2) of the height of these
six students is: [Jan. 9, 2019 (I)]
[April 08, 2019 (I)]
(a) 45 (b) 49 (c) 48 (d) 40 (a) 16 (b) 22 (c) 20 (d) 18
38. A student scores the following marks in five tests: 45, 54, 46. The mean and the standard deviation (s.d.) of five
41, 57, 43. His score is not known for the sixth test. If the observations are 9 and 0, respectively.
mean score is 48 in the six tests, then the standard If one of the observations is changed such that the mean
deviation of the marks in six tests is : [April. 08, 2019 (II)] of the new set of five observations becomes 10, then their
10 100 10 100 s.d. is? [Online April 16, 2018]
(a) (b) (c) (d) (a) 0 (b) 4 (c) 2 (d) 1
3 3 3 3

Downloaded from @Freebooksforjeeneet


EBD_8344
M-232 Mathematics

47. If the mean of the data : 7, 8, 9, 7, 8, 7, l, 8 is 8, then the 55. Let X and M.D. be the mean and the mean deviation about
variance of this data is [Online April 15, 2018]
X of n observations xi, i = 1, 2, ........, n. If each of the
9 7 observations is increased by 5, then the new mean and
(a) (b) 2 (c) (d) 1 the mean deviation about the new mean, respectively, are:
8 8
[Online April 12, 2014]
9 9 (a) X, M.D. (b) X + 5, M.D.
48. If å (xi - 5) = 9 and å (xi - 5)2 = 45 , then the standard
i =1 i=1 (c) X, M.D. + 5 (d) X + 5, M.D. + 5
deviation of the 9 items x1, x2, ..., x9 is : [2018] 56. All the students of a class performed poorly in
(a) 4 (b) 2 (c) 3 (d) 9 Mathematics. The teacher decided to give grace marks of
10 to each of the students. Which of the following statistical
49. The sum of 100 observations and the sum of their squares measures will not change even after the grace marks were
are 400 and 2475, respectively. Later on, three observations, given ? [2013]
3, 4 and 5, were found to be incorrect. If the incorrect (a) mean (b) median (c) mode (d) variance
observations are omitted, then the variance of the remaining 57. In a set of 2n observations, half of them are equal to 'a' and
observations is : [Online April 9, 2017] the remaining half are equal to ' –a'. If the standard deviation
(a) 8.25 (b) 8.50 (c) 8.00 (d) 9.00 of all the observations is 2 ; then the value of | a | is :
[Online April 25, 2013]
50. If the standard deviation of the numbers 2, 3, a and 11 is
3.5, then which of the following is true? [2016] (a) 2 (b)2 (c) 4 (d) 2 2
58. Mean of 5 observations is 7. If four of these observations
(a) 3a2 – 34a + 91 = 0 (b) 3a2 – 23a + 44 = 0
are 6, 7, 8, 10 and one is missing then the variance of all the
(c) 3a2 – 26a + 55 = 0 (d) 3a2 – 32a + 84 = 0 five observations is : [Online April 22, 2013]
(a) 4 (b) 6 (c) 8 (d) 2
51. The mean of 5 observations is 5 and their variance is 124.
If three of the observations are 1, 2 and 6; then the mean 59. Let x1 , x2,...., xn be n observations, and let x be their
deviation from the mean of the data is : arithmetic mean and s2 be the variance.
Statement-1 : Variance of 2x1, 2x2, ..., 2xn is 4s2.
[Online April 10, 2016]
Statement-2 : Arithmetic mean 2x1, 2x2, ..., 2xn is 4 x .
(a) 2.5 (b) 2.6 (c) 2.8 (d) 2.4 [2012]
52. If the mean deviation of the numbers 1, 1 + d, ..., 1 + 100d (a) Statement-1 is false, Statement-2 is true.
from their mean is 255, then a value of d is : (b) Statement-1 is true, statement-2 is true; statement-2 is
a correct explanation for Statement-1.
[Online April 9, 2016]
(c) Statement-1 is true, statement-2 is true; statement-2 is
(a) 10.1 (b) 5.05 (c) 20.2 (d) 10 not a correct explanation for Statement-1.
53. The variance of first 50 even natural numbers is [2014] (d) Statement-1 is true, statement-2 is false.
60. Statement 1: The variance of first n odd natural numbers
437 833
(a) 437 (b)
4
(c)
4
(d) 833 n2 - 1
is
3
54. Let x , M and s2 be respectively the mean, mode and
Statement 2: The sum of first n odd natural number is n2
variance of n observations x1, x2, ...., xn and di = – xi – a ,
and the sum of square of first n odd natural numbers is
i = 1, 2, ...., n, where a is any number.
Statement I: Variance of d1, d2,... dn is s2. (
n 4n2 + 1 ). [Online May 26, 2012]
3
Statement II: Mean and mode of d1, d2, .... dn are - x - a
(a) Statement 1 is true, Statement 2 is false.
and – M – a, respectively. [Online April 19, 2014]
(b) Statement 1 is true, Statement 2 is true; Statement 2 is
(a) Statement I and Statement II are both false not a correct explanation for Statement 1.
(b) Statement I and Statement II are both true (c) Statement 1 is false, Statement 2 is true.
(c) Statement I is true and Statement II is false (d) Statement 1 is true, Statement 2 is true, Statement 2 is
a correct explanation for Statement 1.
(d) Statement I is false and Statement II is true

Downloaded from @Freebooksforjeeneet


Statistics M-233

61. If the mean of 4, 7, 2, 8, 6 and a is 7, then the mean deviation (c) Statement-1 is false, Statement-2 is true.
from the median of these observations is (d) Statement-1 is true, Statement-2 is true. Statement-2 is
[Online May 12, 2012] a correct explanation for Statement-1.
(a) 8 (b) 5 (c) 1 (d) 3 67. The mean of the numbers a, b, 8, 5, 10 is 6 and the variance
62. A scientist is weighing each of 30 fishes. Their mean weight is 6.80. Then which one of the following gives possible
worked out is 30 gm and a standarion deviation of 2 gm. values of a and b? [2008]
Later, it was found that the measuring scale was misaligned (a) a = 0, b = 7 (b) a = 5, b = 2
and always under reported every fish weight by 2 gm. The (c) a = 1, b = 6 (d) a = 3, b = 4
correct mean and standard deviation (in gm) of fishes are
respectively : [2011RS] 68. Suppose a population A has 100 observations 101, 102,
............., 200 and another population B has 100 obsevrations
(a) 32, 2 (b) 32, 4 (c) 28, 2 (d) 28, 4 151, 152, ................ 250. If VA and VB represent the variances
63. If the mean deviation about the median of the numbers a,
2a,.......,50a is 50, then | a | equals [2011] VA
of the two populations, respectively then is [2006]
VB
(a) 3 (b) 4 (c) 5 (d) 2
64. For two data sets, each of si e 5, the variances are given to
9 4 2
be 4 and 5 and the corresponding means are given to be 2 (a) 1 (b) (c) (d)
4 9 3
and 4, respectively. The variance of the combined data set
is [2010] 69. In a series of 2 n observations, half of them equal a and
remaining half equal –a. If the standard deviation of the
11 13 5 observations is 2, then |a| equals. [2004]
(a) (b) 6 (c) (d)
2 2 2
2 1
65. If the mean deviation of the numbers 1, 1 + d, (a) (b) 2 (c) 2 (d)
n n
1 + 2d, .... 1 + 100d from their mean is 255, then d is equal to:
70. Consider the following statements :
[2009]
(A) Mode can be computed from histogram
(a) 20.0 (b) 10.1 (c) 20.2 (d) 10.0
(B) Median is not independent of change of scale
66. Statement-1 : The variance of first n even natural numbers
(C) Variance is independent of change of origin and scale.
n2 – 1 Which of these is / are correct ? [2004]
is .
4 (a) (A), (B) and (C) (b) Only (B)
Statement-2 : The sum of first n natural numbers is (c) Only (A) and (B) (d) Only (A)
n(n + 1) 71. In an experiment with 15 observations on x, the following
and the sum of squares of first n natural numbers results were available:
2
Sx 2 = 2830, Sx = 170
is n (n + 1)(2n + 1) . [2009]
6 One observation that was 20 was found to be wrong and
(a) Statement-1 is true, Statement-2 is true. Statement-2 is was replaced by the correct value 30. The corrected
not a correct explanation for Statement-1. variance is [2003]
(b) Statement-1 is true, Statement-2 is false. (a) 8.33 (b) 78.00 (c) 188.66 (d) 177.33

Downloaded from @Freebooksforjeeneet


EBD_8344
M-244 Mathematics

15
Probability

Random Experiment, Sample 7 5


(a) 20 (b)
Space, Events, Probability of an 2 220
TOPIC Ć Event, Mutually Exclusive &
Exhaustive Events, Equally Likely 4 6
(c) 20 (d)
1. Out of 11 consecutive natural numbers if three numbers 2 220
are selected at random (without repetition), then the 5. Two different families A and B are blessed with equal
probability that they are in A.P. with positive common number of children. There are 3 tickets to be distributed
difference, is: [Sep. 06, 2020 (I)] amongst the children of these families so that no child
gets more than one ticket.
15 5 If the probability that all the tickets go to the children of
(a) (b)
101 101 1
the family B is , then the number of children in each
5 10 12
(c) (d) family is? [Online April 16, 2018]
33 99
(a) 4 (b) 6
2. If 10 different balls are to be placed in 4 distinct boxes at
(c) 3 (d) 5
random, then the probability that two of these boxes
contain exactly 2 and 3 balls is : [Jan. 9, 2020 (II)] 6. A box ¢A¢ contanis 2 white, 3 red and 2 black balls. Another
box ¢B¢ contains 4 white, 2 red and 3 black balls. If two
965 965
(a) (b) balls are drawn at random, without replacement, from a
211 210 randomly selected box and one ball turns out to be white
945 945 while the other ball turns out to be red, then the probability
(c)
210
(d)
211 that both balls are drawn from box ¢B¢ is
[Online April 15, 2018]
3. If three of the six vertices of a regular hexa on are chosen
at random, then the probability that the triangle formed 7 9
(a) (b)
with these chosen vertices is equilateral is : 16 32
[April 12, 2019 (I)]
7 9
(c) (d)
1 1 8 16
(a) (b)
10 5
7. If the lengths of the sides of a triangle are decided by
3 3 the three throws of a single fair die, then the probability
(c) (d) that the triangle is of maximum area given that it is an
10 20
4. Let S = {1, 2, ....., 20}. A subset B of S is said to be “nice”, isosceles triangle, is : [Online April 11, 2015]
if the sum of the elements of B is 203. Than the probability 1 1
(a) (b)
that a randomly chosen subset of S is “nice” is : 21 27
[Jan. 11, 2019 (II)] 1 1
(c) (d)
15 26

Downloaded from @Freebooksforjeeneet


Probability M-245

8. A number x is chosen at random from the set {1, 2, 3, 4, ...., Statement -2 : If the four chosen numbers form an AP,
100}. Define the event: A = the chosen number x satisfies then the set of all possible values of common difference is
( x - 10 )( x - 50) ³ 0 (±1, ±2, ±3, ±4, ±5) .
( x - 30 ) (a) Statement -1 is true, Statement -2 is true; Statement -2
Then P (A) is: [Online April 12, 2014] is not a correct explanation for Statement -1
(a) 0.71 (b) 0.70 (b) Statement -1 is true, Statment -2 is false
(c) 0.51 (d) 0.20 (c) Statement -1 is false, Statment -2 is true.
9. A set S contains 7 elements. A non-empty subset A of S (d) Statement -1 is true, Statement -2 is true ; Statement -
and an element x of S are chosen at random. Then the
2 is a correct explanation for Statement -1.
probability that x Î A is: [Online April 11, 2014]
14. An urn contains nine balls of which three are red, four are
1 64 blue and two are green. Three balls are drawn at random
(a) (b) without replacement from the urn. The probability that the
2 127
three balls have different colours is [2010]
63 31
(c) (d) 2 1
128 128
(a) (b)
10. There are two balls in an urn. Each ball can be either white 7 21
or black. If a white ball is put into the urn and there after a
ball is drawn at random from the urn, then the probability 2 1
(c) (d)
that it is white is [Online May 26, 2012] 23 3
1 2 15. Five horses are in a race. Mr. A selects two of the horses
(a) (b)
4 3 at random and bets on them. The probability that Mr. A
1 1 selected the winning horse is [2003]
(c) (d)
5 3 2 4
(a) (b)
11. If six students, including two particular students A and B, 5 5
stand in a row, then the probability that A and B are
separated with one student in between them is 3 1
(c) (d)
[Online May 19, 2012] 5 5

8 4
(a) (b) Odds Against & Odds in Favour of
15 15
TOPIC n an Event, Addition Theorem,
2 1 Boole's Inequality, Demorgan's Law
(c) (d)
15 15
16. The probabilities of three events A, B and C are given by
12. A number n is randomly selected from the set
P(A) = 0.6, P(B) = 0.4 and P(C) = 0.5. If P (A È B) = 0.8, P
n
åi
2 (A Ç C) = 0.3, P (A Ç B Ç C) = 0.2, P (B Ç C) = b and
{1, 2, 3, ..... , 1000}. The probability that i =1 is an integer P (A È B È C) = a, where 0.85 £ a £ 0.95 , then b lies in
n
åi the interval: [Sep. 06, 2020 (II)]
i =1 (a) [0.35, 0.36] (b) [0.25, 0.35]
is [Online May 12, 2012] (c) [0.20, 0.25] (d) [0.36, 0.40]
(a) 0.331 (b) 0.333 17. Let A and B be two events such that the probability that
(c) 0.334 (d) 0.332 2
exactly one of them occurs is and the probability that
13. Four numbers are chosen at random (without replacement) 5
from the set {1, 2, 3, ...20}. [2010]
1
Statement -1: The probability that the chosen numbers A or B occurs is , then the probability of both of them
2
1 occur together is: [Jan. 8, 2020 (II)]
when arranged in some order will form an AP is .
85 (a) 0.02 (b) 0.20
(c) 0.01 (d) 0.10

Downloaded from @Freebooksforjeeneet


EBD_8344
M-246 Mathematics

18. In a class of 60 students, 40 opted for NCC, 30 opted for 23. If the events A and B are mutually exclusive events such
NSS and 20 opted for both NCC and NSS. If one of these 3x + 1 1- x
students is selected at random, then the probability that the that P(A) = and P(B) = , then the set of
3 4
student selected has opted neither for NCC nor for NSS is : possible values of x lies in the interval :
[Jan. 12, 2019 (II)]
[Online April 25, 2013]
1 1
(a) (b) é1 2 ù
6 3 (a) [0, 1] (b) ê , ú
ë3 3û
2 5
(c) (d) é 1 5ù é 7 4ù
3 6 (c) ê - , ú (d) ê - , ú
ë 3 9û ë 9 9û
19. For three events A, B and C, 24. Let X and Y are two events such that
P(Exactly one of A or B occurs)
= P(Exactly one of B or C occurs) P ( X È Y ) = P ( X Ç Y ).
1
= P(Exactly one of C or A occurs) = and Statement 1: P ( X Ç Y ') = P ( X 'Ç Y ) = 0
4
Statement 2: P ( X ) + P (Y ) = 2P ( X Ç Y )
1
P(All the three events occur simultaneously) = .
16
Then the probability that at least one of the events [Online May 7, 2012]
occurs, is : [2017] (a) Statement 1 is false, Statement 2 is true.
3 7 (b) Statement 1 is true, Statement 2 is true, Statement 2 is
(a) (b)
16 32 not a correct explanation of Statement 1.
7 7 (c) Statement 1 is true, Statement 2 is false.
(c) (d) (d) Statement 1 is true, Statement 2 is true; Statement 2 is
16 64
20. From a group of 10 men and 5 women, four member a correct explanation of Statement 1.
committees are to be formed each of which must contain at 25. A die is thrown. Let A be the event that the number
least one woman. Then the probability for these committees obtained is greater than 3. Let B be the event that the
to have more women than men, is : number obtained is less than 5. Then P(AÈB) is [2008]
[Online April 9, 2017]
3
21 3 (a) (b) 0
(a) (b) 5
220 11
1 2 2
(c) (d) (c) 1 (d)
11 23 5
21. If 12 identical balls are to be placed in 3 identical boxes, 26. Events A, B, C are mutually exclusive events such that
then the probability that one of the boxes contains exactly 1 - 2x
3x + 1 1- x
3 balls is : (2015) P ( A) = , P( B) = and P (C ) = The set
12 11 3 4 2
æ 1ö æ 1ö
(a) 220 ç ÷ (b) 22 ç ÷ of possible values of x are in the interval. [2003]
è 3ø è 3ø
55 æ 2 ö
11 10 é1 1 ù
æ 2ö (a) [0 , 1] (b) ê 3 , 2 ú
(c) ç ÷ (d) 55 ç ÷ ë û
3 è 3ø è 3ø
22. If A and B are two events such that é1 2 ù é 1 13 ù
(c) ê 3 , 3 ú (d) ê 3 , 3 ú
P ( A È B) = P ( A Ç B ) , then the incorrect statement ë û ë û
amongst the following statements is: 27. A and B are events such that P(A È B)=3/4, P(A Ç B)=1/4,
[Online April 9, 2014] P( A ) =2/3 then P ( A Ç B) is [2002]
(a) A and B are equally likely (a) 5/12 (b) 3/8
(b) P ( A Ç B¢ ) = 0 (c) 5/8 (d) 1/4

(c) P ( A¢ Ç B ) = 0
(d) P(A) + P(B) = 1

Downloaded from @Freebooksforjeeneet


Probability M-247

1. (c) For an A.P. 2b = a + c (even), so both a and c even 2 1


Hence, required probability = =
numbers or odd numbers from given numbers and b number C3 6 10
will be fixed automatically.
4. (2) Since total number of subsets of the set S = 220
6
C2 + 5 C225 5 20 ´ 21
Required probability = 11
== Now, the sum of all number from 1 to 20 = = 210
C3 165 33 2
2. (Bonus) Total number of ways placing 10 different balls in Then, find the sets which has sum 7.
4 distinct boxes = 410 (1) {7}
Since, two of the 4 distinct boxes contains exactly 2 and 3 (2) {1, 6}
balls. (3) {2, 5}
Then, there are three cases to place exactly 2 and 3 balls in (4) {3, 4}
2 of the 4 boxes. (5) {1, 2, 4}
Case-1: When boxes contains balls in order 2, 3, 0, 5 Then, there is only 5 sets which has sum 203
Then, number of ways of placing the balls
5
10! Hence required probability =
= ´ 4! 220
2! ´ 3! ´ 0! ´ 5! 5. (d) Let the number of children in each family be x.
Case-2: When boxes contains ball in order 2, 3, 1, 4. Thus the total number of children in both the families are
Then, number of ways of placing the balls 2x
Now, it is given that 3 tickets are distributed amongst the
10!
= ´ 4! children of these two families.
2! ´ 3! ´1! ´ 4! Thus, the probability that all the three tickets go to the
Case-3: When boxes contains ball in order 2, 3, 2, 3 children in family B
Then, number of ways of placing the balls x
C3 1
10! = =
= ´ 4! 2x
C3 12
(2!)2 ´ (3!)2 ´ 2! ´ 3!
x ( x - 1) ( x - 2) 1
Therefore, number of ways of placing the balls that Þ =
contains exactly 2 and 3 balls. 2 x (2 x - 1) (2 x - 2) 12

=
10!
´ 4! +
10!
´ 4! ( x - 2) 1
Þ =
2! ´ 3! ´ 0! ´ 5! 2! ´ 3! ´1! ´ 4! (2 x - 1) 6
10! Þ x=5
+ ´ 4! Thus, the number of children in each family is 5.
(2!) ´ 2! ´ (3!)2 ´ 2!
2
6. (a) Probability of drawing a white ball and then a red ball
= 25 × 17 × 945 4
C1 ´ 2 C1 2
Hence, the required probability from bag B is given by 9
=
C2 9
25 ´ 17 ´ 945 17 ´ 945
= = Probability of drawing a white ball and then a red ball from
410 215
3. (1) Total no. of triangles = 6C3
2
C1 ´ 3C1 2
bag A is given by =
Favorable no. of triangle i.e, equilateral triangles (DAEC C2 7
7
and DBDF) = 2.
Hence, the probability of drawing a white ball and then a
E D
2
9 2´7 7
red ball from bag B = = =
2 2 18 + 14 16
F C +
7 9

7. (b) Favourable case = (6, 6, 6)


A B Total case = {(1, 1, 1) (2, 2, 1), (2, 2, 2), (2, 2, 3), (3, 3, 1) .....
(3, 3, 5) (4, 4, 1).... (4, 4, 6) (5, 5, 1).... (5, 5, 6)
(6, 6, 1).... (6, 6, 6)}

Downloaded from @Freebooksforjeeneet


EBD_8344
M-248 Mathematics

which satisfies condition a + b > c Now the group of three students (student A, student B
Number of total case = 27 and a student in between A and B) and the remaining
1 3 students can be stand in a row in 4! ways.
Probability = Hence total number of ways to stand in a row such that
27
A and B are separated with one student in between them
( x - 10)( x - 50) = 4 × 2 × 4!
8. (a) Given ³0
( x - 30) Now total number of ways to stand 6 student stand in
a row without any restriction = 6!
Let x ³ 10, x ³ 50 equation will be true " x ³ 50
Hence required probability
æ x - 50 ö 4 ´ 2 ´ 4! 4 ´ 2 4
as ç ³ 0, " x Î[10, 30) = =
è x - 30 ÷ø =
6! 6 ´ 5 15
( x - 10)( x - 50)
³ 0 " x Î[10, 30) n
x - 30 å i2 n(n + 1)(2 n + 1)
Total value of x between 10 to 30 is 20. i =1 6 2n + 1
12. (c) = =
Total values of x between 50 to 100 including 50 and 100 n n(n + 1) 3
is 51. åi 2
i =1
Total values of x = 51 + 20 = 71
71 For n = 1, 2, 3, ......, 1000
P (A) = = 0.71
100
2n + 1 3 5 7 2001
9. (b) Let S = {x1, x2, x3, x4, x5, x6, x7} Value of = , , ,........, respectively. Out
Let the chosen element be xi. 3 3 3 3 3
Total number of subsets of S = 27 = 128
No. of non-empty subsets of S = 128 – 1 = 127 of
3 5 7
, , ,........,
2001 æ3 ö
only first term ç = 1÷ , fourth
We need to find number of those subsets that contains xi. 3 3 3 3 è3 ø

2 2 2 2 1 2 2 æ9 ö æ 2001 ö
term ç = 3 ÷ , 667th term ç = 667 ÷ are integers.
x1 x2 ------- xi ---- x7 è3 ø è 3 ø
For those subsets containing xi, each element has 2
choices. 2n + 1
Hence, out of 1000 values of ,
i.e., (included or not included) in subset, 3
However as the subset must contain xi, xi has only one
2n + 1
choice. (included one) total number of integral values of
So, total no. of subsets containing 3
xi = 2 × 2 × 2 × 2 × 1 × 2 × 2 = 64 = 333 + 1 = 334
No. of subsets containing xi 334
Required prob = \ Required probability = = 0.334
Total no. of non-empty subsets 1000
13. (b) Four numbers are chosen from {1, 2, 3...20}
64
= n(S) = 20C4
127
Statement-1:
10. (b) Total possible event when one ball is taken out Common difference is 1; total number of ways = 17
= 3C1
common difference is 2; total number of ways = 14
Let E : The event of 1 white ball coming out
common difference is 3; total number of ways = 11
No. of ways to 1 white ball coming out
common difference is 4; total number of ways = 8
= 2C1
common difference is 5; total number of ways = 5
2
C1 2 common difference is 6; total number of ways = 2
\ P(E) = 3
=
C1 3
17 + 14 + 11 + 8 + 5 + 2 1
11. (b) Consider a group of three students A, B and an Prob. =
20
=
C4 85
other student in between A and B. Choice for a student
between A and B is 4. A and B can interchange their Statement -2 is false, because common difference can be 6
places in the group in 2 ways. also.

Downloaded from @Freebooksforjeeneet


Probability M-249

P (Exactly one of B or C occurs)


14. (a) n( S ) = 9 C3
1
n( E ) = 3C1 ´ 4 C1 ´ 2 C1 = P(B) + P (C) – 2P (B Ç C) = ...(2)
4
3´ 4´ 2 24 ´ 3! 24 ´ 6 2 P (Exactly one of C or A occurs)
Probability = = ´ 6! = =
9
C3 9! 9 ´8´ 7 7 1
= P(C) + P(A) – 2P (C Ç A) = ...(3)
15. (a) Let 5 horses are H1, H2, H3, H4 and H5. 4
Total ways of selecting pair of horses be Adding (1), (2) and (3),we get
3
= 5C2 = 10[i.e. H1H2 ,H1H3 , H1H4 , H1 H5 , 2SP(A) – 2SP (A Ç B) =
4
H 2H3 , H 2 H 4 , H 2H5 , H3H 4 , H3H5 , H 4H5 ] 3
\ SP(A) – SP (A Ç B) =
Any horse can win the race in 4 ways 8
(e.g. for H1 : H1H2, H1H3, H1H4, H1H5)
1
Now, P (A Ç B Ç C) =
4 2 16
Hence required probability = =
10 5 \ P (A È B È C)
= SP (A) – SP (A Ç B) + P (A Ç B Ç C)
16. (b) Q P ( A Ç B) = P ( A) + P ( B ) - P ( A È B )
3 1 7
= 1 – 0.8 = 0.2 = + =
Now, 8 16 16
20. (c) Probability of 4 member committee which contain
Q P ( A È B È C ) = P ( A) + P ( B ) + P (C ) - P ( A Ç B )
atleast one woman.
- P( B Ç C ) - P(C Ç A) + P ( A Ç B Ç C ) Þ P(3M, 1W) + P(2M, 2W) + P(1M, 3W) + P(0M, 4W)
Þ a = 0.6 + 0.4 + 0.5 - 0.2 - b - 0.3 + 0.2 10
C3 5C1 10
C2 5C2 10
C1 5C3 10
C0 5 C4
Þ + + +
Þ b = 1.2 - a 15
C4 15
C4 15
C4 15
C4
Q a Î[0.85, 0.95] then b Î[0.25, 0.35]
600 450 100 5
2 Þ + + +
17. (4) P(exactly one) = 1365 1365 1365 1365
5
1155
2 Þ
Þ P(A) + P(B) – 2P(A Ç B) = 1365
5
\ Probability of committees to have more women than
1
P(A or B) = P(A È B) = men.
2
1 P (1M,3W) + P ( 0M,4W)
Þ P(A) + P(B) – P(A Ç B) = =
2 P ( 3M,1W) + P ( 2M,2W) + P (1M,3W) + P ( 0M,4W)
1 2 5-4 1
\ P(A Ç B) = - = = = 0.10 105
2 5 10 10 1
1365
= =
18. (1) P = Set of students who opted for NCC 1155 11
Q = Set of Students who opted for NSS 1365
n(P) = 40, n(Q) = 30, n(P Ç Q) = 20 21. (c) Note:- The question should state ‘3 different’ boxes
n(P È Q) = n(P) + n(Q) – n(P Ç Q) instead of ‘3 identical boxes’ and one particular box has 3
= 40 + 30 – 20 balls. Then the solution would be:
= 50
12
50 C3 ´ 29
\ Hence, required probability = 1 - Required probability =
60 312
1 11
= 55 æ 2 ö
6 = ç ÷
19. (c) P (exactly one of A or B occurs) 3 è 3ø
1 22. (d) Let A and B be two events such that
= P(A) + P (B) – 2P (A Ç B) = ...(1) P(A È B) = P(A Ç B)
4

Downloaded from @Freebooksforjeeneet


EBD_8344
M-250 Mathematics

and P(A È B) = P (A) + P (B) – P(A Ç B) \ A Ç B = {4}


option (a) : since P(A È B) = P(A Ç B) (given) 1
Þ P( A Ç B) =
therefore A and B are equally likely 6
Suppose option (b) and option (c) are correct. \ P(A È B) = P(A) + P(B) – P(A Ç B)
\ P(A Ç B¢ ) = 0 and P(A¢ Ç B) = 0 1 2 1 3 + 4 –1
= + – = =1
Þ P(A) - P(A Ç B) = 0 and P(B) - P(A Ç B) = 0 2 3 6 6

Þ P (A) = P(A Ç B) and P(B) = P(A Ç B) 3x + 1 1- x


26. (b) Given that P ( A) = , P( B) = and
3 4
Thus P(A) = P (B) = P(A Ç B) = P(A È B)
1 - 2x
[Q Given P(A Ç B) = P(A È B) ] P (C ) =
2
Also, we know
We know that 0 £ P ( E ) £ 1
P(A È B) = P(A) + P (B) – P(A Ç B)
Þ 0 £ 3x + 1 £ 1, ³ –1 £ 3 x £ 2
= P(A Ç B) + P(A Ç B) – P(A Ç B) 3
= P(A Ç B) 1 2
Þ - £x£ ...(i)
which is true from given condition 3 3
Hence, option (a), (b) and (c) are correct. 1- x
23. (c) Since events A and B are mutually exclusive 0£ £ 1 Þ -3 £ x £ 1 ...(ii)
4
\ P(A) + P(B) = 1
1 - 2x
3x + 1 1 - x and 0 £ £ 1 Þ -1 £ 2 x £ 1
Þ + =1 2
3 4 1 1
Þ 12x + 4 + 3 – 3x = 12 Þ- £x£ ...(iii)
2 2
5 Also for mutually exclusive events A, B, C,
Þ 9x = 5 Þ x =
9 P ( A È B È C ) = P ( A) + P ( B ) + P ( C )

é 1 5ù 3x + 1 1 - x 1 - 2 x
\ x Î ê- , ú Þ P( A È B È C) = + +
ë 3 9û 3 4 2
24. (b) Let X and Y be two events such that 1 + 3x 1 - x 1 - 2 x
\ 0£ + + £1
P ( X ÈY ) = P ( X ÇY ) ...(1) 3 4 2
We know 0 £ 13 - 3x £ 12 Þ 1 £ 3x £ 13

P ( X È Y ) = P ( X ) + P (Y ) - P ( X Ç Y ) 1
Þ £x£
13
...(iv)
3 3
P ( X Ç Y ) = P ( X ) + P (Y ) - P ( X Ç Y ) (from (1) From (i), (ii), (iii) and (iv), we get

Þ P ( X ) + P (Y ) = 2 P ( X Ç Y ) 1 1 é1 1 ù
£ x £ Þ xÎê , ú
Hence, Statement - 2 is true.
3 2 ë3 2 û
27. (a) We know that P (A È B) = P (A) + P (B) – P (A Ç B)
Now, P ( X Ç Y ') = P ( X ) - P ( X Ç Y )
3 1
and P ( X 'Ç Y ) = P (Y ) - P ( X Ç Y )
Þ
4
=1 – P( A ) + P(B) –
4 ëéQ P ( A) = 1 - P( A)ûù
This implies statement-1 is also true. 2 2
25. (c) A (number is greater than 3) = {4, 5, 6} Þ 1=1– + P(B) Þ P(B) = ;
3 3
3 1 Now, P( A Ç B ) = P(B) – P ( A Ç B )
Þ P( A) = =
6 2 2 1 5
= – = .
4 2 3 4 12
B (number is less than 5) = {1, 2, 3, 4} Þ P( B) = =
6 3

Downloaded from @Freebooksforjeeneet


Relations and Functions M-251

16
Relations and
Functions
Types of Relations, Inverse of a (a) not inective
Relation, Mappings, Mapping of (b) neither inective nor surective
TOPIC Ć Functions, Kinds of Mapping of (c) surective but not inective
Functions (d) inective but not surective
1. Let A = {a, b, c} and B = {1, 2, 3, 4}. Then the number of é 1 1ù x
6. The function f : R ® ê - , ú defined as f(x) = , is :
elements in the set C = { f : A ® B | 2 Î f ( A) and f is not ë 2 2û 1 + x2
one-one} is ___________. [NA Sep. 05, 2020 (II)] [2017]
f : ( 0, ¥ ) ® ( 0, ¥ ) be defined by
(a) neither inective nor surective
2. Let a function
(b) invertible
1 (c) inective but not surective
f ( x) = 1- . Then f is : [Jan. 11, 2019 (II)]
x (d) surective but not inective
(a) not inective but it is surective éx ù
7. The function f : N ® N defined by f ( x ) = x - 5 ê ú , where
(b) inective only ë5û
(c) neither inective nor surective N is set of natural numbers and [x] denotes the greatest
(d) both inective as well as surective integer less than or equal to x, is :
3. The number of functions f from {1, 2, 3, ..., 20} onto [Online April 9, 2017]
{1, 2, 3, ...., 20} such that f (k) is a multiple of 3, whenever k (a) one-one and onto.
is a multiple of 4 is : [Jan. 11, 2019 (II)]
(b) one-one but not onto.
(a) 65 × (15)! (b) 5! × 6!
(c) onto but not one-one.
(c) (15)! × 6! (d) 56 × 15
(d) neither one-one nor onto.
4. Let N be the set of natural numbers and two functions
8. Let A = {x1, x2, ........., x7} and B = {y1, y2, y3} be two sets
f and g be defined as f, g : N ® N such that
containing seven and three distinct elements respectively.
ì n +1 Then the total number of functions f : A ® B that are
ïï if n is odd
onto, if there exist exactly three elements x in A such that
f (n) = í 2
ï n f(x) = y2, is equal to : (Online April 11, 2015)
if n is even
ïî 2
(a) 14.7C3 (b) 16.7C3 (c) 14.7C2 (d) 12.7C2
and g(n) = n – (– 1)n. Then fog is: [Jan. 10, 2019 (II)] x -1
(a) onto but not one-one. 9. Let f : R ® R be defined by f(x) = x + 1 then f is:
(b) one-one but not onto.
(c) both one-one and onto. [Online April 19, 2014]
(d) neither one-one nor onto. (a) both one-one and onto
5. Let A = {xÎR : x is not a positive integer}. Define a func- (b) one-one but not onto
(c) onto but not one-one
2x
tion f: A ® R as f(x) = , then f is:[Jan. 09, 2019 (II)] (d) neither one-one nor onto.
x -1

Downloaded from @Freebooksforjeeneet


EBD_8344
M-252 Mathematics

10. Let P be the relation defined on the set of all real numbers 17. Consider the following relations:
such that R = {(x, y) | x, y are real numbers and x = wy for some
P = {(a, b) : sec2a – tan2b = 1}. Then P is: æ m pö
[Online April 9, 2014] rational number w}; S = {ç , ÷ | m,n, p and q are
è n qø
(a) reflexive and symmetric but not transitive.
integers such that n, q ¹ 0 and qm = pn}.
(b) reflexive and transitive but not symmetric.
Then [2010]
(c) symmetric and transitive but not reflexive.
(a) Neither R nor S is an equivalence relation
(d) an equivalence relation.
(b) S is an equivalence relation but R is not an equivalence
11. Let R = {(x, y) : x, y Î N and x2 – 4xy + 3y2 = 0}, where N relation
is the set of all natural numbers. Then the relation R is : (c) R and S both are equivalence relations
[Online April 23, 2013] (d) R is an equivalence relation but S is not an equivalence
(a) reflexive but neither symmetric nor transitive. relation
(b) symmetric and transitive. 18. Let R be the real line. Consider the following subsets of
(c) reflexive and symmetric, the plane R × R:
(d) reflexive and transitive. S ={(x, y): y = x + 1 and 0 < x < 2}
12. Let R = {(3, 3) (5, 5), (9, 9), (12, 12), (5, 12), (3, 9), (3, 12), (3, 5)} T ={(x, y): x – y is an integer},
be a relation on the set A = {3, 5, 9, 12}. Then, R is : Which one of the following is true? [2008]
[Online April 22, 2013] (a) Neither S nor T is an equivalence relation on R
(a) reflexive, symmetric but not transitive. (b) Both S and T are equivalence relation on R
(b) symmetric, transitive but not reflexive. (c) S is an equivalence relation on R but T is not
(c) an equivalence relation. (d) T is an equivalence relation on R but S is not
(d) reflexive, transitive but not symmetric. 19. Let W denote the words in the English dictionary. Define
the relation R by R = {(x, y) Î W × W| the words x and y
13. Let A = {1, 2, 3, 4} and R : A ® A be the relation defined have at least one letter in common.} Then R is [2006]
by R = {(l, 1), (2, 3), (3, 4), (4, 2)}. The correct statement is : (a) not reflexive, symmetric and transitive
[Online April 9, 2013] (b) relexive, symmetric and not transitive
(a) R does not have an inverse. (c) reflexive, symmetric and transitive
(b) R is not a one to one function. (d) reflexive, not symmetric and transitive
(c) R is an onto function. 20. Let R = {(3, 3), (6, 6), (9, 9), (12, 12), (6, 12), (3, 9),
(d) R is not a function. (3, 12), (3, 6)} be a relation on the set
14. If P(S) denotes the set of all subsets of a given set S, then A = {3, 6, 9, 12}. The relation is [2005]
the number of one-to-one functions from the set (a) reflexive and transitive only
S = {1, 2, 3} to the set P(S) is [Online May 19, 2012] (b) reflexive only
(a) 24 (b) 8 (c) 336 (d) 320 (c) an equivalence relation
(d) reflexive and symmetric only
15. If A = {x Î z + : x < 10 and x is a multiple of 3 or 4}, where
21. Let f : (– 1, 1) ® B, be a function defined by
z+ is the set of positive integers, then the total number of
symmetric relations on A is [Online May 12, 2012] -1 2x
f (x) = tan , then f is both one - one and onto when
(a) 25 (b) 215 (c) 210 (d) 220 1 - x2
16. Let R be the set of real numbers. [2011] B is the interval [2005]
Statement-1: A = {(x, y) Î R × R : y – x is an integer} is an æ pö é pö
equivalence relation on R. (a) ç 0, ÷ (b) ê0, ÷
è 2ø ë 2ø
Statement-2: B = {(x, y) Î R × R : x = ay for some rational
number a} is an equivalence relation on R. æ p pö
(c) éê- p , p ùú (d) ç - , ÷
(a) Statement-1 is true, Statement-2 is true; Statement-2 is ë 2 2 û è 2 2ø
not a correct explanation for Statement-1. 22. Let R = {(1,3), (4, 2), (2, 4), (2, 3), (3,1)} be a relation on the
(b) Statement-1 is true, Statement-2 is false.
set A = {1, 2,3, 4}. . The relation R is [2004]
(c) Statement-1 is false, Statement-2 is true.
(d) Statement-1 is true, Statement-2 is true; Statement-2 is (a) reflexive (b) transitive
a correct explanation for Statement-1. (c) not symmetric (d) a function

Downloaded from @Freebooksforjeeneet


Relations and Functions M-253

23. If f : R ® S , defined by f ( x) = sin x - 3 cos x + 1, is æ 3ö 1- x2


28. For x Î ç 0, ÷ , let f(x) = x , g(x) = tan x and h(x) = .
onto, then the interval of S is [2004] è 2ø 1 + x2
(a) [ –1, 3] (b) [–1, 1] (c) [ 0, 1] (d) [0, 3] æpö
If f (x) = ((hof)og) (x), then f ç ÷ is equal to :
24. A function f from the set of natural numbers to integers è3ø
[April 12, 2019 (I)]
defined by [2003]
p 11p 7p 5p
(a) tan (b) tan (c) tan (d) tan
ìn -1 12 12 12 12
ïï 2 , when n is odd 29. Let f ( x) = x2 , x Î R . For any A Í R , define g(A) =
f (n) = í is
ï - n , when n is even {x Î R : f ( x) Î A} . If S = [0, 4], then which one of the
ïî 2
following statements is not true ? [April 10, 2019 (I)]
(a) neither one -one nor onto (a) g (f (S)) ¹ S (b) f (g (S)) = S
(b) one-one but not onto (c) g (f (S)) = g (S) (d) f (g (S)) ¹ f (S)
(c) onto but not one-one 1
30. For xÎ R – {0, 1}, let f1 (x) = , f (x) = 1 – x and
(d) one-one and onto both. x 2
1
f3 (x) = be three given functions. If a function, J(x)
Composite Functions & Relations, 1- x
TOPIC n Inverse of a Function, Binary satisfies (f2oJof1) (x) = f3(x) then J(x) is equal to:
Operations
[Jan. 09, 2019 (I)]
1
-2 x (a) f3 (x) (b) f (x) (c) f2 (x) (d) f1 (x)
8 -8
2x
x 3
25. The inverse function of f ( x) = , x Î(-1,1), is
8 2 x + 8 -2 x 31. Let N denote the set of all natural numbers. Define two
________. [Jan. 8, 2020 (I)] binary relations on N as R1 = {(x, y) Î N × N : 2x + y = 10}
and R2 = {(x, y) Î N × N : x + 2y = 10}. Then
1 æ1+ x ö 1 æ 1- x ö [Online April 16, 2018]
loge ç (log8 e)log e ç
(a)
4 è 1 - x ÷ø (b)
4 è 1 + x ÷ø (a) Both R1 and R2 are transitive relations
(b) Both R1 and R2 are symmetric relations
1 æ1- x ö 1 æ 1+ x ö (c) Range of R2 is {1, 2, 3, 4}
loge ç (log8 e)log e ç
(c)
4 è 1 + x ÷ø (d)
4 è 1 - x ÷ø (d) Range of R1 is {2, 4, 8}
32. Consider the following two binary relations on the set
A = {a, b, c} : R1 = {(c, a) (b, b) , (a, c), (c, c), (b, c), (a, a)}
æ 5ö and R2 = {(a, b), (b, a), (c, c), (c, a), (a, a), (b, b), (a, c). Then
26. If g(x) = x2 + x – 1 and (gof) (x) = 4x2 – 10x + 5, then f çè ÷ø [Online April 15, 2018]
4
(a) R2 is symmetric but it is not transitive
is equal to: [Jan. 7, 2020 (I)]
(b) Both R1 and R2 are transitive
3 1 1 3 (c) Both R1 and R2 are not symmetric
(a) (b) - (c) (d) - (d) R1 is not symmetric but it is transitive
2 2 2 2
27. For a suitably chosen real constant a, let a function, x -1
33. Let f : A ® B be a function defined as f (x) = , where
x-2
a-x
f : R –{– a}® R be defined by f (x) = . Further sup- A = R – {2} and B = R – {1}. Then f is
a+x
[Online April 15, 2018]
pose that for any real number x ¹ – a and f (x) ¹ – a,
2y +1
(a) invertible and f –1 (y) =
æ 1ö y -1
( fof ) (x) = x. Then f ç - ÷ is equal to:
è 2ø 3y - 1
(b) invertible and f –1 (y) =
[Sep. 06, 2020 (II)] y -1
(c) no invertible
1 1
(a) (b) - (c) – 3 (d) 3 2y -1
3 3 (d) invertible and f –1 (y) =
y -1

Downloaded from @Freebooksforjeeneet


EBD_8344
M-254 Mathematics

34. Let f (x) = 210·x + 1 and g(x) = 310·x – 1. If (fog)(x)=x, then 38. Let f be a function defined by
x is equal to : [Online April 8, 2017]
f ( x) = ( x -1) +1, ( x ³1) .
2
[2011RS]
10
3 -1 2 10
-1
{ }
(a) (b)
310 - 2 -10 210 - 3-10 Statement - 1 : The set x : f ( x ) = f ( x ) = {1, 2} .
-1

-10 -10
1- 3 1- 2 Statement - 2 : f is a biection and
(c) -10 (d)
10
-3 3 - 2 -10
10
f -1 ( x ) = 1 + x - 1, x ³ 1.
2
1
35. For x Î R, x ¹ 0 , let f0(x) = and fn+1 (x) = f0(fn(x)), (a) Statement-1 is true, Statement-2 is true; Statement-2 is
1- x a correct explanation for Statement-1.
æ 2ö æ 3ö (b) Statement-1 is true, Statement-2 is true; Statement-2 is
n = 0, 1, 2, .... Then the value of f100(3) + f1 çè ÷ø + f 2 çè ÷ø is NOT a correct explanation for Statement-1.
3 2
equal to : [Online April 9, 2016] (c) Statement-1 is true, Statement-2 is false.
8 4 5 1 (d) Statement-1 is false, Statement-2 is true.
(a) (b) (c) (d)
3 3 3 3 39. Let f(x) = ( x + 1)2 –1, x ³ –1
1
36. If g is the inverse of a function f and f ' ( x ) = , then Statement -1 : The set {x : f(x) = f –1(x) = {0, –1}
1+ x 5
Statement-2 : f is a biection. [2009]
g ¢ ( x ) is equal to: [2014] (a) Statement-1 is true, Statement-2 is true. Statement-2 is
not a correct explanation for Statement-1.
1
(b) 1 + { g ( x )}
5 (b) Statement-1 is true, Statement-2 is false.
(a)
1 + { g ( x )}
5
(c) Statement-1 is false, Statement-2 is true.
(c) 1 + x5 (d) 5x4 (d) Statement-1 is true, Statement-2 is true. Statement-2 is
37. Let A and B be non empty sets in R and f : A ® B is a a correct explanation for Statement-1.
biective function. [Online May 26, 2012] 40. Let f: N®Y be a function defined as f(x) = 4x + 3 where
Statement 1: f is an onto function. Y = {y Î N : y = 4x + 3 for some x Î N}.
Statement 2: There exists a function g : B ® A such that Show that f is invertible and its inverse is [2008]
fog = IB.
3y + 4 y+3
(a) Statement 1 is true, Statement 2 is false. (a) g ( y ) = (b) g ( y ) = 4 +
(b) Statement 1 is true, Statement 2 is true; Statement 2 is 3 4
a correct explanation for Statement 1. y+3 y –3
(c) Statement 1 is false, Statement 2 is true. (c) g ( y ) = (d) g ( y ) =
4 4
(d) Statement 1 is true, Statement 2 is true, Statement 2 is
not the correct explanation for Statement 1.

Downloaded from @Freebooksforjeeneet


Inverse Trigonometric Functions M-261

17
Inverse Trigonometric
Functions
Trigometric Functions & Their 6. The largest interval lying in æç -p , p ö÷ for which the
Inverses, Domain & Range of Inverse è 2 2ø
Trigonometric Functions, Principal 2 æx ö
TOPIC Ć Value of Inverse Trigonometric function, f ( x ) = 4- x + cos -1 ç - 1÷ + log (cos x) , is defined, is
è2 ø
Functions, Intervals for Inverse
Trigonometric Functions [2007]

p
(a) é - p , p ö (b) êé 0, ö÷
æ 3ö æ1ö p ê 4 2÷ ë 2ø
1. If a = cos–1 ç ÷ , b = tan–1 ç 3 ÷ , where 0 < a, b < , then ë ø
è5ø è ø 2
p p
a – b is equal to : [April 8, 2019 (I)] (c) [0, p] (d) æç - , ö÷
è 2 2ø
æ 9 ö æ 9 ö
(a) tan–1 ç ÷ (b) cos–1 ç ÷
è 5 10 ø è 5 10 ø sin -1 ( x - 3)
7. The domain of the function f ( x) = is
9 - x2
æ 9ö æ 9 ö
(c) tan–1 ç ÷ (d) sin–1 ç ÷ (a) [1, 2] (b) [2, 3) [2004]
è 14 ø è 5 10 ø
2. A value of x satisfying the equation sin[cot–1(1 + x) ] = cos (c) [1, 2 ] (d) [2, 3]
[tan–1x], is : [Online April 9, 2017] 8. The trigonometric equation sin -1 x = 2 sin -1 a has a
1 1 solution for [2003]
(a) - (b) –1 (c) 0 (d)
2 2 1 1 1
(a) a £ (b) < a <
æ 43p ö 2 2 2
3. The principal value of tan -1 ç cot ÷ is:
è 4 ø 1
[Online April 19, 2014] (c) all real values of a (d) a <
2
3p 3p p p
(a) - (b) (c) - (d) 9. cot -1 ( cos a ) - tan -1 ( cos a ) = x ,
4 4 4 4
4. The number of solutions of the equation, then sin x =
sin–1 x = 2 tan –1x (in principal values) is : æaö a
[Online April 22, 2013] (a) tan2 ç ÷ (b) cot2 çæ ÷ö [2002]
è2ø è2ø
(a) 1 (b) 4 (c) 2 (d) 3
æaö
-1
æ æ æ 2 ööö (c) tan a (d) cot ç 2 ÷
ç sin ç cos -1 ç ÷ is è ø
ç 3 ÷÷ ÷÷ ÷
5. A value of tan
ç ç è øøø
è è 10. The domain of sin-1 [log3 (x/3)] is [2002]
[Online May 19, 2012] (a) [1, 9] (b) [–1, 9]
p p p p
(a) (b) (c) (d) (c) [–9, 1] (d) [–9, –1]
4 2 3 6

Downloaded from @Freebooksforjeeneet


EBD_8344
M-262 Mathematics

Properties of Inverse Trigonometric (b) ( cot 2, ¥ )


TOPIC n Functions, Infinite Series of Inverse
Trigonometric Functions (c) ( -¥, cot 5) È ( cot 2, ¥ )
(d) (cot 5, cot 4)
æ 4 5 16 ö
11. 2p - ç sin -1 + sin -1 + sin -1 ÷ is equal to :
è 5 13 65 ø æ 19 æ n öö
17. The value of cot ç å cot -1 ç1 + å 2p ÷ ÷ is:
[Sep. 03, 2020 (I)] ç n =1 ç p =1 ÷÷
è è øø
p 5p 3p 7p [Jan. 10, 2019 (II)]
(a) (b) (c) (d)
2 4 2 4
21 19
12. If S is the sum of the first 10 terms of the series (a) (b)
19 21
æ1ö æ1ö æ1ö æ 1ö
tan -1 ç ÷ + tan -1 ç ÷ + tan -1 ç ÷ + tan -1 ç ÷ + . . . . , 22 23
è 3ø è7ø è 13 ø è 21 ø (c)
23
(d)
22
then tan (S) is equal to: [Sep. 05, 2020 (I)]
18. If x = sin –1 (sin10) and y = cos–1 (cos10), then y – x is equal
5 5 to: [Jan. 09, 2019 (II)]
(a) (b)
6 11 (a) 0 (b) 10 (c) 7 p (d) p

6 -1 æ 2 ö –1 æ 3 ö pæ 3ö
If cos ç ÷ + cos ç ÷ = ç x > ÷ ,then x is equal
10
(c) - (d) 19.
5 11 è 3x ø è 4x ø 2 è 4ø
to: [Jan. 09, 2019 (I)]
-1 æ 12 ö -1 æ 3 ö
13. The value of sin ç ÷ - sin ç ÷ is equal to :
è 13 ø è 5ø 145 145 146 145
(a) (b) (c) (d)
[April 12, 2019 (I)] 12 10 12 11

p é 1 + x2 + 1 - x 2 ù
-1 æ 63 ö æ 56 ö 1
(a) p - sin ç ÷ (b) - sin -1 ç ÷ 20. The value of tan–1 ê ú , | x |< , x ¹ 0 ,
è 65 ø 2 è 65 ø ê 1 + x 2 - 1 - x2 ú 2
ë û
p æ 9 ö -1 æ 33 ö
(c) - cos -1 ç ÷ (d) p - cos ç ÷ is equal to [Online April 8, 2017]
2 è 65 ø è 65 ø
p 1 p
(a) + cos -1 x 2 (b) + cos -1 x 2
-1 -1 y 4 2 4
14. If cos x - cos = a , where – 1 < x < 1, –2 < y < 2,
2
p 1 p
(c) - cos -1 x 2 (d) - cos -1 x 2
y 4 2 4
x < , then for all x, y, 4x2 –4xy cosa + y2 is equal to:
2 21. Let
æ 2x ö
[April 10, 2019 (II)] tan –1 y = tan –1 x + tan –1 ç
(a) 4 sin a
2
(b) 2 sin a
2 è 1 - x 2 ÷ø ,
(c) 4 sin2a – 2x2y2 (d) 4 cos2 a + 2x2y2 1
where or x < . Then a value of y is : [2015]
15. Considering only the principal values of inverse functions, 3
ì -1 -1 pü 3x - x 3 3x + x 3
the set A = í x ³ 0 : tan ( 2 x ) + tan ( 3x ) = ý (a) (b)
î 4þ 1 + 3x 2 1 + 3x 2
[Jan. 12, 2019 (I)]
3x – x 3 3x + x 3
(a) contains two elements (c) (d)
1 – 3x 2 1 - 3x 2
(b) contains more than two elements
æ 2x ö
2 tan -1 x + sin -1 ç
(c) is a singleton 22. If f (x) = ÷ , x > 1 th en
(d) is an empty set è 1+ x 2 ø
16. All x satisfying the inequality (cot–1x)2 – 7 (cot–1x) + 10 > f (5) is equal to : [Online April 10, 2015]
0, lie in the interval : [Jan. 11, 2019 (II)] p
(a) tan -1 æç 65 ö÷ (b)
(a) ( -¥, cot 5) È ( cot 4, cot 2 ) è 156 ø 2
(c) p (d) 4 tan–1(5)

Downloaded from @Freebooksforjeeneet


Inverse Trigonometric Functions M-263

23. Statement I: The equation (sin–1x)3 + (cos–1 x)3 – ap3 = 0 æ 1 ö -1 æ 1 ö


26. S = tan -1 ç ÷ + tan ç 2 ÷ + ...
1 è n + n +1 ø
2
è n + 3n + 3 ø
has a solution for all a ³ .
32
æ 1 ö
p
Statement II: For any x Î R , sin -1 x + cos-1 x = and + tan -1 ç ÷ , then tan S is equal to :
2 è 1 + ( n + 19)( n + 20) ø
[Online April 23, 2013]
2
æ pö 9 p2
0 £ ç sin -1 x - ÷ £ [Online April 12, 2014] (a)
20
(b)
n
è 4ø 16 401 + 20 n 2
n + 20n + 1
(a) Both statements I and II are true. 20 n
(b) Both statements I and II are false. (c) (d)
2
n + 20n + 1 401 + 20 n
(c) Statement I is true and statement II is false. 27. A value of x for which sin (cot–1(1 + x)) = cos (tan–1 x), is
(d) Statement I is false and statement II is true. : [Online April 9, 2013]
24. If x, y, z are in A.P. and tan –1x, tan–1y and tan–1z are also in
1 1
A.P., then [2013] (a) - (b) 1 (c) 0 (d)
2 2
(a) x = y = z (b) 2x = 3y = 6z
æ xö æ 5ö p
(c) 6x = 3y = 2z (d) 6x = 4y = 3z 28. If sin -1 ç ÷ + cosec -1 ç ÷ = , then the values of x is
è 5ø è 4ø 2
25. Let x Î (0, 1). The set of all x such that (a) 4 (b) 5 [2007]
sin–1x > cos–1x, is the interval: [Online April 25, 2013] (c) 1 (d) 3
æ1 1 ö æ 1 ö 29.
y
If cos -1 x - cos -1 = a , then 4 x 2 - 4 xy cos a + y 2 is
(a) ç , ÷ (b) ç ,1÷ 2
è2 2ø è 2 ø
equal to [2005]
æ 3ö (a) 2 sin 2 a (b) 4
(c) (0, 1) (d) çç 0, 2 ÷÷ (c) 4 sin 2 a (d) – 4 sin 2 a
è ø

Downloaded from @Freebooksforjeeneet


Matrices M-269

18
Matrices

Order of Matrices, Types of Matrices, é1 0 0ù


TOPIC Ć
Addition & Subtraction of Matrices, 5. Let P = êê3 1 0úú and Q = [qi] be two 3 × 3 matrices
Scalar Multiplication of Matrices,
Multiplication of Matrices êë9 3 1úû

q 21 + q31
1. Let a be a root of the equation x2 + x + 1 = 0 such that Q – P5 = I3. Then is equal to :
q32
[Jan. 12, 2019 (I)]
é1 1 1ù
(a) 10 (b) 135 (c) 15 (d) 9
1 ê ú
and the matrix A = ê1 a a 2 ú , then the matrix A31
3 ê ú é1 0 0 ù
ë1 a a4 û
2
6. Let A = êê1 1 0 úú and B = A20. Then the sum of the
is equal to: [Jan. 7, 2020 (I)] êë1 1 1 úû
elements of the first column of B is?
(a) A (b) I3 (c) A2 (d) A3 [Online April 16, 2018]
(a) 211 (b) 210 (c) 231 (d) 251
é cos q i sin q ù æ p ö éa b ù
2. If A = ê ú , çq = ÷ and A5 = ê ú, é0 -1ù
ëi sin q cos q û è 24 ø ëc d û 7. If A=ê ú , then which one of the following
ë1 0 û
where i = -1, then which one of the following is not statements is not correct? [Online April 10, 2015]
true? [Sep. 04, 2020 (I)] 2 2
(a) A + I = A(A – I) (b) A4 – I = A2 + I
3 3
(c) A + I = A(A – I) (d) A3 – I = A(A – I)
(a) 0 £ a 2 + b2 £ 1 (b) a 2 - d 2 = 0
é yù
ê x ú be such that AB = é ù ,
é1 2 x ù 6
1 8. If A = ê ú and B = ê ú
(c) a 2 - c 2 = 1 (d) a 2 - b 2 = ê ú
2 ë3 - 1 2 û êë 1 úû
ë8 û

then: [Online April 12, 2014]


é x 1ù
3. Let A = ê ú , x Î R and A = [aij]. If a11 = 109, then a22
4
(a) y = 2x (b) y = – 2x
ë 1 0û
(c) y = x (d) y = – x
is equal to __________. [NA Sep. 03, 2020 (I)] 9. If p, q, r are 3 real numbers satisfying the matrix equation,

écos a - sin a ù é0 -1ù é3 4 1ù


4. Let A = ê ú , (aÎ R) such that A32
= ê1 0 ú . [ p q r ] êê 3 2 3úú = [3 0 1] then
ësin a cos a û ë û
Then a value of a is : [April 8, 2019 (I)] ëê 2 0 2úû
2p + q – r equals : [Online April 22, 2013]
p p p
(a) (b) 0 (c) (d) (a) – 3 (b) – 1
32 64 16 (c) 4 (d) 2

Downloaded from @Freebooksforjeeneet


EBD_8344
M-270 Mathematics

10. The matrix A2 + 4A – 5I, where I is identity matrix and


éa b ù éa b ù
17. If A = ê ú and A2 = ê ú , then [2003]
é1 2 ù ëb a û ëb aû
A= ê ú , equals [Online April 9, 2013]
ë 4 -3û
(a) a = 2 ab, b = a 2 + b 2
é2 1ù é 0 -1ù (b) a = a 2 + b 2 , b = ab
(a) 4 ê ú (b) 4 ê ú
ë2 0û ë2 2 û
(c) a = a 2 + b 2 , b = 2ab
é2 1ù é1 1 ù (d) a = a 2 + b 2 , b = a 2 - b 2 .
(c) 32 ê ú (d) 32 ê ú
ë2 0û ë1 0 û
Transpose of Matrices, Symmetric
é 1 0 0ù é 1 0 0ù & Skew Symmetric Matrices,
11. If A = ê 2 1 0 ú and B = ê -2 1 0ú then AB TOPIC n Inverse of a Matrix by Elementary
ê ú ê ú
êë -3 2 1 úû êë 7 -2 1úû Row Operations

equals [Online May 26, 2012] 18. Let a, b, c Î R be all non- ero and satisfy
(a) I (b) A (c) B (d) 0
12. If w ¹ 1is the complex cube root of unity and matrix æa b cö
ç ÷
éω 0 ù a + b + c = 2. If the matrix A = ç b c a ÷ satisfies
3 3 3
H = ê ú , then H70 is equal to [2011RS] çc a b÷
ë 0 ωû è ø
(a) 0 (b) –H (c) H2 (d) H ATA = I, then a value of abc can be : [Sep. 02, 2020 (II)]
13. The number of 3 × 3 non-singular matrices, with four entries
1 1 2
as 1 and all other entries as 0, is [2010] (a) - (b) (c) 3 (d)
(a) 5 (b) 6 3 3 3
(c) at least 7 (d) less than 4 19. The number of all 3 ´ 3 matrices A, with enteries from the
æ 1 2ö æ a 0ö
14. Let A = ç ÷ and B = ç , a, b Î N. Then [2006] set {–1, 0, 1} such that the sum of the diagonal elements
è 3 4ø è 0 b÷ø
(a) there cannot exist any B such that AB = BA of AAT is 3, is _______. [NA Jan. 8, 2020 (I)]
(b) there exist more than one but finite number of B¢s
such that AB = BA æ 2 2ö æ 1 0ö
20. If A = ç 9 4÷ and I = ç 0 1÷ , then 10A–1 is equal to:
(c) there exists exactly one B such that AB = BA è ø è ø
(d) there exist infinitely many B¢s such that AB = BA [Jan. 8, 2020 (II)]
15. If A and B are square matrices of si e n × n such that
(a) A – 4I (b) 6I – A (c) A – 6I (d) 4I – A
A2 - B 2 = ( A - B)( A + B) , then which of the following
will be always true? [2006] 21. If A is a symmetric matrix and B is a skew-symmetrix matrix
(a) A = B é2 3 ù
(b) AB = BA such that A + B = ê 5 -1ú , then AB is equal to :
ë û
(c) either of A or B is a ero matrix [April 12, 2019 (I)]
(d) either of A or B is identity matrix
é - 4 -1ù é 4 -2 ù
é1 0ù é1 0 ù (a) ê -1 4 ú (b) ê -1 - 4ú
16. If A = ê ú and I = ê ú , then which one of the ë û ë û
ë1 1 û ë0 1û
following holds for all n ³ 1, by the principle of é 4 -2 ù é - 4 2ù
(c) ê1 - 4ú (d) ê 1 4ú
mathematical induction [2005] ë û ë û
(a) An = nA – (n – 1) I æ 0 2y 1 ö
n n -1 A = ç 2 x y -1÷
(b) A = 2 A – (n – 1) I 22. The total number of matrices ç 2 x - y 1 ÷ , (x, y Î
n è ø
(c) A = nA + (n – 1) I
R, x ¹ y) for which ATA = 3I3 is: [April 09, 2019 (II)]
(d) A = 2 n - 1 A + (n – 1) I
n
(a) 2 (b) 3 (c) 6 (d) 4

Downloaded from @Freebooksforjeeneet


Matrices M-271

æ 0 2q r ö é1 2 2 ù
23. Let A = ç p q -r ÷ . If AAT = I3, then |p| is : 26. If A = ê 2 1 -2ú is a matrix satisfying the equation
ç ÷ ê ú
ç p -q r ÷ êë a 2 b úû
è ø
[Jan. 11, 2019 (I)] AAT = 9I, where I is 3 × 3 identity matrix, then the ordered
pair (a, b) is equal to: [2015]
1 1 (a) (2, 1) (b) (–2, – 1)
(a) (b)
5 3 (c) (2, – 1) (d) (–2, 1)
27. Let A and B be any two 3 × 3 matrices. If A is symmetric
1 1 and B is skewsymmetric, then the matrix AB – BA is:
(c) (d)
2 6 (a) skewsymmetric [Online April 19, 2014]
(b) symmetric
24. For two 3 × 3 matrices A and B, let A + B = 2BT and 3A + 2B =
I3, where BT is the transpose of B and I3 is 3 × 3 identity (c) neither symmetric nor skewsymmetric
matrix. Then : [Online April 9, 2017] (d) I or – I, where I is an identity matrix.
(a) 5A + 10B = 2I3 (b) 10A + 5B = 3I3 æ a - 1ö æ a + 1ö
(c) B + 2A = I3 (d) 3A + 6B = 2I3 28. If A = 0 , B = ç 0 ÷ be two matrices, then ABT is a
ç ÷
ç ÷ ç ÷
è 0 ø è 0 ø
é 3 1 ù non- ero matrix for |a| not equal to [Online May 7, 2012]
ê ú
ê 2 2 ú é1 1ù (a) 2 (b) 0 (c) 1 (d) 3
25. If P = ê 1 3 ú , A = êë 0 1úû and Q = PAP
APT, then 29. Let A and B be two symmetric matrices of order 3. [2011]
ê- ú
ë 2 2 û Statement-1: A(BA) and (AB)A are symmetric matrices.
Statement-2: AB is symmetric matrix if matrix multiplication
PT Q2015 P is ; [Online April 9, 2016] of A with B is commutative.
é 0 2015ù é 2015 0 ù (a) Statement-1 is true, Statement-2 is true; Statement-2 is
(a) ê 0 0 úû (b) ê 1 2015úû
not a correct explanation for Statement-1.
ë ë
(b) Statement-1 is true, Statement-2 is false.
(c) Statement-1 is false, Statement-2 is true.
é1 2015ù é 2015 1 ù
(c) ê 0 1 úû (d) ê 0 2015úû
(d) Statement-1 is true, Statement-2 is true; Statement-2 is
ë ë a correct explanation for Statement-1.

Downloaded from @Freebooksforjeeneet


Determinants M-277

19
Determinants
Minor & Co-factor of an Element of a 5. The sum of the real roots of the equation
Determinant, Value of a Determinant,
TOPIC Ć Property of Determinant of Matrices, x -6 -1
Singular & Non-Singular Matrices, 2 -3x x - 3 = 0, is equal to : [April 10, 2019 (II)]
Multiplication of two Determinants -3 2 x x+2
(a) 6 (b) 0 (c) 1 (d) –4
p cos q sin q ù
1. Let q = and A = é - sin q 4
cos qúû . If B = A + A ,
5 êë
é2 b 1ù
then det (B): [Sep. 06, 2020 (II)] ê ú
(a) is one (b) lies in (2, 3) 6. Let A = ê b b2 + 1 b ú where b > 0. Then the minimum
(c) is ero (d) lies in (1, 2) ê1 b 2úû
ë
x - 2 2x - 3 3x - 4
2. If D = 2 x - 3 3x - 4 4 x - 5 = Ax3 + Bx2 + Cx + D, det (A)
value of is: [Jan. 10, 2019 (II)]
3x - 5 5x - 8 10 x - 17 b
then B + C is equal to : [Sep. 03, 2020 (I)]
(a) 2 3 (n) - 2 3 (c) - 3 (d) 3
(a) –1 (b) 1 (c) –3 (d) 9
3. Let a – 2b + c = 1. x - 4 2x 2x
x+a x + 2 x +1 7. If 2x x - 4 2x = (A + Bx)(x - A) 2 , then the
If f(x) = x + b x + 3 x + 2 , then : [Jan. 9, 2020 (II)] 2x 2x x -4
x+c x+ 4 x+3
ordered pair (A, B) is equal to : [2018]
(a) f (–50) = 501 (b) f (–50) = –1 (a) (– 4, 3) (b) (– 4, 5)
(c) (4, 5) (d) (– 4, – 5)
(c) f (50) = –501 (d) f (50) = 1
x sin q cos q ì 0 cos x - sin x ü
ï ï
4. If D1 = - sin q -x 1 and 8. If S = í x Î [ 0, 2p] : sin x 0 cos x = 0 ý , then
cos q 1 x ï cos x sin x 0 ï
î þ
x sin 2q cos 2q
æp ö
D 2 = - sin 2q -x 1 ,x ¹ 0 ; å tan çè 3 + x ÷ø is equal to [Online April 8, 2017]
cos 2q 1 x xÎS
æ pö (a) 4 + 2 3 (b) -2 + 3
then for all qÎ ç 0, ÷ : [April 10, 2019 (I)]
è 2ø (c) -2 - 3 (d) -4 - 2 3
(a) D1 - D 2 = -2x3
é -4 -1ù
(b) D1 - D 2 = x (cos 2q - cos 4q) 9. If A = ê3 1 ú , then the determinant of the matrix
ë û
(c) D1 ´ D 2 = - 2( x3 + x - 1)
(A2016 – 2A2015 – A2014) is : [Online April 10, 2016]
(d) D1 + D2 = -2x3 (a) –175 (b) 2014 (c) 2016 (d) –25

Downloaded from @Freebooksforjeeneet


EBD_8344
M-278 Mathematics

16. Statement - 1: [2011RS]


x2 + x x +1 x–2 Determinant of a skew-symmetric matrix of order 3 is ero.
if 2 x + 3x –1
2 Statement - 2 :
10. 3x 3 x – 3 = ax –12, then ‘a’ is
x2 + 2 x + 3 2 x –1 2 x –1 For any matrix A, det (A)T= det (A) and det (– A) = – det (A).
Where det (B) denotes the determinant of matrix B. Then :
equal to : [Online April 11, 2015] (a) Both statements are true
(a) 24 (b) –12 (c) –24 (d) 12 (b) Both statements are false
11. The least value of the product xyz for which the (c) Statement-1 is false and statement-2 is true
x 1 1 (d) Statement-1 is true and statement-2 is false
1 y 1 17. Let A be a 2 × 2 matrix with non- ero entries and let A2 = I ,
determinant is non-negative, is : where I is 2 × 2 identity matrix. Define
1 1
Tr(A) = sum of diagonal elements of A and
[Online April 10, 2015]
|A| = determinant of matrix A.
(a) -2 2 (b) –1 Statement - 1 : Tr(A) = 0.
(c) -16 2 (d) –8 Statement -2 : |A| = 1. [2010]
(a) Statement-1 is true, Statement-2 is true ; Statement-2
1 cos q 1 is not a correct explanation for Statement -1.
12. If f(q) = - sin q 1 - cos q and (b) Statement -1 is true, Statement -2 is false.
-1 sin q 1 (c) Statement -1 is false, Statement -2 is true .
(d) Statement - 1 is true, Statement 2 is true ; Statement -2
A and B are respectively the maximum and the minimum is a correct explanation for Statement -1.
values of f(q), then (A, B) is equal to: 18. Let A be a 2 × 2 matrix with real entries. Let I be the 2 × 2
[Online April 12, 2014] identity matrix. Denote by tr(A), the sum of diagonal entries
of a. Assume that A2 = I.
( 4, 2 - 2 )
[2008]
(a) (3, – 1) (b)
Statement-1 : If A ¹ I and A ¹ –I, then det (A) = –1
Statement-2 : If A ¹ I and A ¹ –I, then tr (A) ¹ 0.
(c) (2 + 2, 2 - 2 ) (d) (2 + 2, -1 ) (a) Statement -1 is false, Statement-2 is true
13. If B is a 3 × 3 matrix such that B 2 = 0, then (b) Statement -1 is true, Statement-2 is true; Statement -2
det. [(I + B)50 – 50B] is equal to: [Online April 9, 2014] is a correct explanation for Statement-1
(a) 1 (b) 2 (c) 3 (d) 50 (c) Statement -1 is true, Statement-2 is true; Statement -2
is not a correct explanation for Statement-1
ìæ a11 a12 ö ü
14. Let S = íç ÷ : aij Î{0,1, 2}, a11 = a22 ý (d) Statement -1 is true, Statement-2 is false
îè a21 a22 ø þ
5 5a a
Then the number of non-singular matrices in the set S is : 19.
2
Let A = 0 a 5a . If A = 25 , then a equals [2007]
[Online April 25, 2013] 0 0 5
(a) 27 (b) 24
(a) 1/5 (b) 5
(c) 10 (d) 20
(c) 52 (d) 1
15. Let A, other than I or – I, be a 2 × 2 real matrix such that
A2 = I, I being the unit matrix. Let Tr (A) be the sum of 20. If 1, w, w 2 are the cube roots of unity, then
diagonal elements of A. [Online April 23, 2013]
Statement-1: Tr (A) = 0 1 wn w 2n
Statement-2: det (A) = – 1 D = wn w2n 1 is equal to [2003]
(a) Statement-1 is true; Statement-2 is false. 2n n
w 1 w
(b) Statement-1 is true; Statement-2 is true; Statement-2
is not a correct explanation for Statement-1.
(c) Statement-1 is true; Statement-2 is true; Statement-2 (a) w 2 (b) 0
is a correct explanation for Statement-1. (c) 1 (d) w
(d) Statement-1 is false; Statement-2 is true.

Downloaded from @Freebooksforjeeneet


Determinants M-279

27. Let the numbers 2, b, c be in an A.P. and


Properties of Determinants,
TOPIC n Area of a Triangle é1 1 1ù
ê2 b cú
21. If the minimum and the maximum values of the function A= ê 2 ú . If det(A)?[2, 16], then c lies in the
ë4 b c2 û
ép pù
f : ê , ú ® R , defined by interval : [April 08, 2019 (II)]
ë4 2û
(a) [2, 3) (b) (2 + 23/4, 4)
- sin 2 q -1 - sin 2 q 1 (c) [4, 6] (d) [3, 2 + 23/4]
f (q) = - cos 2 q -1 - cos 2 q 1 are m and M respec-
12 10 -2 é 1 sin q 1 ù
ê ú æ 3p 5p ö
tively, then the ordered pair (m, M) is equal to : 28. If A = - sin q
ê 1 sin qú ; then for all qÎ ç , ÷,
è 4 4 ø
[Sep. 05, 2020 (I)] êë -1 - sin q 1 úû
(a) (0, 2 2 ) (b) (– 4, 0 )
(c) (– 4, 4) (d) (0, 4) det (A) lies in the interval : [Jan. 12, 2019 (II)]
22. If a + x = b + y = c + z + 1, where a, b, c, x, y, z are non- ero
æ 5ù é5 ö æ 3ù æ3 ù
x a+ y x+a (a) ç 1, ú (b) ê , 4 ÷ (c) ç 0, ú (d) ç ,3ú
è 2û ë2 ø è 2û è2 û
distinct real numbers, then y b + y y + b is equal to :
z c+ y z+c a–b–c 2a 2a
[Sep. 05, 2020 (II)] 2b b–c–a 2b
29. If
(a) y (b – a) (b) y (a – b) 2c 2c c–a–b
(c) 0 (d) y (a – c)
23. Let two points be A(l, – 1) and B(0, 2). If a point P(x¢, y¢) be = (a + b + c) (x + a + b + c)2, x ¹ 0 and a + b + c ¹ 0, then x
is equal to : [Jan. 11, 2019 (II)]
such that the area of DPAB = 5 sq. units and it lies on the
(a) abc (b) – (a + b +c)
line, 3x + y – 4l = 0, then a value of l is: [Jan. 8, 2020 (I)]
(c) 2 (a + b +c) (d) – 2 (a + b +c)
(a) 4 (b) 3 (c) 1 (d) –3 30. Let dÎR, and
24. Let A = [aij] and B = [bij] be two 3 ´ 3 real matrices such that
é- 2 4+d (sin q) -2 ù
bij = (3)(i + j – 2) aij, where i, j = 1, 2, 3. If the determinant of B ê ú
1 (sin q) + 2
A= ê ú,
d
is 81, then the determinant of A is: [Jan. 7, 2020 (II)]
ê 5 (2 sin q) - d (- sin q) + 2 + 2d ú
(a) 1/3 (b) 3 (c) 1/81 (d) 1/9 ë û
25. A value of q Î (0, p/3), for which q Î [0, 2p]. If the minimum value of det (A) is 8, then a
value of d is: [Jan 10, 2019 (I)]
1 + cos 2 q sin 2 q 4 cos 6q
(a) – 5 (b) – 7
cos q 1 + sin q
2 2
4 cos 6q
= 0, is :
cos 2 q sin 2 q 1 + 4cos 6q (c) 2 ( 2 +1 ) (d) 2 ( 2 +2 )
[April 12, 2019 (II)] 31. Let a1, a2, a3, ..., a10 be in G.P. with ai > 0 for i = 1, 2, ..., 10 and
p p 7p 7p S be the set of pairs (r, k), r, kÎN (the set of natural num-
(a) (b) (c) (d) bers) for which
9 18 24 36
26. Let a and b be the roots of the equation x2 + x + 1 = 0. Then
log e a1r a k2 log e a 2r a 3k log e a 3r a k4
y +1 a b log e a r4 a 5k log e a 5r a k6 log e a 6r a 7k = 0
a y+b 1 log e a r7 a 8k log e a 8r a 9k log e a 9r a10
k
for y �벜 0 in R, is equal to:
b 1 y+a
Then the number of elements in S, is : [Jan. 10, 2019 (II)]
[April 09, 2019 (I)]
(a) 4 (b) infinitely many
(a) y(y2 – 1) 2
(b) y(y – 3)
(c) 2 (d) 10
(c) y3 (d) y3 – 1

Downloaded from @Freebooksforjeeneet


EBD_8344
M-280 Mathematics

32. If (a) depends only on a


(b) depends only on n
éet e - t cos t e - t sin t ù
ê t -t -t -t -t
ú (c) depends both on a and n
A = êe -e cos t - e sin t -e sin t + e cos t ú (d) is independent of both a and n
ê t ú
ëê e 2e- t sin t -2e- t cos t ûú
38. If

then A is: [Jan. 09, 2019 (II)] a2 b2 c2 a2 b2 c2


(a) invertible for all tÎR.
( a + l )2 ( b + l)2 ( c + l )2 = kl a b c ,l ¹ 0
(b) invertible only if t = p.
(c) not invertible for any tÎR. ( a - l )2 ( b - l)2 ( c - l )2 1 1 1

p then k is equal to: [Online April 12, 2014]


(d) invertible only if t = . (a) 4labc (b) – 4labc (c) 4l2 (d) – 4l2
2
39. If a, b, c are sides of a scalene triangle, then the value of
33. Let k be an integer such that triangle with vertices
(k, –3k), (5, k) and (–k, 2) has area 28 sq. units. Then the a b c
orthocentre of this triangle is at the point : [2017] b c a is : [Online April 9, 2013]
æ 1ö æ 1ö æ 3ö æ 3ö c a b
(a) ç 2, ÷ (b) ç 2, - ÷ (c) ç1, ÷ (d) ç1, - ÷
è 2ø è 2ø è 4ø è 4ø
(a) non - negative (b) negative
34. Let w be a complex number such that 2w + 1 = where =
(c) positive (d) non-positive
1 1 1 40. If a, b, c, are non ero complex numbers satisfying
2 2 a2 + b2 + c2 = 0 and
-3 . If 1 -w - 1 w = 3k, then k is equal to :
1 w2 w7 b2 + c 2 ab ac
[2017] 2 2
(a) 1 (b) – (c) (d) –1 ab c +a bc = ka2b2c2, then k is equal to
35. The number of distinct real roots of the equaiton, ac bc a 2 + b2
cos x sin x sin x
[Online May 19, 2012]
é p pù
= 0 in the interval ê - , ú is :
sin x cos x sin x (a) 1 (b) 3 (c) 4 (d) 2
sin x sin x cos x ë 4 4û
-2a a+b a+c
If b + a -2b b + c = a ( a + b)( b + c)( c + a ) ¹ 0
[Online April 9, 2016]
41.
(a) 1 (b) 4 (c) 2 (d) 3
c + a b + c -2c
36. If a, b ¹ 0, and f ( n ) = a n + b n and
then a is equal to [Online May 12, 2012]
3 1 + f (1) 1 + f ( 2 ) (a) a + b + c (b) abc
1 + f (1) 1 + f ( 2 ) 1 + f ( 3) = K (1 - a ) (1 - b ) ( a - b) (c) 4 (d) 1
2 2 2
,
1 + f ( 2 ) 1 + f ( 3) 1 + f ( 4 ) 42. The area of the triangle whose vertices are complex numbers
z, iz, z + iz in the Argand diagram is [Online May 12, 2012]
then K is equal to: [2014] (a) 2|z|2 (b) 1/2|z|2 (c) 4|z|2 (d) |z|2
1 43. The area of triangle formed by the lines oining the vertex
(a) 1 (b) –1 (c) ab (d) of the parabola, x2 = 8y, to the extremities of its latus rectum
ab is [Online May 12, 2012]
(a) 2 (b) 8 (c) 1 (d) 4
44. Let a, b, c be such that b(a + c) ¹ 0 if [2009]
r 2r - 1 3r - 2
n a a + 1 a –1 a +1 b +1 c –1
37. If D r = n -1 a
2 –b b + 1 b –1 a –1 b -1 c + 1 = 0,
+
1 1 c –1 c + 1 n+2 n +1
n(n - 1) (n - 1)2 (n - 1)(3n - 4) c (-1) a (-1) b (-1)n c
2 2
then the value of n is :
n -1 (a) any even integer (b) any odd integer
then the value of å Dr [Online April 19, 2014] (c) any integer (d) ero
r =1

Downloaded from @Freebooksforjeeneet


Determinants M-281

1 1 1 Adjoint of a Matrix, Inverse of a


45. If D = 1 1 + x 1 for x ¹ 0, y ¹ 0 , then D is TOPIC Đ Matrix, Some Special Cases of
Matrix, Rank of a Matrix
1 1 1+ y
(a) divisible by x but not y [2007] é 2 -1 1 ù
(b) divisible by y but not x = ê -1 0 2 úú
(c) divisible by neither x nor y 51. Let A be a 3 × 3 matrix such that ad A ê and
(d) divisible by both x and y ëê 1 -2 -1ûú

46. If a1 , a2 , a3 , ............, an , ...... are in G. P., then the B = ad(ad A). If | A |= l and | ( B-1 )T |= m, then the ordered
determinant pair, (| l |, m) is equal to : [Sep. 03, 2020 (II)]
log an log an + 1 log an + 2 æ 1ö æ 1ö
(a) ç 3, ÷ (b) ç 9, ÷
D = log an + 3 log an + 4 log an + 5 è 81 ø è 9ø
log an + 6 log an + 7 log an + 8 æ 1ö
(c) (3, 81) (d) ç 9, ÷
is equal to [2005] è 81 ø
(a) 1 (b) 0 (c) 4 (d) 2
é1 1 2ù
2 2 2 ê ú
47. If a + b + c = – 2 and [2005] 52. If the matrices A = ê1 3 4ú , B = ad A
êë1 -1 3úû
1 + a2 x (1 + b 2 ) x (1 + c 2 ) x
2 2 2 ad B
f (x) = (1 + a ) x 1 + b x (1 + c ) x , and C = 3A, then is equal to : [Jan. 9, 2020 (I)]
C
(1 + a2 ) x (1 + b 2 ) x 1 + c 2 x
(a) 8 (b) 16 (c) 72 (d) 2
then f (x) is a polynomial of degree é 5 2a 1 ù
(a) 1 (b) 0 (c) 3 (d) 2 ê ú
53. If B = ê 0 2 1 ú is the inverse of a 3 × 3 matrix A, then
48. If a1, a2 , a3 ,......, an ,.... are in G.P., then the value of the ë a 3 -1û
determinant [2004] the sum of all values of a for which det (A) + 1 = 0, is :
[April 12, 2019 (I)]
log an log an+1 log an+ 2
(a) 0 (b) –1 (c) 1 (d) 2
log an+ 3 log an+ 4 log an +5
, is
log an+ 6 log an+ 7 log an+8 é1 1ù é1 2 ù é1 3ù é1 n – 1ù é1 78 ù
If ê ú ×ê ú ×ê ú ê0 =
1 úû êë0 1 úû
54. ............ ,
ë 0 1û ë0 1 û ë 0 1 û ë
(a) –2 (b) 1
(c) 2 (d) 0 é1 n ù
If a > 0 and discriminant of ax2+2bx+c is –ve, then     � 矄 ᨨ ê 0 1 ú �矄 [April 09, 2019 (II)]
49. ë û
a b ax + b
é 1 0ù é1 -13ù
b bx + c is equal to
c [2002] (a) ê12 1 ú
ë û
(b) ê ú
ë0 1 û
ax + b bx + c 0
é1 -12 ù é 1 0ù
(a) +ve (b) (ac-b2)(ax2+2bx+c) (c) ê0 1 ú (d) ê13 1 ú
ë û ë û
(c) –ve (d) 0
50. l, m, n are the p , q and rth term of a G. P. all positive,
th th 55. Let A and B be two invertible matrices of order 3 × 3. If
det (ABAT) = 8 and det (AB–1) = 8, then det (BA–1 BT) is
log l p 1 equal to : [Jan. 11, 2019 (II)]
then log m q 1 equals [2002]
1
log n r 1 (a) (b) 1
4
(a) –1 (b) 2
1
(c) 1 (d) 0 (c) (d) 16
16

Downloaded from @Freebooksforjeeneet


EBD_8344
M-282 Mathematics

Then :
écos q - sin qù (a) Both the statements are true.
56. If A = ê ú , then the matrix A–50 when
ë sin q cos q û (b) Both the statements are false.
(c) Statement–I is true, but Statement-II is false.
p
q= , is equal to: [Jan 09, 2019 (I)] (d) Statement I is false, but Statement-II is true.
12 63. If A is a 3 × 3 matrix such that |5.adA| = 5, then |A| is
é 1 3ù é 3 1ù equal to : [Online April 11, 2015]
ê - ú ê - ú
2 2 ú 2 2ú 1 1
ê ê (a) ± (b) ± (c) ±1 (d) ±5
(a) ê 3 1 ú (b) ê 1 3ú 5 25
ê ú ê ú
ë 2 2 û ë 2 2 û 64. If A is an 3 × 3 non-singular matrix such that AA' = A'A and
é 3 B = A–1A', then BB' equals: [2014]
1 ù é 1 3ù
ê ú ê ú
ê 2 2 ú ê 2 2 ú (a) B –1 (b) ( B )¢
-1
(c) I + B (d) I
(c) ê 1 3ú (d) ê 3 1 ú
ê- ú ê- ú 65. Let A be a 3 × 3 matrix such that
ë 2 2 û ë 2 2 û
é 1 2 3ù é 0 0 1 ù
é1 2ù
57. Let A be a matrix such that A . ê ú is a scalar matrix and A êê0 2 3úú = êê1 0 0úú
ë 0 3û
ëê0 1 1úû ëê0 1 0ûú
|3A| = 108. Then A2 equals [Online April 15, 2018]
Then A–1 is: [Online April 11, 2014]
é 4 – 32 ù é 4 0ù
(a) ê ú (b) ê ú
ë 0 36 û ë – 32 36 û é3 1 2 ù é3 2 1 ù
ê3 0 2 ú ê3 2 0 ú
é 36 0 ù é36 – 32 ù (a) ê ú (b) ê ú
(c) ê ú (d) ê
ë – 32 4 û ë0 4 úû ëê1 0 1 úû ëê1 1 0 úû
58. Suppose A is any 3 × 3 non-singular matrix and
(A – 3I) (A – 5I) = O, where I = I3 and O = O3. If aA + bA –1 = 4I, é 0 1 3ù é 1 2 3ù
ê 0 2 3ú ê0 1 1ú
then a + b is equal to [Online April 15, 2018] (c) ê ú (d) ê ú
(a) 8 (b) 12 (c) 13 (d) 7 êë1 1 1úû êë0 2 3úû
é 2 -3ù
59. If A = ê 2
ú , then ad (3A + 12A) is equal to : [2017] é 1 a 3ù
ë -4 1 û
66. If P = êê 1 3 3úú is the adoint of a 3 × 3 matrix A and
é 72 -63ù é 72 -84 ù êë 2 4 4úû
(a) ê ú (b) ê ú
ë -84 51 û ë -63 51 û |A| = 4, then a is equal to : [2013]
é 51 63ù é 51 84 ù (a) 4 (b) 11 (c) 5 (d) 0
(c) ê ú (d) ê ú 67. Let P and Q be 3 ´ 3 matrices P ¹ Q. If P3= Q3 and
ë84 72 û ë 63 72 û
P2Q = Q2P then determinant of (P2 + Q2) is equal to :
60. Let A be any 3 × 3 invertible matrix. Then which one of the (a) – 2 (b) 1 [2012]
following is not always true ? [Online April 8, 2017] (c) 0 (d) – 1
(a) ad (A)= |A| . A–1
æ1 0 0ö
(b) ad (ad(A)) = |A|.A
(c) ad (ad(A)) = |A|2 .(ad(A))–1 68. Let A = çç 2 1 0 ÷÷ . If u1 and u2 are column matrices such
(d) ad (ad(A)) = |A|.(ad(A))–1 ç 3 2 1÷
è ø
é5a - b ù
61. If A = ê T
ú and A ad A = A A , then 5a + b is equal to: æ1ö æ0ö
ë 3 2 û ç ÷ ç ÷
that Au1 = ç 0 ÷ and Au2 = ç 1 ÷ , then u1 + u2 is equal to :
[2016] ç0÷ ç0÷
(a) 4 (b) 13 (c) –1 (d) 5 è ø è ø
62. Let A be a 3 × 3 matrix such that A2 – 5A + 7I = 0. [2012]
-1 1 æ -1 ö æ -1 ö æ -1 ö æ1ö
Statement–I : A = (5I - A) .
7 ç ÷ ç ÷ ç ÷ ç ÷
(c) ç -1 ÷ (d) ç -1 ÷
1
Statement II : the polynomial A3 – 2A2 – 3A + a can be (a) ç 1 ÷ (b) ç ÷
ç0÷ ç -1 ÷ ç0÷ ç -1 ÷
reduced to 5 (A – 4I). [Online April 10, 2016] è ø è ø è ø è ø

Downloaded from @Freebooksforjeeneet


Determinants M-283

2
é0 0 a ù 74. If A – A + I = 0 , then the inverse of A is [2005]
69. If A denotes the transpose of the matrix A = êê 0 b c úú ,
T
(a) A + I (b) A (c) A – I (d) I – A
êëd e f úû
æ 1 -1 1 ö æ 4 2 2ö
where a, b, c, d, e and f are integers such that abd ¹ 0, then 75. ç ÷
Let A = 2 1 -3 . and B = ç -5 0 a ÷ . If B is the
the number of such matrices for which A–1 = AT is ç ÷ ç ÷
è1 1 1 ø è 1 -2 3 ø
[Online May 19, 2012]
(a) 2(3!) (b) 3(2!) (c) 23 (d) 32 inverse of matrix A, then a is [2004]
éa 0 ù (a) 5 (b) –1 (c) 2 (d) –2
70. Let A and B be real matrices of the form ê ú and
ë 0 bû æ 0 0 -1ö
é0 g ù 76. Let A = ç 0 -1 0 ÷ . The only correct
ç ÷
ê d 0 ú , respectively. [Online May 12, 2012] è -1 0 0 ø
ë û
Statement 1: AB – BA is always an invertible matrix. statement about the matrix A is [2004]
Statement 2: AB – BA is never an identity matrix.
(a) A2 = I
(a) Statement 1 is true, Statement 2 is false.
(b) Statement 1 is false, Statement 2 is true. (b) A = (–1) I, where I is a unit matrix
(c) Statement 1 is true, Statement 2 is true; Statement 2 is -1
(c) A does not exist
a correct explanation of Statement 1.
(d) A is a ero matrix
(d) Statement 1 is true, Statement 2 is true, Statement 2 is
not a correct explanation of Statement 1. Solution of System of Linear
71. Consider the following relation R on the set of real square TOPIC Ė Equations
matrices of order 3. [2011RS]
R = { ( A, B ) A = P BP for some invertible matrix P}
–1 77. The values of l and m for which the system of linear
equations [Sep. 06, 2020 (I)]
Statement-1 : R is equivalence relation.
x+y+z=2
Statement-2 : For any two invertible 3 ´ 3 matrices M and
x + 2y + 3z = 5
N, ( MN )
-1
= N -1 M -1 . x + 3y + lz = m
(a) Statement-1 is true, statement-2 is true and statement- has infinitely many solutions are, respectively :
2 is a correct explanation for statement-1. (a) 6 and 8 (b) 5 and 7
(b) Statement-1 is true, statement-2 is true; statement-2 is
not a correct explanation for statement-1. (c) 5 and 8 (d) 4 and 9
(c) Statement-1 is true, stement-2 is false. 78. The sum of distinct values of l for whcih the system of
(d) Statement-1 is false, statement-2 is true. equations
72. Let A be a 2 × 2 matrix
(l - 1) x + (3l + 1) y + 2lz = 0
Statement -1 : ad (ad A) = A
Statement -2 : |ad A |= |A| [2009] (l - 1) x + (4l - 2) y + (l + 3) z = 0
(a) Statement-1 is true, Statement-2 is true. Statement-2 2x + (3l + 1) y + 3(l – 1) z = 0,
is not a correct explanation for Statement-1.
has non- ero solutions, is ______. [NA Sep. 06, 2020 (II)]
(b) Statement-1 is true, Statement-2 is false.
(c) Statement -1 is false, Statement-2 is true. 79. Let l Î R . The system of linear equations
(d) Statement-1 is true, Statement -2 is true. 2 x1 - 4 x2 + lx3 = 1 [Sep. 05, 2020 (I)]
Statement-2 is a correct explanation for Statement-1.
73. Let A be a square matrix all of whose entries are integers. x1 - 6 x2 + x3 = 2
Then which one of the following is true? [2008] lx1 - 10 x2 + 4 x3 = 3
(a) If det A = ± 1, then A–1 exists but all its entries are not
(a) exactly one negative value of l
necessarily integers
(b) If det A ¹ ± 1, then A–1 exists and all its entries are non (b) exactly one positive value of l
integers (c) every value of l
(c) If det A = ± 1, then A–1 exists but all its entries are
integers (d) exactly two value of l
(d) If det A = ± 1, then A–1 need not exists

Downloaded from @Freebooksforjeeneet


EBD_8344
M-284 Mathematics

80. If the system of linear equations 85. Let S be the set of all l Î R for which the system of linear
x + y + 3z = 0 equations [Sep. 02, 2020 (I)]
2x - y + 2z = 2
x + 3y + k2z = 0
x - 2 y + lz = -4
3x + y + 3z = 0
x + ly + z = 4
has a non- ero solution (x, y, z) for some k ÎR, then has no solution. Then the set S
æ yö (a) contains more than two elements.
x+ç ÷ is equal to : [Sep. 05, 2020 (II)] (b) is an empty set.
è zø
(c) is a singleton.
(a) – 3 (b) 9 (c) 3 (d) – 9 (d) contains exactly two elements.
81. If the system of equations x - 2 y + 3 z = 9 , 2 x + y + z = b 86. Let A = {X = (x, y, z)T : PX = 0 and x2 + y 2 + z 2 = 1},
x - 7 y + az = 24, has infinitely many solutions, then
é1 2 1ù
where P = êê -2 3 -4 úú , then the set A :
a – b is equal to __________. [NA Sep. 04, 2020 (I)]
82. Suppose the vectors x1, x2 and x3 are the solutions of the
system of linear equations, Ax = b when the vector b on êë 1 9 -1úû
[Sep. 02, 2020 (II)]
the right side is equal to b1, b2 and b3 respectively. If
(a) is a singleton
é1ù é 0ù é0ù é1ù é 0ù (b) is an empty set
x1 = 1 , x2 = 2 , x3 = 0 , b1 = 0 , b2 = ê 2ú
ê ú ê ú ê ú ê ú and (c) contains more than two elements
êú ê ú ê ú ê ú ê ú (d) contains exactly two elements
êë1úû êë1 úû êë1úû êë0úû êë 0úû
é0ù 87. The following system of linear equations
b3= ê0ú , then the determinant of A is equal to : 7x + 6y – 2z = 0
ê ú 3x + 4y + 2z = 0
êë 2úû
x – 2y – 6z = 0, has [Jan. 9, 2020 (II)]
[Sep. 04, 2020 (II)] (a) infinitely many solutions, (x, y, z) satisfying y = 2z.
(a) 4 (b) 2 (b) no solution.
(c) infinitely many solutions, (x, y, z) satisfying x = 2z.
1 3
(c) (d) (d) only the trivial solution.
2 2
83. If the system of equations 88. For which of the following ordered pairs (m, d), the system
x+ y+ z = 2 of linear equations
x + 2y + 3z = 1
2x + 4 y - z = 6 3x + 4y + 5z = m
3x + 2 y + lz = m 4x + 4y + 4z = d
is inconsistent? [Jan. 8, 2020 (I)]
has infinitely many solutions, then : [Sep. 04, 2020 (II)]
(a) (4, 3) (b) (4, 6)
(a) l + 2m = 14 (b) 2l - m = 5 (c) (1, 0) (d) (3, 4)
(c) l - 2m = -5 (d) 2l + m = 14 89. The system of linear equations
84. Let S be the set of all integer solutions, (x, y, z), of the lx + 2y + 2z = 5
system of equations
2lx + 3y + 5z = 8
x - 2 y + 5z = 0
4x + ly + 6z = 10 has: [Jan. 8, 2020 (II)]
-2 x + 4 y + z = 0
(a) no solution when l = 8
-7 x + 14 y + 9 z = 0
(b) a unique solution when l = –8
such that 15 £ x2 + y 2 + z 2 £ 150. Then, the number of
elements in the set S is equal to ____________. (c) no solution when l = 2
[NA Sep. 03, 2020 (II)] (d) infinitely many solutions when l = 2

Downloaded from @Freebooksforjeeneet


Determinants M-285

90. If the system of linear equations 95. The greatest value of c Î R for which the system of linear
equations
2x + 2ay + az = 0 x – cy – cz = 0; cx – y + cz = 0; cx + cy – z = 0
2x + 3by + bz = 0 has a non-trivial solution, is : [April 08, 2019 (I)]
1
2x + 4cy + cz = 0, (a) –1 (b) (c) 2 (d) 0
2
where a, b, c ÎR are non- ero and distinct; has a non- ero 96. If the system of linear equations
solution, then: [Jan. 7, 2020 (I)] x – 2y + kz = 1
2x + y + z = 2
1 1 1 3x – y – kz = 3
(a) , , are in A.P..
a b c has a solution (x, y, z), z ¹ 0, then (x, y) lies on the straight
(b) a, b, c are in G.P. line whose equation is : [April 08, 2019 (II)]
(a) 3x – 4y – 1 = 0 (b) 4x – 3y – 4 = 0
(c) a + b + c = 0
(c) 4x – 3y – 1 = 0 (d) 3x – 4y – 4 = 0
(d) a, b, c are in A.P. 97. An ordered pair (a, b) for which the system of linear
equations
91. If the system of linear equations,
(1 +a) x + by + = 2
x+y+z=6 ax + (1 + b)y + = 3
ax + by + 2 = 2
x + 2y + 3z = 10
has a unique solution, is : [Jan. 12, 2019 (I)]
3x + 2y + lz = m (a) (2, 4) (b) (–3, 1)
(c) (–4, 2) (d) (1, – 3)
has more than two solutions, then m – l2 is equal to
98. The set of all values of l for which the system of linear
_________. [NA Jan. 7, 2020 (II)] equations
92. If the system of linear equations x – 2y – 2 = lx
x + y+ = 5 x + 2y + = ly
–x – y = l2
x + 2y + 2 = 6
has a non-trivial solution : [Jan. 12, 2019 (II)]
x + 3y + l = m, (l, m Î R), has infinitely many solutions, (a) is a singleton
then the value of l + m is : [April 10, 2019 (I)] (b) contains exactly two elements
(a) 12 (b) 9 (c) 7 (d) 10 (c) is an empty set
93. Let l be a real number for which the system of linear (d) contains more than two elements
equations: 99. If the system of linear equations
2x + 2y + 3 = a
x+y+z=6
3x – y + 5 = b
4x + ly – lz = l –2 x – 3y + 2 = c
3x + 2y – 4z = –5 where, a, b, c are non- ero real numbers, has more than one
has infinitely many solutions. Then l is a root of the solution, then : [Jan. 11, 2019 (I)]
quadratic equation : [April 10, 2019 (II)] (a) b – c + a = 0 (b) b – c – a = 0
(c) a + b + c = 0 (d) b + c – a = 0
(a) l2 + 3l – 4 = 0 (b) l2 – 3l – 4 = 0
100. The number of values of q Î (0, p) for which the system of
(c) l2 +l – 6 = 0 (d) l2 – l – 6 = 0
linear equations
94. If the system of equations 2x + 3y – z =0, x + ky – 2z = 0 and
x + 3y + 7z = 0
2x – y + z = 0 has a non-trivial solution (x, y, z), then
– x + 4y + 7z = 0
x y z
+ + + k is equal to: [April 09, 2019 (II)] (sin 3q)x + (cos 2q)y + 2z = 0
y z x
has a non-trivial solution, is: [Jan. 10, 2019 (II)]
3 1 1 (a) three (b) two
(a) (b) (c) - (d) –4
4 2 4 (c) four (d) one

Downloaded from @Freebooksforjeeneet


EBD_8344
M-286 Mathematics

101. If the system of equations [Jan 10, 2019 (I)] (a) a singleton
x+y+z=5 (b) an empty set
x + 2y + 3z = 9 (c) an infinite set
(d) a finite set containing two or more elements
x + 3y + az = b
108. The number of real values of l for which the system of
has infinitely many solutions, then b – a equals: linear equations
(a) 21 (b) 8 (c) 18 (d) 5 2x + 4y – l = 0
102. If the system of linear equations 4x + ly + 2 = 0
x – 4y + 7z = g lx + 2y + 2 = 0
3y – 5z = h has infinitely many solutions, is : [Online April 8, 2017]
– 2x + 5y – 9z = k (a) 0 (b) 1 (c) 2 (d) 3
is consistent, then : [Jan. 09, 2019 (II)] 109. The system of linear equations
(a) g + 2h + k = 0 x + ly – = 0
(b) g + h + 2k = 0 lx – y – = 0
(c) 2g + h + k = 0 x + y– l = 0
(d) g + h + k = 0 has a non-trivial solution for: [2016]
103. If the system of linear equations (a) exactly two values of l.
x + ky + 3 = 0 (b) exactly three values of l.
3x + ky – 2 = 0 (c) infinitely many values of l.
2x + 4y – 3 = 0 (d) exactly one value of l.
110. The set of all values of l for which the system of linear
x
has a non- ero solution (x, y, ), then is equal to : equations : [2015]
y2
2x1 – 2x2 + x3 = lx1
[2018] 2x1 – 3x2 + 2x3 = lx2
(a) 10 (b) – 30 (c) 30 (d) – 10 –x1 + 2x2 = lx3
104. The number of values of k for which the system of linear has a non-trivial solution,
equations, (k + 2) x + 10y = k, kx + (k + 3) y = k – 1 has no (a) contains two elements.
solution, is [Online April 16, 2018]
(b) contains more than two elements
(a) Infinitely many (b) 3
(c) is an empty set.
(c) 1 (d) 2
(d) is a singleton
105. Let S be the set of all real values of k for which the system
111. If a, b, c are non- ero real numbers and if the system of
of linear equations
equations [Online April 9, 2014]
x+y+z=2
(a – 1)x = y + ,
2x + y – z = 3
(b – 1)y = + x,
3x + 2y + kz = 4
(c – 1) = x + y,
has a unique solution. Then S is [Online April 15, 2018]
has a non-trivial solution, then ab + bc + ca equals:
(a) an empty set (b) equal to R – {0}
(a) a + b + c (b) abc
(c) equal to {0} (d) equal to R
(c) 1 (d) – 1
106. If the system of linear equations
112. The number of values of k, for which the system of equations:
x + ay + z = 3
(k + 1) x + 8y = 4k
x + 2y + 2z = 6
kx + (k + 3)y = 3k – 1
x + 5y + 3z = b
has no solution, is [2013]
has no solution, then [Online April 15, 2018]
(a) infinite (b) 1
(a) a = 1, b ¹ 9 (b) a ¹ – 1, b = 9
(c) 2 (d) 3
(c) a = – 1, b = 9 (d) a = – 1, b ¹ 9
113. Consider the system of equations :
107. If S is the set of distinct values of ‘b’ for which the
following system of linear equations [2017] x + ay = 0, y + az = 0 and z + ax = 0. Then the set of all real
values of ‘a’ for which the system has a unique solution
x + y+ = 1
is: [Online April 25, 2013]
x + ay + = 1
(a) R – {1} (b) R – { – 1}
ax + by + = 0
(c) {1, – 1} (d) {1, 0, –1}
has no solution, then S is :

Downloaded from @Freebooksforjeeneet


Determinants M-287

114. Statement-1: The system of linear equations 117. If the system of equations [Online May 7, 2012]
x + (sin a) y + (cos a) z = 0 x+y+z=6
x + (cos a) y + (sin a) z = 0 x + 2y + 3z = 10
x + 2y + lz = 0
x – (sin a) y – (cos a) z = 0
has a unique solution, then l is not equal to
has a non-trivial solution for only one value of a lying in
(a) 1 (b) 0 (c) 2 (d) 3
æ pö 118. If the trivial solution is the only solution of the system of
the interval ç 0, ÷ .
è 2ø equations [2011RS]

Statement-2: The equation in a x - ky + z = 0


kx + 3 y - kz = 0
cos a sin a cos a
3x + y - z = 0
sin a cos a sin a =0
cos a - sin a - cos a then the set of all values of k is :
(a) R - { 2, -3} (b) R - { 2}
æ pö
has only one solution lying in the interval ç 0, ÷ . (c) R - { -3} (d) { 2, -3}
è 2ø
119. The number of values of k for which the linear equations
[Online April 23, 2013]
4x + ky + 2z = 0 , kx + 4y + z = 0 and 2x + 2y + z = 0 possess
(a) Statement-1 is true, Statement-2 is true, Statement-2 is a non- ero solution is [2011]
not correct explantion for Statement-1. (a) 2 (b) 1 (c) ero (d) 3
(b) Statement-1 is true, Statement-2 is true, Statement-2 is 120. Consider the system of linear equations; [2010]
a correct explantion for Statement-1. x1 + 2x2 + x3 = 3
(c) Statement-1 is true, Statement-2 is false. 2x1 + 3x2 + x3 = 3
(d) Statememt-1 is false, Statement-2 is true. 3x1 + 5x2 + 2x3 = 1
115. If the system of linear equations : The system has
(a) exactly 3 solutions
x1 + 2x2 + 3x3 = 6
(b) a unique solution
x1 + 3x2 + 5x3 = 9
(c) no solution
2x1 + 5x2 + ax3 = b (d) infinite number of solutions
is consistent and has infinite number of solutions, then : 121. Let a, b, c be any real numbers. Suppose that there are real
[Online April 22, 2013] numbers x, y, z not all ero such that x = cy + bz, y = az + cx,
(a) a = 8, b can be any real number and z = bx + ay. Then a2 + b2 + c2 + 2abc is equal to
(b) b = 15, a can be any real number (a) 2 (b) –1 [2008]
(c) 0 (d) 1
(c) a Î R - {8} and b Î R - {15} 122. The system of equations
(d) a = 8, b = l5 ax+y + = a –1
116. Statement 1: If the system of equations x + ky + 3z = 0, x + a y+ = a – 1
3x + ky – 2z = 0, 2x + 3y – 4z = 0 has a non-trivial solution, x+ y+ a = a –1
31 has infinite solutions, if a is [2005]
then the value of k is .
2 (a) – 2 (b) either – 2 or 1
Statement 2: A system of three homogeneous equations (c) not – 2 (d) 1
in three variables has a non trivial solution if the determinant 123. If the system of linear equations [2003]
of the coefficient matrix is ero. [Online May 26, 2012] x + 2ay + az = 0 ; x + 3by + bz = 0 ;
(a) Statement 1 is false, Statement 2 is true.
x + 4cy + cz = 0 has a non - ero solution, then a, b, c.
(b) Statement 1 is true, Statement 2 is true, Statement 2 is
a correct explanation for Statement 1. (a) satisfy a + 2b + 3c = 0
(c) Statement 1 is true, Statement 2 is true, , Statement 2 is (b) are in A.P
not a correct explanation for Statement 1. (c) are in G..P
(d) are in H.P.
(d) Statement 1 is true, Statement 2 is false.

Downloaded from @Freebooksforjeeneet


Continuity and Differentiability M-309

20
Continuity and
Differentiability
æ p pö
TOPIC Ć Continuity 5. If the function f defined on ç , ÷ by
è6 3ø
ì 2 cos x – 1 p
éxù ïï ,x ¹
1. Let f (x) = x. ê ú , for –10 < x < 10, where [t] denotes the cot x – 1 4
ë 2û í
f(x) = ï p
k, x=
greatest integer function. Then the number of points of ïî 4
discontinuity of f is equal to ______. is continuous, then k is equal to: [April 09, 2019 (I)]
[NA Sep. 05, 2020 (I)]
1
2. If a function f (x) defined by (a) 2 (b) 1 (c) 1 (d)
2 2
ìae x + be - x , - 1 £ x < 1 éxù
ïï 6. If f ( x) = [ x] - ê ú , x Î R, where [x] denotes the greatest
f ( x) = í cx 2 , 1 £ x £ 3 be continuous for some ë4û
ï 2 integer function, then: [April 09, 2019 (II)]
ïî ax + 2cx , 3 < x £ 4
(a) f is continuous at x = 4.
a, b, c Î R and f '(0) + f '(2) = e, then the value of a is :
(b) xlim
®4+
f(x) exists but xlim
®4-
f(x) does not exist.
[Sep. 02, 2020 (I)]
(c) Both xlim
®4-
f(x) and xlim
®4+
f(x) exist but are not equal.
1 e
(a) (b)
e 2 - 3e + 13 e 2 - 3e - 13 (d) xlim
®4-
f(x) exists but xlim
®4+
f(x) does not exist.

e e 7. If the function

{
(c) (d)
e 2 + 3e + 13 e 2 - 3e + 13 a | p - x | +1, x £ 5
f(x) = b | x - p | +3, x > 5
é4ù
Let [t] denote the greatest integer £ t and xlim x ú = A.
®0 ê
3. is continuous at x = 5, then the value of a – b is:
ëxû
2 [April 09, 2019 (II)]
Then the function, f(x) = [x ] sin(px) is discontinuous, when
x is equal to : [Jan. 9, 2020 (II)] 2 -2 2 2
(a) (b) (c) (d)
(a) A +1 (b) A+5 p+5 p+5 p -5 5-p
8. Let f : [– 1, 3] ® R be defined as
(c) A + 21 (d) A
ì x + [ x ] , - 1£ x < 1
æ 1 1ö ï
4. If the function f defined on ç - , ÷ by f(x) = í x + x , 1 £ x < 2
è 3 3ø ï x + [ x ] , 2 £ x £ 3,
î
ì1 æ 1 + 3x ö where [t] denotes the greatest integer less than or equal
ï log e ç , when x ¹ 0
f(x) = í x è 1 - 2 x ÷ø is continuous, then k to t. Then, f is discontinuous at : [April 08, 2019 (II)]
ïk , when x = 0
î (a) only one point (b) only two points
is equal to __________. [NA Jan. 7, 2020 (II)] (c) only three points (d) four or more points

Downloaded from @Freebooksforjeeneet


EBD_8344
M-310 Mathematics

9. Let f : R ® R be a function defined as 14. Let k be a non– ero real number.


[Online April 11, 2015]
ì 5, if x £1
ï a + bx, if 1 < x < 3 ì
ï ï
f ( x) = í ï (e x - 1)
ï b + 5 x, if 3 £ x < 5 ïï æ x ö æ xö
, x¹0
ïî 30, x³5 log ç1 + ÷
If f(x) = í çè k ÷ø
if sin
ï è 4ø
Then, f is : [Jan 09, 2019 (I)] , x =0
ï 12
(a) continuous if a = 5 and b = 5 ï
(b) continuous if a = – 5 and b = 10 ïî
(c) continous if a = 0 and b = 5 is a continuous function then the value of k is:
(d) not continuous for any values of a and b (a) 4 (b) 1 (c) 3 (d) 2
10. If the function f defined as 15. If the function

1 k -1 ì 2 + cos x - 1
f (x) = - ï ,x ¹ p
x e2 x - 1 f ( x ) = í ( p - x)2
x ¹ 0, is continuous at x = 0, ï
îk ,x = p
then the ordered pair (k, f (0)) is equal to?
[Online April 16, 2018] is continuous at x = p, then k equals:
[Online April 19, 2014]
æ1 ö 1 1
(a) (3, 1) (b) (3, 2) (c) ç , 2 ÷ (d) (2, 1)
è3 ø (a) 0 (b)
2
(c) 2 (d)
4
ì 1 æ9ö 2
ï ( x - 1) 2 - x , x > 1, x ¹ 2 16. If f(x) is continuous and fç ÷= , then
11. Let f (x) = í è2ø 9
ïî k, x=2 æ 1 - cos 3x ö
lim f ç ÷ is equal to:
The value of k for which f is continuous at x = 2 is x ®0 è x2 ø
[Online April 15, 2018] [Online April 9, 2014]
(a) e –2 (b) e (c) e –1 (d) 1 9 2 8
(a) (b) (c) 0 (d)
12. The value of k for which the function 2 9 9
17. Consider the function :
ì tan 4x
f (x) = [ x] + | 1 – x |, -1 £ x £ 3 where [x] is the greatest
ï æç 4 ö÷ tan 5x , 0 < x < p
ï p integer function.
f (x) = í è 5 ø2 2
p is continuous at x = , is :
ï 5k + , x = 2 Statement 1 : f is not continuous at x = 0, 1, 2 and 3.
2
ïî
æ - x, -1 £ x < 0
ç
[Online April 9, 2017] 1 - x, 0 £ x < 1
Statement 2 : f (x) = ç
2 3 2 ç 1 + x, 1 £ x < 2
17 ç
(a) (b) (c) (d) -
20 5 5 5 è 2 + x, 2 £ x £ 3
13. Let a, b Î R, (a ¹ 0) . if the function f defined as [Online April 25, 2013]
(a) Statement 1 is true ; Statement 2 is false,
ì 2x 2 (b) Statement 1 is true; Statement 2 is true; Statement 2 is
ï , 0 £ x <1
ï a not correct explanation for Statement 1.
ï (c) Statement 1 is true; Statement 2 is true; Statement It is
ía , 1£ x < 2
f(x) = ï 2 a correct explanation for Statement 1.
ï 2b - 4b , 2£x<¥ (d) Statement 1 is false; Statement 2 is true.
ïî x 3 18. Let f be a composite function of x defined by
is continuous in the interval [0, ¥) , then an ordered pair f (u ) =
1 1
, u ( x) =
.
(a, b) is : [Online April 10, 2016] 2
u +u-2 x -1
(a) ( - 2,1 - 3) (b) ( 2, -1 + 3) Then the number of points x where f is discontinuous is :
[Online April 23, 2013]
(c) ( 2,1 - 3) (d) ( - 2,1 + 3) (a) 4 (b) 3 (c) 2 (d) 1

Downloaded from @Freebooksforjeeneet


Continuity and Differentiability M-311

19. Let f (x) = – 1 + | x – 2 |, and g (x) = 1 – | x |; then the set of Statement - 1 : f (x) is continuous on R.
all points where fog is discontinuous is :
[Online April 22, 2013] Statement - 2 : f1 ( x) and f 2 ( x) are continuous on R.
(a) {0, 2} (b) {0, 1, 2} (a) Statement -1 is true, Statement-2 is true; Statement-2
(c) {0} (d) an empty set is a correct explanation for Statement-1.
20. If f : R ® R is a function defined by f (x) = [x] (b) Statement-1 is true, Statement-2 is true; Statement-2 is
æ 2x - 1 ö NOT a correct explanation for Statement-1
cos ç ÷ p , where [x] denotes the greatest integer
è 2 ø (c) Statement-1 is true, Statement-2 is false
function, then f is . [2012] (d) Statement-1 is false, Statement-2 is true
(a) continuous for every real x.
24. The values of p and q for which the function [2011]
(b) discontinuous only at x = 0
(c) discontinuous only at non- ero integral values of x. ì
(d) continuous only at x = 0. ï sin( p + 1) x + sin x , x < 0
ï x
21. Let f : [1, 3] ® R be a function satisfying ï
f ( x) = í q , x = 0 is continuous for all x in R,
ï
x
£ f ( x) £ 6 - x , for all x ¹ 2 and f (2) = 1, ï x+ x - x
2
,x > 0
[ x] ïî x 3/2

where R is the set of all real numbers and [x] denotes the are
largest integer less than or equal to x.
5 1 3 1
Statement 1: lim f ( x ) exists. [Online May 19, 2012] (a) p = , q = (b) p = - , q =
2 2 2 2
x® 2 -
Statement 2: f is continuous at x = 2. 1 3 1 3
(a) Statement 1 is true, Statement 2 is true, Statement 2 is (c) p = , q = (d) p = , q = -
2 2 2 2
a correct explanation for Statement 1.
(b) Statement 1 is false, Statement 2 is true. 25. The function f : R /{0} ® R given by [2007]
(c) Statement 1 is true, Statement 2 is true, Statement 2 is
1 2
not a correct explanation for Statement 1. f ( x) = -
(d) Statement 1 is true, Statement 2 is false. x e2 x - 1
22. Statement 1: A function f : R ® R is continuous at x0 if and can be made continuous at x = 0 by defining f (0) as
only if lim f ( x ) exists and lim f ( x ) = f ( x0 ). (a) 0 (b) 1
x ® x0 x® x0
(c) 2 (d) – 1
Statement 2: A function f : R ® R is discontinuous at x0 if
1 - tan x p é pù
and only if, lim f ( x ) exists and lim f ( x ) ¹ f ( x0 ) . 26. Let f ( x) = , x ¹ , x Î ê 0, ú .
x ® x0 x ® x0 4x - p 4 ë 2û
[Online May 12, 2012]
(a) Statement 1 is true, Statement 2 is true, Statement 2 is é pù æ pö
If f (x) is continuous in ê 0, ú , then f çè ÷ø is [2004]
not a correct explanation of Statement 1. ë 2û 4
(b) Statement 1 is false, Statement 2 is true.
(c) Statement 1 is true, Statement 2 is true, Statement 2 is 1
(a) –1 (b)
a correct explanation of Statement 1. 2
(d) Statement 1 is true, Statement 2 is false.
1
23. Define f (x) as the product of two real functions (c) - (d) 1
[2011RS] 2
27. f is defined in [-5, 5] as [2002]
ì 1
ïsin , if x ¹ 0 f (x) = x if x is rational
f1 ( x ) = x, x Î R, and f 2 ( x ) = í x
ï0, if x = 0
= – x if x is irrational. Then
î
(a) f (x) is continuous at every x, except x = 0
as follows :
(b) f (x) is discontinuous at every x, except x = 0
ïì f1 ( x ) . f 2 ( x) , if x = 0 (c) f (x) is continuous everywhere
f ( x) = í
ïî 0 if x = 0 (d) f (x) is discontinuous everywhere

Downloaded from @Freebooksforjeeneet


EBD_8344
M-312 Mathematics

34. Let f and g be differentiable functions on R such that fog


TOPIC n Differentiability
is the identity function. If for some a, b Î R, g¢ (a) = 5 and
28. Let f : R ® R be a function defined by f (x) = max{x, x }. Let 2
g (a) = b, then f ¢ (b) is equal to: [Jan. 9, 2020 (II)]
S denote the set of all points in R, where f is not
1 2
differentiable. Then: [Sep. 06, 2020 (II)] (a) (b) 1 (c) 5 (d)
5 5
(a) {0, 1} (b) {0}
(c) f (an empty set) 35. Let S be the set of all functions f : [0,1] ® R, which are
(d) {1}
ì k1 ( x - p)2 - 1, x £ p continuous on [0, 1] and differentiable on (0,1). Then for
29. If the function f ( x) í is twice dif-
î k2 cos x, x>p
ferentiable, then the ordered pair (k1, k2) is equal to: every f in S, there exists a c Î (0,1), depending on f, such
[Sep. 05, 2020 (I)] that: [Jan. 8, 2020 (II)]

æ1 ö (a) | f (c) – f (l)| < (l – c)|f ¢(c)|


(a) ç ,1÷ (b) (1, 0)
è2 ø f (1) - f (c )
(b) = f ¢(c)
1- c
æ1 ö
(c) ç , -1÷ (d) (1, 1) (c) | f (c) + f (1)| < (1 + c) |f ¢(c)|
è 2 ø
30. Let f be a twice differentiable function on (1, 6). If f (2) = 8, (d) | f (c) – f (1)| < | f ¢(c)|
f '(2) = 5, f '(x) ³ 1 and f ''( x ) ³ 4, for all x Î (1, 6), then : 36. Let the function, f: [–7, 0] ® R be continuous on [ –7, 0]
[Sep. 04, 2020 (I)] and differentiable on (–7, 0). If f(–7) = –3 and f ¢ (x) d” 2, for
(a) f (5) + f '(5) £ 26 (b) f (5) + f '(5) ³ 28
all xÎ(–7, 0), then for all such functions f, f ¢(–1) + f(0) lies
(c) f '(5) + f ''(5) £ 20 (d) f (5) £ 10
in the interval: [Jan. 7, 2020 (I)]
31. Suppose a differentiable function f (x) satisfies the identity
(a) (– ฀ , 20] (b) [–3, 11]
f ( x + y ) = f ( x) + f ( y ) + xy 2 + x 2 y , for all real x and y. If
(c) (– ฀, 11] (d) [–6, 20]
f ( x)
lim = 1, then f '(3) is equal to ___________.
x ®0 x 37. Let S be the set of points where the function,
[NA Sep. 04, 2020 (I)]
f(x) = |2 – |x – 3||, xÎR, is not differentiable.
ìp -1
ïï 4 + tan x, | x |£ 1 Then å f(f(x)) is equal to ¾¾¾. [NA Jan. 7, 2020 (I)]
32. The function f ( x) = í xÎS
ï 1 (| x | -1) , | x |> 1
ïî 2 ì sin (p + 1) x + sin x
is : [Sep. 04, 2020 (II)] ï ,x < 0
x
(a) continuous on R – {1} and differentiable on ï
38. If f (x) = í q ,x=0
R – {–1, 1}. ï
ï x + x2 - x
(b) both continuous and differentiable on R – {1}. , x >0
(c) continuous on R – {–1} and differentiable on î x3/2
R – {–1, 1}. is continuous at x = 0, then the ordered pair (p, q) is equal to:
(d) both continuous and differentiable on R – {–1}.
[April 10, 2019 (I)]
ì sin(a + 2) x + sin x æ 3 1ö æ 1 3ö
ï ; x<0 (a) ç - , - ÷ (b) ç - , ÷
ïï x è 2 2ø è 2 2ø
33. If f ( x) = í b ; x =0
ï æ 3 1ö æ5 1ö
(c) ç - , ÷ (d) ç , ÷
ï ( x + 3x ) - x
2 1/3 1/3
; x>0 è 2 2ø è2 2ø
ïî x 4/3
39. Let f(x) = loge (sinx), (0 < x < p) and g(x) = sin–1 (e–x), (x > 0).
is continuous at x = 0, then a + 2b is equal to: If a is a positive real number such that a = (fog)¢ (a) and
b = (fog) ( a), then: [April 10, 2019 (II)]
[Jan. 9, 2020 (I)]
2 2
(a) aa + ba + a = 0 (b) aa – ba – a =1
(a) 1 (b) –1 (c) 0 (d) –2
(c) aa2 – ba – a = 0 (d) aa2 + ba – a = – 2a2

Downloaded from @Freebooksforjeeneet


Continuity and Differentiability M-313

40. Let f : R ® R be differentiable at c Î R and f (c) = 0. If (a) is an empty set


g (x) = f (x) , then at x = c, g is : [April 10, 2019 (I)] (b) equals {– 2, – 1, 0, 1, 2}

(a) not differentiable if f '(c) = 0 (c) equals {– 2, – 1, 1, 2}

(b) differentiable if f "(c) ¹ 0 (d) equals {– 2, 2}


(c) differentiable if f ' (c) = 0 48. Let f : (– 1, 1) ® R be a function defined by f (x) = max

41.
(d) not differentiable
Let f(x) = 15 – |x – 10|; x Î R. Then the set of all values of x, { - | x |, - }
1 - x 2 . If K be the set of all points at which f

at which the function, g(x) = f (f(x)) is not is not differentiable, then K has exactly:
differentiable, is: [April 09, 2019 (I)] [Jan. 10, 2019 (II)]
(a) five elements (b) one element
(a) {5, 10, 15} (b) {10, 15}
(c) three elements (d) two elements
(c) {5, 10, 15, 20} (d) {10}
42. If f (1) = 1, f¢(1) = 3, then the derivative of 49. Let S = { t Î R : f (x) = | x - p | (e|x| - 1)sin | x | is not
differentiable at t}. Then the set S is equal to : [2018]
f (f (f (x))) + (f (x))2 at x = 1 is : [April 08, 2019 (II)]
(a) {0} (b) {p}
(a) 33 (b) 12 (c) 15 (d) 9
(c) {0, p} (d) f (an empty set)
43. Let f be a differentiable function such that f(1) = 2 and 50. Let S = {(l, m) Î R × R : f (t) = (|l|e|t| – m). sin (2|t|), t Î R, is
f ¢ (x) = f (x) for all x Î R. If h (x) = f ( f (x)), then h¢ (1) is equal a differentiable function}. Then S is a subest of?
to : [Jan. 12, 2019 (II)]
2
[Online April 15, 2018]
(a) 2e (b) 4e (c) 2e (d) 4e2
(a) R × [0, ¥) (b) (–¥, 0) × R
ìï -1, -2 £ x < 0 (c) [0, ¥) × R (d) R × (–¥, 0)
44. Let f ( x ) = í 2 and 51. If the function
ïî x - 1, 0 £ x £ 2
g(x) = |f(x)| + f(|x|). Then, in the interval (–2, 2), g is : ìï- x, x <1
f(x) = í -1 is differentiable at
[Jan. 11, 2019 (I)] ïîa + cos (x + b), 1 £ x £ 2
(a) differentiable at all points
(b) not continuous a
x = 1, then is equal to : [Online April 9, 2016]
(c) not differentiable at two points b
(d) not differentiable at one point p+2 p-2
(a) (b)
dy 2 2
45. If x log e ( log e x ) - x 2 + y 2 = 4 ( y > 0 ) , then at x = e is
dx -p - 2
equal to : [Jan. 11, 2019 (I)] (c) (d) –1 – cos–1(2)
2
(1 + 2e ) ( 2e –1) 52. If the function.
(a) (b)
2 4 + e2 2 4 + e2 ìï k x + 1, 0 £ x £ 3
g(x) = í is differentiable, then the
(1 + 2e ) e ïî m x + 2, 3 < x £ 5
(c) (d) value of k + m is : [2015]
4+e 2
4 + e2
10
46. Let K be the set of all real values of x where the function (a) (b) 4
3
f (x) = sin | x | – | x | + 2 (x – p) cos | x | is not differentiable. 16
Then the set K is equal to : [Jan. 11, 2019 (II)] (c) 2 (d)
5
(a) f (an empty set) (b) {p}
(c) {0} (d) {0, p} 53. Let f: R ® R be a function such that f ( x ) £ x 2 , for all
x Î R . Then, at x = 0, f is: [Online April 19, 2014]
ìïmax {| x |, x 2 } | x|£ 2
47. Let f (x) = í (a) continuous but not differentiable.
ïî 8 - 2 | x |, 2 <| x|£ 4 (b) continuous as well as differentiable.
Let S be the set of points in the interval (– 4, 4) at which f (c) neither continuous nor differentiable.
is not differentiable. Then S: [Jan 10, 2019 (I)] (d) differentiable but not continuous.

Downloaded from @Freebooksforjeeneet


EBD_8344
M-314 Mathematics

54. Let f, g: R ® R be two functions defined by 60. Let f : R ® R be a function defined by


ì æ1ö f (x) = min {x + 1, x + 1} ,Then which of the following is true?
ï x sin ç ÷ , x ¹ 0
f (x) = í èxø , and g(x) = x f(x) (a) f (x) is differentiable everywhere [2007]
ï0, ,x = 0
î (b) f (x) is not differentiable at x = 0
Statement I: f is a continuous function at x = 0. (c) f (x) ³ 1 for all x Î R
Statement II: g is a differentiable function at x = 0. (d) f (x) is not differentiable at x = 1
[Online April 12, 2014]
x
(a) Both statement I and II are false. 61. The set of points where f ( x ) = is differentiable is
(b) Both statement I and II are true. 1+ | x |
(c) Statement I is true, statement II is false. [2006]
(d) Statement I is false, statement II is true. (a) (-¥,0) È (0, ¥) (b) (-¥,-1) È (-1, ¥)
55. Consider the function, f (x) = | x – 2 | + | x – 5 |, x Î R.
(c) (-¥, ¥) (d) (0, ¥)
Statement-1 : f ¢(4) = 0
Statement-2 : f is continuous in [2,5], differentiable in (2,5) 62. If f is a real valued differentiable function satisfying
and f (2) = f (5). [2012] | f (x) – f (y) | £ ( x - y )2 , x, y Î R and f (0) = 0, then f (1)
(a) Statement-1 is false, Statement-2 is true. equals [2005]
(b) Statement-1 is true, statement-2 is true; statement-2 is (a) – 1 (b) 0
a correct explanation for Statement-1. (c) 2 (d) 1
(c) Statement-1 is true, statement-2 is true; statement-2 is
63. Suppose f (x) is differentiable at x = 1 and
not a correct explanation for Statement-1.
(d) Statement-1 is true, statement-2 is false. 1
lim f (1 + h) = 5 , then f '(1) equals [2005]
56. If f(x) = a |sinx| + be |x| + c|x|3, where a, b, c Î R, is h®0 h
differentiable at x = 0, then [Online May 26, 2012] (a) 3 (b) 4 (c) 5 (d) 6
(a) a = 0, b and c are any real numbers
(b) c = 0, a = 0, b is any real number ì æ 1 1ö
ï -ç + ÷
(c) b = 0, c = 0, a is any real number 64. If f ( x ) = í xe è x x ø , x ¹ 0 then f(x) is
(d) a = 0, b = 0, c is any real number ï0 ,x = 0
î
57. If x + | y | = 2y, then y as a function of x, at x = 0 is
[Online May 7, 2012] (a) discontinuous every where [2003]
(a) differentiable but not continuous (b) continuous as well as differentiable for all x
(b) continuous but not differentiable (c) continuous for all x but not differentiable at x = 0
(c) continuous as well as differentiable (d) neither differentiable nor continuous at x = 0
(d) neither continuous nor differentiable
58. If function f (x) is differentiable at x = a, Chain Rule of Differentiation,
Differentiation of Explicit & Implicit
x2 f (a) - a2 f ( x) Functions, Parametric & Composite
then xlim
®a
is : [2011RS] TOPIC Đ Functions, Logarithmic & Exponential
x-a Functions, Inverse Functions,
(a) -a 2 f ' ( a ) (b) a f (a ) - a 2 f ' ( a )
Differentiation by Trigonometric
Substitution
(c) 2af ( a) - a 2 f ' ( a) (d) 2a f (a) +a f '(a)
2

æ 1 + x2 - 1 ö
ì 1 65. The derivative of tan -1 ç ÷ with respect to
ï( x –1) sin if x ¹ 1 ç x ÷
59. Let f ( x) = í x –1 [2008] è ø
ïî 0 if x = 1
æ 2x 1- x2 ö
Then which one of the following is true? tan -1 ç ÷ at x = 1 is : [Sep. 05, 2020 (II)]
ç 1 - 2x2 ÷ 2
(a) f is neither differentiable at x = 0 nor at x =1 è ø
(b) f is differentiable at x = 0 and at x =1
(c) f is differentiable at x = 0 but not at x = 1 2 3 3 2 3 3
(a) (b) (c) (d)
(d) f is differentiable at x = 1 but not at x = 0 5 12 3 10

Downloaded from @Freebooksforjeeneet


Continuity and Differentiability M-315

66. If (a + 2b cos x)(a - 2b cos y) = a 2 - b 2 , where a > b > 0, 2 1


(a) 1 (b) (c) (d) 2
3 2
dx æ p p ö
then at ç , ÷ is : [Sep. 04, 2020 (I)] 2
dy è 4 4 ø æ -1 æ 3 cos x + sin x ö ö æ pö dy
73. If 2y = çç cot çç ÷÷ ÷÷ , x Î ç 0, ÷ then
a - 2b a -b a+b 2a + b è è cos x - 3 sin x ø ø è 2ø dx
(a) (b) (c) (d)
a + 2b a+b a -b 2a - b is equal to : [April 08, 2019 (I)]
6
-1 ì 3 ü p p p p
å k cos
4 dy
67. If y = í cos kx - sin kx ý, then at x = 0 is (a) – x (b) x – (c) –x (d) 2x –
î 5 5 þ dx 6 6 3 3
k =1
74. Let S be the set of all points in (–p,p) at which the function
___________. [NA Sep. 02, 2020 (II)] f(x) = min {sinx, cosx} is not differentiable. Then S is a
68. If x = 2sinq – sin2q and y = 2cosq – cos2q, q Î [0, 2p], then subset of which of the following? [Jan. 12, 2019 (I)]

d2y ì p pü ì 3p p 3p p ü
at q = p is : (a) í- , 0, ý (b) í- , - , , ý
dx 2
[Jan. 9, 2020 (II)] î 4 4þ î 4 4 4 4þ

3 3 3 3 ì p p p pü ì 3p p p 3p ü
(a) (b) - (c) (d) - (c) í- , - , , ý (d) í- , - , , ý
4 8 2 4 î 2 4 4 2þ î 4 2 2 4 þ
dy
æ tan a + cot a ö æ 3p ö For x > 1, if ( 2 x )2 y = 4e 2 x- 2 y , then (1 + log e 2 x )
1 2
If y (a) = 2 ç ÷ + 2 , a Î ç 4 , p ÷ , then
75. is
69. dx
2
è 1 + tan a ø sin a è ø
equal to : [Jan. 12, 2019 (I)]
dy 5p x log e 2 x - log e 2
at a = is: [Jan. 7, 2020 (I)] (a) (b) loge 2x
da 6 x
4 1 x log e 2 x + log e 2
(a) 4 (b) (c) –4 (d) - (c) (d) x loge 2x
3 4 x
70. Let y = y(x) be a function of x satisfying 76. Let f : R ® R be a function such that
f (x) = x3 + x2f¢(1) + xf ²(2) + f ¢²(3), xÎR. Then f (2) equals:
= k – x 1 - y where k is a constant and
2
y 1 - x2 [Jan 10, 2019 (I)]
(a) – 4 (b) 30 (c) – 2 (d) 8
æ 1ö 1 dy 1
y ç ÷ = - . Then at x = , is equal to:
d2y
è 2ø 4 dx 2
77. If x = 3 tan t and y = 3 sec t, then the value of at
[Jan. 7, 2020 (II)] dx 2
p
5 5 2 5 t= , is: [Jan. 09, 2019 (II)]
(a) - (b) - (c) (d) 4
4 2 5 2 1 1 3 1
(a) (b) (c) (d)
æ dy d 2 y ö 3 2 6 2 2 2 6
71. If ey + xy = e, the ordered pair çç dx , 2 ÷÷ at x = 0 is
è dx ø
78. If x = 2cosec
–1
t
and y = 2sec
–1
t dy
(| t | ³ 1), then
is
equal to : [April 12, 2019 (I)] dx
equal to. [Online April 16, 2018]
æ1 1 ö æ 1 1 ö
(a) ç , - 2 ÷ (b) ç - , 2 ÷ y y x x
èe e ø è e e ø (a) (b) – (c) – (d)
x x y y
æ1 1 ö æ 1 1ö cos x x 1
(c) ç , 2 ÷ (d) ç - , - 2 ÷ f ¢ (x)
èe e ø è e e ø 79. If f (x) = 2 sin x x 2
2 x , then lim
x®0 x
-1 æ sin x - cos x ö x tan x x 1
72. The derivative of tan ç ÷ , with respect to ,
è sin x + cos x ø 2 [Online April 15, 2018]
(a) Exists and is equal to – 2
æ æ p öö (b) Does not exist
where ç x Î ç 0, 2 ÷ ÷ is : [April 12, 2019 (II)]
è è øø (c) Exist and is equal to 0
(d) Exists and is equal to 2

Downloaded from @Freebooksforjeeneet


EBD_8344
M-316 Mathematics

æ 2 ´ 3x ö æ 1ö x2 y2 x2 + y2 x2 + y2
80. If f (x) = sin -1 çç x ÷
÷ , then f ¢ ç - ÷ equals. (a) (b) (c) (d)
è1+ 9 ø è 2ø y2 x2 y2 x2
[Online April 15, 2018] x2 - x d
88. Let f (x) = .x ¹ 0, -2 . Then [ f -1 ( x )] (wherever
(a) 3 log e 3 (b) - 3 log e 3 x + 2x
2 dx
it is defined) is equal to : [Online April 9, 2013]
(c) - 3 loge 3 (d) 3 loge 3 -1 3
(a) (b)
2
(1 - x ) (1 - x )2
d2 y
81. If x2 + y2 + sin y = 4, then the value of at the point
dx 2 1 -3
(c) 2
(d)
(– 2, 0) is [Online April 15, 2018] (1 - x ) (1 - x )2
(a) – 34 (b) – 32 (c) – 2 (d) 4
æ 2 x + 3ö dy
If f ¢(x) = sin (log x) and y = f ç
è 3 - 2 x ÷ø
89. , then equals
æ 1ö æ 6x x ö dx
82. If for x Îç 0, ÷ , the derivative of tan -1 ç ÷ is
è 4ø è 1 - 9x 3 ø [Online May 12, 2012]
é æ 2x + 3ö ù
x.g ( x ) , then g(x) equals : (a) sin ê log ç
è 3 - 2 x ÷ø úû
[2017]
ë
3 9 12
(a) (b) (b)
( 3 - 2 x)2
3
1 + 9x 1 + 9x3

3x x 3x 12 é æ 2 x + 3ö ù
(c) (d) sin êlog ç ÷ú
ë è 3 - 2x ø û
(c)
1 - 9x 3 1 - 9x 3
( 3 - 2 x) 2

83. For x Î R, f(x) = |log2 – sinx| and g(x) = f(f(x)), then : 12 é æ 2 x + 3ö ù


[2016] (d) cos êlog ç ÷ú
( 3 - 2 x) 2
ë è 3 - 2x ø û
(a) g'(0) = – cos(log2)
(b) g is differentiable at x = 0 and g'(0) = – sin(log2) 90. Let f : (–1, 1) ® R be a differentiable function with f (0) = –
(c) g is not differentiable at x = 0 1 and f ¢ (0) = 1. Let g(x) = [f (2f (x) + 2)]2. Then g¢(0) =
(d) g'(0) = cos(log2) [2010]
(a) –4 (b) 0 (c) –2 (d) 4
1 Let y be an implicit function of x defined by
84. If f (x) = x2 – x + 5, x > , and g(x) is its inverse function, 91.
2 x2x – 2xx cot y – 1= 0. Then y¢(1) equals [2009]
then g¢(7) equals: [Online April 12, 2014] (a) 1 (b) log 2 (c) –log 2 (d) –1
dy
(a) -
1
(b)
1
(c)
1
(d) -
1 92. If x m . y n = ( x + y ) m+ n , then is [2006]
3 13 3 13 dx

(b) x + y
y x
dy (a) (c) xy (d)
85. If y = sec(tan–1x), then at x = 1 is equal to : [2013] x xy y
dx
y +L to ¥ dy
1 1 93. If x = e y + e , x > 0, then
is [2004]
(a) (b) (c) 1 (d) dx
2 2 2
1+ x 1 1- x x
(a) (b) (c) (d)
x2 y 2 x x x 1+ x
86. If the curves + = 1 and y3 = 16x intersect at right 94. Let f (x) be a polynomial function of second degree.
a 4
If f(1) = f(-1) and a, b, c are in A. P , then f '(a), f ¢ (b), f '(c)
angles, then a value of a is : [Online April 23, 2013]
are in [2003]
4 1 3 (a) Arithmetic -Geometric Progression
(a) 2 (b) (c) (d)
3 2 4 (b) A.P
(c) G..P
æ pö -1 cos-1 t
87. For a > 0, t Î ç 0, ÷ , let x = asin t and y = a , (d) H.P.
è 2ø
95. If f ( x + y ) = f ( x ). f ( y )"x. y and f (5) = 2,
2
Then, 1 + æç ö÷ equals : f '(0) = 3, then f ¢ (5) is
dy [2002]
[Online April 22, 2013]
è dx ø (a) 0 (b) 1 (c) 6 (d) 2

Downloaded from @Freebooksforjeeneet


Continuity and Differentiability M-317

102. Let f be a polynomial function such that f (3x) = f ¢ (x) , f ²


Differentiation of Infinite Series, (x), for all x Î R. Then : [Online April 9, 2017]
Successive Differentiation, nth (a) f (b) + f ¢ (b) = 28
Derivative of Some Standard (b) f ² (b) – f ¢ (b) = 0
TOPIC Ė Functions, Leibnitz’s Theorem, (c) f ² (b) – f ¢ (b) = 4
Rolle’s Theorem, Lagrange’s Mean (d) f (b) – f ¢ (b) + f ² (b) = 10
Value Theorem
15 15
103. If y = é x + x 2 - 1ù + éê x - x 2 - 1ùú , then
96. For all twice differentiable functios f : R®R, with êë úû ë û
f(0) = f(1) = f’(0) = 0 [Sep. 06, 2020 (II)]
d2 y dy
(x 2 - 1) +x is equal to [Online April 8, 2017]
(a) f "( x ) ¹ 0 at every point x Î (0,1) dx 2 dx

(b) f "(x) = 0, for some x Î (0,1) (a) 12 y (b) 224 y2 (c) 225 y2 (d) 225 y
104. If Rolle’s theorem holds for the function f (x) 2x3 + bx2
(c) f "(0) = 0
1
(d) f "(x) = 0, at every point x Î (0,1) + cx, x Î [–1, 1], at the point x = , then 2b + c equals :
2
æ p pö [Online April 10, 2015]
If y + log e (cos x) = y, x Î ç - , ÷ , then :
2 2
97. (a) –3 (b) –1 (c) 2 (d) 1
è 2 2ø
105. If f and g are differentiable functions in [0, 1] satisfying
[Sep. 03, 2020 (I)]
f (0) = 2 = g(1), g(0) = 0 and f (1) = 6, then for some c Î]0,1[
(a) y ''(0) = 0 (b) | y '(0) | + | y ''(0) |= 1 [2014]
(c) | y ''(0) |= 2 (d) | y '(0) | + | y ''(0) |= 3 (a) f ¢(c) = g¢(c) (b) f ¢(c) = 2g ¢(c)
(c) 2f ¢(c) = g ¢(c) (d) 2f ¢(c) = 3g¢(c)
98. If c is a point at which Rolle’s theorem holds for the 106. Let f(x) = x|x|, g(x) = sin x and h(x) = (gof) (x). Then
æ x2 + a ö [Online April 11, 2014]
function, f ( x ) = log e ç ÷ in the interval [3, 4], where (a) h(x) is not differentiable at x = 0.
è 7x ø (b) h(x) is differentiable at x = 0, but h¢(x) is not continuous
a Î R, then f ²(c) is equal to: [Jan. 8, 2020 (I)] at x = 0
(c) h¢(x) is continuous at x = 0 but it is not differentiable at
1 1 1 3 x= 0
(a) - (b) (c) - (d)
12 12 24 7 (d) h¢(x) is differentiable at x = 0
1 107. Let for i = 1, 2, 3, pi(x) be a polynomial of degree 2 in x, p¢i(x)
dy æ y ö 3 and p¢¢i(x) be the first and second order derivatives of pi(x)
99. k k k
Let x + y = a , (a, k > 0) and + ç ÷ = 0, then k is:
dx è x ø respectively. Let,
[Jan. 7, 2020 (I)] é p ( x ) p ¢ ( x ) p ¢¢ ( x ) ù
3 4 2 1 ê 1 1 1
ú
(a) (b) (c) (d) A ( x ) = p2 ( x ) p2 ( x ) p2¢¢ ( x ) ú
ê ¢
2 3 3 3 ê ú
ê p3 ( x ) p3¢ ( x ) p3¢¢ ( x ) ú
100. The value of c in the Lagrange’s mean value theorem for ë û
the function f(x) = x3 – 4x2 + 8x + 11, when x Î [0,1] is: and B(x) = [A(x)]T A(x). Then determinant of B(x):
[Jan. 7, 2020 (II)] [Online April 11, 2014]
(a) is a polynomial of degree 6 in x.
4- 5 4- 7 (b) is a polynomial of degree 3 in x.
(a) (b)
3 3 (c) is a polynomial of degree 2 in x.
2 7-2 (d) does not depend on x.
(c) (d) 108. If the Rolle’s theorem h olds for the function
3 3
f(x) = 2x3 + ax2 + bx in the interval [– 1, 1] for the point
1 1
- d2y dy
101. If 2 x = y5 +y 5 = and (x2 – 1) + lx
+ ky = 0, 1
2 dx c= , then the value of 2a + b is: [Online April 9, 2014]
dx 2
then l + k is equal to : [Online April 9, 2017] (a) 1 (b) – 1 (c) 2 (d) – 2
(a) – 23 (b) – 24 (c) 26 (d) – 26

Downloaded from @Freebooksforjeeneet


EBD_8344
M-318 Mathematics

109. If f (x) = sin (sin x) and f "(x) + tan x f¢ (x) + g(x) = 0, then 114. Let f be differentiable for all x. If f (1) = – 2 and
g(x) is : [Online April 23, 2013] f '( x) ³ 2 for x Î [1, 6], then [2005]
(a) cos2 x cos (sin x) (b) sin2 x cos (cos x)
(c) sin2 x sin (cos x) (d) cos2 x sin (sin x) (a) f (6) ³ 8(b) f (6) < 8 (c) f (6) < 5 (d) f (6) = 5
110. Consider a quadratic equation ax2 + bx + c = 0, where 115. If the equation an x n + an -1 x n -1 + ............. + a1 x = 0
x3 x2 a1 ¹ 0, n ³ 2, has a positive root x = a , then the equation
2a + 3b + 6c = 0 and let g ( x ) = a + b + cx .
3 2
n -2
[Online May 19, 2012] nan x n -1 + (n – 1) an -1 x + ......... + a1 = 0 has a positive
Statement 1: The quadratic equation has at least one root root, which is [2005]
in the interval (0, 1). (a) greater than a
Statement 2: The Rolle’s theorem is applicable to function (b) smaller than a
g(x) on the interval [0, 1]. (c) greater than or equal to a
(a) Statement 1 is false, Statement 2 is true. (d) equal to a
(b) Statement 1 is true, Statement 2 is false. 116. If 2a + 3b + 6c = 0, then at least one root of the equation
(c) Statement 1 is true, Statement 2 is true, Statement 2 is
not a correct explanation for Statement 1. ax 2 + bx + c = 0 lies in the interval [2004]
(d) Statement 1 is true, Statement 2 is true, , Statement 2 is (a) (1, 3) (b) (1, 2) (c) (2, 3) (d) (0, 1)
a correct explanation for Statement 1.
117. If f ( x) = x n , then the value of [2003]
2
d x
111. equals : [2011] f ' (1) f ' ' (1) f ' ' ' (1) ( -1) n f n (1)
dy 2 f (1) - + - + .......... is
1! 2! 3! n!
-1
æ d2y ö æ dy ö
-3 æ d 2 y ö æ dy ö -2
(a) - ç 2 ÷ çè ÷ø (b) ç 2 ÷ çè dx ÷ø (a) 1 (b) 2 n (c) 2 n - 1 (d) 0.
è dx ø dx è dx ø
118. Let f (a) = g (a) = k and their nth derivatives
-1
æ d 2 y ö æ dy ö -3 æd2yö
- f n (a) , g n (a) exist and are not equal for some n. Further
(c) ç 2 ÷ çè dx ÷ø (d) ç 2 ÷
è dx ø è dx ø if
112. Let f (x) = x | x | and g (x) = sin x. f ( a ) g ( x) - f ( a ) - g ( a ) f ( x ) + f ( a )
Statement-1 : gof is differentiable at x = 0 and its derivative lim =4
x ®a g ( x) - f ( x)
is continuous at that point.
Statement-2 : gof is twice differentiable at x = 0. [2009] then the value of k is [2003]
(a) Statement-1 is true, Statement-2 is true; Statement-2 is (a) 0 (b) 4 (c) 2 (d) 1
not a correct explanation for Statement-1.
d2y dy
(b) Statement-1 is true, Statement-2 is false. 119. If y = (x + 1 + x 2 )n, then (1 + x2) 2
+x is [2002]
dx dx
(c) Statement-1 is false, Statement-2 is true.
(d) Statement-1 is true, Statement-2 is true; Statement-2 is (a) n 2y (b) – n2y
(c) –y (d) 2x2y
a correct explanation for Statement-1. 120. If 2a + 3b + 6c = 0, (a, b, c Î R) then the quadratic equation
113. A value of c for which conclusion of Mean Value Theorem ax2 + bx + c = 0 has [2002]
holds for the function f (x) = loge x on the interval [1, 3] is (a) at least one root in [0, 1]
[2007] (b) at least one root in [2, 3]
(c) at least one root in [4, 5]
1
(a) log3e (b) loge3 (c) 2 log3e (d) log e (d) None of these
2 3

Downloaded from @Freebooksforjeeneet


21
Applications
of Derivatives
5. A 2 m ladder leans against a vertical wall. If the top of the
ladder begins to slide down the wall at the rate 25 cm/sec.,
TOPIC Ć Rate of Change of Quantities then the rate (in cm/sec.) at which the bottom of the ladder
slides away from the wall on the hori ontal ground when
1. The position of a moving car at time t is given by the top of the ladder is 1 m above the ground is:
f (t) = at2 + bt + c, t > 0, where a, b and c are real numbers greater [April 12, 2019 (I)]
than 1. Then the average speed of the car over the time interval
25
[t1, t2] is attained at the point : [Sep. 06, 2020 (I)] (a) 25 3 (b)
3
(a) (t2 – t1)/2 (b) a(t2 – t1) + b
(c) (t1 + t2)/2 (d) 2a(t1 + t2) + b 25
(c) (d) 25
2. If the surface area of a cube is increasing at a rate of 3.6 3
cm2/sec, retaining its shape; then the rate of change of its 6. A spherical iron ball of radius 10 cm is coated with a layer of
ice of uniform thickness that melts at a rate of 50 cm3/min.
volume (in cm3/sec.), when the length of a side of the cube
When the thickness of the ice is 5 cm, then the rate at
is 10 cm, is : [Sep. 03, 2020 (II)]
which the thickness (in cm/min) of the ice decreases, is :
(a) 18 (b) 10 [April 10, 2019 (II)]
(c) 20 (d) 9
3. If a function f (x) defined by [Sep. 02, 2020 (I)] 1 1
(a) (b)
18p 36p
ìae x + be - x , - 1 £ x < 1
ïï 5 1
f ( x) = í cx 2 , 1 £ x £ 3 be continuous for some (c) (d)
ï 2 6p 9p
ïî ax + 2cx , 3 < x £ 4 7. A water tank has the shape of an inverted right circular
a, b, c Î R and f '(0) + f '(2) = e, then the value of a is : cone, whose semi-vertical angle is tan –1 . Water is poured
into it at a constant rate of 5 cubic meter per minute. Then
1 e the rate (in m/min.), at which the level of water is rising at
(a) (b)
e 2 - 3e + 13 e 2 - 3e - 13 the instant when the depth of water in the tank is 10m; is:
[April 09, 2019 (II)]
e e (a) 1/15 p (b) 1/10 p
(c) (d) (d) 1/5 p
e + 3e + 13
2
e - 3e + 13
2 (c) 2/p
8. If the volume of a spherical ball is increasing at the rate of
4. A spherical iron ball of 10 cm radius is coated with a layer 4p cc/sec, then the rate of increase of its radius (in cm/sec),
of ice of uniform thickness that melts at a rate of 50 cm3/ when the volume is 288 p cc, [Online April 19, 2014]
min. When the thickness of ice is 5 cm, then the rate (in
cm/min.) at which of the thickness of ice decreases, is: 1 1
[Jan. 9, 2020 (I)] (a) (b)
6 9
5 1
(a) (b) 1 1
6p 54p
(c) (d)
1 1 36 24
(c) (d)
36p 18p

Downloaded from @Freebooksforjeeneet


EBD_8344
M-340 Mathematics

9. Two ships A and B are sailing straight away from a fixed 16. If a circular iron sheet of radius 30 cm is heated such that
point O along routes such that ÐAOB is always 120 . At a its area increases at the uniform rate of 6p cm2/hr, then the
certain instance, OA = 8 km, OB = 6 km and the ship A is rate (in mm/hr) at which the radius of the circular sheet
sailing at the rate of 20 km/hr while the ship B sailing at the increases is [Online May 7, 2012]
rate of 30 km/hr. Then the distance between A and B is (a) 1.0 (b) 0.1
changing at the rate (in km/hr): [Online April 11, 2014] (c) 1.1 (d) 2.0
17. Two points A and B move from rest along a straight line
260 260
(a) (b) with constant acceleration f and f ' respectively. If A takes
37 37 m sec. more than B and describes ‘n’units more than B in
acquiring the same speed then [2005]
80 80
(c) (d) (a) ( f - f ')m2 = ff ' n
37 37
10. A spherical balloon is being inflated at the rate of 35cc/ (b) ( f + f ')m2 = ff ' n
min. The rate of increase in the surface area (in cm 2/min.)
of the balloon when its diameter is 14 cm, is : 1
(c) ( f + f ')m = ff ' n 2
[Online April 25, 2013] 2
(a) 10 (b) 10 1
(d) ( f '- f )n = ff ' m2
(c) 100 (d) 10 10 2
18. A li ard, at an initial distance of 21 cm behind an insect,
11. If the surface area of a sphere of radius r is increasing
uniformly at the rate 8 cm2/s, then the rate of change of its moves from rest with an acceleration of 2 cm / s 2 and
volume is : [Online April 9, 2013] pursues the insect which is crawling uniformly along a
(a) constant (b) proportional to r straight line at a speed of 20 cm/s. Then the li ard will
catch the insect after [2005]
(c) proportional to r2 (d) proportional to r
(a) 20 s (b) 1 s
12. A spherical balloon is filled with 4500p cubic meters of
(c) 21 s (d) 24 s
helium gas. If a leak in the balloon causes the gas to escape
19. A spherical iron ball 10 cm in radius is coated with a layer
at the rate of 72p cubic meters per minute, then the rate (in
meters per minute) at which the radius of the balloon of ice of uniform thickness that melts at a rate of 50 cm 3 /min.
decreases 49 minutes after the leakage began is: When the thickness of ice is 5 cm,then the rate at which
9 7 the thickness of ice decreases is [2005]
(a) (b) [2012]
7 9 1 1
2 9 (a) cm/min. (b) cm/min.
(c) (d) 36 p 18 p
9 2
1 5
13. If a metallic circular plate of radius 50 cm is heated so that (c) cm/min. (d) cm/min
its radius increases at the rate of 1 mm per hour, then the 54 p 6p
rate at which, the area of the plate increases (in cm 2/hour) 20. A point on the parabola y 2 = 18x at which the ordinate
is [Online May 26, 2012] increases at twice the rate of the abscissa is [2004]
(a) 5 p (b) 10 p
(c) 100 p (d) 50 p æ9 9ö
(a) ç 8 , 2 ÷ (b) (2, - 4)
14. The weight W of a certain stock of fish is given by W = nw, è ø
where n is the si e of stock and w is the average weight of æ -9 9ö
a fish. If n and w change with time t as n = 2t2 + 3 and (c) ç 8 , 2 ÷ (d) (2, 4)
è ø
w = t2 – t + 2, then the rate of change of W with respect to
t at t = 1 is [Online May 19, 2012]
(a) 1 (b) 8 TOPIC n Increasing & Decreasing Functions
(c) 13 (d) 5
15. Consider a rectangle whose length is increasing at the 21. The function, f ( x) = (3x - 7) x 2 / 3 , x Î R, is increasing
uniform rate of 2 m/sec, breadth is decreasing at the uniform
for all x lying in : [Sep. 03, 2020 (I)]
rate of 3 m/sec and the area is decreasing at the uniform
rate of 5 m2/sec. If after some time the breadth of the æ3 ö
(a) (-¥, 0) È æç , ¥ ö÷
14
rectangle is 2 m then the length of the rectangle is (b) (-¥, 0) È ç , ¥ ÷
è 15 ø è7 ø
[Online May 12, 2012]
(a) 2 m (b) 4 m æ 14 ö æ 14 ö
(c) ç -¥, ÷ (d) ç -¥, - ÷ È (0, ¥)
(c) 1 m (d) 3 m è 15 ø è 15 ø

Downloaded from @Freebooksforjeeneet


Applications of Derivatives M-341

22. Let f be any function continuous on [a, b] and twice


differentiable on (a, b). If for all x Î (a, b), f 2 (x) > 0 and x d-x
28. Let f ( x ) = - , x Î R where a, b
b + (d - x)
2 2 2
f (c ) - f ( a ) a +x 2
f ²(x) < 0, then for any c Î (a, b), is greater
f (b) - f ( c )
and d are non- ero real constants. Then :
than: [Jan. 9, 2020 (I)] [Jan. 11, 2019 (II)]
b+ a (a) f is an increasing function of x
(a) (b) 1
b-a (b) f is a decreasing function of x
(c) f ¢ is not a continuous function of x
b-c c-a
(c) (d) (d) f is neither increasing nor decreasing function of x
c-a b-c
29. The function f defined by
é p pù f(x) = x3 – 3x2 + 5x + 7, is : [Online April 9, 2017]
23. Let f(x) = x cos–1 (–sin |x|), x Î ê - , ú , then which of the (a) increasing in R.
ë 2 2û (b) decreasing in R.
following is true? [Jan. 8, 2020 (I)] (c) decreasing in (0, ¥) and increasing in (– ¥ , 0).
æ p ö æ pö (d) increasing in (0, ¥) and decreasing in (– ¥, 0).
(a) f ’ is increasing in çè - ,0÷ø and decreasing in çè 0, ÷ø 30. Let f(x) = sin4x + cos4x. Then f is an increasing function in
2 2
the interval :
p
(b) f’ (0) = – ù 5p 3p ù ù p 5p ù
2 (a) ú , ú (b) ú , ú
(c) f ’ is not differentiable at x = 0 û 8 4û û2 8 û

æ p ö æ pö ùp pù ù pù
(d) f ’ is decreasing in çè - ,0÷ø and increasing in çè 0, ÷ø (c) ú , ú (d) ú 0, ú
2 2 û 4 2û û 4û
24. Let f(x) = ex – x and g(x) = x2 – x, x Î R. Then the set of all 31. Let f and g be two differentiable functions on R such that
x Î R, where the function h(x) = (fog) (x) is increasing, is : f ¢(x) > 0 and g¢(x) < 0 for all x Î R . Then for all x:
[April 10, 2019 (I)]
[Online April 12, 2014]
é -1ù é 1 ö é 1ù
(a) ê -1, ú È ê , ¥ ÷ (b) ê 0, ú È [1, ¥ ) (a) f (g (x)) > f (g (x – 1)) (b) f (g (x)) > f (g (x + 1))
ë 2 û ë2 ø ë 2û (c) g(f (x)) > g (f (x – 1)) (d) g(f (x)) < g (f (x + 1))
é -1 ù 32. The real number k for which the equation, 2x3 + 3x + k = 0
(c) [0,¥ ) (d) ê ,0ú È [1, ¥ ) has two distinct real roots in [0, 1] [2013]
ë2 û
(a) lies between 1 and 2
25. If the function f : R – {1, –1} ® A defin ed by
(b) lies between 2 and 3
x2 (c) lies between .1 and 0
f(x) = , is surective, then A is equal to:
1 - x2 (d) does not exist.
[April 09, 2019 (I)] 33. Statement-1: The function x2 (ex + e–x) is increasing for
(a) R – {–1} (b) [0, 벜) all x > 0.
(c) R – [–1, 0) (d) R – (–1, 0) Statement-2: The functions x2ex and x2e–x are increasing
26. Let f: [0 : 2] ® R be a twice differentiable function such for all x > 0 and the sum of two increasing functions in any
that f''(x) > 0, for all xÎ(0, 2). If f(x) = f(x) + f(2 – x), then f is : interval (a, b) is an increasing function in (a, b).
[April 08, 2019 (I)] [Online April 22, 2013]
(a) increasing on (0, 1) and decreasing on (1, 2). (a) Statement-1 is false; Statement-2 is true.
(b) decreasing on (0, 2) (b) Statement-1is true; Statement-2 is true; Statement-2 is
(c) decreasing on (0, 1) and increasing on (1, 2). not a correct explanation for Statement-1.
(d) increasing on (0, 2) (c) Statement-1 is true; Statement-2 is false.
27. If the function f given by (d) Statement-1is true; Statement-2 is true; Statement-2 is
f (x) = x3 – 3(a – 2)x2 + 3ax + 7, for some aÎR is increasing a correct explanation for statement-1.
in (0, 1] and decreasing in [1, 5), then a root of the equation, 34. Statement-1: The equation x log x = 2 – x is satisfied by at
least one value of x lying between 1 and 2.
f ( x ) - 14
= 0( x ¹ 1) is [Jan. 12, 2019 (II)] Statement-2: The function f (x) = x log x is an increasing
( x - 1) 2 function in [l, 2] and g (x) = 2 – x is a decreasing function in
(a) –7 (b) 5 [1, 2] and the graphs represented by these functions
(c) 7 (d) 6 intersect at a point in [1, 2] [Online April 9, 2013]

Downloaded from @Freebooksforjeeneet


EBD_8344
M-342 Mathematics

(a) Statement-1 is true; Statement-2 is true; Statement-2 41. Which of the following points lies on the tangent to the
is a correct explanation for Statement-1. curve x4 e y + 2 y + 1 = 3 at the point (1, 0)?
(b) Statement-1 is true; Statement-2 is true; Statement-2
is not correct explanation for Statement-1. [Sep. 05, 2020 (II)]
(c) Statement-1 is false, Statement-2 is true. (a) (2, 2) (b) (2, 6)
(d) Statement-1 is true, Statement-2 is false. (c) (– 2, 6) (d) (– 2, 4)
35. If f(x) = xex(1 – x), x Î R , then f(x) is 42. If the lines x + y = a and x – y = b touch the curve
[Online May 12, 2012] y = x2 – 3x + 2 at the points where the curve intersects the
(a) decreasing on [–1/2, 1] a
(b) decreasing on R x-axis, then is equal to _____. [NA Sep. 05, 2020 (II)]
b
(c) increasing on [–1/2, 1] 43. If the tangent to the curve, y = ex at a point (c, ec) and the
(d) increasing on R normal to the parabola, y2 = 4x at the point (1, 2) intersect
36. For real x, let f (x) = x3 + 5x + 1, then [2009] at the same point on the x-axis, then the value of c is
(a) f is onto R but not one-one ____________. [NA Sep. 03, 2020 (II)]
(b) f is one-one and onto R
6
-1 ì 3 ü
(c) f is neither one-one nor onto R
å k cos
4 dy
(d) f is one-one but not onto R 44. If y = í cos kx - sin kx ý, then at x = 0 is
k =1 î 5 5 þ dx
37. How many real solutions does the equation
x7 + 14x5 + 16x3 + 30x – 560 = 0 have? [2008] ___________. [NA Sep. 02, 2020 (II)]
(a) 7 (b) 1 45. Let the normal at a point P on the curve y2 – 3x2 + y + 10 = 0
(c) 3 (d) 5
38. The function f (x) = tan –1(sin x + cos x) is an increasing æ 3ö
intersect the y-axis at ç 0, ÷ . If m is the slope of the
function in [2007] è 2ø
tangent at P to the curve, then |m| is equal to ______.
p æ p pö
(a) æç 0, ö÷ (b) ç - , ÷ [NA Jan. 8, 2020 (I)]
è 2ø è 2 2ø
46. The length of the perpendicular from the origin, on the
p p
(c) æç p , p ö÷ (d) æç - , ö÷
normal to the curve, x 2 + 2xy – 3y2 = 0 at the point (2, 2)
è 4 2ø è 2 4ø is: [Jan. 8, 2020 (II)]
39. A function is matched below against an interval where it is (a) 2 (b) 4 2
supposed to be increasing. Which of the following pairs is (c) 2 (d) 2 2
incorrectly matched? [2005]

( )
Interval Function x
47. If the tangent to the curve y = , x Î R, x ¹ ± 3 ,
(a) (– ¥ , ¥ ) 3 2
x - 3x + 3x + 3 x2 - 3
at a point (a, b) (0, 0) on it is parallel to the line
(b) [2, ¥ ) 2 x3 - 3 x 2 - 12 x + 6 2x + 6y – 11 = 0, then : [April 10, 2019 (II)]
æ 1ù (a) |6a + 2b| = 19 (b) |6a + 2b| = 9
(c) ç - ¥, ú 3x 2 - 2 x + 1
è 3û (c) |2a + 6b| = 19 (d) |2a + 6b| = 11
48. If the tangent to the curve, y = x3 + ax – b at the point
(d) (– ¥ , – 4) x3 + 6 x 2 + 6 (1, –5) is perpendicular to the line, – x + y + 4 = 0, then
which one of the following points lies on the curve?
[April 09, 2019 (I)]
TOPIC Đ Tangents & Normals
(a) (–2, 1) (b) (–2, 2)
(c) (2, –1) (d) (2, –2)
40. If the tangent to the curve, y = f (x) = xlogex, (x > 0) at a 49. Let S be the set of all values of x for which the tangent to
point (c, f(c)) is parallel to the line segement oining the the curve y = f(x) = x3 – x2 – 2x at (x, y) is parallel to the line
points (1, 0) and (e, e), then c is equal to: segment oining the points (1, f(1)) and (– 1, f(– 1)), then S
[Sep. 06, 2020 (II)] is equal to: [April 09, 2019 (I)]
æ 1 ö ì1 ü
(b) ìí– , –1üý
1
e -1 ç ÷ (a) í ,1ý
(a) (b) eè e-1 ø î3 þ î 3 þ
e
æ 1 ö
ç ÷
ì1 ü ì 1 ü
e (c) í , –1ý (d) í– ,1ý
(c) eè 1-e ø (d) î3 þ î 3 þ
e -1

Downloaded from @Freebooksforjeeneet


Applications of Derivatives M-343

50. The tangent and the normal lines at the point ( 3 , 1) to


5 1 7
the circle x2 + y2 = 4 and the x-axis form a triangle. The area (a) (b)
of this triangle (in square units) is : [April 08, 2019 (II)] 6 3 3

4 1
(a) (b) 1 7 1
3 3 (c) (d)
6 3 2
2 1 56. If q denotes the acute angle between the curves,
(c) (d)
3 3 y = 10 – x2 and y = 2 + x2 at a point of their intersection,
51. The maximum area (in sq. units) of a rectangle having its then |tan q| is equal to: [Jan. 09, 2019 (I)]
base on the x-axis and its other two vertices on the parabola, 4 8
(a) (b)
y = 12 – x2 such that the rectangle lies inside the parabola, 9 15
is: [Jan. 12, 2019 (I)] 7 8
(c) (d)
(a) 36 (b) 20 2 17 17
(c) 32 (d) 18 3 57. If the curves y2 = 6x,9x 2 + by 2 = 16 intersect each other
52. The tangent to the curve y = x2 – 5x + 5, parallel to the line at right angles, then the value of b is : [2018]
2y = 4x + 1, also passes through the point : 7
(a) (b) 4
[Jan. 12, 2019 (II)] 2

æ7 1ö æ1 ö 9
(a) ç , ÷ (b) ç , -7 ÷ (c) (d) 6
è2 4ø è8 ø 2
58. Let P be a point on the parabola,x2 = 4y. If the distance of
æ 1 ö æ1 7ö P from the centre of the circle, x2 + y2 + 6x + 8 = 0 is
(c) ç - , 7 ÷ (d) ç , ÷ minimum, then the equation of the tangent to the parabola
è 8 ø è4 2ø at P, is [Online April 16, 2018]
(a) x + 4y – 2 = 0 (b) x + 2y = 0
æ3 ö (c) x + y + 1 = 0 (d) x – y + 3 = 0
53. The shortest distance between the point ç , 0 ÷ and the
è2 ø 59. If the tangents drawn to the hyperbola 4y2 = x2 + 1 intersect
the co-ordinate axes at the distinct points A and B, then
curve y = x , ( x > 0), is: [Jan. 10, 2019 (I)]
the locus of the mid point of AB is[Online April 15, 2018]
(a) x2 – 4y2 + 16 x2 y2 = 0
5 3 (b) 4x2 – y2 + 16 x2 y2 = 0
(a) (b)
2 2 (c) 4x2 – y2 – 16 x2 y2 = 0
3 5 (d) x2 – 4y2 – 16 x2 y2 = 0
(c) (d) 60. If b is one of the angles between the normals to the ellipse,
2 4
x 2 + 3y 2 = 9 at the points (3cosq, 3 sin q) and
x2
54. The tangent to the curve, y = xe passing through the æ pö 2 cot b
(– 3sin q, 3 cos q); Î ç 0, ÷ ; then is equal to
point (1, e) also passes through the point: è 2ø sin 2q
[Jan. 10, 2019 (II)] [Online April 15, 2018]

æ4 ö 2
(a) (2, 3e) (b) ç , 2e ÷ (a) 2 (b)
è3 ø 3

1 3
æ5 ö (c) (d)
(c) ç , 2e ÷ (d) (3, 6e) 3 4
è3 ø 61. A normal to the hyperbola, 4x2 – 9y2 = 36 meets the co-
55. A helicopter is flyin g along the curve given by ordinate axes x and y at A and B, respectively. If the
parallelogram OABP (O being the origin) is formed, then
æ1 ö the locus of P is [Online April 15, 2018]
y – x3/2 = 7, (x ³ 0). A soldier positioned at the point ç , 7 ÷ (a) 4x2 – 9y2 = 121
è2 ø
(b) 4x2 + 9y2 = 121
wants to shoot down the helicopter when it is nearest to (c) 9x2 – 4y2 = 169
him. Then this nearest distance is: [Jan. 10, 2019 (II)] (d) 9x2 + 4y2 = 169

Downloaded from @Freebooksforjeeneet


EBD_8344
M-344 Mathematics

62. The normal to the curve y(x – 2)(x – 3) = x + 6 at the point 68. If the tangent at a point P, with parameter t, on the curve
where the curve intersects the y-axis passes through the x = 4t2 + 3, y = 8t3 – 1, t Î R, meets the curve again at a
point: [2017] point Q, then the coordinates of Q are :
[Online April 9, 2016]
æ 1 1ö æ 1 1ö
(a) çè , ÷ø (b) ç - , - ÷ (a) (16t2 + 3, –64t3 – 1) (b) (4t2 + 3, –8t3 – 2)
2 3 è 2 2ø
(c) (t2 + 3, t3 – 1) (d) (t2 + 3, – t3 – 1)
æ 1 1ö æ 1 1ö
(c) ç , ÷ (d) ç , - ÷ 69. The normal to the curve, x2 + 2xy – 3y2 = 0, at (1, 1) [2015]
è 2 2ø è 2 3ø (a) meets the curve again in the third quadrant.
63. The eccentricity of an ellipse whose centre is at the (b) meets the curve again in the fourth quadrant.
1 (c) does not meet the curve again.
origin is . If one of its directices is x = – 4, then the (d) meets the curve again in the second quadrant.
2
70. The equation of a normal to the curve,
æ 3ö æp ö
equation of the normal to it at ç1, ÷ is : [2017]
è 2ø sin y = x sin ç + y ÷ at x = 0, is :
(a) x + 2y = 4 (b) 2y – x = 2 è3 ø
[Online April 11, 2015]
(c) 4x – 2y = 1 (d) 4x + 2y = 7
64. A tangent to the curve, y = f (x) at P(x, y) meets x-axis at A (a) 2x – 3y = 0 (b) 2x + 3y = 0
and y-axis at B. If AP : BP = 1 : 3 and f (a) = 1 , then the curve
(c) 2y – 3x = 0 (d) 2y + 3x = 0
also passes through the point : [Online April 9, 2017]
æ1 ö 71. If the tangent to the conic, y – 6 = x2 at (2, 10) touches the
(a) æç , 24 ö÷
1
(b) ç , 4 ÷ circle, x2 + y2 + 8x – 2y = k (for some fixed k) at a point
è3 ø è2 ø
(a, b) ; then (a, b) is : [Online April 10, 2015]
æ 1ö æ 1 ö æ 7 6ö æ 4 1ö
(c) ç 2, ÷ (d) ç 3, ÷ (a) çè - , ÷ø (b) çè - , ÷ø
è 8 ø è 28 ø 17 17 17 17
65. The tangent at the point (2, –2) to the curve,
æ 6 10 ö æ 8 2ö
x2y2 – 2x = 4 (1–y) does not pass through the point : (c) çè - , ÷ø (d) çè - , ÷ø
17 17 17 17
[Online April 8, 2017]
72. The distance, from the origin, of the normal to the curve,
æ 1ö
(a) ç 4, ÷ (b) (8, 5) p
è 3ø x = 2 cost + 2t sint, y = 2 sint – 2t cost at t = , is :
4
(c) (– 4, – 9) (d) (– 2, – 7) [Online April 10, 2015]
(a) 2 (b) 4
66. Consider
(c) 2 (d) 2 2
æ 1 + sin x ö æ pö For the curve y = 3 sinq cosq, x = eq sin q, 0 £ q £ p, the
f ( x ) = tan -1 ç ÷÷ , x Î ç 0, ÷ .
73.
ç 1 - sin x è 2ø
[2016] tangent is parallel to x-axis when q is:
è ø
[Online April 11, 2014]
p
A normal to y = f(x) at x = also passes through the point: 3p p
6 (a) (b)
4 2
æp ö æp ö
(a) ç , 0 ÷ (b) ç , 0 ÷ p p
è6 ø è4 ø (c) (d)
4 6
æ 2p ö 74. If an equation of a tangent to th e curve,
(c) (0, 0) (d) ç 0, ÷
è 3 ø y – cos(x + f), – 1 -1 £ x £ 1 + p, is x + 2y = k then k is equal
3 to : [Online April 25, 2013]
67. Let C be a curve given by y(x) = 1 + 4x - 3, x >
. If P is (a) l (b) 2
4
2 p p
a point on C, such that the tangent at P has slope , then (c) (d)
3 4 2
a point through which the normal at P passes, is : 75. The equation of the normal to the parabola,
[Online April 10, 2016] x2 = 8y at x = 4 is [Online May 19, 2012]
(a) (1, 7) (b) (3, –4) (a) x + 2y = 0 (b) x + y = 2
(c) (4, –3) (d) (2, 3) (c) x – 2y = 0 (d) x + y = 6

Downloaded from @Freebooksforjeeneet


Applications of Derivatives M-345

4 83. The set of all real values of l for which the function
76. The equation of the tangent to the curve y = x + 2 , that æ p pö
x f ( x) = (1 - cos2 x ).(l + sin x) , x Î ç - , ÷ , has exactly
one maxima and exactly minima, is: è
2 2ø
is parallel to the x-axis, is [2010]
(a) y = 1 (b) y = 2 [Sep. 06, 2020 (II)]
(c) y = 3 (d) y = 0
æ 1 1ö æ 3 3ö
77. Angle between the tangents to the curve y = x 2 - 5 x + 6 (a) ç - , ÷ - {0} (b) ç - , ÷
è 2 2ø è 2 2ø
at the points (2, 0) and (3, 0) is [2006]
p æ 1 1ö æ 3 3ö
(a) p (b) (c) ç - , ÷ (d) ç - , ÷ - {0}
2 è 2 2ø è 2 2ø
p p 84. If x = 1 is a critical point of the function
(c) (d)
6 4
f ( x) = (3x 2 + ax - 2 - a)e x , then : [Sep. 05, 2020 (II)]
78. The normal to the curve [2005]
x = a (cos q + q sin q ), y = a (sin q – q cos q ) at any 2
point q is such that (a) x = 1 and x = - are local minima of f.
3
(a) it passes through the origin
2
p (b) x = 1 and x = - are local maxima of f.
(b) it makes an angle + q with the x- axis 3
2
2
æ p ö (c) x = 1 is a local maxima and x = - is a local minima of f.
(c) it passes through ç a , - a÷ 3
è 2 ø
(d) It is at a constant distance from the origin 2
(d) x = 1 is a local minima and x = - is a local maxima of f.
79. The normal to the curve x = a(1 + cos q), y = a sinq at ‘q’ 3
always passes through the fixed point [2004] 85. The area (in sq. units) of the largest rectangle ABCD whose
(a) (a, a) (b) (0, a) vertices A and B lie on the x-axis and vertices C and D lie
(c) (0, 0) (d) (a, 0) on the parabola, y = x2 – 1 below the x-axis, is :
80. A function y = f ( x ) has a second order derivative [Sep. 04, 2020 (II)]
f "( x) = 6( x - 1). If its graph passes through the point 2 1
(a) (b)
(2,1) and at that point the tangent to the graph is y = 3x – 3 3 3 3
5, then the function is [2004]
4 4
(a) ( x + 1)2 (b) ( x - 1)3 (c) (d)
3 3 3
(c) ( x + 1)3 (d) ( x - 1)2 86. Suppose f(x) is a polynomial of degree four, having critical
points at –1, 0, 1. If T = {x Î R | f ( x ) = f (0)}, then the
TOPIC Ė Approximations, Maxima & Minima sum of squares of all the elements of T is :
[Sep. 03, 2020 (II)]
81. Let m and M be respectively the minimum and maximum (a) 4 (b) 6
values of [Sep. 06, 2020 (I)] (c) 2 (d) 8
87. Let f(x) be a polynomial of degree 3 such that f(–1) = 10,
cos 2 x 1 + sin 2 x sin 2 x
f(1)= –6, f(x) has a critical point at x = –1 and f ¢(x) has a
1 + cos 2 x sin 2 x sin 2 x critical point at x = 1. Then f(x) has a local minima at
2 2 x = ________. [NA Jan. 8, 2020 (II)]
cos x sin x 1 + sin 2 x
88. Let f(x) be a polynomial of degree 5 such that x = ±1 are its
Then the ordered pair (m, M) is equal to : critical points. If = 4, then which one of the following is
(a) (– 3, 3) (b) (– 3, – 1) not true ? [Jan. 7, 2020 (II)]
(c) (– 4, – 1) (d) (1, 3) (a) f is an odd function.
82. Let AD and BC be two vertical poles at A and B respectively (b) f(l) – 4f(–l) = 4.
on a hori ontal ground. If AD = 8 m, BC = 11 m and AB = 10 (c) x = 1 is a point of maxima and x = –1 is a point of
m; then the distance (in meters) of a point M on AB from minima of f.
the point A such that MD2 + MC2 is minimum is ______. (d) x = 1 is a point of minima and x = –1 is a point of
[NA Sep. 06, 2020 (I)] maxima of f.

Downloaded from @Freebooksforjeeneet


EBD_8344
M-346 Mathematics

89. If m is the minimum value of k for which the function 96. Let x, y be positive real numbers and m, n positive integers.

f ( x ) = x kx - x 2 is increasing in the interval [0,3] and M xm y n


The maximum value of the expression
is the maximum value of f in [0,3] when k = m, then the
ordered pair (m, M) is equal to : [April 12, 2019 (I)]
(1 + x2m )(1 + y2n )
(a) (4,3 2) (b) (4,3 3) is : [Jan. 11, 2019 (II)]
1
(c) (3,3 3) (d) (5,3 6) (a) 1 (b)
2
90. Let a1, a2, a3, …. be an A. P. with a6 = 2. Then the common 1 m+n
difference of this A.P., which maximises the product a1 a 4 (c) (d)
4 6mn
a 5, is : [April 10, 2019 (II)] 97. The maximum volume (in cu.m) of the right circular cone
3 8 having slant height 3 m is: [Jan. 09, 2019 (I)]
(a) (b)
2 5 (a) 6 p (b) 3 3 p
6 2 4
(c)
5
(d)
3 (c) p (d) 2 3 p
3
91. If S1 and S2 are respectively the sets of local minimum and
1 1
local maximum points of the function, 98. Let f (x) = x 2 +
2
and g(x) = x - , x Î R - {-1,0,1} .
f(x) = 9x4 + 12x3 – 36x2 + 25, xÎR, then : x x
[April 08, 2019 (I)] f (x)
If h(x) = , then the local minimum value of h(x) is :
(a) S1 = {–2}; S2 = {0, 1} (b) S1 = {–2, 0}; S2 = {1} g(x)
(c) S1 = {–2, 1}; S2 = {0} (d) S1 = {–1}; S2 = {0, 2} [2018]
92. The height of a right circular cylinder of maximum volume (a) – 3 (b) -2 2
inscribed in a sphere of radius 3 is : [April 08, 2019 (II)]
(c) 2 2 (d) 3
2 99. Let M and m be respectively the absolute maximum and
(a) 6 (b) 3
3 the absolute minimum values of the function,
(c) 2 3 (d) 3 f (x) = 2x3 – 9x2 + 12x + 5 in the interval [0, 3]. Then
M – m is equal to [Online April 16, 2018]
æ pö (a) 1 (b) 5
93. The maximum value of 3cos q + 5sin ç q - ÷ for any real (c) 4 (d) 9
è 6ø
value of q is: [Jan. 12, 2019 (I)] 100. If a right circularcone having maximum volume, is inscribed
in a sphere of radius 3 cm, then the curved surface area
79 (in cm2) of this cone is [Online April 15, 2018]
(a) 19 (b)
2 (a) 8 3p (b) 6 2p
(c) 34 (d) 31 (c) 6 3p (d) 8 2p
94. Let P(4, –4) and Q(9, 6) be two points on the parabola, 101. Twenty metres of wire is available for fencing off a flower-
bed in the form of a circular sector. Then the maximum area
y2 = 4x and let this X be any point arc POQ of this
(in sq. m) of the flower-bed, is : [2017]
parabola, where O is vertex of the parabola, such that (a) 30 (b) 12.5
the area of DPXQ is maximum. Then this minimum area (c) 10 (d) 25
(in sq. units) is: [Jan. 12, 2019 (I)] 102. A wire of length 2 units is cut into two parts which are bent
respectively to form a square of side = x units and a circle
75 125
(a) (b) of radius = r units. If the sum of the areas of the square and
2 4 the circle so formed is minimum, then: [2016]
625 125 (a) x = 2r (b) 2x = r
(c) (d) (c) 2x = (p+ 4)r (d) (4– p) x= pr
4 2
103. The minimum distance of a point on the curve y = x2 – 4
95. The maximum value of the function f(x) = 3x3 – 18x2 + 27 x – 40 from the origin is : [Online April 9, 2016]
{ 2
on the set S = x Î R : x + 30 £ 11x is : } (a)
15
(b)
19
[Jan. 11, 2019 (I)] 2 2
(a) – 122 (b) – 222 15 19
(c) 122 (d) 222 (c) (d)
2 2

Downloaded from @Freebooksforjeeneet


Applications of Derivatives M-347

104. Let k and K be the minimum and the maximum values of 111. Let a, b Î R be such that the function f given by
f (x) = ln | x | + bx2 + ax, x ¹ 0 has extreme values at x = –1
(1 + x )0.6 and x = 2
the function f(x) = in [0, 1] respectively, then
1 + x 0.6 Statement-1 : f has local maximum at x = –1 and at x = 2.
the ordered pair (k, K) is equal to : 1 -1
Statement-2 : a = and b = [2012]
[Online April 11, 2015] 2 4
(a) (2–0.4, 1) (b) (2–0.4, 20.6) (a) Statement-1 is false, Statement-2 is true.
(c) (2–0.6, 1) (d) (1, 20.6) (b) Statement-1 is true, statement-2 is true; statement-2 is
105. From the top of a 64 metres high tower, a stone is thrown a correct explanation for Statement-1.
upwards vertically with the velocity of 48 m/s. The (c) Statement-1 is true, statement-2 is true; statement-2 is
greatest height (in metres) attained by the stone, not a correct explanation for Statement-1.
(d) Statement-1 is true, statement-2 is false.
assuming the value of the gravitational acceleration g =
112. A line is drawn through the point (1,2) to meet the
32 m s2, is: [Online April 11, 2015]
coordinate axes at P and Q such that it forms a triangle
(a) 128 (b) 88
OPQ, where O is the origin. If the area of the triangle OPQ
(c) 112 (d) 100 is least, then the slope of the line PQ is : [2012]
106. If x = –1 and x = 2 are extreme points of
1
(a) - (b) – 4
f ( x ) = a log x + b x 2 + x then [2014] 4
1
1 1 (c) – 2 (d) -
(a) a = 2, b = - (b) a = 2, b = 2
2 2
113. Let f: ( -¥, ¥ ) ® ( -¥, ¥ ) be defined by
1 1
(c) a = -6, b = (d) a = -6, b = - f(x) = x3 + 1. [Online May 26, 2012]
2 2
Statement 1: The function f has a local extremum at x = 0
107. The minimum area of a triangle formed by any tangent to
Statement 2: The function f is continuous and
x 2 y2
the ellipse + = 1 and the co-ordinate axes is: differentiable on ( -¥, ¥ ) and f ¢(0) = 0
16 81
(a) Statement 1 is true, Statement 2 is false.
[Online April 12, 2014]
(a) 12 (b) 18 (b) Statement 1 is true, Statement 2 is true, Statement 2 is
a correct explanation for Statement 1.
(c) 26 (d) 36
(c) Statement 1 is true, Statement 2 is true, Statement 2 is
108. The volume of the largest possible right circular cylinder
not the correct explanation for Statement 1.
that can be inscribed in a sphere of radius = 3 is: (d) Statement 1 is false, Statement 2 is true.
[Online April 11, 2014] 114. Let f be a function defined by - [2011RS]
4 8 ì tan x
(a) 3p (b) 3p ï , x¹0
3 3 f (x) = í x
ï1, x=0
(c) 4p (d) 2p î
æ bö Statement - 1 : x = 0 is point of minima of f
109. The cost of running a bus from A to B, is ` ç av + ÷ ,
è vø Statement - 2 : f ¢ ( 0) = 0.
where v km/h is the average speed of the bus. When the (a) Statement-1 is true, statement-2 is true; statement-2 is
bus travels at 30 km/h, the cost comes out to be ` 75 while a correct explanation for statement-1.
at 40 km/h, it is ` 65. Then the most economical speed (in (b) Statement-1 is true, statement-2 is true; statement-2 is
km/ h) of the bus is : [Online April 23, 2013] NOT a correct explanation for statement-1.
(a) 45 (b) 50 (c) Statement-1 is true, statement-2 is false.
(c) 60 (d) 40 (d) Statement-1 is false, statement-2 is true.
110. The maximum area of a right angled triangle with æ 5p ö
x
hypotenuse h is : [Online April 22, 2013] 115. For x Î ç 0, ÷ , define f ( x) = ò t sin t dt . Then f has
è 2 ø
0
h2 h2 [2011]
(a) (b) (a) local minimum at p and 2p
2 2 2
(b) local minimum at p and local maximum at 2p
h2 h2 (c) local maximum at p and local minimum at 2p
(c) (d) (d) local maximum at p and 2p
2 4

Downloaded from @Freebooksforjeeneet


EBD_8344
M-348 Mathematics

116. Let f : R ® R be a continuous function defined by


p p
1 (a) The cubic has minima at and maxima at –
f ( x) = [2010] 3 3
e x + 2e - x
p p
1 (b) The cubic has minima at – and maxima at
Statement -1 : f (c) = , for some c Î R. 3 3
3
p p
1 (c) The cubic has minima at both and –
Statement -2 : 0 < f (x) £ , for all x Î R 3 3
2 2
(a) Statement -1 is true, Statement -2 is true ; Statement - p p
(d) The cubic has maxima at both and –
2 is not a correct explanation for Statement -1. 3 3
(b) Statement -1 is true, Statement -2 is false. x 2
(c) Statement -1 is false, Statement -2 is true . 120. The function f ( x ) = + has a local minimum at
2 x
(d) Statement - 1 is true, Statement 2 is true ; Statement -2
is a correct explanation for Statement -1. (a) x = 2 (b) x = -2 [2006]
117. Let f : R ® R be defined by (c) x = 0 (d) x = 1

{ k2-x+23,x, ifif xx £>--11


121. The real number x when added to its inverse gives the
f ( x) = minimum value of the sum at x equal to
(a) –2 (b) 2 [2003]
If f has a local minimum at x = – 1 , then a possible value of (c) 1 (d) –1
k is [2010]
122. If the function f ( x) = 2 x3 - 9ax 2 + 12a 2 x + 1 , where
1 a > 0 , attains its maximum and minimum at p and q
(a) 0 (b) -
2 respectively such that p 2 = q , then a equals [2003]
(c) –1 (d) 1
118. Given P(x) = x4 + ax3 + bx2 + cx + d such that x = 0 is the 1
(a) (b) 3
only real root of P' (x) = 0. If P(–1) < P(1), then in the interval 2
[ –1, 1] : [2009] (c) 1 (d) 2
(a) P(–1) is not minimum but P(1) is the maximum of P 123. The maximum distance from origin of a point on the curve
(b) P(–1) is the minimum but P(1) is not the maximum of P
(c) Neither P(–1) is the minimum nor P(1) is the maximum æ at ö æ at ö
x = a sin t–b sin çè ÷ø , y = a cos t – b cos çè ÷ø , both
of P b b
(d) P(–1) is the minimum and P(1) is the maximum of P a, b > 0 is
119. Suppose the cubic x3 – px + q has three distinct real roots (a) a – b (b) a + b [2002]
where p > 0 and q > 0. Then which one of the following
holds? [2008] (c) a 2 + b2 (d) a 2 - b2

Downloaded from @Freebooksforjeeneet


EBD_8344
M-374 Mathematics

22
Integrals
x
Standard Integrals, Integration by
TOPIC Ć Substitution, Integration by Parts 5. Let f ( x) = ò dx ( x ³ 0). Then f (3) – f (1) is equal
(1 + x)2
to : [Sep. 04, 2020 (I)]
æ ( x -1) 2 ö p 1
ç ò0 t cos(t 2 )dt ÷ p 1 3 3
(a) - + + (b) + -
1. lim ç ÷ [Sep. 06, 2020 (I)] 12 2 4 6 2 4
x ®1 ç ( x - 1)sin( x - 1) ÷
ç ÷
è ø p 1 3 p 1 3
(c) - + + (d) + -
1 6 2 4 12 2 4
(a) is equal to (b) is equal to 1
2 æ x ö
(c) is equal to –
1
(d) does not exist
6. ò
If sin -1 çç
è 1 + x ÷÷
ø
dx = A( x) tan -1 ( x ) + B ( x ) + C , wher e
2
C is a constant of integration, then the ordered pair (A(x),
-x (e x + e - x ) x -x
If ò (e + 2e - e - 1) e dx = g ( x)e(e + e ) + c ,
2x x B(x)) can be : [Sep. 03, 2020 (II)]
2.
where c is a constant of integeration, then g (0) is equal to: (a) ( x + 1, - x ) (b) ( x + 1, x)
[Sep. 05, 2020 (I)] (c) ( x - 1, - x ) (d) ( x - 1, x)
(a) e (b) e 2 (c) 1 (d) 2
cos q dx
3. If ò 5 + 7 sin q - 2 cos 2 qd q = A loge | B(q) | +C , where 7. The integral ò ( x + 4)8/7 ( x - 3)6/7 is equal to:
(where C is a constant of integration) [Jan. 9, 2020 (I)]
B (q )
C is a constant of integration, then can be : 1/7 -1/7
A æ x-3 ö æ x -3ö
(a) ç ÷ +C (b) - ç ÷ +C
[Sep. 05, 2020 (II)] è x+4ø è x+4ø
2sin q + 1 -13/7
2sin q + 1 1æ x-3 ö
3/7
1 æ x-3 ö
+C (d) - +C
2 çè x + 4 ÷ø 13 çè x + 4 ÷ø
(a) (b) (c)
sin q + 3 5(sin q + 3)
5(sin q + 3) 5(2 sin q + 1) dq
(c)
2 sin q + 1
(d)
sin q + 3 8. If ò cos2 q(tan 2q + sec 2q) = ltanq + 2loge|f(q)| + C where

æ x ö
2 C is a constant of integration, then the ordered pair
4. The integral ò ç ÷ dx is equal to (l, f(q)) is equal to: [Jan. 9, 2020 (II)]
è x sin x + cos x ø (a) (1, 1 – tanq) (b) (–1, 1 – tanq)
(where C is a constant of integration) : [Sep. 04, 2020 (I)] (c) (–1, 1 + tanq) (d) (1, 1 + tanq)
x sec x
(a) tan x - +C cos x dx 1/l
ò sin3 x(1 + sin 6 x)2/3 = f ( x)(1 + sin
6
x sin x + cos x 9. If x) + c where c is
x tan x
(b) sec x + +C
æ pö
x sin x + cos x
a constant of integration, then lf çè ÷ø is equal to:
x tan x 3
(c) sec x - +C [Jan. 8, 2020 (II)]
x sin x + cos x
x sec x 9 9
(d) tan x + +C (a) - (b) 2 (c) (d) –2
x sin x + cos x 8 8

Downloaded from @Freebooksforjeeneet


Integrals M-375

If ò esec (sec x tan x f ( x ) + (sec x tan x + sec 2 x )) dx


x
2 x3 - 1 15.
10. The integral ò x4 + x dx is equal to : = esec x f(x) + C, then a possible choice of f(x) is:
(Here C is a constant of integration) [April 12, 2019 (I)] [April 09, 2019 (II)]
( x 3 + 1)2 1 1
1 x3 + 1 1
log e +C (a) sec x + tan x + (b) sec x - tan x -
(a) log e +C (b) 2 x3
2 2
2 x2 1 1
(c) sec x + x tan x - (d) x sec x + tan x +
x3 + 1 x3 + 1 2 2
(c) log e +C (d) log e +C
x x2 5x
sin
Let a Î (0, p/2) be fixed. If the integral
11.
tan x + tan a
16. ò 2x dx is equal to :
ò tan x - tan a dx = A(x) cos2a+B(x) sin2a+C, where C is
sin
2
a constant of integration, then the functions A(x) and (where c is a constant of integration.) [April 08, 2019 (I)]
B(x) are respectively : [April 12, 2019 (II)] (a) 2x + sinx + 2 sin2x + c (b) x + 2 sinx + 2 sin2x + c
(c) x + 2 sinx + sin2x + c (d) 2x + sinx + sin2x + c
(a) x + a and loge sin( x + a) 1
dx
(b) x - a and loge sin( x - a) 17. If ò 3 = xf ( x ) (1 + x ) + C , where C is a
6 3
x (1 + x )
6 2/3
(c) x - a and loge cos( x - a) constant of integration, then the function f(x) is equal to :
(d) x + a and loge sin( x - a) [April 08, 2019 (II)]
3 1 1 1
dx (a) 2 (b) – 3 (c) – 2 (d) – 3
12. If ò ( x2 - 2 x + 10) 2 x 6x 2x 2x
18. Theintegral ò cos ( log e x )dx is equal to :
æ x -1 ö ö
= A ç tan -1 çæ
f ( x)
÷+ 2 ÷ + C where C is a (where C is a constant of integration) [Jan. 12, 2019 (I)]
è è 3 ø x - 2 x + 10 ø
x
ésin ( log e x ) - cos ( log e x ) ûù + C

constant of integration, then : [April 10, 2019 (I)] (a)
1
(a) A = and f(x) = 3 (x – 1) (b) x éëcos ( log e x ) + sin ( log e x ) ùû + C
54
x
écos ( loge x ) + sin ( log e x ) ûù + C
1
A=

(b) and f(x) = 3 (x – 1) (c)
81
1 (d) x éëcos ( log e x ) - sin ( log e x ) ùû + C
(c) A = and f(x) = 9 (x – 1)
27
3x13 + 2 x11
(d) A =
1
and f(x) = 9 (x – 1)2
19. The integral ò (2 x 4 + 3x 2 + 1) 4 dx is equal to:
54
13. If f(x) is a non- ero polynomial of degree four, having (where C is a constant of integration) [Jan. 12, 2019 (II)]
local extreme points at x = –1, 0, 1; then the set
S = {x Î R : f(x) = f(0)}contains exactly: x4 x12
(a) + C (b) +C
[April 09, 2019 (I)] 6(2 x 4 + 3x 2 + 1)3 6(2 x 4 + 3x 2 + 1)3
(a) four irrational numbers.
(b) four rational numbers.
x4 x12
(c) two irrational and two rational numbers. (c) + C (d) +C
(d) two irrational and one rational number. (2 x 4 + 3 x 2 + 1)3 (2 x 4 + 3 x 2 + 1)3
14. The integral ò sec 2/3 x cosec 4/3 xdx is equal to:
1 - x2 m
[April 09, 2019 (I)] 20. If ò dx = A(x) æç 1 - x 2 ö÷ + C , for a suitable
x4 è ø
3
(a) –3 tan–1/3 x + C (b) – tan–4/3 x + C chosen integer m and a function A (x), where C is a constant
4
(c) –3 cot–1/3 x + C (d) 3 tan–1/3 x + C of integration, then (A(x))m equals : [Jan. 11, 2019 (I)]
(Here C is a constant of integration) -1 -1 1 1
(a) 9 (b) 3 (c) 6 (d)
27x 3x 27x 9x 4

Downloaded from @Freebooksforjeeneet


EBD_8344
M-376 Mathematics

x +1 25. The integral


21. If ò dx = f ( x ) 2 x - 1 + C , where C is a constant
2x -1 sin 2 x cos 2 x
of integration, then f(x) is equal to: [Jan. 11, 2019 (II)]
ò (sin5 x + cos3x sin 2 x + sin 3x cos2 x + cos5x)2 dx is equal

1 2 to: [2018]
(a) ( x + 1) (b) ( x + 2) -1
3 3 1
(a) +C (b) +C
2 1 3(1 + tan x)
3
1 + cot 3 x
(c) ( x - 4) (d) ( x + 4 )
3 3 -1 1
+C +C
p (c)
1 + cot 3 x
(d)
3(1 + tan3 x)
22. Let n ³ 2 be a natural number and 0 < q <
2 (where C is a constant of integration)
1 26. If
(sin n q + sin q) n cos q
Then ò dq is equal to: tan x Kæ K tan x + 1 ö
sin n + 1 q ò 1 + tan x + tan 2 x dx = x - A
tan -1 ç
è A
÷ + C,
ø
[Jan 10, 2019(I)]
n +1 (C is a constant of integration), then the ordered pair
n æ 1 ö n (K, A) is euqal to [Online April 16, 2018]
(a) ç1 - ÷ +C (a) (2, 3) (b) (2, 1) (c) (– 2, 1) (d) (– 2, 3)
n2 - 1 è sin n - 1 q ø
n +1 æ x – 4ö
n æ 1 ö 27. If f ç ÷ = 2x + 1, (x Î R = {1, – 2}), then ò f (x) dx is
è x + 2ø
n
(b) 2 ç1 - n -1 ÷
+C
n +1è sin qø equal to (where C is a constant of integration)
n +1 [Online April 15, 2018]
n æ 1 ö n
ç1 + +C (a) 12 loge |1 – x| – 3x + c
(c) 2 n -1 ÷
n -1 è sin qø (b) – 12 loge |1 – x| – 3x + c
n +1 (c) – 12 loge |1 – x| + 3x + c
n æ 1 ö n (d) 12 loge |1 – x| + 3x + c
(d) 2 ç1 - ÷ +C
n -1 è sin n + 1 q ø 2x + 5 æ x +3ö
(where C is a constant of integration)
28. ò 7 - 6x - x
dx = A 7 - 6 x - x 2 + B sin -1 ç
2 è 4 ø
÷+C
23. For x2 ¹ np + 1, nÎN (the set of natural numbers), the (where C is a constant of integration), then the ordered
integral [Jan. 09, 2019(I)] pair (A, B) is equal to [Online April 15, 2018]

( ) dx
(a) (– 2, – 1) (b) (2, – 1)
2 sin ( x 2 - 1) - sin 2 x 2 - 1 (c) (– 2, 1) (d) (2, 1)
òx 2 sin ( x 2 - 1) + sin 2 ( x 2
- 1)
is equal to:
æ 3x - 4 ö 4
29. If f ç ÷ = x + 2, x ¹ - , and
è 3 x + 4 ø 3
(a) log e
1
2
(
sec 2 x 2 - 1 + c ) ò f (x) dx = A log |1 – x | + Bx + C, then the ordered pair
(A, B) is equal to : [Online April 9, 2017]
1
(b) log e | sec ( x 2 - 1) | + c (where C is a constant of integration)
2
æ8 2ö æ 8 2ö
1 2 æ x -1ö
2
(a) ç , ÷ (b) ç - , ÷
(c) 2 log e sec çç 2 ÷÷ + c è3 3ø è 3 3ø
è ø
æ 8 2ö æ8 2ö
æ x2 - 1 ö (c) ç - , - ÷ (d) ç , - ÷
è3 3ø
ç 2 ÷÷ + c
(d) log e sec ç è 3 3ø
è ø
(where c is a constant of integration) 30. The integral ò 1 + 2 cot x(cosec x + cot x)dx

5 x8 + 7 x 6 æ pö
24. If f ( x ) = ò dx, ( x ³ 0 ) , ç 0 < x < ÷ is equal to : [Online April 8, 2017]
(x ) è 2ø
2
2
+ 1 + 2 x7
(where C is a constant of integration)
and f(0) = 0, then the value of f(1) is: [Jan. 09, 2019 (II)]
x x
1 1 (a) 2 log sin +C (b) 4 log sin +C
(a) – (b) – 2 2
2 4
x x
1 1 (c) 2 log cos +C (d) 4 log cos +C
(c) (d) 2 2
2 4

Downloaded from @Freebooksforjeeneet


Integrals M-377

ò f ( x)dx = y(x), then ò x


5
dx f ( x 3 )dx is equal to [2013]
31. If ò cos3 x2sin 2x
= (tan x) A + C(tan x) B + k , where k 37. If
1é 3
is a constant of integration, then A + B + C equals : (a) x y ( x 3 ) - ò x 2 y ( x3 )dx ùû + C
[Online April 9, 2016] 3ë
1 3
(a)
16
(b)
27
(c)
7
(d)
21 (b) x y ( x3 ) - 3ò x 3 y ( x 3 )dx + C
5 10 10 5 3
1 3
log(t + 1 + t 2 ) 1 (c) x y ( x3 ) - ò x 2 y ( x3 )dx + C
32. If ò dt = ( g (t )) 2 + C , where C is a
2
3
1+ t2 1é 3
(d) x y ( x 3 ) - ò x3 y ( x3 ) dx ùû + C
constant, then g(b) is equal to : 3ë
[Online April 11, 2015] 38. If the integral
1 1 cos8 x + 1
log(2 + 5) log(2 + 5)
ò cot 2 x - tan 2 x dx = A cos8 x + k ,
(a) (b)
5 2
(c) 2 log(2 + 5) (d) log(2 + 5) where k is an arbitrary constant, then A is equal to :
[Online April 25, 2013]
1
x+
33. The integral ò æç1 + x - 1 ö÷ e x dx is equal to [2014] (a) -
1
(b)
1
(c)
1
(d) -
1
è xø 16 16 8 8
If the 5tan x dx = x + a ln sin x - 2cos x + k, then a is
1
x+
ò tan x - 2
1 39.
x+
(a) ( x + 1) e x +c (b) - xe x +c
1
equal to : [2012]
1 x+
x+ (a) – 1 (b) – 2 (c) 1 (d) 2
(c) ( x - 1) e x +c (d) xe x +c
æ x 2 + sin 2 x ö
2 2 40. If f ( x ) = ò ç 2
÷ sec x dx and f (0) = 0, then f (1)
sin x cos x è 1+ x 2
ø
34. The integral ò dx is equal to:
( )
2
sin 3 x + cos3 x equals [Online May 19, 2012]
p
[Online April 12, 2014] (a) tan1 - (b) tan1 + 1
4
1 1 p p
+c (b) - +c (c) (d) 1 -
(1 + cot x) ( )
(a) 4 4
3
3 1 + tan 3 x
x2 - x
41. The integral of 3 w.r.t. x is
sin 3 x cos3 x x - x2 + x - 1
(d) - +c
( )
+c
( )
(c)
1 + cos3 x 3 1 + sin 3 x [Online May 12, 2012]

æ
-1 1 - x
2ö (a)
1
2
(
log x 2 + 1 + C) (b)
1
2
log x 2 - 1 + C
35. The integral ò x cos ç ÷
ç 1 + x 2 ÷ dx (x > 0) is equal to:
è ø
[Online April 11, 2014]
2
(
(c) log x + 1 + C ) 2
(d) log x - 1 + C
(a) – x + (1 + x2) tan–1 x + c 42. Let f(x) be an indefinite integral of cos3x.
(b) x – (1 + x2) cot–1 x + c Statement 1:f(x) is a periodic function of period p.
(c) – x + (1 + x2) cot–1x + c
(d) x – (1 + x2) tan– 1 x + c Statement 2: cos3x is a periodic function.
[Online May 7, 2012]
sin8 x - cos8 x
ò (a) Statement 1 is true, Statement 2 is false.
(1 - 2sin2 x cos2 x)
36. dx is equal to:
(b) Both the Statements are true, but Statement 2 is not
[Online April 9, 2014] the correct explanation of Statement 1.
(c) Both the Statements are true, and Statement 2 is correct
1 1
(a) sin 2x + c (b) - sin 2x + c explanation of Statement 1.
2 2 (d) Statement 1 is false, Statement 2 is true.
1
(c) - sin x + c (d) - sin 2 x + c 43. The value of 2ò
sin xdx
is [2008]
2 æ pö
sin ç x – ÷
è 4ø

Downloaded from @Freebooksforjeeneet


EBD_8344
M-378 Mathematics

æ pö Integration of the Forms: òex(f(x) +


(a) x + log | cos ç x – ÷ | + c
è 4ø f'(x))dx, òekx(df(x) + f'(x))dx,
Integration by Partial Fractions,
æ pö TOPIC n Integration of Some Special
(b) x – log | sin ç x – ÷ | + c
è 4ø Irrational Algebraic Functions,
Integration of Different
æ pö Expressions of ex
(c) x + log | sin ç x – ÷ | +c
è 4ø
2 x x
æ pö 48. The integral ò1 e . x (2 + log e x) dx equals:
(d) x – log | cos ç x - ÷ | + c
è 4ø [Sep. 06, 2020 (II)]
(a) e (4e +1) (b) 4e2 – 1
dx
ò cos x +
44. equals [2007] (c) e (4e–1) (d) e (2e – 1)
3 sin x 49. A value of a such that

æx pö a+1
dx æ9ö
(a) log tan ç + ÷ + C
è 2 12 ø ò ( x + a)( x + a + 1)
= log e ç ÷ is : [April 12, 2019 (II)]
è8ø
a

(b) log tan æç x - p ö÷ + C 1 1


è 2 12 ø (a) –2 (b) (c) - (d) 2
2 2
x p
log tan çæ + ÷ö + C
1 5 - x2
òx e dx = g ( x)e - x + c , where c is a constant of
2
(c) 50. If
2 è 2 12 ø
integration, then g(–1) is equal to : [April 10, 2019 (II)]
1 æx pö 5 1
(d) log tan ç - ÷ + C (a) –1 (b) 1 (c) - (d) -
2 è 2 12 ø 2 2
dx 1 - 4 x3
45. ò cos x - sin x is equal to [2004] 51. If òx
5 - 4 x3
e
48
dx =
e f ( x) + C, where C is a

constant of integration, then f (x) is equal to:


1 æ x 3p ö
(a) log tan ç + ÷ + C [Jan. 10, 2019 (II)]
2 è2 8 ø
(a) – 2x3 – 1 (b) – 4x3 – 1
(c) – 2x3 + 1 (d) 4x3 + 1
1 æ xö
(b) log cot ç ÷ + C dx
è 2ø
2 52. The integral ò is equal to :
(1 + x ) x - x 2
1 æ x 3p ö (where C is a constant of integration)
(c) log tan ç - ÷ + C
2 è2 8 ø [Online April 10, 2016]

1 æ x pö 1+ x 1- x
(d) log tan ç - ÷ + C (a) -2 +C (b) - +C
2 è 2 8ø 1- x 1+ x

sin x 1- x 1+ x
46. If ò sin( x - a) dx = Ax + B log sin( x - a), +C , then value (c) -2
1+ x
+C (d) 2
1- x
+C

of (A, B) is [2004]
dx
(a) (- cos a, sin a) (b) (cos a, sin a) 53. The integral ò x 2 (x 4 + 1)3/4 equals : [2015]

(c) (- sin a, cos a) (d) (sin a, cos a) 1


1 æ x 4 + 1ö 4
47. f(x) and g(x) are two differentiable functions on [0, 2] such (a) - (x 4 + 1) 4 +c (b) - ç 4 ÷ + c
è x ø
that f ¢¢( x ) - g ¢¢ ( x) = 0, f ¢(1) = 2 g ¢(1) = 4 f(2) = 3g(2) = 9
1
then f (x)–g(x) at x = 3/2 is [2002]
æ x 4 + 1ö 4 1
(a) 0 (b) 2 (c) 10 (d) 5 (c) ç 4 ÷ + c (d) (x 4 + 1) 4 + c
è x ø

Downloaded from @Freebooksforjeeneet


Integrals M-379

dx
54. The integral ò 3 5 is equal to : Evaluation of Definite Integral
TOPIC Đ by Substitution, Properties of
( x + 1) 4 (x - 2) 4 Definite Integrals
[Online April 10, 2015]
1 1
dx and I2 = ò (1 - x ) dx such
ò0 (1 - x
1 1 50 100 50 101
60. If I1 = )
x +1ö 4 x +1ö 4
(a) - çæ (b) 4 æç
4 0
÷ +C +C
3 è x - 2ø è x - 2 ÷ø that I2 = aI1 then a equals to : [Sep. 06, 2020 (I)]
1 1 5049 5050 5050 5051
(a) (b) (c) (d)
æ x - 2 ö4 4 æ x - 2 ö4 5050 5049 5051 5050
(c) 4 ç ÷ +C (d) - ç ÷ +C
è x +1 ø 3 è x +1 ø p/2
1
55. If m is a non- ero number and 61. The value of ò sin x
dx is: [Sep. 05, 2020 (I)]
5m -1 4m-1 -p / 2 1 + e
x + 2x
ò dx = f ( x ) + c , p 3p
p
( )
3
x 2m + x m + 1 (a) (b) p (c) (d)
4 2 2
then f(x) is: [Online April 19, 2014] 62. Let f ( x) =| x - 2 | and g ( x) = f ( f ( x)), x Î [0, 4].
5m 4m
x x 3
(a) (b) Then ò ( g ( x) - f ( x)) dx is equal to :
( ) ( )
2 2 [Sep. 04, 2020 (I)]
2m x 2m + x m + 1 2m x 2m + x m + 1 0

2m ( x5m + x 4m ) ( x5m - x4m ) 1 3


(a) 1 (b) 0 (c) (d)
(c) (d) 2 2
( x 2m + xm + 1) 2m ( x 2m + x m + 1)
2 2 63. The integral
p /3
xdx ò tan 3 x × sin 2 3 x (2 sec 2 x × sin 2 3 x + 3 tan x × sin 6 x ) dx
56. The integral ò equals : p /6
2 - x 2 + 2 - x2 is equal to : [Sep. 04, 2020 (II)]
[Online April 23, 2013]
7 1 1 9
(a) (b) - (c) - (d)
(a) log 1 + 2 + x 2 + c (b) - log 1 + 2 - x 2 + c 18 9 18 2
64. Let {x} and [x] denote the fractional part of x and the
greatest integer £ x respectively of a real number x. If
(c) - x log 1 - 2 - x2 + c (d) x log 1 - 2 + x +c
2
n n

x - x + 1 cot
2 -1 -1
ò0 {x} dx, ò0 [ x] dx and 10(n2 – n), (n Î N, n > 1) are
ò dx = A( x )ecot + C , then A(x) is
x x
57. If e three consecutive terms of a G.P., then n is equal to
x2 + 1
_______________. [NA Sep. 04, 2020 (II)]
equal to : [Online April 22, 2013]
p
(a) – x (b) x (c) 1 - x (d) 1 + x
65. ò | p- | x || dx is equal to : [Sep. 03, 2020 (I)]
dx x6 -p
58. If ò x + x7 = p( x) then, ò x + x7 dx is equal to:
[Online April 9, 2013] p2
(a) 2p2 (b) 2p2 (c) p2 (d)
(a) ln | x | – p (x) + c (b) ln | x | + p (x) + c 2
(c) x – p (x) + c (d) x + p (x) + c
1/ 2 x2 k
ïì (log x - 1) ïü
2 66. If the value of the integral ò (1 - x )
dx is , then
ò íï1 + (log x)2 ýï dx is equal to 0 2 3/ 2 6
59. [2005]
î þ k is equal to : [Sep. 03, 2020 (II)]
log x x (a) 2 3 - p (b) 2 3 + p
(a) +C (b) +C
(log x)2 + 1 x2 + 1 (c) 3 2 + p (d) 3 2 - p
x x 2
xe +C
+C
(c)
1+ x 2
(d)
(log x) + 12 67. The integral ò | x - 1| - x dx is equal to ________.
0
[NA Sep. 02, 2020 (I)]

Downloaded from @Freebooksforjeeneet


EBD_8344
M-380 Mathematics

68. Let [t] denote the greatest integer less than or equal to t. 75. Let f : R ® R be a continuously differentiable function
2 1
Then the value of
ò
1
| 2 x - [3x] | dx is ___________. such that f(2) = 6 and f’(2) = .
48
[NA Sep. 02, 2020 (II)] f ( x)
69. If for all real triplets (a, b, c), f(x) = a + bx + cx2; then If ò6 4t3dt = (x – 2) g (x), then lim g(x) is equal to :
x ®2
1 [April 12, 2019 (I)]
ò f ( x) dx is equal to: [Jan. 9, 2020 (I)] (a) 18
x
(b) 24 (c) 12 (d) 36
0 cot x
ì
76. If ò 2
cot x + cosec x
dx = m (p + n) , then m.n is equal to :
æ 1 öü 1ì æ 1 öü
0
(a) 2 í3 f (1) + 2 f ç ÷ý (b) 2 í f (1) + 3 f ç ÷ý [April 12, 2019 (I)]
î è 2 øþ î è 2 øþ
1 1
1ì æ 1 öü 1ì æ 1 öü (a) - (b) 1 (c) (d) –1
(c) 3 í f (0) + f ç ÷ý (d) 6 í f (0) + f (1) + 4 f ç 2 ÷ý 2 2
î è 2 øþ î è øþ 2p
2p
x sin8 x
77. The value of ò éësin 2x (1 + cos3x )ùû dx , where [t] denotes
70. The value of ò sin8 x + cos8 x
dx is equal to: 0
the greatest integer function, is: [April 10, 2019 (I)]
0
[Jan. 9, 2020 (I)] (a) p (b) –p (c) –2p (d) 2p
(b) 2p2 (c) p2 p/3
òp/6 sec
(a) 2p (d) 4p 78. The integral 2/3
x cos ec 4/3 x dx is equal to :
2
dx [April 10, 2019 (II)]
71. If I = ò , then: [Jan. 8, 2020 (II)]
1 2 x - 9 x + 12 x + 4
3 2
(a) 3 - 3 5/6 2/3 (b) 3 - 31/3
4/3

1 1 1 1 (c) 37/6 - 35/6 (d) 35/3 - 31/3


(a) < I2 < (b) < I2 <
8 4 9 8 p /2
sin 3 x
1 1 1 1 79. The value of ò sin x + cos x
dx is: [April 9, 2019 (I)]
(c) < I2 < (d) < I2 < 0
16 9 6 2
p-2 p-2 p -1
72. If f (a + b + 1 – x) = f(x), for all x, where a and b are fixed (a) (b) p - 1 (c) (d)
b
8 4 4 2
1
positive real numbers, then a + b ò x(f (x) + f (x + l))dx
1
-1
The value of the integral ò x cot (1 - x + x )dx is:
2 4
a 80.
0
is equal to: [Jan. 7, 2020 (I)]
[April 09, 2019 (II)]
b +1 b -1
(a) òa+1 f ( x)dx (b) òa-1 f ( x)dx (a)
p 1
- loge 2 (b)
p
- loge 2
2 2 4
b -1 b +1
(c) òa-1 f ( x + 1)dx (d) òa+1 f ( x + 1)dx (c)
p
- loge 2 (d)
p 1
- loge 2
2 4 2
2
- a| x| 81. If f : R ® R is a differentiable function and f (2) = 6, then
73. The value of a for which 4a òe dx = 5 , is :
f (x)
-1 2t dt
[Jan. 7, 2020 (II)]
lim
x ®2 ò ( x - 2) is: [April 09, 2019 (II)]
6
(a) 24 f ¢ (2) (b) 2 f ¢ (2) (c) 0 (d) 12 f ¢ (2)
(b) loge æç ö÷ (c) loge (d) loge æç ö÷
3 4
(a) loge2
è 2ø 2 è 3ø 2 - x cos x
If q1 and q2 be respectively the smallest and the largest 82. If f ( x) = and g ( x) = loge x, (x > 0) then the
74. 2 + x cos x
values of q in (0,2p) – {p} which satisfy the equation,
p
q2 4
5
2cot 2 q - + 4 = 0 , then
ò cos ò
2
3q dq is equal to: value of the integral g ( f ( x))dx is :
sin q
q1 p
-
4
[Jan. 7, 2020 (II)]
[April 8, 2019 (I)]
p 2p p 1 p (a) loge3 (b) logee (c) loge2 (d) loge1
(a) (b) (c) + (d)
3 3 3 6 9

Downloaded from @Freebooksforjeeneet


Integrals M-381

83.
x
Let f(x) = ò g ( t ) dt , where g is a non- ero even function. If
(a) ( 0, 2 ) (b) (- 2, 0 )
0
(c) ( 2, - 2 ) (d) (- 2, 2 )
x

ò f ( t ) dt equals : [April 08, 2019 (II)]


x 1
f(x + 5) = g(x), then
0
89. If ò f (t) dt = x 2 + ò t 2 f (t) dt, then f ¢ (1/2) is:
0 x
5 x +5 [Jan. 10, 2019 (II)]
(a) ò g ( t ) dt (b) ò g ( t ) dt 24 18 4 6
x +5 5
(a) (b) (c) (d)
25 25 5 25
x +5 5 p /2
g ( t ) dt dx
(c) 2 ò (d) 5 ò g ( t ) dt 90. The value of ò [ x ] + [sin x] + 4
, where [t] denotes the
5 x +5 - p /2
84. Let f and g be continuous functions on [0, a] such that greatest integer less than or equal to t, is:
a [Jan. 10, 2019 (II)]
f(x) = f(a – x) and g(x) + g(a – x) = 4, then ò f ( x ) g ( x ) dx (a)
1
(7p + 5) (b)
1
(7p - 5)
0 12 12
is equal to: [Jan. 12, 2019 (I)]
3 3
a a (c) (4p - 3) (d) (4p - 3)
20 10
(a) 4 ò f ( x ) dx (b) ò f ( x ) dx p

ò cos x
0 0 3
91. The value of dx is: [Jan 9, 2019 (I)]
a a 0
(c) 2 ò f ( x ) dx (d) –3ò f ( x ) dx 4 2 4
0 0 (a) 0 (b) (c) (d) -
3 3 3
eì 92. Let f be a differentiable function from R to R such that |f (x)
æ e ö üï
2x x
ïæ x ö – f (y)| £ 2|x – y|3/2, for all x, y, Î R. If f (0) = 1 then
85. The integral ò íç ÷ - ç ÷ ý loge x dx is equal to :
1îïè e ø è x ø þï 1

ò f ( x ) dx is equal to :
2
[Jan. 12, 2019 (II)] [Jan. 09, 2019 (II)]
0
1 1 1 1 1
(a) -e- 2 (b) - + - 1
2 e 2 e 2e2 (a) 1 (b) 2 (c) (d) 0
2
3 1 1 3 1 p /3
- - -e- 2 tan q 1
, ( k > 0 ) then the value of k
(c) (d)
2 e 2e 2 2 2e 93. If ò
2k sec q
dq = 1 -
2
0
2
sin 2 x is: [Jan. 09, 2019 (II)]
86. The value of the integral 1
dx ò éxù 1
-2
êë p úû + 2
(a) 4 (b) (c) 1 (d) 2
2
(where [x] denotes the greatest integer less than or equal p
2
to x) is : [Jan. 11, 2019 (I)] sin 2 x
(a) 0 (b) sin 4 (c) 4 (d) 4 –sin 4 94. The value of ò x
dx is : [2018]
p 1+ 2
p /4 dx -
òp/6 sin 2 x 2
( tan5 x + cot5 x )
87. The integral equals :
p p p
(a) (b) 4p (c) (d)
[Jan. 11, 2019 (II)] 2 4 8
x
1 æ 1 ö
tan -1 ç
1 æp -1 æ 1 ö ö
95. If f (x) = ò0 t (sin x - sin t ) dt then
(a) ÷ (b) 10 ç 4 - tan ç ÷÷
20 è9 3ø è è 9 3 øø [Online April 16, 2018]
(a) f ²¢(x) + f ¢(x) = cos x – 2x sin x
p 1æp -1 æ 1 ö ö (b) f ²¢(x) + f ²(x) – f ¢(x) = cos x
(c) (d) 5 ç 4 - tan ç ÷÷ (c) f ²¢(x) – f ²(x) = cos x – 2x sin x
40 è è 3 3 øø
(d) f ²¢ (x) + f ²(x) = sin x
b
88. Let I = ò ( x 4 - 2 x 2 )dx. If I is minimum then the ordered
a
pair (a, b) is: [Jan 10, 2019 (I)]

Downloaded from @Freebooksforjeeneet


EBD_8344
M-382 Mathematics

3p 104. For x Î R, x ¹ 0, if y(x) is a differentiable function such


x
96. The value of integral ò p
4
1 + sin x
dx is x x
4 that x ò y(t)dt = (x + 1) ò ty(t)dt , then y(x) equals :
[Online April 15, 2018] 1 1
p (where C is a constant) [Online April 10, 2016]
(a) ( 2 + 1) (b) p ( 2 - 1)
2 1 1 1
1 C - C -x C -
(c) 2p ( 2 - 1) (d) p 2 (a) (b) e x (c) e (d) e x
Cx 3e x x2 x x3
1 1 2 1 3
97. If I1 = ò e - x cos2 x dx; I2 = ò e - x cos2 x dx and I3 = ò e - x 105. The value of the integral
0 0 0
10
dx; then [Online April 15, 2018] [x 2 ]dx
(a) I2 > I3 > I1 (b) I3 > I1 > I2 ò é x 2 - 28x + 196ù + [x 2 ] , where [x]
(c) I2 > I1 > I3 (d) I3 > I2 > I1 4ë û
p denotes the greatest integer less than or equal to x, is :
æ æ 2 + sin x ö ö
ò sin 4 x çç1 + log ç [Online April 10, 2016]
÷ ÷÷ dx
98. The value of the integral 2
p

2 è è 2 – sin x ø ø 1
(a) (b) 6 (c) 7 (d) 3
is [Online April 15, 2018] 3
-1
1 -1 1
106. If 2 ò tan xdx = ò cot (1 - x + x )dx , then
3 3 3 2
(a) p (b) 0 (c) p (d) 0 0
16 8 4
1 -1
3p
4
ò0 tan (1 - x + x 2 )dx is equal to :
dx
99. The integral ò 1 + cos x is equal to : [2017] [Online April 9, 2016]
p p
(a) + log 2 (b) log2
4 2
(a) –1 (b) –2 (c) 2 (d) 4 p
(c) - log 4 (d) log4
ò tan
n
100. Let In = x dx,(n > 1) . I4 + I6 = a tan5x + bx5 + C, 2
where C is constant of integration, then the ordered pair 107. The integral [2015]
(a, b) is equal to : [2017] 4
log x 2

æ 1 ö æ 1 ö æ1 ö æ1 ö ò log x 2 + log(36 - 12 x + x 2 ) dx is equal to :


(a) çè - , 0÷ø (b) çè - ,1÷ø (c) çè , 0÷ø (d) çè , -1÷ø 2
5 5 5 5 (a) 1 (b) 6 (c) 2 (d) 4
2 108. Let f : R ® R be a function such that
dx k
101. If ò 3
=
k +5
then k is equal to : f(2 – x) = f(2 + x) and f(4 – x) = f(4 + x), for all xÎR and
1 2 2
( x - 2 x + 4) 2 50
[Online April 9, 2017] ò f (x) dx = 5. Then the value of ò10 f (x) dx is :
0
(a) 1 (b) 2 (c) 3 (d) 4
[Online April 11, 2015]
p (a) 125 (b) 80 (c) 100 (d) 200
8cos 2x
102. The integral ò 4
p
(tan x + cot x)3
dx equals : 109. Let f : (–1, 1) ® R be a continuous function. If
sin x
12 3 æ 3ö
[Online April 8, 2017] ò f (t )dt =
2
x, then f ç
ç 2 ÷÷ is equal to :
0 è ø
15 15 13 15
(a) (b) (c) (d) [Online April 11, 2015]
128 64 32 256
1 3 3
2x12 + 5x 9 (a) (b) (c) (d) 3
103. The integral ò dx is equal to : [2016] 2 2 2

(x )
5 3
+ x3 + 1 x
dt. Then f (x) + f æç ö÷ is equal to:
log t 1
5 10
110. For x > 0, let f (x) = ò 1+ t è xø
x -x 1
(a) +C (b) +C
( ) ( )
2 2 [Online April 10, 2015]
2 x5 + x3 +1 2 x5 + x3 +1
1
5 10
(a) ( log x ) 2 (b) log x
-x x 4
(c) +C (d) +C
( ) ( )
2 2 1 1
x5 + x3 +1 2 x5 + x3 +1 (c) ( log x ) 2 (d) log x 2
2 4

Downloaded from @Freebooksforjeeneet


Integrals M-383

p (a) ± 1 (b) ± 2 (c) ± 3 (d) ± 4


x x
111. The integral ò 1 + 4sin 2
2
- 4sin dx equals:
2
[2014] 118. Statement-1 : The value of the integral
p /3
0 dx
p ò is equal to p/6
(a) 4 3 - 4 (b) 4 3 - 4 - p/61+ tan x
3
b b
2p
(c) p - 4 (d)
3
- 4- 4 3 Statement-2 : ò f ( x)dx = ò f (a + b - x)dx. [2013]
a a
x t
e (a) Statement-1 is true; Statement-2 is true; Statement-2
112. Let function F be defined as F(x) = ò dt, x > 0 then the
t is a correct explanation for Statement-1.
1 (b) Statement-1 is true; Statement-2 is true; Statement-2
x t
e is not a correct explanation for Statement-1.
value of the integral ò t + a dt, where a > 0, is: (c) Statement-1 is true; Statement-2 is false.
1 (d) Statement-1 is false; Statement-2 is true.
[Online April 19, 2014]
p
(a) e ëé F ( x ) - F (1 + a ) ûù
a 119. For 0 £ x £ , the value of
2
(b) e -a éë F ( x + a ) - F ( a ) ùû sin 2 x cos 2 x
-1
ò sin ( t )dt + ò cos -1 ( t ) dt equals :
(c) e éë F ( x + a ) - F (1 + a ) ùû
a
0 0
[Online April 25, 2013]
(d) e -a éëF ( x + a ) - F (1 + a ) ùû
p p
t
(a) (b) 0 (c) 1 (d) -
4 4
113. If for a continuous function f(x), ò ( f ( x ) + x ) dx = p2 – p/2
sin 2 x
-p
120. The value of ò dx is :
-p / 2 1 + 2
x
æ pö
t2, for all t ³ – p, then f ç - ÷ is equal to: [Online April 23, 2013]
è 3ø
[Online April 12, 2014] p p
(a) p (b) (c) 4p (d)
p p p 2 4
(a) p (b) (c) (d)
2 3 6 7p / 3
114. If [ ] denotes the greatest integer function, then the integral
p
121. The integral ò tan 2 x dx is equal to :
7p / 4
ò [ cos x ] dx is equal to: [Online April 12, 2014] [Online April 22, 2013]
0
(a) log 2 2 (b) log 2
p p
(a) (b) 0 (c) –1 (d) - (c) 2 log 2 (d) log 2
2 2
y
e dt d2y
115. If for n ³ 1, Pn = ò ( log x ) dx , then P10 – 90P8 is equal to:
n 122. If x = ò , then is equal to :
0 1+ t
2 dx 2
1
[Online April 11, 2014] [Online April 9, 2013]
(a) – 9 (b) 10e (c) – 9 e (d) 10
(a) y (b) 1+ y 2
1
2 ln
(1 + 2x ) x
(d) y 2
116. The integral ò 1 + 4x 2 dx , equals: (c)
1+ y 2
0
[Online April 9, 2014] x

p
ln2 (b)
p
ln2
p
ln2 (d)
p
ln2
ò
123. If g (x) = cos 4t dt , then g (x + p) equals [2012]
(a) (c) 0
4 8 16 32
g ( x)
117. The intercepts on x-axis made by tangents to the curve, (a) (b) g (x) + g (p)
x
g ( p)
y= ò t dt , x Î R, which are parallel to the line y = 2x, are (c) g (x) – g (p) (d) g (x) . g (p)
0
equal to : [2013]

Downloaded from @Freebooksforjeeneet


EBD_8344
M-384 Mathematics

124. If [x] is the greatest integer £ x, then the value of the integral 2 2
(a) I > and J > 2 (b) I < and J < 2
0.9 æ 2 æ 2 - xöö
ò ç éë x ùû + log ç
3 3
è 2 + x ÷ø ÷ø
dx is [Online May 26, 2012]
-0.9 è 2 2
(a) 0.486 (b) 0.243 (c) 1.8 (d) 0 (c) I < and J > 2 (d) I > and J < 2
3 3
0.9 133. The solution for x of the equation
125. The value of the integral ò ëé x - 2 [ x ]ùû dx , x
dt p
0
ò 2
=
2
is [2007]
where [.] denotes the greatest integer function is 2 t t -1
[Online May 19, 2012]
(a) 0.9 (b) 1.8 (c) – 0.9 (d) 0 3
(a) (b) 2 2
d e tan x 2
126. If G( x ) = , x Î (0, p 2), then (c) 2 (d) None of these
dx x
x
134. Let F(x) = f (x) + f æç ö÷ ,where f ( x) = ò
12 2 tan(p x 2 ) 1 log t
dt , Then
ò .e dx is equal to [Online May 12, 2012] è xø 1+ t
14 x l
F(e) equals [2007]
(a) G (p 4) - G (p 16) (b) 2[G (p 4) - G(p 16)] (a) 1 (b) 2 (c) 1/2 (d) 0,
(c) p[G (1 2) - G (1 4)] (d) G (1 2) - G (1 2) a
135. The value of ò [ x ] f '( x )dx , a > 1 where [x] denotes the
x x2
127. If ò t f ( t ) dt = sin x - x cos x - , for all x Î R - { 0} , 1
e 2 greatest integer not exceeding x is [2006]
æ pö (a) af (a) - { f (1) + f (2) + .............. f ([ a])}
then the value of f ç ÷ is [Online May 7, 2012]
è 6ø (b) [a ] f (a) - { f (1) + f (2) + .............. f ([a ])}
(a) 1/2 (b) 1 (c) 0 (d) – 1/2 (c) [a ] f ([a ]) - { f (1) + f (2) + .............. f ( a)}
128. Let [.] denote the greatest integer function then the value
1.5
(d) af ([ a]) - { f (1) + f (2) + .............. f ( a )}
ò x éë x

of û dx is :. [2011 RS] p
-
0 2

(a) 0 (b)
3
(c)
3
(d)
5 136. ò [( x + p )3 + cos 2 ( x + 3p )]dx is equal to [2006]
2 4 4 3p
-
1 2
8log(1 + x)
129. The value of
1+ x2
ò
dx is [2011] p4 p4 p p p
0 (a) (b) + (c) (d) -1
p p 32 32 2 2 4
(a) log 2 (b) log 2 (c) log 2 (d) p log 2
8 2 p
130. Let p(x) be a function defin ed on R such that
p¢(x) = p¢(1 – x), for all x Î [0, 1], p (0) = 1 and p (1) = 41.
137. ò xf (sin x)dx is equal to [2006]
0
1
Then ò p( x) dx equals [2010] p p
0 (a) p ò f (cos x )dx (b) p ò f (sin x )dx
0 0
(a) 21 (b) 41 (c) 42 (d) 41
p p /2 p /2
ò [cot x] dx , where [ . ] denotes the greatest integer (c) p (d) p ò
131.
2 ò f (sin x)dx f (cos x )dx
0 0 0
function, is equal to : [2009]
p p 6
x
(a) 1 (b) –1 (c) - (d) I =ò
138. The value of integral, dx is [2006]
2 2
3
9- x + x
1 1
sin x cos x
132. Let I = ò dx and J = ò dx. Then which one of (a)
1
(b)
3
(c) 2 (d) 1
0
x 0
x 2 2
the following is true? [2008]

Downloaded from @Freebooksforjeeneet


Integrals M-385

p 147. Let f(x) be a function satisfying f '(x) = f(x) with


cos 2 x
ò
139. The value of dx , a > 0, is [2005] f(0)=1 and g(x) be a function that satisfies
x
- p 1+ a f(x) + g(x) = x 2 . Then the value of the integral
p p 1
(a) a p (b) (c) (d) 2p ò f ( x ) g ( x ) dx, is [2003]
2 a 0
1
2
1
3 2 e2 5 e2 5
(a) e + + (b) e - -
ò 2 dx , I3 =
2
ò 2 dx , I 2 = ò 2 dx and
x x x
140. If I1 = 2 2 2 2
0 0 1
e2 3 e2 3
2 (c) e + - (d) e - - .
3 2 2 2 2
I 4 = ò 2 dx then
x
[2005] b
1 148. If f (a + b - x) = f ( x) then ò xf ( x )dx is equal to [2003]
a
(a) I 2 > I1 (b) I1 > I 2 (c) I3 = I 4 (d) I3 > I 4
141. Let f : R ® R be a differentiable function having f (2) = 6, a+b
b
a+b b
f ( x)
4t 3
(a)
2 ò f (a + b + x )dx (b)
2 a
ò f (b - x)dx
æ 1 ö
ò
a
f '(2) = ç ÷ . Then lim dt equals [2005]
è 48 ø x®2 x-2
6 a +b b b-a b
(c) ò f ( x)dx (d) ò f ( x )dx .
(a) 24 (b) 36 (c) 12 (d) 18 2 a 2 a
f ( a)
ex x2
142. If f ( x) =
1+ ex
, I1 = ò xg{x (1 - x )}dx 2
ò sec tdt
f (-a ) 0
149. The value of lim is [2003]
f ( a) x ®0 x sin x
and I 2 = ò g{x (1 - x )}dx, (a) 0 (b) 3 (c) 2 (d) 1
f (-a ) 150. If f ( y ) = e y , g ( y ) = y; y > 0 and
I2 t
then the value of
I1
is [2004] F (t ) = ò f (t - y ) g ( y ) dy , then [2003]
0
(a) 1 (b) –3 (c) –1 (d) 2
p p/2 (a) F (t ) = te -t (b) F (t ) = 1 - te -t (1 + t )
143. If ò xf (sin x)dx = A ò f (sin x )dx, then A is [2004]
(c) F (t ) = e t - (1 + t ) (d) F (t ) = tet .
0 0

p p 2 x (1 + sin x )
(a) 2p (b) p
ò
(c) (d) 0
4 151. dx is [2002]
-p 1 + cos 2 x
p/2
(sin x + cos x ) 2
144. The value of I = ò 1 + sin 2 x
dx is [2004]
(a)
p2
(b) p 2 (c) ero (d)
p
0 2
4
(a) 3 (b) 1 (c) 2 (d) 0
2
ò [x
3 2
152. ]dx is [2002]
ò
2
145. The value of | 1 - x |dx is [2004]
0
-2
1 14 7 28 (a) 2 – 2 (b) 2 + 2
(a) (b) (c) (d)
3 3 3 3 (c) 2 –1 (d) - 2 - 3 + 5
1 p/4
146. The value of the integral I = ò x(1 - x ) n dx is [2003] 153. I n = ò tan n x dx then lim n[ I n + I n + 2 ] equals
n®¥
[2002]
0 0
1 1 1
(a) + (b) (a) 1 (b) 1 (c) ¥ (d) ero
n +1 n + 2 n +1
2
1 1 1 10 p
-
(c)
n+2
(d)
n +1 n + 2
. 154. ò0 | sin x | dx is [2002]
(a) 20 (b) 8 (c) 10 (d) 18

Downloaded from @Freebooksforjeeneet


EBD_8344
M-386 Mathematics

1
Reduction Formulae for Definite
Integration, Gamma & Beta æ ( n + 1) (n + 2)...3n ö n
159. lim ç ÷ is equal to: [2016]
TOPIC Ė Function, Walli's Formula, n ®¥ è n 2n ø
Summation of Series by
Integration 9
(a) (b) 3 log 3 – 2
e2
155. Let a function f : [0, 5] ® R be continuous, f (1) = 3 and
F be defined as: 18 27
(c) 4 (d)
x x e e2
F(x) = òt
2
g (t )dt , where g(t) = ò f (u )du 160. f ( x ) = ò
dx
is a polynomial of degree
1
1
sin 6 x
Then for the function F, the point x = 1 is :
[Jan. 9, 2020 (II)] [Online May 26, 2012]
(a) 5 in cot x (b) 5 in tan x
(a) a point of local minima.
(c) 3 in tan x (d) 3 in cot x
(b) not a critical point.
é1 1 2 4 1 ù
(c) a point of local maxima. 161. nLim ê sec2 + sec 2 ............ + sec 2 1ú
®¥ ë n2 n2 n2 n2 n û
(d) a point of inflection.
equals [2005]
æ (n + 1)1/3 (n + 2)1/3 (2n)1/3 ö 1 1
156. nlim çç + + ..... + 4/3 ÷ is equal to : (a) sec 1 (b) cosec 1
®¥ 4/3 4/3 ÷
è n n n ø 2 2
[April 10, 2019 (I)] 1
(c) tan 1 (d) tan 1
3 4/3 3 4 4/3 2
(a) (2) - (b) (2)
4 4 3 n r
1
(c)
3 4/3 4
(2) - (d)
4 3/4
(2) 162.
Lim
n®¥ å n e n is [2004]
2 3 3 r =1
(a) e + 1 (b) e – 1 (c) 1 – e (d) e
æ n n n 1 ö
157. lim ç 2 2 + 2 2
+
2 2
+ ..... + ÷ is equal 1 + 2 + 3 + ...n
4 4 4
1 + 2 + 3 + ...n3
3 3
n ®¥ è n + 1 n +2 n +3 5n ø 163. lim - lim [2003]
to : [Jan. 12, 2019 (II)] n®¥ n5 n®¥ n5
p p 1 1 1
(a) (b) tan–1(3) (c) (d) tan–1(2) (a) (b) (c) Zero (d)
4 2 5 30 4
1a + 2a + ......... + na 1 1 p + 2 p + 3 p + ..... + n p
158. If lim = 164. lim is [2002]
n ®¥ ( n + 1)a -1 [ (na + 2) + .....(na + n) ] 60 n ®¥ n p +1
for some positive real number a, then a is equal to : 1 1
[Online April 9, 2017] (a) (b)
p +1 1- p
15 17
(a) 7 (b) 8 (c) (d) 1 1 1
2 2 (c) - (d)
p p -1 p+2

Downloaded from @Freebooksforjeeneet


23
Applications of
Integrals
4. The area (in sq. units) of the region
Curve & X-axis Between two A = {(x, y) ÎR × R|0 d” x d”3, 0 d” y d” 4, y d” x2 + 3x} is :
Ordinates, Area of the Region [April 8, 2019 (I)]
TOPIC Ć Bounded by a Curve & Y-axis
Between two Abscissa 53 59 26
(a) (b) 8 (c) (d)
6 6 3
1. The area (in sq. units) of the region 5. The area of the region
A = {(x, y): 0 £ y £ x |x| + 1 and – 1 £ x £ 1} in sq. units is:
A= {( x, y) : ( x - 1)[ x] £ y £ 2 x , 0 £ x £ 2}, where [t] [Jan. 09, 2019 (II)]
denotes the greatest integer function, is :
2 4 1
[Sep. 05, 2020 (II)] (a) (b) 2 (c) (d)
3 3 3
8 1 4
(a) 2- (b) 2 +1 6. Let g(x) = cos x 2 ,f (x) = x , and a, b (a < b) be the
3 2 3
8 4 1 roots of the quadratic equation 18x 2 - 9 px + p 2 = 0 . Then
(c) 2 -1 (d) 2-
3 3 2 the area (in sq. units) bounded by the curve y = (gof)(x)
2. The area (in sq. units) of the region
and the lines x = a, x = b and y = 0 , is : [2018]
1
{( x, y ) : 0 £ y £ x + 1, 0 £ y £ x + 1, £ x £ 2} is :
2
1 1
2 (a) ( 3 + 1) (b) ( 3 - 2)
[Sep. 03, 2020 (I)] 2 2

23 79 79 23 1 1
(a) (b) (c) (d) (c) ( 2 - 1) (d) ( 3 - 1)
16 24 16 6 2 2
7. Let f : [ – 2, 3] ® [0, ¥ ) be a continuous function such
ì 1
ï x , 0£x< that f (1– x) = f (x) for all x Î [-2, 3] .
2
ï If R1 is the numerical value of the area of the region
ï 1 1
3. Given: f(x) = í , x= bounded by y = f (x), x = –2, x = 3 and the axis of x and
ï 2 2
ï 1 3
ï1 - x , 2 < x £ 1 R2 = ò x f ( x) dx, then : [Online April 25, 2013]
î
-2
2 (a) 3R1 = 2R2 (b) 2R1 = 3R2
æ 1ö
and g(x) = ç x - ÷ , x Î R. Then the area (in sq. units) of (c) R1 = R2 (d) R1 = 2R2
è 2ø 8. Let f (x) be a non – negative continuous function such that
the region bounded by the curves, y = f(x) and y = g(x) the area bounded by the curve y = f (x), x - axis and the
between the lines, 2x = 1 and 2x = 3, is : p p
[Jan. 9, 2020 (II)] ordinates x = and x = b > is
4 4
1
+
3 3 1
-
1
-
3 1
+
3 æ p ö æ pö
(a) (b) (c) (d) ç b sin b + cos b + 2b ÷ . Then f ç ÷ is [2005]
3 4 4 3 2 4 2 4 è 4 ø è 2ø

Downloaded from @Freebooksforjeeneet


EBD_8344
M-418 Mathematics

(a) x6 – 6x3 + 4 = 0 (b) x6 – 12x3 + 4 = 0


æp ö æp ö
(a) ç + 2 - 1÷
6 3
(c) x + 6x – 4 = 0 (d) x6 – 12x3 – 4 = 0
(b) ç - 2 + 1÷
è4 ø è4 ø 17. The area (in sq. units) of the region
{(x, y) Î R2: x2 £ y £ |3 – 2x|, is: [Jan. 8, 2020 (II)]
æ p ö æ p ö
(c) ç1 - - 2 ÷ (d) ç1 - + 2 ÷ 32 34 29 31
è 4 ø è 4 ø (a) (b) (c) (d)
3 3 3 3
9. The area enclosed between the curve y = log e ( x + e) and 18. If the area (in sq. units) of the region {(x, y) : y2 £ 4x,
the coordinate axes is [2005] x + y £ 1, x ³ 0, y ³ 0} is a 2 + b, then a – b is equal to
(a) 1 (b) 2 (c) 3 (d) 4
: [April 12, 2019 (I)]
10. If y = f(x) makes +ve intercept of 2 and 0 unit on x and y
axes and encloses an area of 3/4 square unit with the axes 10 8 2
(a) (b) 6 (c) (d) -
2 3 3 3
then ò xf ¢( x)dx is [2002] 19. If the area (in sq. units) bounded by the parabola y2 = 4lx
0 1
and the line y = lx, l > 0, is , then l is equal to :
(a) 3/2 (b) 1 (c) 5/4 (d) –3/4 9
[April 12, 2019 (II)]
Different Cases of Area Bounded (a) 2 6 (b) 48 (c) 24 (d) 4 3
TOPIC n Between the Curves
20. The region represented by x - y £ 2 and x + y £ 2 is
11. The area (in sq. units) of the region bounded by a : [April 10, 2019 (I)]
A = {(x, y) : |x| + |y| £ 1, 2y2 ³ |x|} is : [Sep. 06, 2020 (I)] (a) square of side length 2 2 units
1 7 1 5 (b) rhombus of side length 2 units
(a) (b) (c) (d) (c) square of area 16 sq. units
3 6 6 6
12. The area (in sq. units) of the region enclosed by the curves (d) rhombus of area 8 2 sq. units
y = x2 – 1 and y = 1 – x2 is equal to: [Sep. 06, 2020 (II)] 21. The area (in sq. units) of the region bounded by the curves
y = 2x and y = |x + 1|, in the first quadrant is :
4 8 7 16 [April 10, 2019 (II)]
(a) (b) (c) (d)
3 3 2 3
3 3
(a) loge 2 + (b)
13. Consider a region R = {( x, y)ÎR 2 : x 2 £ y £ 2 x}. If a line 2 2
y = a divides the area of region R into two equal parts, 3 1
1
then which of the following is true? [Sep. 02, 2020 (II)] (c) (d) 2 - log 2
2 e
(a) a3 - 6a 2 + 16 = 0 (b) 3a 2 - 8a 3/ 2 + 8 = 0 22. The area (in sq. units) of the region
(c) 3a 2 - 8a + 8 = 0 (d) a3 - 6a3/ 2 - 16 = 0 A = {(x, y) : x2 £ y £ x + 2} is: [April 9, 2019 (I)]
14. The area of the region, enclosed by the circle x2 + y2 = 2 10 9 31 13
which is not common to the region bounded by the (a) (b) (c) (d)
3 2 6 6
parabola y2 = x and the straight line y = x, is: 23. The area (in sq. units) of the region
[Jan. 7, 2020 (I)]
(a) (24p – 1) (b) (6p – 1) y2
A = {( x, y) : £ x £ y + 4} is: [April 09, 2019 (II)]
(c) (12p – 1) (d) (12p – 1)/6 2
15. The area (in sq. units) of the region
{(x, y) ÎR2|4x2 £ y £ 8x + 12} is: [Jan. 7, 2020 (II)] 53
(a) (b) 30 (c) 16 (d) 18
3
125 128 124 127
(a) (b) (c) (d) 24. Let S(a) = {(x, y) : y2 £ x, 0 £ x £ a} and A(a) is area of the
3 3 3 3 region S(a). If for a l, 0 < l < 4, A(l) : A(a) = 2 : 5, then l
16. For a > 0, let the curves C1: y2 = ax and C2: x2= ay intersect equals : [April 08, 2019 (II)]
at origin O and a point P. Let the line x = b (0 < b < a) 1 1
intersect the chord OP and the x-axis at points Q and R, æ 4 ö3
(b) 2 æç ö÷
2 3
(a) 2 ç ÷
respectively. If the line x = b bisects the area bounded by è 25 ø è5ø
1
the curves, C1 and C2, and the area of DOQR = , then 1 1
2 æ 2 ö3
(d) 4 æç ö÷
4 3
(c) 4 ç ÷
‘a’ satisfies the equation: [Jan. 8, 2020 (I)] è5ø è 25 ø

Downloaded from @Freebooksforjeeneet


Applications of Integrals M-419

25. The area (in sq. units) of the region bounded by the
1 p 1 2p
parabola, y = x2 + 2 and the lines, y = x + 1, x = 0 and x = 3, (a) + (b) +
is : [Jan. 12, 2019 (I)] 2 3 3 3 3
1 2p 1 4p
(a)
15
(b)
21
(c)
17
(d)
15
(c) + (d) +
4 2 4 2 2 3 3 3 3
26. The area (in sq. units) of the region bounded by the curve 34. The area (in sq. units) of the region {(x, y) : y2 ³ 2x and
x2 + y2 £ 4x, x ³ 0, y ³ 0} is : [2016]
x 2 = 4 y and the straight line x = 4 y – 2 is :
4 2 p 2 2
[Jan. 11, 2019 (I)] (a) p - (b) -
3 2 3
5 9 7 3
(a) (b) (c) (d) 4 8
4 8 8 4 (c) p - (d) p -
3 3
27. The area (in sq. units) in the first quadrant bounded by the 35. The area (in sq. units) of the region described by
parabola, y = x2 + 1, the tangent to it at the point (2, 5) and A = {(x, y)| y ³ x2 – 5x + 4, x + y ³ 1, y £ 0} is:
the coordinate axes is : [Jan. 11, 2019 (II)] [Online April 9, 2016]
8 37 187 14 19 17 7 13
(a) (b) (c) (d) (a) (b) (c) (d)
3 24 24 3 6 6 2 6
28. If the area enclosed between the curves y = kx2 and 36. The area (in sq. units) of the region described by
x = ky2, (k > 0), is 1 square unit. Then k is: {(x, y) : y2 £ 2x and y ³ 4x – 1} is [2015]
[Jan. 10, 2019 (I)] 15 9 7 5
(a) (b) (c) (d)
64 32 32 64
3 1 2
(a) (b) (c) 3 (d) 37. The area (in square units) of the region bounded by the
2 3 3 curves y + 2x2 = 0 and y + 3x2 = 1, is equal to :
29. The area (in sq. units) bounded by the parabola y = x2 –1, the [Online April 10, 2015]
tangent at the point (2, 3) to it and the y-axis is: 3 1 4 3
[Jan. 9, 2019 (I)] (a) (b) (c) (d)
5 3 3 4
8 32 56 14 38. The area of the region described by
(a) (b) (c) (d)
3 3 3 3
30. If the area of the region bounded by the curves, y = x2,
A= {( x, y ) : x 2
}
+ y 2 £ 1 and y 2 £ 1 - x is: [2014]

1 p 2 p 2 p 4 p 4
y= and the lines y = 0 and x = t (t > 1) is 1 sq. unit, then (a) - (b) + (c) + (d) -
x 2 3 2 3 2 3 2 3
t is equal to [Online April 16, 2018] 39. The area of the region above the x-axis bounded by the
p
(a)
4
(b) e 2/3 (c)
3
(d) e 3/2 curve y = tan x, 0 £ x £ and the tangent to the curve at
3 2 2
31. The area (in sq. units) of the region p
x= is: [Online April 19, 2014]
{ x Î R : x ³ 0, y ³ 0, y ³ x – 2 and y £ x , is} 4
1æ 1ö 1æ 1ö
[Online April 15, 2018] (a) ç log 2 - ÷ (b) ç log 2 + ÷
2è 2ø 2è 2ø
13 10 5 8
(a) (b) (c) (d) 1 1
3 3 3 3 (c) (1 - log 2 ) (d) (1 + log 2 )
32. The area (in sq. units) of the region 2 2
40. Let A = {(x, y): y2 £ 4x, y – 2x ³ – 4}. The area (in square
{(x, y) : x ³ 0, x + y £ 3, x2 £ 4y and y £ 1 + x } is : units) of the region A is: [Online April 9, 2014]
[2017] (a) 8 (b) 9 (c) 10 (d) 11
41. The area (in square units) bounded by the curves
5 59 3 7
(a) (b) (c) (d) y = x , 2y – x + 3 = 0, x-axis, and lying in the first quadrant
2 12 2 3
33. The area (in sq. units) of the smaller portion enclosed is : [2013]
between the curves, x2 + y2 = 4 and y2 = 3x, is : 27
[Online April 8, 2017] (a) 9 (b) 36 (c) 18 (d)
4

Downloaded from @Freebooksforjeeneet


EBD_8344
M-420 Mathematics

52. The area of the region enclosed by the curves


p
42. The area under the curve y = | cos x – sin x |, 0 £ x £ , 1
2 y = x, x = e, y = and the positive x-axis is [2011]
and above x-axis is : [Online April 23, 2013] x
3
(a) 2 2 (b) 2 2 - 2 (a) 1 square unit (b) square units
2
(c) 2 2 + 2 (d) 0
5 1
43. The area of the region (in sq. units), in the first quadrant (c) square units (d) square unit
2 2
bounded by the parabola y = 9x2 and the lines x = 0, y = l
and y = 4, is : [Online April 22, 2013] 53. The area bounded by the curves y = cos x and
(a) 7/9 (b) 14/3 (c) 7/3 (d) 14/9 3p
y = sin x between the ordinates x = 0 and x = is
44. The area bounded by the curve y = ln (x) and the lines 2
y = 0, y = ln (c) and x = 0 is equal to :[Online April 9, 2013] [2010]
(a) 3 (b) 3 ln (c) – 2
(a) 4 2 + 2 (b) 4 2 - 1
(c) 3 ln (c) + 2 (d) 2
45. The area between the parabolas x 2 =
y
and (c) 4 2 + 1 (d) 4 2 - 2
4 54. The area of the region bounded by the parabola
x2 = 9y and the straight line y = 2 is : [2012] (y – 2)2 = x –1, the tangent of the parabola at the point
10 2 20 2 (2, 3) and the x-axis is: [2009]
(a) 20 2 (b) (c) (d) 10 2 (a) 6 (b) 9 (c) 12 (d) 3
3 3
46. 2
The area bounded by the parabola y = 4x and the line 55. The area of the plane region bounded by the curves
2x – 3y + 4 = 0, in square unit, is [Online May 26, 2012] x + 2y2 = 0 and x + 3y2 = 1is equal to [2008]
2 1 1 5 1 4
(a) (b) (c) 1 (d) (a) (b) (c) 2 (d)
5 3 2 3 3 3 3
47. The area of the region bounded by the cur ve 56. The area enclosed between the curves y2 = x and y = | x | is
y = x3, and the lines, y = 8, and x = 0, is [2007]
[Online May 19, 2012] (a) 1/6 (b) 1/3 (c) 2/3 (d) 1
(a) 8 (b) 12 (c) 10 (d) 16
2
57. The parabolas y 2 = 4 x and x = 4 y divide the square
48. If a straight line y – x = 2 divides the region x2 + y 2 £ 4
region bounded by the lines x = 4, y = 4 and the coordinate
into two parts, then the ratio of the area of the smaller part
axes. If S1 , S2 , S3 are respectively the areas of these parts
to the area of the greater part is [Online May 12, 2012]
(a) 3p – 8 : p + 8 (b) p – 3 : 3p + 3 numbered from top to bottom; then S1 : S2 : S3 is [2005]
(c) 3p – 4 : p + 4 (d) p – 2 : 3p + 2 (a) 1 : 2 : 1 (b) 1 : 2 : 3 (c) 2 : 1 : 2 (d) 1 : 1 : 1
49. The area enclosed by the curves y = x 2 , y = x 3 ,
58. The area of the region bounded by the curves
x = 0 and x = p, where p > 1, is 1/6. The p equals
[Online May 12, 2012] y =| x - 2 |, x = 1, x = 3 and the x-axis is [2004]
(a) 8/3 (b) 16/3 (c) 2 (d) 4/3 (a) 4 (b) 2 (c) 3 (d) 1
50. The parabola y2 = x divides the circle x2 + y2 = 2 into two 59. The area of the region bounded by the curves
parts whose areas are in the ratio [Online May 7, 2012]
(a) 9p + 2 : 3p – 2 (b) 9p – 2 : 3p + 2 y = x - 1 and y = 3 - x is [2003]
(c) 7p – 2 : 2p – 3 (d) 7p + 2 : 3p + 2 (a) 6 sq. units (b) 2 sq. units
51. The area bounded by the curves [2011 RS] (c) 3 sq. units (d) 4 sq. units.
y 2 = 4 x and x2 = 4y is: 60. The area bounded by the curves y = lnx, y = ln |x|,y = | ln x
| and y = | ln |x| | is [2002]
32 16 (a) 4sq. units (b) 6 sq. units
(a) sq units (b) sq units
3 3 (c) 10 sq. units (d) none of these
8
(c) sq. units (d) 0 sq. units
3

Downloaded from @Freebooksforjeeneet


Differential Equations M-435

24
Differential
Equations
(c) Statement-1 is true; Statement-2 is false.
Ordinary Differential Equations, (d) Statement-1 is false; Statement-2 is true.
Order & Degree of Differential
TOPIC Ć Equations, Formation of
5. Statement 1: The degrees of the differential equations
Differential Equations dy d2 y
+ y 2 = x and + y = sin x are equal.
dx dx 2
1. The differential equation of the family of curves,
Statement 2: The degree of a differential equation, when it
x2 = 4b (y + b), b Î R, is: [Jan. 8, 2020 (II)] is a polynomial equation in derivatives, is the highest
(a) x(y¢)2 = x + 2yy¢ (b) x(y¢)2 = 2yy¢ – x positive integral power of the highest order derivative
involved in the differential equation, otherwise degree is
(c) xy² = y¢ (d) x(y¢)2 = x – 2yy¢ not defined. [Online May 12, 2012]
2. The differential equation representing the family of ellipse (a) Statement 1 is true, Statement 2 is true, Statement 2 is
having foci either on the x-axis or on the y-axis centre at not a correct explanation of Statement 1.
the origin and passing through the point (0, 3) is: (b) Statement 1 is false, Statement 2 is true.
[Online April 16, 2018]
(a) xyy' + y2 – 9 = 0 (b) x + yy" = 0 (c) Statement 1 is true, Statement 2 is false.
(c) xyy" + x (y')2 – yy' = 0 (d) xyy' – y2 + 9 = 0 (d) Statement 1 is true, Statement 2 is true; Statement 2 is
a correct explanation of Statement 1.
3. If the differential equation representing the family of all
circles touching x-axis at the origin is 6. The differential equation which represents the family of
curves y = c1ec2 x , where c1, and c2 are arbitrary constants,
( x 2 - y2 ) dy
dx
= g ( x ) y , then g(x) equals:
is [2009]
[Online April 9, 2014] (a) y" = y'y (b) yy" = y'
1 (c) yy" = (y')2 (d) y' = y2
(a) x (b) 2x2 7. The differential equation of the family of circles with fixed
2
radius 5 units and centre on the line y = 2 is [2009]
1 2
(c) 2x (d) x (a) (x – 2)y'2 = 25 –(y – 2)2
2
(b) (y – 2)y'2 = 25 –(y – 2)2
4. Statement-1: The slope of the tangent at any point P on a
parabola, whose axis is the axis of x and vertex is at the (c) (y – 2)2y'2 = 25 –(y – 2)2
origin, is inversely proportional to the ordinate of the point P. (d) (x – 2)2 y'2 = 25 –(y – 2)2
Statement-2: The system of parabolas y2 = 4ax satisfies 8. The differential equation of all circles passing through the
a differential equation of degree 1 and order 1. origin and having their centres on the x-axis is [2007]
[Online April 9, 2013]
dy dy
(a) Statement-1 is true; Statement-2 is true; Statement-2 (a) y 2 = x 2 + 2 xy (b) y 2 = x 2 - 2 xy
dx dx
is a correct explanation for statement-1.
(b) Statement-1 is true; Statement-2 is true; Statement-2 dy dy
(c) x 2 = y 2 + xy (d) x 2 = y 2 + 3 xy
is not a correct explanation for statement-1. dx dx

Downloaded from @Freebooksforjeeneet


EBD_8344
M-436 Mathematics

9. The differential equation whose solution is Ax 2 + By 2 = 1 1 æ 1 + x 2 - 1ö


(c) 1 + y 2 + 1 + x 2 = log e ç ÷ +C
where A and B are arbitrary constants is of [2006] 2 çè 1 + x 2 + 1÷ø
(a) second order and second degree
(b) first order and second degree æ 1 + x 2 - 1ö
1
(c) first order and first degree (d) 1 + y 2 - 1 + x2 = log e ç ÷ +C
(d) second order and first degree 2 çè 1 + x 2 + 1÷ø
10. The differential equation representing the family of curves
æ2 ö
y 2 = 2c ( x + c ) , where c > 0, is a parameter, is of order 15. If y = ç x - 1÷ cosec x is the solution of the differential
èp ø
and degree as follows : [2005]
(a) order 1, degree 2 (b) order 1, degree 1 dy 2 p
equation, + p( x ) y = cosec x , 0 < x < , then the
(c) order 1, degree 3 (d) order 2, degree 2 dx p 2
11. The differential equation for the family of circle function p(x) is equal to: [Sep. 06, 2020 (II)]
2 2
x + y - 2ay = 0, where a is an arbitrary constant is (a) cot x (b) cosec x
[2004] (c) sec x (d) tan x
2 2 2 2
(a) ( x + y ) y ¢ = 2 xy (b) 2( x + y ) y ¢ = xy 16. If y = y (x) is the solution of the differential equation
2 2
(c) ( x - y ) y ¢ = 2 xy (d) 2( x - y ) y ¢ = xy 2 2
5 + e x . dy x
+ e = 0 satisfying y (0) = 1, then a value of
12. The degree and order of the differential equation of the 2 + y dx
family of all parabolas whose axis is x - axis, are respectively.
[2003] y(loge 13) is : [Sep. 05, 2020 (I)]
(a) 2, 3 (b) 2, 1 (a) 1 (b) – 1
(c) 1, 2 (d) 3, 2. (c) 0 (d) 2
13. The order and degree of the differential equation 17. The solution of the differential equation
2/3 3
æ dy ö d y y + 3x
çè1 + 3 ÷ø = 4 are [2002] dy
- + 3 = 0 is : [Sep. 04, 2020 (II)]
dx dx3 dx log e ( y + 3 x)
2
(a) (1, ) (b) (3, 1) (where C is a constant of integration.)
3
1
(a) x - (log e ( y + 3 x))2 = C
(c) (3, 3) (d) (1, 2) 2

General & Particular Solution of (b) x - log e ( y + 3x ) = C


Differential Equation, Solution of
Differential Equation by the Method 1
TOPIC n (c) y + 3x - (loge x )2 = C
of Separation of Variables, Solution 2
of Homogeneous Differential
Equations (d) x - 2 log e ( y + 3x ) = C

14. The general solution of the differential equation 18. Let f : (0, ¥) ® (0, ¥) be a differentiable function such

dy
1+ x 2 + y 2 + x 2 y 2 + xy t 2 f 2 ( x ) - x 2 f 2 (t )
= 0 is : [Sep. 06, 2020 (I)] that f (1) = e and lim = 0.
dx t ®x t-x
(where C is a constant of integration) If f (x) = 1, then x is equal to : [Sep. 04, 2020 (II)]
1 æ 1 + x 2 + 1ö
1
(a) 1 + y 2 + 1 + x 2 = log e ç ÷ +C (a) (b) 2e
2 çè 1 + x 2 - 1÷ø e

1
1 æ 1 + x 2 + 1ö (c) (d) e
2 2
(b) 1 + y - 1 + x = log e ç ÷ +C 2e
2 çè 1 + x 2 - 1÷ø

Downloaded from @Freebooksforjeeneet


Differential Equations M-437

19. The solution curve of the differential equation, p +1 1


(a) (b)
dy 4 4
(1 + e - x )(1 + y 2 ) = y 2 , which passes through the point
dx p -1 p+ 2
(c) (d)
(0, 1), is : [Sep. 03, 2020 (I)] 4 4
dy xy
æ æ 1 + e- x ö ö 24. If = ; y (1) = 1; then a value of x satisfying
(a) y 2 + 1 = y ç log e ç ÷÷ + 2 ÷÷ dx x 2 + y 2
ç ç 2
è è ø ø
y(x) = e is: [Jan. 9, 2020 (II)]
æ æ 1+ ex ö ö 1 e
(b) y 2 + 1 = y ç log e çç ÷÷ + 2 ÷÷ (a) 3e (b)
ç 2 2
è è 2 ø ø
(c) 2e (d) 3e
æ 1 + ex ö
(c) y 2 = 1 + y log e çç ÷÷ 25. Let f (x) = (sin(tan x) + sin(cot x))2 – 1, |x| > 1. If
–1 –1

è 2 ø

æ 1 + e- x ö
dy 1 d
=
dx 2 dx
(sin–1 (f (x))) and y ( 3 ) = p6 , then y (- 3) is
(d) y 2 = 1 + y loge çç ÷÷
è 2 ø equal to: [Jan. 8, 2020 (I)]
2p p
20. If x3dy + xy dx = x 2 dy + 2 y dx; y (2) = e and x > 1, then (a) (b) -
3 6
y(4) is equal to : [Sep. 03, 2020 (II)]
5p p
3 3 (c) (d)
(a) + e (b) e 6 3
2 2
26. Let y = y(x) be a solution of the differential equation,
1 e dy
(c) + e (d) 1 - x2 + 1 - y 2 = 0,| x |< 1.
2 2 dx
21. Let y = y(x) be the solution of the differential equation,
æ 1ö 3 æ -1 ö
If y ç ÷ = , then y ç
è 2 ÷ø
2 + sin x dy is equal to:
è 2ø 2
× = - cos x, y > 0, y (0) = 1. If y (p) = a and
y + 1 dx [Jan. 8, 2020 (I)]
dy
at x = p is b, then the ordered pair (a, b) is equal to : 3 1
dx (a) (b) –
[Sep. 02, 2020 (I)] 2 2
æ 3ö 1 3
(a) ç 2, ÷ (b) (1, –1) (c) (d) –
è 2ø 2 2
(c) (1, 1) (d) (2, 1) 27. If y = y(x) is the solution of the differential equation,
22. If a curve y = f (x), passing through the point (1, 2), is the
solution of the differential equation, ey = ex such that y(0) = 0, then y(l) is equal to:

æ1ö [Jan. 7, 2020 (I)]


2 x 2 dy = (2 xy + y 2 ) dx, then f ç ÷ is equal to :
è 2ø (a) l + loge 2 (b) 2 + loge 2
[Sep. 02, 2020 (II)]
1 1 (c) 2e (d) loge 2
(a) (b)
1 + loge 2 1 - log e 2 28. The general solution of the differential equation (y2 – x3)
-1 dx – xydy = 0 ( x ¹ 0) is : [April 12, 2019 (II)]
(c) 1 + log e 2 (d)
1 + loge 2 2 2
(a) y – 2x + cx = 0 3
(b) y2 + 2x3 + cx2 = 0
p p (c) y2 + 2x2 + cx3 = 0 (d) y2 – 2x3 + cx2 = 0
23. If f 2 (x) = tan–1 (sec x + tan x), – <x< ,
2 2 (where c is a constant of integration)
and f(0) = 0, then f(1) is equal to: [Jan. 9, 2020 (I)]

Downloaded from @Freebooksforjeeneet


EBD_8344
M-438 Mathematics

dy p æpö (a) a circle with centre on the x-axis.


29. If cos x - y sin x = 6 x,(0 < x < ) and y ç ÷ = 0, then (b) an ellipse with maor axis along the y-axis.
dx 2 è3ø
(c) a circle with centre on the y-axis.
æpö (d) a hyperbola with transverse axis along the x-axis.
y ç ÷ is equal to: [April. 09, 2019 (II)]
è6ø 34. Let f : [0, 1] ® R be such that f (xy) = f (x).f (y), for all
p2 p2 x, y Î [0, 1], and f (0) ¹ 0. If y = y(x) satisfies the differential
(a) (b) -
2 3 2
dy æ1ö æ 3ö
equation, = f (x) with y(0) = 1, then y ç ÷ + y ç ÷ is
p2 p2 dx è ø
4 è 4ø
(c) - (d) -
2 3 4 3
equal to: [Jan. 09, 2019 (II)]
30. Given that the slope of the tangent to a curve y = y(x) at (a) 3 (b) 4
2y (c) 2 (d) 5
any point (x, y) is . If the curve passes through the
x2 35. The curve satisfying the differential equation,
centre of the circle x2 + y2 – 2x – 2y = 0, then its equation is: (x2 – y2) dx + 2xydy = 0 and passing through the point
(1, 1) is [Online April 15, 2018]
[April. 08, 2019 (II)] (a) a circle of radius two (b) a circle of radius one
(a) x loge | y | = 2(x – 1) (c) a hyprbola (d) an ellipse
(b) x loge | y | = – 2(x – 1) dy æ pö
36. If (2 + sin x) + (y + 1) cos x = 0 and y(0) = 1, then y ç ÷
(c) x2 loge | y | = – 2(x – 1) dx è 2ø
(d) x loge | y | = x – 1 is equal to : [2017]
4 1
dy (a) (b)
= (x - y) ,
2 3 3
31. The solution of the differential equation,
dx
2 1
(c) - (d) -
when y (1) = 1, is : [Jan. 11, 2019 (II)] 3 3
37. If a curve y = f(x) passes through the point (1, –1) and
2- x satisfies the differential equation, y(1 + xy) dx = x dy, then
(a) log e = x- y
2- y
æ 1ö
f ç - ÷ is equal to : [2016]
è 2ø
1- x + y
(b) - log e = 2 ( x - 1) 2 4
1+ x - y (a) (b)
5 5
1+ x - y 2 4
(c) - log e =x+ y-2 (c) - (d) -
1- x + y 5 5
38. If f(x) is a differentiable function in the interval ( (0, ¥) such
2- y
(d) loge = 2 ( y - 1) t 2 f (x) - x 2f (t)
2-x that f(a) = 1 and lim = 1 , for each x > 0,
t ®x t-x
dy 3 1 æ- p pö æpö 4 æ3ö
32. If + y= , x Îç , ÷ , and y ç ÷ = ,
2
dx cos x 2
cos x è 3 3ø è4ø 3 then f ç ÷ is equal to : [Online April 9, 2016]
è2ø
æ pö
then y ç - ÷ equals: [10 Jan 2019 I] 23 13
è 4ø (a)
18
(b)
6
1 1
(a) + e6 (b) 25 31
3 3 (c) (d)
9 18
4
-
1 3
+e 39. The solution of the differential equation ydx – (x + 2y2)dy
(c) (d)
3 3 = 0 is x = f(y). If f(–1) = 1, then f(a) is equal to :
33. The curve amongst the family of curves represented by [Online April 11, 2015]
the differential equation, (x2 – y2) dx + 2xydy = 0 which (a) 4 (b) 3
(c) 1 (d) 2
passes through (1, 1), is: [Jan. 10, 2019 (II)]

Downloaded from @Freebooksforjeeneet


Differential Equations M-439

40. If y(x) is the solution of the differential equation Statement-1: The substitution z = y2 transforms the above
equation into a first order homogenous differential
( x + 2 ) dy = x 2 + 4x - 9, x ¹ -2 and y(0) = 0, then y (–4) equation.
dx Statement-2: The solution of this differential equation is
is equal to : [Online April 10, 2015]
- y2
(a) 0 (b) 2 y2 e x =C.
(c) 1 (d) –1
41. Let the population of rabbits surviving at time t be (a) Both statements are false.
(b) Statement-1 is true and statement-2 is false.
dp ( t ) 1
p ( t ) – 200.
(c) Statement-1 is false and statement-2 is true.
governed by the differential equation =
dt 2 (d) Both statements are true.
If p(0) = 100, then p(t) equals: [2014]
46. The population p (t) at time t of a certain mouse species
(a) 600 - 500 e t 2 (b) 400 - 300 e -t 2
dp ( t )
(c) 400 - 300 et 2 (d) 300 - 200 e -t 2 satisfies the differential equation = 0.5 p(t) – 450. If
dt
42. If the general solution of the differential equation
p (0) = 850, then the time at which the population becomes
y æ xö ero is : [2012]
y¢ = + F ç ÷ , for some function F, is given by
x è yø (a) 2ln 18 (b) ln 9
y ln |cx| = x, where c is an arbitrary constant, then F(2) is
equal to: [Online April 11, 2014] 1
(c) ln 18 (d) ln 18
1 2
(a) 4 (b)
4 ( 2 + sin x ) dy = cos x
47. Let y (x) be a solution of . If y (0) = 2,
1 (1 + y ) dx
(c) – 4 (d) -
4 æ pö
then y ç ÷ equals [Online May 7, 2012]
43. At present, a firm is manufacturing 2000 items. It is estimated è 2ø
that the rate of change of production P w.r.t. additional
5
dP (a) (b) 2
number of workers x is given by = 100 – 12 x . If the 2
dx
firm employs 25 more workers, then the new level of 7
production of items is [2013] (c) (d) 3
2
(a) 2500 (b) 3000 48. The curve that passes through the point (2, 3), and has the
property that the segment of any tangent to it lying between
(c) 3500 (d) 4500 the coordinate axes is bisected by the point of contact is
given by : [2011RS]
If a curve passes through the point æç 2, ö÷ and has slope
7
44.
è 2ø 6
(a) 2 y - 3x = 0 (b) y =
x
æ 1 ö
ç 1 - 2 ÷ at any point (x, y) on it, then the ordinate of the
2 2
æxö æ yö
è x ø (c) x2 + y 2 = 13 (d) ç ÷ + ç ÷ = 2
è 2ø è 3ø
point on the curve whose abscissa is – 2 is :
[Online April 23, 2013] 49. Let I be the purchase value of an equipment and V (t) be
the value after it has been used for t years. The value V(t)
3 3
(a) – (b) depreciates at a rate given by differential equation
2 2
dV (t )
5 5 = - k (T - t ), where k > 0 is a constant and T is the
(c) (d) – dt
2 2 total life in years of the equipment. Then the scrap value
45. Consider the differential equation : V(T) of the equipment is [2011]

dy y3 kT 2 k (T - t )2
= I- I-
( )
(a) (b)
dx 2 xy 2 - x 2 2 2
1
(d) T -
[Online April 22, 2013] 2
(c) e– kT
k

Downloaded from @Freebooksforjeeneet


EBD_8344
M-440 Mathematics

dy 56. Let y = y (x) be the solution of the differential equation,


50. If = y + 3 > 0 and y (0) = 2, then y (ln 2) is equal to :
dx xy '- y = x 2 ( x cos x + sin x), x > 0. If y (p) = p, then
[2011]
(a) 5 (b) 13 æpö æpö
y '' ç ÷ + y ç ÷ is equal to : [Sep. 04, 2020 (I)]
(c) – 2 (d) 7 è ø
2 è2ø
51. The solution of the differential equation dy = x + y
dy x p p p2
(a) 2+ (b) 1 + +
satisfying the condition y(1) =1 is [2008] 2 2 4
(a) y = ln x + x (b) y = x ln x + x2
p p2 p
(c) 2+ + (d) 1 +
(c) y = xe(x – 1) (d) y = x ln x + x 2 4 2
52. The normal to a curve at P(x, y) meets the x-axis at G. If the
distance of G from the origin is twice the abscissa of P, 57. If for x ³ 0, y = y(x) is the solution of the differential equation,
then the curve is a [2007] (x + 1)dy = ((x + 1)2 + y – 3)dx, y(2) = 0,
(a) circle (b) hyperbola
(c) ellipse (d) parabola. then y(3) is equal to _____. [NA Jan. 09, 2020 (I)]

dy 58. Let y = y(x) be the solution curve of the differential


53. If x = y (log y – log x + 1), then the solution of the
( ) dydx = 1, satisfying y(0) = 1. This curve
dx
equation, y - x
2
equation is [2005]
æ xö æ yö intersects the x-axis at a point whose abscissa is:
(a) y log ç ÷ = cx (b) x log ç ÷ = cy
è yø è xø
[Jan. 7, 2020 (II)]
æ yö æ xö (a) 2 – e (b) – e
(c) log ç ÷ = cx (d) log ç ÷ = cy
è xø è yø
(c) 2 (d) 2 + e
2
d y æ 1ö
= e -2 x
Consider the differential equation, y dx + ç x - ÷ dy = 0 .
54. The solution of the equation [2002] 2
dx 2 59.
è yø

e -2 x e -2 x If value of y is 1 when x = 1, then the value of x for which


(a) (b) + cx + d y = 2, is : [April 12, 2019 (I)]
4 4
5 1 3 1
1 -2 x 1 -4 x
+ cx + d + -
(c) e + cx 2 + d (d) e (a)
2 e
(b)
2 e
4 4
1 1 3
(c) + - e
(d)
2 e 2
TOPIC Đ d 2 y 60. If y = y(x) is the solution of the differential equation

dx 2 dy æ p pö
= (tan x - y )sec2 x , x Î ç - 2 , 2 ÷ , such that y (0) = 0,
dx è ø
55. Let y = y(x) be the solution of the differential equation
æ pö
dy æ pö then y ç - ÷ is equal to : [April 10, 2019 (I)]
cos x + 2 y sin x = sin 2 x, x Î ç 0, ÷ . è 4ø
dx è 2ø
1
If y (p / 3) = 0, then y (p / 4) is equal to : (a) e – 2 (b) -e
2
[Sep. 05, 2020 (II)]
1 1
(c) 2+ (d) -2
(a) 2- 2 (b) 2+ 2 e e
1
(c) 2 -2 (d) -1
2

Downloaded from @Freebooksforjeeneet


Differential Equations M-441

61. Let y = y(x) be the solution of the differential equation, 66. If y(x) is the solution of the differential equation
æ p pö dy æ 2 x + 1 ö -2 x 1 -2
dy
+ y tan x = 2 x + x 2 tan x , x Î ç - , ÷ , such that y (0) +ç ÷ y = e , x > 0, where y (1) = e , then
dx è 2 2ø dx è x ø 2
: [Jan. 11, 2019 (I)]
= 1. Then : [April 10, 2019 (II)]
(a) y ( log e 2 ) = loge 4
æpö æ pö p
2
(a) yç ÷ + yç- ÷ = +2 log e 2
è4ø è 4ø 2 (b) y ( log e 2 ) =
4
æpö æ pö
(b) y 'ç ÷ + y ' ç - ÷ = - 2 æ1 ö
è ø
4 è 4ø (c) y ( x ) is decreasing in ç ,1÷
è2 ø
æpö æ pö (d) y (x) is decreasing in (0, 1)
(c) yç ÷ - yç - ÷ = 2
è ø
4 è 4ø 3 f ( x)
67. Let f be a differentiable function such that f ¢(x) = 7 - ,
æpö æ pö 4 x
(d) y¢ ç ÷ - y ¢ ç - ÷ = p - 2
è ø
4 è 4ø æ1ö
(x > 0) and f (1) ¹ 4. Then lim x f ç ÷ :
x ® 0+ è xø
dy
62. The solution of the differential equation x + 2y = x2 [Jan. 10, 2019 (II)]
dx
(x ¹ 0) with y(1) = 1, is: [April 09, 2019 (I)] 4
(a) exists and equals . (b) exists and equals 4.
7
4 3 1 3
(a) y = x + 2 (b) y = x + 1 (c) does not exist. (d) exists and equals 0.
5 5x 5 5 x2 68. If y = y(x) is the solution of the differential equation,
x2 3 3 2 1 æ1ö
(c) y = + 2 (d) y = x + 2 x
dy
+ 2 y = x 2 satisfying y(1) = 1, then y ç ÷ is equal to:
4 4x 4 4x dx è2ø
63. Let y = y(x) be the solution of the differential equation, [Jan. 09, 2019 (I)]
dy 7 1
(x2 + 1)2 + 2x(x2 + 1)y = 1 such that y(0) = 0. If a (a) (b)
dx 64 4
p 49 13
y(1) = , then the value of ‘a’ is : [April 08, 2019 (I)] (c) (d)
32 16 16
69. Let y - y(x) be the solution of the differential equation
1 1
(a) (b) dy æ pö
4 2 sin x + y cos x = 4x, x Î (0, p) . If y ç ÷ = 0 , then
dx è 2ø
1
(c) 1 (d) æ pö
16 y ç ÷ is equal to : [2018]
è6ø
64. Let y = y(x) be the solution of the differential equation,
-8 2 8
x
dy
+ y = x log x, (x > 1). If 2y(2) = log 4 –1, then y(e) is (a) p (b) - p2
dx e e 9 3 9
equal to : [Jan. 12, 2019 (I)] 4 4 2
(c) - p2 (d) p
e 2 9 9 3
e
(a) - (b) - 70. Let y = y (x) be the solution of the differential equation
2 2
dy ì1, x Î [0, 1]
e e2 + 2 y = f (x), where f (x) = í
(c) (d) dx î0, otherwise
4 4
65. If a curve passes through the point (1, – 2) and has slope æ 3ö
If y (0) = 0, then y ç ÷ is [Online April 15, 2018]
x2 - 2 y è 2ø
of the tangent at any point (x, y) on it as , then the
x
e2 – 1 e2 – 1
curve also passes through the point :[Jan. 12, 2019 (II)] (a) (b)
2e 3 e3
(a) (3, 0) (b) ( 3, 0 ) 1 e2 + 1
(c) (–1, 2) (d) ( - 2,1) (c)
2e
(d)
2e 4

Downloaded from @Freebooksforjeeneet


EBD_8344
M-442 Mathematics

71. The curve satisfying the differential equation, ydx–(x +


1
3y2) dy = 0 and passing through the point (1, 1), also (a) (b) x2 – 1
2
passes through the point : [Online April 8, 2017] x -1

æ1 1ö æ 1 1ö x2 - 1 x
(a) ç ,- ÷ (b) ç- , ÷ (c) (d) 2
è4 2ø è 3 3ø x x -1
78. The general solution of the differential equation
æ1 1ö æ1 1ö
(c)ç ,- ÷ (d) ç , ÷ dy 2
+ y = x2 is
è 3 3ø è4 2ø dx x
[Online May 19, 2012]
72. The solution of the differential equaiton
x2 x2
(a) y = cx -3 - (b) y = cx 3 -
dy y tan x p 4 4
+ sec x = , where 0 £ x < , and y(0) = 1, is
dx 2 2y 2 x3 x3
given by : [Online April 10, 2016] (c) y = cx 2 + (d) y = cx -2 +
5 5
x x 79. Consider the differential equation [2011RS]
(a) y2 = 1 + (b) y = 1 + æ 1ö
sec x + tan x sec x + tan x y 2 dx + ç x - ÷ dy = 0. If y (1) = 1, then x is given by :
è yø
x x
(c) y = 1 – (d) y2 = 1 – 1 1
sec x + tan x sec x + tan x
2 ey 1 ey
73. Let y(x) be the solution of the differential equation (a) 4- - (b) 3- +
dy y e y e
(x log x) + y = 2x log x, (x ³ 1). Then y (e) is equal to: 1 1
dx
y y
[2015] 1 e 1 e
(c) 1 + - (d) 1 - +
(a) 2 (b) 2e y e y e
(c) e (d) 0 80. Solution of the differential equation
p
dy cos x dy = y (sin x - y ) dx, 0 < x < is [2010]
74. If + y tan x = sin 2x and y(0) = 1, then y(p) is equal to: 2
dx (a) y sec x = tan x + c
[Online April 19, 2014] (b) y tan x = sec x + c
(a) 1 (b) – 1 (c) tan x = (sec x + c) y
(c) – 5 (d) 5 (d) sec x = (tan x + c) y
75. The general solution of the differential equation, 81. Solution of the differential equation
æ dy ö ydx + ( x + x 2 y )dy
= 0 is [2004]
sin 2x ç - tan x ÷ - y = 0 , is :
è dx ø
1
[Online April 12, 2014] (a) log y = Cx (b) - + log y = C
xy
(a) y tan x = x + c (b) y cot x = tan x + c
1 1
(c) y tan x = cot x + c (d) y cot x = x + c (c) + log y = C (d) - =C
xy xy
76. The equation of the curve passing through the origin and
82. The solution of the differential equation
satisfying the differential equation
-1 dy
dy (1 + y 2 ) + ( x - e tan y
) = 0 , is [2003]
(1 + x ) + 2 xy = 4 x 2 is
2 [Online April 25, 2013] dx
dx
-1 -1
(a) (1 + x 2 ) y = x3 (b) 3(1 + x 2 ) y = 2 x3 (a) xe 2 tan y
= e tan y
+k
-1
(c) (1 + x 2 ) y = 3x3 (d) 3(1 + x 2 ) y = 4 x3 (b) ( x - 2) = ke 2 tan y

77. The integrating factor of the differential equation -1 -1


(c) 2 xe tan y
= e 2 tan y
+k
( x2 - 1 )
dy
dx
+ 2 xy = x is [Online May 26, 2012] (d) xe tan
-1
y
= tan -1 y + k

Downloaded from @Freebooksforjeeneet


Vector Algebra M-459

25
Vector Algebra
5. If the volume of parallelopiped formed by the vectors
Algebra of Vectors, Section Formula, $i + l $j + k$ , $j + l k$ and l$i + k$ is minimum, then l is equal
Linear Dependence & Independence
of Vectors, Position Vector of a Point, to : [April 12, 2019 (I)]
TOPIC Ć Modulus of a Vector, Collinearity of
Three points, Coplanarity of Three 1 1
Vectors & Four Points, Vector (a) - (b) (c) 3 (d) - 3
3 3
Inequality
r
6. Let a Î R and the three vectors a = ai$ + $j + 3k$ ,
1. Let a, b, c Î R be such that a 2 + b 2 + c 2 = 1. If r r
b = 2$i + $j - a k$ an d c = a$i - 2 $j + 3k$ . Then the set
æ 2p ö æ 4p ö p r
a cos q - b cos ç q + ÷ = c cos ç q + ÷ , where q = , r r
è 3 ø è 3 ø 9 S = (a : a, b and c are coplanar) [April 12, 2019 (II)]
then the angle between the vectors ai + bj + ck and (a) is singleton
(b) is empty
bi + cj + ak is : [Sep. 03, 2020 (II)]
(c) contains exactly two positive numbers
p 2p p
(a) (b) (c) (d) 0 (d) contains exactly two numbers only one of which is
2 3 9
positive
2. Let the position vectors of points 'A' and 'B' be i + j + k r
7. If a unit vector a makes angles p 3 with $i , p 4 with $j
and 2i + j + 3k, respectively. A point 'P' divides the line
segment AB internally in the ratio l :1 (l > 0) . If O is the and q Î (0, p) with k$ , then a value of  is:
uuur uuur uuur uuur [April 09, 2019 (II)]
region and OB × OP - 3 | OA ´ OP |2 = 6, then l is equal to
5p p 5p 2p
___________. [NA Sep. 02, 2020 (II)] (a) (b) (c) (d)
ur r 6 4 12 3
3. If the vectors, p = (a + 1)i + aj + ak, q = ai + (a +1) j + ak 8. The sum of the distinct real values of m, for which the
r
and r = ai + aj + (a + 1)k (a Î R) are coplanar and vectors, m i + j + k, i + m j + k, i + j + m k, are co-planar,
ur r r r
3( p . q) 2 - l | r ´ q |2 = 0, then the value of l is _____. is : [Jan. 12, 2019 (I)]
[NA Jan. 9, 2020 (I)] (a) –1 (b) 0 (c) 1 (d) 2
r r
r r
4. Let a = 3$i + 2 $j + 2k$ and b = $i + 2 $j - 2k$ be two vectors. If 9. Let a = i + 2 j + 4k, b = i + l j + 4k and
r r r r r
a vector perpendicular to both the vectors a + b and a - b ( )
c = 2i + 4 j + l 2 - 1 k be coplanar vectors. Then the
has the magnitude 12 then one such vector is : r r
[April 12, 2019 (I)] non- ero vector a ´ c is : [Jan. 11, 2019 (I)]

(
(a) 4 2$i + 2 $j + 2k$ ) (
(b) 4 2$i - 2 $j - k$ ) (a) –10i - 5 j (b) –14i - 5 j

(c) 4 ( 2$i + 2 $j - k$ ) (d) 4 ( -2$i - 2 $j + k$ ) (c) –14i + 5 j (d) –10i + 5 j

Downloaded from @Freebooksforjeeneet


EBD_8344
M-460 Mathematics
r r r
10. Let 3i + j, i + 3 j and bi + (1 - b ) j respectively be the 17. Let a, b, c be three non- ero vectors which are pairwise
r r r r r
position vectors of the points A, B and C with respect to non-collinear. If a + 3b is collinear with c and b + 2c is
the origin O. If the distance of C from the bisector of the r r r r
collinear with a , then a + 3b + 6c is : [2011RS]
3 r r r r r
acute angle between OA and OB is , then the sum of (a) a (b) c (c) 0 (d) a + c
2
all possible values of b is : [Jan. 11, 2019 (II)] 18. If the pi$ + $j + k$ , $i + q $j + k$ and i + j + rk ( p ¹ q ¹ r ¹ 1)
(a) 4 (b) 3 (c) 2 (d) 1 vector are coplanar, then the value of pqr - ( p + q + r ) is
ur r r r r r
11. Let a = (l - 2) a + b and b = (4l - 2) a + 3b be two [2011RS]
r r (a) 2 (b) 0 (c) – 1 (d) – 2
given vectors where vectors a and b are non-collinear..
ur r r
19. The vector a = a i + 2 j + bk lies in the plane of the
The value of l for which vectors a and b are collinear, is:
r r
[Jan. 10, 2019 (II)] vectors b = i + j and c = j + k and bisects the angle
(a) – 4 (b) – 3 (c) 4 (d) 3 r
r between b and cr . Then which one of the following gives
12. Let u be a vector coplanar with the vectors
r r r r possible values of a and b? [2008]
a = 2i + 3 - k and b =  + k . If u is perpendicular to a (a) a = 2, b = 2 (b) a = 1, b = 2
r r r (c) a = 2, b = 1 (d) a = 1, b = 1
and u × b - 24 , then | u |2 is equal to : [2018]
20. ABC is a triangle, right angled at A. The resultant of the
(a) 315 (b) 256 (c) 84 (d) 336
13. Let ABC be a triangle whose circumcentre is at P. If the 1
forces acting along AB, BC with magnitudes and
r r r AB
r r r a +b+c
position vectors A, B, C and P are a, b, c and
4 1
respectively is the force along AD , where D is the
respectively, then the position vector of the orthocentre AC
of this triangle, is : [Online April 10, 2016] foot of the perpendicular from A onto BC. The magnitude
r r r
æ a + b + cö r of the resultant is [2006]
(a) - ç 2 ÷ø (b) ar + b + cr
è
r r r (a) AB 2 + AC 2 (b) ( AB )( AC )
æ a + b + cö r 2 2 AB + AC
(c) ç ( AB) ( AC )
è 2 ÷ø (d) 0
uuur uuur 1 1 1
14. If the vectors AB = 3i + 4k and AC = 5i – 2 j + 4k are (c) + (d)
AB AC AD
the sides of a triangle ABC, then the length of the median
through A is [2013] 21. If C is the mid point of AB and P is any point outside AB,
then [2005]
(a) 18 (b) 72 (c) 33 (d) 45
uuur uuur uuur
r r (a) PA + PB = 2 PC
15. If a and b are non-collinear vectors, then the value of a
r r r uuur uuur uuur
for which the vectors u = (a - 2)a + b and (b) PA + PB = PC
r r r uuur uuur uuur
v = (2 + 3a)a - 3b are collinear is : [Online April 23, 2013] (c) PA + PB + 2 PC = 0
uuur uuur uuur
3 2 3 2 (d) PA + PB + PC = 0
(a) (b) (c) – (d) –
2 3 2 3 22. Let a, b and c be distinct non- negative numbers. If the
® ®
16. If a = i - 2 j + 3k , b = 2i + 3 j - k and vectors ai + aj + ck , i + k and ci + cj + bk lie in a plane,
® ®
c = ri + j + ( 2r - 1) k are three vectors such that c is
then c is [2005]
® ® (a) the Geometric Mean of a and b
parallel to the plane of a and b , then r is equal to
(b) the Arithmetic Mean of a and b
[Online May 19, 2012] (c) equal to ero
(a) 1 (b) – 1 (c) 0 (d) 2
(d) the Harmonic Mean of a and b

Downloaded from @Freebooksforjeeneet


Vector Algebra M-461

r r r r r r r r
23. If a, b , c are non-coplanar vectors and l is a real number,, 30. Let the vectors a, b , c be such that | a |= 2, | b |= 4 and
r r r
then the vectors a + 2b + 3c , lb + 4c and (2l - 1)c are | c |= 4. If the proection of b on a is equal to the
r r r r
non coplanar for [2004] proection of c on a and b is perpendicular to c , then
r r r
(a) no value of l the value of | a + b - c | is _______.
(b) all except one value of l [NA Sep. 05, 2020 (II)]
(c) all except two values of l r r r
31. Let a, b and c be three unit vectors such that
(d) all values of l
r r r r r r r r
r r r | a - b |2 + | a - c |2 = 8. Then | a + 2b |2 + | a + 2c |2 is
24. Let a, b and c be three non- ero vectors such that no two
r r equal to _________. [NA Sep. 02, 2020 (I)]
of these are collinear. If the vector a + 2b is collinear
32. The proection of the line segment oining the points
r
with cr and b + 3cr is collinear with ar (l being some non- (1, –1, 3) and (2, –4, 11) on the line oining the points
r
ero scalar) then ar + 2b + 6cr equals [2004] (–1, 2, 3) and (3, –2, 10) is ______ . [NA Jan. 9, 2020 (I)]
r
(a) 0 (b) lb (c) lcr (d) lar 33. Let the volume of a parallelopiped whose coterminous
r r
25. Consider points A, B, C and D with position vectors edges are given by u = i + j + lk, v = i + j + 3k and
7i - 4 j + 7 k, i - 6 j + 10k , - i - 3 j + 4 k an d 5i - j + 5k r
w = 2i + j + k be 1 cu. unit. If q be the angle between the
respectively. Then ABCD is a [2003] r r
edges u and w , then cos q can be: [Jan. 8, 2020 (I)]
(a) parallelogram but not a rhombus
(b) square 7 7 5 5
(a) (b) (c) (d)
(c) rhombus 6 6 6 3 7 3 3
(d) rectangle. r
34. A vector a = a i + 2 j + b k (a, b ÎR) lies in the plane of
r r r
a a2 1 + a3 the vectors, b = i + j and c = i – j + 4 k . If a bisects
26. If b b2 1 + b3 = 0 and vectors (1, a, a 2 ), r r
the angle between b and c , then: [Jan. 7, 2020 (I)]
c c2 1 + c3
(a) ar · i + 3 = 0 (b) ar · i + 1 = 0
r r
(1, b, b2 ) and (1, c, c 2 ) are non- coplanar, then the product (c) a · k + 2 = 0 (d) a · k + 4 = 0
r r
abc equals [2003] 35. Let a = 2 i + l1 j + 3 k,
 b = 4 i + (3 - l ) j + 6 k and
2
(a) 0 (b) 2 (c) –1 (d) 1 r
c = 3i + 6 j + (l3 - 1) k be three vectors such that
  
27. The vectors AB = 3i + 4 k & AC = 5i - 2 j + 4k are the r r r r
b = 2a and a is perpendicular to c Then a possible
sides of a triangle ABC. The length of the median through
A is [2003] value of (l1, l2, l3) is: [Jan. 10, 2019 (I)]
æ 1 ö
(a) 288 (b) 18 (c) 72 (d) 33 (a) (1, 3, 1) (b) ç - , 4, 0 ÷
è 2 ø
Scalar or Dot Product of two Vectors, æ1 ö
TOPIC n Projection of a Vector Along any (c) ç , 4, - 2 ÷ (d) (1, 5, 1)
è 2 ø
other Vector, Component of a Vector
r r
36. Let a = i + j + 2 k, b = b1 i + b 2 j + 2k
® ®
r
28. If a and b are unit vectors, then the greatest value of and c = 5i + j + 2k be three vectors such that the
® ® ® ® r r r
3 | a + b | + | a - b | is ______. [NA Sep. 06, 2020 (I)] proection vector of b on a is a .
r r r
® ® If ar + b is perpendicular to c , then | b | is equal to:
29. If® x ®and ®y be two non- ero vectors such that
® ® ® [Jan. 09, 2019 (II)]
| x + y | = | x | and 2 x + l y is perpendicular to y , then
the value of l is ______. [NA Sep. 06, 2020 (II)] (a) 32 (b) 6 (c) 22 (d) 4

Downloaded from @Freebooksforjeeneet


EBD_8344
M-462 Mathematics

rr r r
r 3 ( p.q ) r
37. In a triangle ABC, right angled at the vertex A, if the position
vectors of A, B and C are respectively
r
(a) r = 3q - r r p ( p .q )
(b) rr = -qr + r r pr
( p.p ) ( p . p)
3i +  - k,
 -i + 3 + pk and 5i + q - 4k , then the point (p,
r r r r
r r (p.q) r r r 3 (p.q) r
q) lies on a line : [Online April 9, 2016] (c) r = q - r r p (d) r = -3q - r r p
(a) making an obtuse angle with the positive direction of
(p.p) (p .p)
r r
x–axis 44. Let a and b be two unit vectors. If the vector s
(b) parallel to x–axis r r
c = aˆ + 2 bˆ and d = 5 aˆ - 4bˆ are perpendicular to each other,,
(c) parallel to y–axis
(d) making an acute angle with the positive direction of then the angle between â and b̂ is : [2012]
x–axis p p p p
uuur uuur (a) (b) (c) (d)
38. In a parallelogram ABD, AB = a, AD = b and 6 2 3 4
uuur uuur uuur ® ® ®
AC = c, then DA. AB has the value : 45. If a + b + c = 0, a = 3 , b = 5 and c = 7 , then the
[Online April 11, 2015]
® ®
1 2 1 angle between a and b is [Online May 19, 2012]
(a) (a + b2 + c2) (b) (a2 – b2 + c2)
2 2 p p p p
(a) (b) (c) (d)
1 2 1 2 3 4 6 2
(c) (a + b2 – c2) (d) (b + c2 – a2)
2 3 46. A unit vector which is perpendicular to the vector
39.  y and  are three unit vectors in three-dimensional
If x,
2i - j + 2k and is coplanar with the vectors i + j - k and
space, then the minimum value of
2 2
x + y + y +  +  + x
2
[Online April 12, 2014] 2i + 2 j - k is [Online May 12, 2012]

3 2 j + k 3i + 2 j - 2 k
(a) (b) 3 (c) 3 3 (d) 6 (a) (b)
2 5 17
® ® ® ® ® ®
40. If a = 2, b = 3 and 2 a - b = 5 , then 2 a + b equals: 3i + 2 j + 2k 2i + 2 j - 2k
(c) (d)
[Online April 9, 2014] 17 3
(a) 17 (b) 7 (c) 5 (d) 1
47. ABCD is parallelogram. The position vectors of A and C
r
41. If a$ , b$ and c$ are unit vectors satisfying a$ - 3 b$ + c$ = 0 ,
are respectively, 3i + 3 j + 5k and i - 5 j - 5k . If M is the
then the angle between the vectors a$ and c$ is : midpoint of the diagonal DB, then the magnitude of the
[Online April 22, 2013] ® ®
p p p p proection of OM on OC , where O is the origin, is
(a) (b) (c) (d) [Online May 7, 2012]
4 3 6 2
r r r
42. Let a = 2$i - $j + k$ , b = $i + 2 $j - k$ and c = $i + $j - 2k$ be three (a) 7 51 (b)
7
(c) 7 50 (d)
7
r r 50 51
vectors. A vector of the type b + lc for some scalar l,
r r
48. If the vectors a = i - j + 2k , b = 2i + 4 j + k% and
r 2
whose proection on a is of magnitude is : r
3 c = li + j + m k are mutually orthogonal, then (l, m) =
[Online April 9, 2013] (a) (2, –3) (b) (–2, 3) [2010]
(c) (3, –2) (d) (–3, 2)
(a) 2i + j + 5k (b) 2i + 3 j - 3k r r r r
r
49. The non- ero vectors are a , b and c are related by a = 8b
(c) 2i - j + 5k (d) 2i + 3 j + 3k r r
uuur r uuuur r and c = –7b . Then the angle between ar and cr is
43. Let ABCD be a parallelogram such that AB = q , AD = p [2008]
r
and ÐBAD be an acute angle. If r is the vector that p p
coincide with the altitude directed from the vertex B to the (a) 0 (b) (c) (d) p
r 4 2
side AD, then r is given by : [2012]

Downloaded from @Freebooksforjeeneet


Vector Algebra M-463

50. The values of a, for which points A, B, C with position r


57. If a = 2i + j + 2k, then the value of
vectors 2i - j + k , i - 3 j - 5k and ai - 3 j + k respectively are r r r
| i ´ (a ´ i) |2 + | j ´ (a ´ j ) |2 + | k ´ (a ´ k) |2 is equal to
p
the vertices of a right angled triangle with C = are _______________. [NA Sep. 04, 2020 (II)]
2 r r r
[2006] 58. Let b , b an d c be three vectors such that
(a) 2 and 1 (b) – 2 and – 1 r r r r r
(c) – 2 and 1 (d) 2 and – 1 | a | = 3, | b | = 5, b . c = 10 and the angle between b

51. Let u , v , w be such that | u |= 1,| v |= 2, | w |= 3. If the r p r r r


and c is . If b is perpendicular to the vector b × c ,
proection v along u is equal to that of w along u and 3
r r r
v , w are perpendicular to each other then then | a ´ (b ´ c) | is equal to _____.[NA Jan. 9, 2020 (II)]
| u - v + w | equals [2004] r r
59. Let a = i - 2 j + k and b = i - j + k be two vectors. If cr
(a) 14 (b) 7 (c) 14 (d) 2 r r r r
is a vector such that b ´ cr = b ´ ar and c . a = 0, then
r r r r r r
52. a , b , c are 3 vectors, such that a + b + c = 0 , r r
c . b is equal to: [Jan. 8, 2020 (II)]
r r r rr rr rr
a = 1, b = 2, c = 3, then a.b + b .c + c .a is equal to
3 1 1
[2003] (a) - (b) (c) - (d) –1
2 2 2
(a) 1 (b) 0 (c) –7 (d) 7 r r r
r r r 60. Let a , b and c be three unit vectors such
53. If | a |= 5, | b |= 4, | c |= 3 thus what will be the value of r r
that ar + b + cr = 0. if
rr rr rr ® ® ®
| a.b + b .c + c.a | , given that a + b + c = 0 [2002] r r r r r r
l = a × b + b × c + c × a and
(a) 25 (b) 50 (c) –25 (d) –50 r r r r r r r
d = a ´ b + b ´ c + c ´ a , then
® ® ® ® ® ®
If sdaa a , b , c are vectors such that a + b + c = 0 and r
54.
( )
the ordered pair, l, d is equal to: [Jan. 7, 2020 (II)]
® ® ® ®
| a |= 7,| b |= 5,| c |= 3 then angle between vector b and
æ 3 r rö æ 3 r rö
®
(a) ç ,3a ´ c ÷ (b) ç - ,3c ´ b ÷
è2 ø è 2 ø
c is [2002]
(a) 60 (b) 30 (c) 45 (d) 90 æ 3 r rö æ 3 r rö
(c) ç ,3b ´ c ÷ (d) ç - ,3a ´ b ÷
è2 ø è 2 ø
Vector or Cross Product of two r r r
r r
TOPIC Đ
vectors, Area of a Parallelogram & 61. Let a = 3i$ + $ and b = 2i$ – $ + 3k$ . If b = b1 – b2 , where
Triangle, Scalar & Vector Tripple r r
ur ur
Product b1 is parallel to a and b2 is perpendicular to a , then
r r
55. If the volume of a parallelopiped, whose coterminus edges b1 ´b2 is equal to: [April 09, 2019 (I)]
r r
are given by the vectors a = i$ + $j + nk$ , b = 2$i + 4 $j - nk$
r (a) –3i$ + 9$ + 5k$ (b) 3i$ – 9$ – 5k$
and c = i$ + n $j + 3k$ ( n ³ 0 ) , is 158 cu.units, then:
[Sep. 05, 2020 (I)] (c)
1
( –3i$ + 9$ + 5k$ ) (d) ( 3i – 9  + 5k )
1 $ $ $
r r r r 2 2
(a) a . c = 17 (b) b . c = 10
62. The magnitude of the proection of the vector 2$i + 3 $j + k$
(c) n = 7 (d) n = 9
r r r on the vector perpendicular to the plane containing the
56. Let x0 be the point of local maxima of f ( x) = a × (b ´ c), where
r r r vectors $i + $j + k$ and $i + 2 $j + 3k$ , is : [April 08, 2019 (I)]
a = xi - 2 j + 3k, b = -2i + xj - k and c = 7i - 2 j + xk.
r r r r r r
Then the value of a × b + b × c + c × a at x = x0 is : 3 3
(a) (b) 6 (c) 3 6 (d)
[Sep. 04, 2020 (I)] 2 2
(a) – 4 (b) – 30 (c) 14 (d) – 22

Downloaded from @Freebooksforjeeneet


EBD_8344
M-464 Mathematics

r r
63. Let a = 3i + 2 j + xk and b = i – j + k , for some real x. 1 25
(a) (b) (c) 2 (d) 5
r r 8 8
Then a ´ b = r is possible if : [April 08, 2019 (II)] r
70. If the vector b = 3 $j + 4k$ is written as the sum of a vec-
3 3 3 uur r uur
(a) <r £3 (b) r ³ 5 tor b1 , parallel to a = i$ + $j and a vector b2 , perpendicu-
2 2 2
r uur uur
lar to a , then b1 ´ b2 is equal to : [Online April 9, 2017]
3 3 3
(c) 0 < r £ (d) 3 <r<5
2 2 2
(a) -3$i + 3 $j - 9k$
9
(b) 6i$ - 6 $j + k$
r r r 2
64. Let a, b and c be three unit vectors, out of which vectors
r r
b and c are non-parallel. If a and b are the angles which 9
(c) -6 $i + 6 $j - k$ (d) 3 $i - 3 $j + 9k$
r r
vector ar makes with vectors b and c respectively and 2
71. The area (in sq. units) of the parallelogram whose diagonals
r r r 1r
( )
a ´ b ´ c = b , then |a – b| is equal to :
2
are along the vectors 8i - 6 and 3i + 4 - 12k , is :
[Jan. 12, 2019 (II)] [Online April 8, 2017]
(a) 30 (b) 90 (c) 60 (d) 45 (a) 26 (b) 65 (c) 20 (d) 52
r   r    r ® ® ®
65. Let a = i - , b = i +  + k and c be a vector such that 72. Let a , b an d c be three unit vectors such that
r r r r rr r
a ´ c + b = 0 and a .c = 4 , then c
2
is equal to: ® æ® ®ö 3 æ ® ®ö ® ®
a ´ çç b ´ c ÷÷ = çç b + c ÷÷ . If b is not parallel to c , then
[Jan 09, 2019]
è ø 2 è ø
19 17
(a) (b) 9 (c) 8 (d) ® ®
2 2 the angle between a and b is: [2016]
66. If the position vectors of the vertices A, B and C of a
2p 5p 3p p
DABC are respectively 4i + 7 j + 8k, 2i + 3 j + 4k and (a) (b) (c) (d)
3 6 4 2
® ® ®
2i + 5 j + 7k , then the position vector of the point, where 73. Let a , b and c be three non- ero vectors such that no two
the bisector of ÐA meets BC is [Online April 15, 2018] ® ® ® 1® ®®
of them are collinear and (a ´ b) ´ c = b c a . If q is the
1  1  3
(a) (4i + 8 j + 11k) (b) (6i + 13 j + 18k) ® ®
2 3 angle between vectors b and c , then a value of sin q is :
1  1 
(c) (8i + 14 j + 9k) (d) (6i + 11 j + 15k) [2015]
4 3
r r r 2 -2 3 2 2 - 2
67. Let a = i + j + k, c = j – k and a vector b be such that (a) (b) (c) (d)
3 3 3 3
r r r r r r
a ´ b = c and a × b = 3. Then | b | equals? ® ® ® ®
[Online April 16, 2018] 74. Let a and b be two unit vectors such that a + b = 3 .

( )
11 11 11 11 ® ® ® ® ®
(a) (b) (c) (d) ®
3 3 3 3 If c = a + 2 b + 3 a ´ b , then 2 c is equal to :
r r r [Online April 10, 2015]
68. If a, b , and c are unit vectors such that
r r r r r r (a) 55 (b) 37 (c) 51 (d) 43
a + 2b + 2c = 0, then a ´ c is equal to r r r r r r rrr 2
[Online April 15, 2018] 75. If éë a ´ b b ´ c c ´ a ùû = l éë a b c ùû then l is equal to

1 15 15 15
[2014]
(a) (b) (c) (d) (a) 0 (b) 1 (c) 2 (d) 3
4 4 16 16 ® ® ®
r r r 76. If x = 3i - 6 - k , y = i + 4 - 3k and = 3i - 4 - 12k ,
69. Let a = 2i +  - 2k and b = i +  . Let c be a vector such
r r r ® ® ®
r r
( )
that | c - a | = 3, a ´ b ´ c = 3 and the angle between rc then the magnitude of the proection of x ´ y on is:
r r rr [Online April 19, 2014]
and a ´ b be 30 . Then a.c is equal to : [2017] (a) 12 (b) 15 (c) 14 (d) 13

Downloaded from @Freebooksforjeeneet


Vector Algebra M-465

® r r r r
® ®
( )
2 84. The vectors a and b are not perpendicular and c and d
77. If c = 60 and c ´ i + 2 + 5k = 0 , then a value of r r r r rr
® are two vectors satisfying b ´ c = b ´ d and a.d = 0 .
(
c . -7i + 2 + 3k is: ) [Online April 11, 2014] r
Then the vector d is equal to [2011]
(a) 4 2 (b) 12 (c) 24 (d) 12 2 r
r r r
rr
r æ a.c ö r r æ b .cr ö r
78. Let a = 2i + j - 2k, b = i + j. If c is a vector such that (a) c + çè r r ÷ø b (b) b + ç r r ÷ c
a.b è a.b ø
r r uur r r
a · c = | c | , | c - a | = 2 2 and the angle between r
r r r r r r
rr
r æ a.c ö r r æ b .cr ö r
a ´ b and c is 30 , then | (a ´ b ) ´ c | equals: (c) c - çè r r ÷ø b (d) b - ç r r ÷ c
a.b è a.b ø
[Online April 25, 2013]

(a)
1
2
3 3
(b) (c) 3 (d)
3
2
85.
r
If a =
1
10
( ) r 1
( )
3i + k and b = 2i + 3 j - 6k , then the
7
2
rr rr rr r r r
79. ( ) ( ) (
The vector $i ´ a.b $i + $j ´ a.b $j + k$ ´ a.b k$ is equal to : ) ( r
)(
r
) (
r
value of 2a - b éë a ´ b ´ a + 2b ùû is ) [2011]
[Online April 9, 2013] (a) –3 (b) 5 (c) 3 (d) –5
r r r r r r r
(a) b ´ a (c) a ´ b r   r   
(b) a (d) b 86. Let a = j - k and c = i - j - k . Then the vector b
® ® ®
r r r r r r
80. Statement 1: The vectors a , b and c lie in the same satisfying a ´ b + c = 0 and a. b = 3 is [2010]
® æ ® ®ö
plane if and only if a . ç b ´ c ÷ = 0 (a) 2i - j + 2k (b) i - j - 2k
è ø
®
(c) i + j - 2k (d) -i + j - 2k
®
Statement 2: The vectors u and v are perpendicular if r r r
87. If u, v, w are non-coplanar vectors and p, q are real
® ® ® ®
and only if u . v = 0 where u ´ v is a vector perpendicular numbers, then the equality
r r r rr r r r r
® ® [3u pv pw] - [ pv w qu ] - [2w qv qu ] = 0
to the plane of u and v . [Online May 26, 2012] holds for : [2009]
(a) Statement 1 is false, Statement 2 is true. (a) exactly two values of (p, q)
(b) Statement 1 is true, Statement 2 is true, Statement 2 is (b) more than two but not all values of (p, q)
correct explanation for Statement 1.
(c) all values of (p, q)
(c) Statement 1 is true, Statement 2 is false.
(d) exactly one value of (p, q)
(d) Statement 1 is true, Statement 2 is true, , Statement 2 is r
r
not a correct explanation for Statement 1. 88. Let a = i + j + k, b = i - j + 2k and
® ® ®
If u = j + 4k , v = i + 3k and w = cos qi + sin qj are r r
81. c = xi + ( x - 2) j - k . If the vector c lies in the plane of
vectors in 3-dimensional space, then the maximum possible r r
® ® ® a and b , then x equals [2007]
value of u ´ v . w is [Online May 12, 2012]
(a) – 4 (b) – 2 (c) 0 (d) 1.
(a) 3 (b) 5 (c) 14 (d) 7
89. If u and v are unit vectors and q is the acute angle
82. Statement 1: If the points (1, 2, 2), (2, 1, 2) and
(2, 2, z) and (1, 1, 1) are coplanar, then z = 2. between them, then 2 u ×3 v is a unit vector for [2007]
Statement 2: If the 4 points P, Q, R and S are coplanar, (a) no value of q
then the volume of the tetrahedron PQRS is 0. (b) exactly one value of q
[Online May 12, 2012]
(c) exactly two values of q
(a) Statement 1 is false,, Statement 2 is true.
(b) Statement 1 is true, Statement 2 is false. (d) more than two values of q
(c) Statement 1 is true, Statement 2 is true, Statement 2 is 90. If ( a ´ b ) ´ c = a ´ (b ´ c ) where a , b and c are any three
a correct explanation of Statement 1.
(d) Statement 1 is true, Statement 2 is true, Statement 2 is vectors such that a .b ¹ 0 , b . c ¹ 0 then a and c are
not a correct explanation of Statement 1. [2006]
® ® p
83. If a = i - 2 j + 3k , b = 2i + 3 j - k and (a) inclined at an angle of between them
3
®
p
c = li + j + ( 2l - 1) k are coplanar vectors, then l is (b) inclined at an angle of between them
equal to [Online May 7, 2012] 6
(a) 0 (b) – 1 (c) 2 (d) 1 (c) perpendicular
(d) parallel

Downloaded from @Freebooksforjeeneet


EBD_8344
M-466 Mathematics
uur uur ®
91. Let a = i - k , b = x i + j + (1 – x) k and ® Ù Ù ® Ù Ù
uur uur uur uur
99. a = 3 i - 5 j and b = 6 i + 3 j are two vectors and c is a
c = y i + x j + (1 + x – y) k . Then [a , b , c ] depends on ® ® ® ® ® ®
[2005] vector such that c = a ´ b then | a |:| b |:| c | [2002]
(a) only y (b) only x
(c) both x and y (d) neither x nor y (a) 34 : 45 : 39 (b) 34 : 45 : 39
uur uur uur
92. If a , b , c are non coplanar vectors and l is a real number (c) 34 : 39 : 45 (d) 39 : 35 : 34
then [2005] r r
uur uur 2 uur uur uur uur uur uur 100. If the vectors c , a = xi + yj + zk and b = j are such that
[l (a + b ) l b l c ] = [a b + c b ] for r r r r
a, c and b form a right handed system then c is:[2002]
(a) exactly one value of l r
(b) no value of l (a) zi - xk (b) 0
(c) exactly three values of l (c) yj (d) - zi + xk
(d) exactly two values of l
® ® ® ®®®
93. For any vector a , the value of 101. If a , b , c are vectors such that [ a b c ] = 4 then
ur ur ur ® ®® ® ® ®
(a ´ i)2 + (a ´ j ) 2 + (a ´ k)2 is equal to[2005] [a´ b b´ c c´ a]= [2002]
ur 2 ur 2 ur 2 ur 2 (a) 16 (b) 64 (c) 4 (d) 8
(a) 3a (b) a (c) 2a (d) 4a r r r r
102. If | a |= 4,| b |= 2 and the angle between a and b is p /6
94. Let a , b an d c be non- ero vectors such that
r r
then (a ´ b )2 is equal to [2002]
1
(a ´ b ) ´ c = | b || c | a . If q is the acute angle between (a) 48 (b) 16
3
®
the vectors b and c , then sinq equals [2004] (c) a (d) None of these

2 2 2 2 1 Scalar Product of Four Vectors,


(a) (b) (c) (d) TOPIC Ė Reciprocal System of Vector,
3 3 3 3
Application of Vectors in Mechanics
r r r
95. If u , v and w are three non- coplanar vectors, then
r r r r r r r 103. A particle ust clears a wall of height b at a distance a and
(u + v - w).(u - v ) ´ (v - w) equals [2003] strikes the ground at a distance c from the point of
rr r proection. The angle of proection is [2007]
(a) 3u .v ´ w (b) 0
r r r rr r
(c) u .(v ´ w) (d) u.w ´ v . (a) tan -1
bc
(b) tan -1
bc
96. A tetrahedron has vertices at O(0, 0, 0), A(1, 2, 1) B(2, 1, 3) a(c - a) a
and C(-1, 1, 2). Then the angle between the faces OAB and
ABC will be b
[2003] (c) tan -1 (d) 45 .
-1 æ 19 ö
ac
(a) 90 o (b) cos ç ÷
è 35 ø 104. A body weighing 13 kg is suspended by two strings 5m
and 12m long, their other ends being fastened to the
(c) cos -1æç
17 ö
÷ (d) 30 o extremities of a rod 13m long. If the rod be so held that the
è 31 ø
body hangs immediately below the middle point, then
r r r tensions in the strings are [2007]
97. Let u = i + j, v = i - j and w = i + 2 j + 3k . If n is a unit
r r r (a) 5 kg and 12 kg (b) 5 kg and 13 kg
vector such that u.n = 0 and v .n = 0 , then w. n is equal (c) 12 kg and 13 kg (d) 5 kg and 5 kg
to [2003] 105. The resultant of two forces Pn and 3n is a force of 7n. If the
(a) 3 (b) 0 (c) 1 (d) 2. direction of 3n force were reversed, the resultant would be
® ® ® ® ® ® ® ® ® 19 n. The value of P is [2007]
98. If a ´ b = b ´ c = c ´ a then a + b + c = [2002]
(a) 3 n (b) 4 n (c) 5 n (d) 6 n.
(a) abc (b) –1 (c) 0 (d) 2

Downloaded from @Freebooksforjeeneet


Vector Algebra M-467

106. A body falling from rest under gravity passes a certain 113. A paticle moves towards east from a point A to a point B at
point P. It was at a distance of 400 m from P, 4s prior to the rate of 4 km/h and then towards north from B to C at
passing through P. If g = 10m / s 2 , then the height above the rate of 5km/hr. If AB = 12 km and BC = 5 km, then its
average speed for its ourney from A to C and resultant
the point P from where the body began to fall is [2006] average velocity direct from A to C are respectively [2004]
(a) 720 m (b) 900 m (c) 320 m (d) 680 m
107. A particle has two velocities of equal magnitude inclined 13 17
(a) km / h and km / h
to each other at an angle q . If one of them is halved, the 9 9
angle between the other and the original resultant velocity 13 17
is bisected by the new resultant. Then q is [2006] (b) km / h and km / h
4 4
(a) 90 (b) 120 (c) 45 (d) 60
108. The resultant R of two forces acting on a particle is at right 17 13
(c) km / h and km / h
angles to one of them and its magnitude is one third of the 9 9
other force. The ratio of larger force to smaller one is:
17 13
[2005] (d) km / h and km / h
4 4
(a) 2 : 1 (b) 3 : 2 (c) 3 : 2 (d) 3 : 2 2 r r
114. Three forces P, Q and R acting along IA, IB and IC, where
109. A and B are two like parallel forces. A couple of moment H
lies in the plane of A and B and is contained with them. The I is the incentre of a DABC are in equilibrium. Then
resultant of A and B after combining is displaced through r r r
P : Q : R is [2004]
a distance [2005]
A B C
2H H (a) cos ec : cos ec : cos ec
(a) (b) 2 2 2
A-B A+ B
A B C
H H (b) sin : sin : sin
(c) (d) 2 2 2
2( A + B ) A-B
110. A particle is proected from a point O with velocity u at an A B C
angle of 60 with the hori ontal. When it is moving in a (c) sec : sec : sec
2 2 2
direction at right angles to its direction at O, its velocity
then is given by [2005] A B C
(d) cos : cos : cos
u u 2u u 2 2 2
(a) (b) (c) (d)
3 2 3 3 115. In a right angle DABC , ÐA = 90° and sides a, b, c are
r
111. If t1 and t2 are the times of flight of two particles having respectively, 5 cm, 4 cm and 3 cm. If a force F has moments
the same initial velocity u and range R on the hori ontal , 0, 9 and 16 in N cm. units respectively about vertices A, B
r
then t12 + t22 is equal to [2004] and C, then magnitude of F is [2004]
(a) 9 (b) 4 (c) 5 (d) 3
(a) 1 (b) 4u 2 / g 2 116. With two forces acting at point, the maximum affect is
obtained when their resultant is 4N. If they act at right
(c) u 2 / 2g (d) u 2 / g angles, then their resultant is 3N. Then the forces are
[2004]
1
112. A velocity m / s is resolved into two components along æ 1 ö æ 1 ö
4 (a) çè 2 + 3 ÷ N and ç 2 - 3÷ N
2 ø è 2 ø
OA and OB making angles 30 and 45 respectively with
the given velocity. Then the component along OB is (b) ( 2 + 3 ) N and ( 2 - 3 ) N
[2004]
æ 1 ö æ 1 ö
1
( 6 - 2 )m / s
1 (c) çè 2 + 2 ÷ N and ç 2 - 2÷ N
(a) (b) ( 3 - 1) m / s 2 ø è 2 ø
8 4

1 1 (d) ( 2 + 2 ) N and ( 2 - 2 ) N
(c) m/s (d) m/s
4 8

Downloaded from @Freebooksforjeeneet


EBD_8344
M-468 Mathematics
r r r r
117. A particle is acted upon by constant forces 4i + j - 3k 122. The resultant of forces P and Q is R . If Q is
r r
and 3i + j - k which displace it from a point i + 2 j + 3k doubled then R is doubled. If the direction of Q is
r
to the point 5i + 4 j + k . The work done in standard units reversed,then R is again doubled. Then P 2 : Q 2 : R 2 is
by the forces is given by [2004] [2003]
(a) 15 (b) 30 (c) 25 (d) 40 (a) 2 : 3 : 1 (b) 3 : 1 : 1 (c) 2 : 3 : 2 (d) 1 : 2 : 3.
r
123. A couple is of moment G and the force forming the couple
118. Let R1 and R2 respectively be the maximum ranges up r r
and down an inclined plane and R be the maximum range is P . If P is turned through a right angle the moment of
r r
on the hori ontal plane. Then R1, R , R2 are in [2003] the couple thus formed is H . If instead , the force P are
(a) H.P (b) A.G..P (c) A.P (d) G..P. turned through an angle a , then the moment of couple
119. Two particles start simultaneously from the same point becomes [2003]
r r r r
and move along two straight lines, one with uniform (a) H sin a - G cos a (b) G sin a - H cos a
r r r r r
velocity u and the other from rest with uniform acceleration (c) H sin a + G cos a (d) G sin a + H cos a .
r
f . Let a be the angle between their directions of
124. A particle acted on by constant forces 4i + j - 3k and
motion. The relative velocity of the second particle w.r.t.
the first is least after a time [2003] 3i + j - k is displaced from the point i + 2 - 3k to the
u sin a f cos a
(a) u cos a (b) (c) (d) u sin a point 5i + 4 + k . The total work done by the forces is
f f u
120. Two stones are proected from the top of a cliff h metres [2003]
high , with the same speed u, so as to hit the ground at (a) 50 units (b) 20 units
the same spot. If one of the stones is proected (c) 30 units (d) 40 units.
hori ontally and the other is proected hori ontally and 125. A bead of weight w can slide on smooth circular wire in a
the other is proected at an angle q to the hori ontal vertical plane. The bead is attached by a light thread to the
highest point of the wire and in equilibrium, the thread is
then tan q equals [2003]
taut and make an angle q with the vertical then tension of
2 2u u u the thread and reaction of the wire on the bead are
(a) u gh (b) gh (c) 2 g (d) 2 h g (a) T = w cos q R = w tan q [2002]
h
121. A body travels a distance s in t seconds. It starts from rest (b) T = 2w cos q R = w
and ends at rest. In the first part of the ourney, it moves (c) T = w R = w sin q
with constant acceleration f and in the second part with
(d) T = w sin q R = wcot q
constant retardation r. The value of t is given by [2003]
126. The sum of two forces is 18 N and resultant whose direction
æ 1 1ö æ1 1ö is at right angles to the smaller force is 12 N. The magnitude
(a) 2sçç + ÷÷ (b) 2 sçç + ÷÷
è f rø è f rø
of the two forces are [2002]
(a) 13, 5 (b) 12, 6 (c) 14, 4 (d) 11, 7
2s
(c) (d) 2s( f + r )
1 1
+
f r

Downloaded from @Freebooksforjeeneet


EBD_8344
M-488 Mathematics

26
Three Dimensional
Geometry
2 1
Direction Ratios & Direction cosines (a) (b)
of a Line, Angle between two lines 3 3
in terms of dc's and dr's, Condition 2 2
TOPIC Ć of Parallelism & Perpendicularity (c) (d)
3 3
of two Lines, Projection of a Point 6. An angle between the lines whose direction cosines are
on a Line, Projection of a Line given by the equations, l + 3m + 5n = 0 and 5lm – 2mn + 6nl
Segment Joining two Points = 0, is [Online April 15, 2018]
æ1ö æ1ö
1. If the length of the perpendicular from the point (b, 0, b) (b (a) cos -1 ç ÷ (b) cos -1 ç ÷
è8ø è6ø
x y -1 z + 1 3
¹ 0) to the line, = = is , then b is equal to:
-1 æ1ö æ1ö
1 0 2 (c) cos -1 ç ÷ (d) cos -1 ç ÷
[April 10, 2019 (I)] è ø
3 è4ø
(a) 1 (b) 2 7. ABC is triangle in a plane with vertices A (2, 3, 5), B (–1, 3,
(c) –1 (d) –2 2) and C (l, 5, m). If the median through A is equally inclined
2. The vertices B and C of a “ABC lie on the line, to the coordinate axes, then the value of (l3 + m3 + 5) is :
x + 2 y -1 z [Online April 10, 2016]
= = such that BC = 5 units. Then the area (a) 1130 (b) 1348
3 0 4
(in sq. units) of this triangle, given that the point A (1, –1, 2), (c) 1077 (d) 676
8. The angle between the lines whose direction cosines
is: [April 09, 2019 (II)]
(a) 5 17 (b) 2 34 satisfy the equations l + m + n = 0 and l 2 + m 2 + n 2 is
(c) 6 (d) 34 [2014]
3. If a point R(4, y, z) lies on the line segment oining the p p
(a) (b)
points P(2, –3, 4) and Q(8, 0, 10), then distance of R from 6 2
p p
the origin is : [April 08, 2019 (II)] (c) (d)
3 4
(a) 2 14 (b) 2 21 9. Let A (2, 3, 5), B (– 1, 3, 2) and C (l, 5, m) be the vertices of
(c) 6 (d) 53 a DABC. If the median through A is equally inclined to the
4. A tetrahedron has vertices P(1, 2, 1), Q(2, 1, 3), R(–1, 1, 2) coordinate axes, then: [Online April 11, 2014]
and O(0, 0, 0). The angle between the faces OPQ and (a) 5l – 8m = 0 (b) 8l – 5m = 0
PQR is: [Jan. 12, 2019 (I)]
(c) 10l – 7m = 0 (d) 7l – 10m = 0
-1 æ 17 ö -1 19 ö
æ 10. A line in the 3-dimensional space makes an angle q
(a) cos ç ÷ (b) cos ç ÷
è 31 ø è 35 ø
æ pö
-1 æ 9 ö -1 æ 7 ö çè 0 < q £ ÷ø with both the x and y axes. Then the set of all
(c) cos ç ÷ (d) cos ç ÷ 2
è 35 ø è 31 ø
5. The length of the proection of the line segment oining values of q is the interval: [Online April 9, 2014]

the points (5, –1, 4) and (4, –1, 3) on the plane, x + y + = 7 æ pù é p pù


(a) ç 0, ú (b) ê , ú
is: [2018] è 4û ë6 3û

Downloaded from @Freebooksforjeeneet


Three Dimensional Geometry M-489

é p pù æ p pù
(c) ê , ú (d) ç , ú Equation of a Straight Line in Cartesian
ë 4 2û è 3 2û
and Vector Form, Angle Between two
11. Let ABC be a triangle with vertices at points A (2, 3, 5), Lines, Condition for Coplanarity of two
B (–1, 3, 2) and C (l, 5, m) in three dimensional space. If the TOPIC n Lines Perpendicular Distance of a Point
median through A is equally inclined with the axes, then from a Line, Shortest Distance between
(l, m) is equal to : [Online April 25, 2013] two Skew Lines, Distance Between two
Parallel Lines.
(a) (10, 7) (b) (7, 5)
(c) (7, 10) (d) (5, 7)
12. If the proections of a line segment on the x, y and z-axes 18. A plane P meets the coordinate axes at A, B and C respec-
in 3-dimensional space are 2, 3 and 6 respectively, then the tively. The centroid of DABC is given to be (1,1,2). Then
length of the line segment is : [Online April 23, 2013] the equation of the line through this centroid and perpen-
(a) 12 (b) 7 dicular to the plane P is: [Sep. 06, 2020 (II)]
(c) 9 (d) 6
x - 1 y -1 z - 2 x - 1 y -1 z - 2
13. The acute angle between two lines such that the direction (a) = = (b) = =
cosines l, m, n, of each of them satisfy the equations 2 1 1 1 1 2
l + m + n = 0 and l2 + m2 – n2 = 0 is :
x - 1 y -1 z - 2 x - 1 y -1 z - 2
[Online April 22, 2013] (c) = = (d) = =
2 2 1 1 2 2
(a) 15 (b) 30
(c) 60 (d) 45 19. If (a, b, c) is the image of the point (1, 2, –3) in the line,
x +1 y - 3 z
14. A line AB in three-dimensional space makes angles 45 = = , then a + b + c is equals to:
and 120 with the positive x-axis and the positive y-axis 2 -2 -1
[Sep. 05, 2020 (I)]
respectively. If AB makes an acute angle q with the positive
-axis, then q equals [2010] (a) 2 (b) – 1
(a) 45 (b) 60 (c) 3 (d) 1
(c) 75 (d) 30 r
20. The lines r = (i - j ) + l (2i + k) and
15. The proections of a vector on the three coordinate axis are
6, –3, 2 respectively. The direction cosines of the vector are r
r = (2i - j ) + m(i + j - k) [Sep. 03, 2020 (I)]
: [2009]
(a) do not intersect for any values of l and m
6 -3 2 6 -3 2 (b) intersect for all values of l and m
(a) , , (b) , ,
5 5 5 7 7 7
1
-6 -3 2 (c) intersect when l = 2 and m =
, , 2
(c) (d) 6, – 3, 2
7 7 7 (d) intersect when l = 1 and m = 2
16. If a line makes an angle of p / 4 with the positive directions 21. The shortest distance between the lines
of each of x- axis and y- axis, then the angle that the line
x-3 y -8 z -3 x+3 y+7 z-6
makes with the positive direction of the z-axis is [2007] = = and = = is:
3 -1 1 -3 2 4
p p [Jan. 08, 2020 (I)]
(a) (b)
4 2 7
(a) 2 30 (b) 30
p p 2
(c) (d) (c) 3 30
6 3 (d) 3
17. A line makes the same angle q, with each of the x and z 22. If the foot of the perpendicular drawn from the point
axis. If the angle b, which it makes with y-axis, is such that
(1, 0, 3) on a line passing through (a, 7, 1) is , then a is
sin 2 b = 3 sin 2 q, then cos2q equals [2004] equal to _________. [NA Jan. 07, 2020 (II)]
2 1 23. A perpendicular is drawn from a point on the line
(a) (b)
5 5 x -1 y +1 z
= = to the plane x + y + z = 3 such that the
2 -1 1
3 2
(c) (d) foot of the perpendicular Q also lies on the plane x – y + z = 3.
5 3
Then the co-ordinates of Q are : [April 10, 2019 (II)]

Downloaded from @Freebooksforjeeneet


EBD_8344
M-490 Mathematics

(a) (1, 0, 2) (b) (2, 0, 1) 30. Equation of the line of the shortest distance between the
(c) (–1, 0, 4) (d) (4, 0, –1) x y x -1 y +1
lines = = and = = is:
24. The length of the perpendicular from the point (2, –1, 4) 1 -1 1 0 -2 1
x+3 y-2 z [Online April 19, 2014]
on the straight line, = = is :
10 -7 1 x y x -1 y +1
[April 08, 2019 (I)] (a) = = (b) = =
1 -1 -2 1 -1 -2
(a) greater than 3 but less than 4 x -1 y +1 x y
(b) less than 2 (c) = = (d) = =
1 -1 1 -2 1 2
(c) greater than 2 but less than 3
x - 2 y -3 z - 4
(d) greater than 4 31. If the lines = = and
1 1 -k
x - 3 y +1 z - 6 x+5 y -2 z -3
25. Two lines = = and = = x -1 y -4 z -5
1 3 -1 7 -6 4 = = are coplanar, then k can have
k 2 1
intersect at the point R. The reflection of R in the xy- plane
[2013]
has coordinates : [Jan. 11, 2019 (II)]
(a) any value (b) exactly one value
(a) (2, –4, –7) (b) (2, 4, 7)
(c) exactly two values (d) exactly three values
(c) (2, –4, 7) (d) (–2, 4, 7)
32. If two lines L1 and L2 in space, are defined by
26. If the lines x = ay + b, z = cy + d and x = a' z + b',
y = c' z + d' are perpendicular, then: [Jan. 09, 2019 (II)] {
L1 = x = l y + ( )
l -1 , z = ( ) }
l - 1 y + l and
(a) ab' + bc' + 1 = 0
(b) cc' + a + a' = 0
(c) bb' + cc' + 1 = 0
{ ( )
L2 = x = m y + 1 - m , z = 1 - m y + m ( ) }
(d) aa' + c + c' = 0 then L1 is perpendicular to L2, for all non-negative reals l
27. If the angle between the lines, and m, such that : [Online April 23, 2013]

x y z 5 – x 7 y – 14 z – 3 (a) l + m =1 (b) l ¹ m
= = and = = is
2 2 1 –2 P 4 (c) l + m = 0 (d) l = m
æ 2ö x +1 y -1 z +1 x+2 y-k z
cos–1 ç ÷ , then P is equal to [Online April 16, 2018] 33. If the lines = = and = = are
è 3ø 2 1 3 2 3 4
7 2 coplanar, then the value of k is : [Online April 9, 2013]
(a) – (b)
4 7 11 11
(a) (b) -
4 7 2 2
(c) – (d)
7 2 9
9
(c) (d) -
28. The number of distinct real values of l for which the lines 2 2
x -1 y - 2 +3 x -3 y - 2 -1 x -1 y +1 z-1
= = 2 = = 34. If the line = = and
and are 2 3 4
1 2 l 1 l2 2
coplanar is : [Online April 10, 2016] x-3 y-k z
= = intersect, then k is equal to: [2012]
(a) 2 (b) 4 1 2 1
(c) 3 (d) 1 2
(a) –1 (b)
9
x y
29. The shortest distance between the lines = = and 9
2 2 1 (c) (d) 0
2
x+2 y-4 -5
= = lies in the interval : 35. The distance of the point -i + 2 j + 6k from the straight
-1 8 4
[Online April 9, 2016] line that passes through the point 2i + 3 j - 4k and is
(a) (3, 4] (b) (2, 3] parallel to the vector 6i + 3 j - 4k is
(c) [1, 2) (d) [0, 1) [Online May 26, 2012]

Downloaded from @Freebooksforjeeneet


Three Dimensional Geometry M-491

(a) 9 (b) 8
23 23
(c) 7 (d) 10 (c) (d)
17 15
36. Statement 1: The shortest distance between the lines
x y z x -1 y - 1 z - 1 x –1 y – 2 z – 3
= = and = = is 2 . 41. If the straight lines = = and
2 -1 2 4 -2 4 k 2 3
Statement 2: The shortest distance between two parallel
x – 2 y – 3 z –1
lines is the perpendicular distance from any point on one = = intersect at a point, then the integer
of the lines to the other line. 3 k 2
[Online May 19, 2012] k is equal to [2008]
(a) –5 (b) 5
(a) Statement 1 is true, Statement 2 is false.
(c) 2 (d) –2
(b) Statement 1 is true, Statement 2 is true, Statement 2 is
42. If non ero numbers a, b, c are in H.P., then the straight line
a correct explanation for Statement 1.
(c) Statement 1 is false, Statement 2 is true. x y 1
+ + = 0 always passes through a fixed point. That
(d) Statement 1 is true, Statement 2 is true, , Statement 2 is a b c
not a correct explanation for Statement 1 point is [2005]
37. The coordinates of the foot of perpendicular from the point (a) (– 1, 2) (b) (– 1, – 2)
(1, 0, 0) to the line æ 1ö
(c) (1, – 2) (d) ç1,- ÷
x - 1 y + 1 z + 10 è 2ø
= = are [Online May 12, 2012]
2 -3 8 43. The angle between the lines 2x = 3y = – and
(a) (2, – 3, 8) (b) (1, – 1, – 10) 6x = – y = – 4 is [2005]
(c) (5, – 8, – 4) (d) (3, – 4, – 2) (a) 0 (b) 90
(c) 45 (d) 30
44. If the straight lines [2004]
38. The length of the perpendicular drawn from the point
x = 1 + s , y = -3 - l s , z = 1 + l s
x y - 2 z -3
(3, -1, 11) to the line = = is : [2011RS] t
2 3 4 and x = , y = 1 + t , z = 2 - t , with parameters s and t
2
(a) 29 (b) 33 respectively, are co-planar, then l equals.
(a) 0 (b) –1
(c) 53 (d) 66
1
39. Statement-1: The point A(1, 0, 7)) is the mirror image of the (c) - (d) –2
2
x y -1 z - 2 45. A line with direction cosines proportional to 2, 1, 2 meets
point B(1, 6, 3) in the line : = =
1 2 3 each of the lines x = y + a = z and x + a = 2 y = 2 z . The
x y -1 z - 2 co-ordinates of each of the points of intersection are given
Statement-2: The line = = bisects the line by [2004]
1 2 3
(a) (2a,3a, 3a), (2a, a, a ) (b) (3a, 2a,3a), (a, a, a)
segment oining A(1, 0, 7) and B(1, 6, 3) . [2011]
(a) Statement-1 is true, Statement-2 is true; Statement-2 is (c) (3a, 2a, 3a), (a, a, 2a ) (d) (3a, 3a,3a), (a, a, a )
not a correct explanation for Statement-1.
(b) Statement-1 is true, Statement-2 is false. 46. The lines x - 2 = y - 3 = z - 4 [2003]
1 1 -k
(c) Statement-1 is false, Statement-2 is true.
(d) Statement-1 is true, Statement-2 is true; Statement-2 is x -1 y - 4 -5
and = = are coplanar if
a correct explanation for Statement-1. k 2 1
40. The line L given by x y = 1 passes through the point (a) k = 3 or –2 (b) k = 0 or –1
+ (c) k = 1 or –1 (d) k = 0 or –3.
5 b
(13, 32). The line K is parallel to L and has the equation 47. The two lines x = ay + b , z = cy + d and x = a¢y + b¢, z = c¢y + d¢
will be perpendicular, if and only if [2003]
x y
+ = 1 . Then the distance between L and K is (a) aa¢ + cc¢ + 1 = 0
c 3 (b) aa¢ + bb¢ + cc¢ + 1 = 0
[2010] (c) aa¢ + bb¢ +cc¢ = 0
17 (d) (a + a¢) (b + b¢) +(c + c¢) = 0.
(a) 17 (b)
15

Downloaded from @Freebooksforjeeneet


EBD_8344
M-492 Mathematics

53. The plane which bisects the line oining the points (4, – 2, 3)
and (2, 4, – 1) at right angles also passes through the
Equation of a Plane in Different
point: [Sep. 03, 2020 (II)]
Forms, Equation of a Plane Passing
Through the Intersection of two (a) (4, 0, 1) (b) (0, –1, 1)
Given Planes, Plane Containing two (c) (4, 0, –1) (d) (0, 1, –1)
Lines, Angle Between two Planes, 54. Let a plane P contain two lines
TOPIC Đ Angle Between a Plane and a Line, r r
Distance Between two Parallel r = i + l(i + j ), l Î R and r = - j + m( j - k), m Î R.
Planes, Position of Point and Line
wrt a Plane, Projection of a Line on a If Q(a, b, g) is the foot of the perpendicular drawn from
Plane the point M(1, 0, 1) to P, then 3(a + b + g) equals
____________. [NA Sep. 03, 2020 (II)]
x -1 y +1 z 55. The plane passing through the points (1, 2, 1), (2, 1, 2) and
48. The shortest distance between the lines = = parallel to the line, 2x = 3y, z = 1 also through the point :
0 -1 1
and x + y + z + 1 = 0, 2x – y + z + 3 = 0 is : [Sep. 02, 2020 (I)]
[Sep. 06, 2020 (I)] (a) (0, 6, –2) (b) (–2, 0, 1)
(c) (0, –6, 2) (d) (2, 0, –1)
1 56. A plane passing through the point (3, 1, 1) contains two
(a) 1 (b)
3 lines whose direction ratios are 1, –2, 2 and 2, 3, –1
respectively. If this plane also passes through the point
1 1
(c) (d) (a, - 3, 5), then a is equal to : [Sep. 02, 2020 (II)]
2 2
(a) 5 (b) –10
x + 1 y - 2 z -1
49. If for some a ÎR , the lines L1 : = = and (c) 10 (d) –5
2 -1 1
57. If for some a and b in R, the intersection of the following
x + 2 y +1 z + 1
L2 : = = are coplanar, then the line L2 three planes
a 5-a 1
passes through the point : [Sep. 05, 2020 (II)] x + 4y – 2z = 1
(a) (10, 2, 2) (b) (2, – 10, – 2) x + 7y – 5z = b
(c) (10, – 2, – 2) (d) (– 2, 10, 2)
50. If the equation of a plane P, passing through the x + 5y + az = 5
intersection of the planes, x + 4y – z + 7 = 0 and 3x + y + 5z is a line in R3, then a + b is equal to: [Jan. 9, 2020 (I)]
= 8 is ax + by + 6z = 15 for some a, b Î R, then the
(a) 0 (b) 10
distance of the point (3, 2, –1) from the plane P is
___________. (c) 2 (d) –10
[Sep. 04, 2020 (I)] 58. If the distance between the plane, 23x – 10y – 2z + 48 = 0
51. The distance of the point (1, –2, 3) from the plane
and the plane containing the lines
x y z
x - y + z = 5 measured parallel to the line = = x +1 y - 3 z +1
2 3 -6 = =
is : [NA Sep. 04, 2020 (II)] 2 4 3

7 x + 3 y + 2 z -1
(a) (b) 1 and = = (l Î R)
5 2 6 l

1 k
(c) (d) 7 is equal to , then k is equal to ______.
7 633
52. The foot of the perpendicular drawn from the point (4, 2, 3) [NA Jan. 9, 2020 (II)]
to the line oining the points (1, –2, 3) and (1,1, 0) lies on
59. The mirror image of the point (1, 2, 3) in a plane is
the plane : [Sep. 03, 2020 (I)]
(a) 2x + y – z = 1 (b) x – y – 2z = 1 æ 7 4 1ö
çè - , - , - ÷ø . Which of the following points lies on
(c) x – 2y + z = 1 (d) x + 2y – z = 1 3 3 3
this plane? [Jan. 8, 2020 (II)]

Downloaded from @Freebooksforjeeneet


Three Dimensional Geometry M-493

(a) (1, 1, 1) (b) (1, –1, 1) (a) 5/2 (b)


2 5
(c) (–1, –1, 1) (d) (–1, –1, –1) (c) 9/2 (d) 7/2
67. A plane passing through the points (0, –1, 0) and (0, 0, 1)
60. Let P be a plane passing through the points (2, 1, 0),
p
(4, 1, 1) and (5, 0, 1) and R be any point (2, 1, 6). Then the and making an angle with the plane y – + 5 = 0, also
4
image of R in the plane P is: [Jan. 7, 2020 (I)] passes through the point: [April 09 2019I]
(a) (– 2 , 1, –4) (b) ( 2 , –1, 4)
(a) (6, 5, 2) (b) (6, 5, –2)
(c) (– 2 , –1, –4) (d) ( 2 , 1, 4)
(c) (4, 3, 2) (d) (3, 4, –2)
68. Let P be the plane, which contains the line of intersection
x - 2 y + 1 z -1
61. If the line = = intersects the plane of the planes, x + y + – 6 = 0 and 2x + 3y + + 5 = 0 and it
3 2 -1 is perpendicular to the xy-plane. Then the distance of the
2x + 3y – z + 13 = 0 at a point P and the plane 3x + y + 4z = point (0, 0, 256) from P is equal to: [April 09, 2019 (II)]
16 at a point Q, then PQ is equal to: [April 12, 2019 (I)]
(a) 17/ 5 (b) 63 5
(a) 14 (b) 14
(c) 205 5 (d) 11/ 5
(c) 2 7 (d) 2 14
69. The equation of a plane containing the line of intersection
62. A plane which bisects the angle between the two given
of the planes 2x – y – 4 = 0 and y + 2z – 4 = 0 and passing
planes 2x – y + 2z – 4 = 0 and x + 2y + 2z – 2 = 0, passes
through the point (1, 1, 0) is : [April 08 2019 I]
through the point : [April 12, 2019 (II)]
(a) x – 3y – 2z = –2 (b) 2x – z = 2
(a) (1, –4, 1) (b) (1, 4, –1)
(c) x – y – z = 0 (d) x + 3y + z = 4
(c) (2, 4, 1) (d) (2, –4, 1)
70. The vector equation of the plane through the line of
63. The length of the perpendicular drawn from the point (2,
1, 4) to the plane containing the lines intersection of the planes x + y + z = 1 and 2x + 3y + 4z = 5
r r
r = ($i + $j ) + l(i$ + 2 $j - k$ ) and r = ($i + $j ) + m(-$i + $j - 2k$ ) which is perpendicular to the plane x – y + z = 0 is :
is : [April 12, 2019 (II)] [April 08, 2019 (II)]
1 r r
(a) 3 (b) (a) r ´ ( i – k ) + 2 = 0 (b) r . ( i – k ) – 2 = 0
3
1 (c) rr ´ ( i + k ) + 2 = 0 (d) rr. ( i – k ) + 2 = 0
(c) 3 (d)
3 71. The perpendicular distance from the origin to the plane
64. If Q (0, –1, –3) is the image of the point P in the plane 3x –
x+ 2 y -2 z +5
y + 4 = 2 and R is the point (3, –1, –2), then the area (in sq. containing the two lines, = = and
units) of DPQR is : [April 10, 2019 (I)] 3 5 7
91 x -1 y - 4 z + 4
(a) 2 13 (b) = = , is : [Jan. 12, 2019 (I)]
4 1 4 7
91 65
(c) (d) 11
2 2 (a) 11 6 (b) 6
1 2
65. If the plane 2x – y + 2z + 3 =0 has the distances and
2 3 (c) 11 (d) 6 11
units from the planes 4x – 2y + 4z + l = 0 and 2x – y + 2z +
m = 0, respectively, then the maximum value of l + m is x +1 y - 2 z - 3
72. If an angle between the line, = = and the
equal to : [April 10, 2019 (II)] 2 1 -2
(a) 9 (b) 15 æ2 2ö
(c) 5 (d) 13 plane, x – 2y – kx = 3 is cos–1 çç ÷÷ then a value of k is
,
è 3 ø
x -1 y +1 -2
66. If the line, = = meets the plane, [Jan. 12, 2019 (II)]
2 3 4
x + 2y + 3 = 15 at a point P, then the distance of P from the 5 3
origin is: [April 09 2019I] (a) (b)
3 5

Downloaded from @Freebooksforjeeneet


EBD_8344
M-494 Mathematics

5 79. The system of linear equations


3
(c) - (d) - x+y+z=2
5 3
2x + 3y + 2z = 5
73. Let S be the set of all real values of l such that a plane
passing through the points (–l2, 1, 1), (1, –l2, 1) and 2x + 3y + (a2 – 1)z = a + 1 [Jan 09 2019I]
(1, 1, –l2) also passes through the point- (–1, –1, 1). Then (a) is inconsistent when a = 4
S is equal to : [Jan. 12, 2019 (II)] (b) has a unique solution for |a| = 3
(a) { 3} (b) { 3, - 3 } (c) has infinitely many solutions for a = 4

(c) {1, -1} (d) {3, -3} (d) is inconsistent when |a| = 3
80. The equation of the line passing through (–4, 3, 1), parallel
x - 3 y + 2 z -1
74. The plane containing the line = = and to the plane x + 2y – – 5 = 0 and intersecting the line
2 -1 3
x +1 y -3 z - 2
also containing its proection on the plane 2x + 3y – = 5, = = is: [Jan 09 2019I]
contains which one of the following points? -3 2 -1
[Jan. 11, 2019 (I)] x - 4 y + 3 z +1
(a) (2, 2, 0) (b) (–2, 2, 2) (a) = =
2 1 4
(c) (0, – 2, 2) (d) (2, 0, –2)
75. The direction ratios of normal to the plane through the x + 4 y - 3 z -1
(b) = =
p 1 1 3
points (0, –1, 0) and (0, 0, 1) and making an angle
4 x + 4 y - 3 z -1
(c) = =
with the plane y – z + 5 = 0 are : 3 -1 1
[Jan. 11, 2019 (I)]
x + 4 y - 3 z -1
(a) 2, –1, 1 (b) 2, 2, - 2 (d) = =
-1 1 1
(c) 2,1, -1 (d) 2 3,1, -1 81. The plane through the intersection of the planes
76. If the point (2, a, b) lies on the plane which passes through x + y + z = 1 and 2x + 3y – z + 4 = 0 and parallel to
the points (3, 4, 2) and (7, 0, 6) and is perpendicular to the y-axis also passes through the point: [Jan 09 2019I]
plane 2x – 5y = 15, then 2a – 3b is equal to : (a) (– 3, 0, – 1) (b) (– 3, 1, 1)
[Jan. 11, 2019 (II)] (c) (3, 3, – 1) (d) (3, 2, 1)
(a) 12 (b) 7 82. The equation of the plane containing the straight line
(c) 5 (d) 17
77. The plane which bisects the line segment oining the points x y z
= = and perpendicular to the plane containing the
(– 3, – 3, 4) and (3, 7, 6) at right angles, passes through 2 3 4
which one of the following points? [Jan. 10, 2019 (II)]
(a) (–2, 3, 5) (b) (4, – 1, 7) x y z x y z
(c) (2, 1, 3) (d) (4, 1, – 2) straight lines = = and = = is:
3 4 2 4 2 3
78. On which of the following lines lies the point of in-
[Jan. 09, 2019 (II)]
x-4 y -5 z -3
ter-section of the line, = = and the plane, (a) x – 2y + z = 0
2 2 1
(b) 3x + 2y – 3z = 0
x + y + z = 2? [Jan. 10, 2019 (II)]
(c) x + 2y – 2z = 0
x+3 4- y +1
(a) = = (d) 5x + 2y – 4z = 0
3 3 -2 83. If L 1 is the line of intersection of the planes
2x - 2y + 3 - 2 = 0, x - y + + 1 = 0 and L2 is the line of
x- 4 y -5 z - 5
(b) = = intersection of the planes x + 2y - - 3 = 0,
1 1 -1
3x - y + 2 - 1 = 0 , then the distance of the origin from
x -1 y - 3 z + 4 the plane, containing the lines L1 and L2, is : [2018]
(c) = = 1 1
1 2 -5
(a) (b)
3 2 2 2
x- 2 y -3 z + 3
(d) = = 1 1
2 2 3 (c) (d)
2 4 2

Downloaded from @Freebooksforjeeneet


Three Dimensional Geometry M-495

84. The sum of the intercepts on the coordinate axes of the 90. If x = a, y = b, z = c is a solution of the system of linear
plane passing through the point (– 2, – 2, 2) and containing equations [Online April 9, 2017]
the line oining the points (1, – 1, 2) and (1, 1, 1) is x + 8y + 7z = 0
[Online April 16, 2018] 9x + 2y + 3z = 0
(a) 12 (b) – 8 x+y+z=0
(c) – 4 (d) 4 such that the point (a, b, c) lies on the plane
85. A variable plane passes through a fixed point (3, 2, 1) and x + 2y + z = 6, then 2a + b + c equals :
meets x, y and z axes at A, B and C respectively. A plane is (a) –1 (b) 0
drawn parallel to yz – plane through A, a second plane is (c) 1 (d) 2
drawn parallel zx – plane through B and a third plane is 91. If a variable plane, at a distance of 3 units from the origin,
intersects the coordinate axes at A, B and C, then the lo-
drawn parallel to xy – plane through C. Then the locus of cus of the centroid of DABC is : [Online April 9, 2017]
the point of intersection of these three planes, is 1 1 1
[Online April 15, 2018] (a) 2 + 2 + 2 = 1
x y z
x y z
(a) x + y + z = 6 (b) + + =1 1 1
(b) 2 + 2 + 2 = 3
1
3 2 1
x y z
3 2 1 1 1 1 11 1
(c) + + =1 (d) + + = (c)
1
+
1
+
1
=
x y z x y z 6
x2 y2 z2 9
86. An angle between the plane, x + y + z = 5 and the line of 1 1 1
intersection of the planes, 3x + 4y + z – 1 = 0 and (d) + =9 +
2 2
x yz2
5x + 8y + 2z + 14 = 0, is [Online April 15, 2018]
x-3 y+2 z+l
æ 3 ö æ 3 ö 92. If the line,
1
=
-1
=
-2
lies in the plane, 2x –
(a) cos –1 ç ÷ (b) cos –1 çç ÷÷
è 17 ø 4y + 3 = 2, then the shortest distance between this line
è 17 ø
x -1 y z
æ 3 ö and the line, = = is :
æ 3 ö 12 9 4
(c) sin –1 ç ÷ (d) sin –1 çç ÷÷
è 17 ø è 17 ø
[Online April 9, 2017]
(a) 2 (b) 1
87. A plane bisects the line segment oining the points (1, 2, 3)
(c) 0 (d) 3
and (– 3, 4, 5) at right angles. Then this plane also passes
93. The coordinates of the foot of the perpendicular from the
through the point. [Online April 15, 2018]
point (1, –2, 1) on the plane containing the lines,
(a) (– 3, 2, 1) (b) (3, 2, 1)
x +1 y -1 -3
(c) (1, 2, – 3) (d) (– 1, 2, 3) = = and
88. If the image of the point P(1, –2, 3) in the plane, 6 7 8
x -1 y - 2 -3
x y = = , is : [Online April 8, 2017]
2x + 3y – 4 + 22 = 0 measured parallel to line, = = 3 5 7
1 4 5 (a) (2, –4, 2) (b) (–1, 2, –1)
(c) (0, 0, 0) (d) (1, 1, 1)
is Q, then PQ is equal to : [2017] 94. The line of intersection of the planes
r   
(a) 6 5 (b) 3 5 r.(3i -  + k) = 1 and
r   
(c) 2 42 (d) r.(i + 4 - 2k) = 2, is : [Online April 8, 2017]
42
4 5
89. The distance of the point (1, 3, –7) from the plane passing x- -
through the point (1, –1, –1), having normal (a) 7 =y = 7
perpendicular to both the lines [2017] -2 7 13
4 5
x- +
x -1 y + 2 -4 x - 2 y +1 +7 7 = y = 7
= = and = = , is : (b)
1 -2 3 2 -1 -1 2 -7 13
6 5
10 20 x- y-
(a) (b) (c) 13 = 13 =
74 74 2 -7 -13
6 5
10 5 x- y-
(c) (d) (d) 13 = 13 =
83 83 2 7 -13

Downloaded from @Freebooksforjeeneet


EBD_8344
M-496 Mathematics

103. If the shortest distance between the lines


x -3 y+2 +4
95. If the line, = = lies in the plane, lx + my – =
-1 x -1 y + 1
2 3 = = , ( a ¹ -1) and x + y + + 1 = 0
9, then l2 + m2 is equal to : [2016] a -1 1
(a) 5 (b) 2 1
(c) 26 (d) 18 = 2x – y + + 3 is , then a value a is :
3
96. The distance of the point (1, –5, 9) from the plane x – y + =
[Online April 10, 2015]
5 measured along the line x = y = is : [2016]
16 19
10 20 (a) - (b) -
(a) (b) 19 16
3 3
32 19
(c) 3 10 (d) 10 3 (c) (d)
19 32
97. The distance of the point (1, –2, 4) from the plane passing
through the point (1, 2, 2) and perpendicular to the planes x -1 y - 3 z - 4
104. The image of the line = = in the plane
x – y + 2 = 3 and 2x – 2y + + 12 = 0, is : 3 1 -5
[Online April 9, 2016] 2 x - y + z + 3 = 0 is the line: [2014]
(a) 2 (b) 2
x-3 y +5 z - 2
(a) = =
1 3 1 -5
(c) 2 2 (d)
2
x-3 y +5 z - 2
98. The equation of the plane containing the line 2x – 5y + = (b) = =
-3 -1 5
3; x + y + 4 = 5, and parallel to the plane, x + 3y + 6 = 1, is:
[2015] x+3 y -5 z - 2
(c) = =
(a) x + 3y + 6 = 7 (b) 2x + 6y + 12 = – 13 3 1 -5
(c) 2x + 6y + 12 = 13 (d) x + 3y + 6 = –7
x+3 y -5 z + 2
99. The distance of the point (1, 0, 2) from the point of (d) = =
-3 -1 5
x - 2 y +1 -2
intersection of the line = = and the plane 105. If the angle between the line 2(x + 1) = y = + 4 and the
3 4 12
p
x – y + = 16, is [2015] plane 2x – y + l + 4 = 0 is 6 , then the value of l is:
(a) 3 21 (b) 13 [Online April 19, 2014]
135 45
(c) 2 14 (d) 8 (a) (b)
7 11
100. The shortest distance between the –axis and the line x 45 135
+ y + 2z – 3 = 0 = 2x + 3y + 4z – 4, is (c) (d)
7 11
[Online April 11, 2015] 106. If the distance between planes, 4x – 2y – 4 + 1 = 0 and
(a) 1 (b) 2 4x – 2y – 4 + d = 0 is 7, then d is:
(c) 4 (d) 3 [Online April 12, 2014]
101. A plane containing the point (3, 2, 0) and the line (a) 41 or – 42 (b) 42 or – 43
(c) – 41 or 43 (d) – 42 or 44
x –1 y - 2 z - 3 107. A symmetrical form of the line of intersection of the planes
= = also contains the point :
1 5 4 x = ay + b and = cy + d is [Online April 12, 2014]
[Online April 11, 2015] x - b y -1 -d
(a) = =
(a) (0, 3, 1) (b) (0, 7, –10) a 1 c
(c) (0, –3, 1) (d) 0, 7, 10 x - b - a y -1 -d-c
102. If the points (1, 1, l) and (–3, 0, 1) are equidistant from (b) = =
a 1 c
the plane, 3x + 4y – 12 + 13 = 0, then l satisfies the
equation : [Online April 10, 2015] (c) x - a = y - 0 = - c
(a) 3x2 + 10x – 13 = 0 (b) 3x2 – 10x + 21 = 0 b 1 d
(c) 3x2 – 10x + 7 = 0 (d) 3x2 + 10x – 7 = 0
(d) x - b - a = y - 1 = - d - c
b 0 d

Downloaded from @Freebooksforjeeneet


Three Dimensional Geometry M-497

x -1 y - 2 -3 115. The equation of a plane containing the line


108. The plane containing the line = = and
1 2 3 x +1 y - 3 z + 2
= = and the point (0, 7, – 7) is
x y -3 2 1
parallel to the line = = passes through the point:
1 1 4 (a) x + y + z = 0 [Online May 26, 2012]
[Online April 11, 2014] (b) x + 2y + z = 21
(a) (1, – 2, 5) (b) (1, 0, 5) (c) 3x – 2y + 5z + 35 = 0
(c) (0, 3, –5) (d) (– 1, – 3, 0) (d) 3x + 2y + 5z + 21 = 0
109. Equation of the plane which passes through the point of
116. Consider the following planes
intersection of lines
P : x + y – 2z + 7 = 0
x -1 y - 2 -3 Q : x + y + 2z + 2 = 0
= = and
3 1 2 R : 3x + 3y – 6z – 11 = 0 [Online May 26, 2012]
x - 3 y -1 -2 (a) P and R are perpendicular
= = (b) Q and R are perpendicular
1 2 3
(c) P and Q are parallel
and has the largest distance from the origin is:
(d) P and R are parallel
[Online April 9, 2014]
117. If the three planes x = 5, 2x – 5ay + 3z – 2 = 0 and
(a) 7x + 2y + 4 = 54 (b) 3x + 4y + 5 = 49
3bx + y – 3z = 0 contain a common line, then(a, b) is equal
(c) 4x + 3y + 5 = 50 (d) 5x + 4y + 3 = 57
to [Online May 19, 2012]
110. Distance between two parallel planes 2x + y + 2z = 8 and
4x + 2y + 4z + 5 = 0 is [2013] æ 8 1ö æ1 8 ö
(a) ç , - ÷ (b) ç , - ÷
3 5 è 15 5 ø è 5 15 ø
(a) (b)
2 2
æ 8 1ö æ 1 8ö
7 9 (c) ç - , ÷ (d) ç - , ÷
(c) (d) è 15 5 ø è 5 15 ø
2 2
111. The equation of a plane through the line of intersection of 118. A line with positive direction cosines passes through the
the planes x + 2y = 3, y –2 + 1= 0, and perpendicular to the point P (2, – 1, 2) and makes equal angles with the
first plane is : [Online April 25, 2013] coordinate axes. If the line meets the plane 2x + y + z = 9 at
(a) 2x – y – 10z = 9 (b) 2x – y + 7z = 11 point Q, then the length PQ equals [Online May 7, 2012]
(c) 2x – y + 10z = 11 (d) 2x – y – 9z = 10
112. Let Q be the foot of perpendicular from the origin to the (a) 2 (b) 2
plane 4x – 3y + z + 13 = 0 and R be a point (– 1, – 6) on the (c) 3 (d) 1
plane. Then length QR is : [Online April 22, 2013]
119. The values of a for which the two points (1, a, 1) and
19 (– 3, 0, a) lie on the opposite sides of the plane 3x + 4y –
(a) 14 (b)
2 12z + 13 = 0, satisfy [Online May 7, 2012]

7 3 1
(c) 3 (d) (a) 0 < a < (b) – 1 < a < 0
2 2 3
r
113. A vector n is inclined to x-axis at 45 , to y-axis at 60 and 1
(c) a < – 1 or a < (d) a = 0
r 3
at an acute angle to z-axis. If n is a normal to a plane
120. The distance of the point (1, – 5, 9) from the plane x – y +
passing through the point ( )
2, -1,1 then the equation z = 5 measured along a straight x = y = z is [2011RS]
of the plane is : [Online April 9, 2013] (a) 10 3 (b) 5 3

(a) 4 2 x + 7 y + z - 2 (b) 2 x + y + 2 z = 2 2 + 1 (c) 3 10 (d) 3 5


y -1 z - 3
(c) 3 2 x - 4 y - 3z = 7 (d) 2 x - y - z = 2 121. If the angle between the line x = = and the
2 l
114. A equation of a plane parallel to the plane
x – 2y + 2z –5 = 0 and at a unit distance from the origin is : æ 5 ö
[2012] plane x + 2y + 3z = 4 is cos–1 ç ÷ , then l equals
è 14 ø
(a) x – 2y + 2z – 3 = 0 (b) x – 2y + 2z + 1 = 0
(c) x – 2y + 2z – 1 = 0 (d) x – 2y + 2z + 5 = 0 [2011]

Downloaded from @Freebooksforjeeneet


EBD_8344
M-498 Mathematics

3 2 x +1 y -1
(a) (b) 128. If the angle q between the line =
2 5 1 2
5 2
(c) (d) z -2
3 3 = and the plane 2x – y + l + 4 = 0 is such that
122. Statement -1 : The point A(3, 1, 6) is the mirror image of 2
the point B(1, 3, 4) in the plane x – y + z = 5.
1
Statement -2: The plane x – y + z = 5 bisects the line sin q = then the value of l is [2005]
segment oining A(3, 1, 6) and B(1, 3, 4). [2010] 3
(a) Statement -1 is true, Statement -2 is true ; Statement -
5 -3
2 is not a correct explanation for Statement -1. (a) (b)
3 5
(b) Statement -1 is true, Statement -2 is false.
(c) Statement -1 is false, Statement -2 is true . 3 -4
(c) (d)
(d) Statement - 1 is true, Statement 2 is true ; Statement -2 4 3
is a correct explanation for Statement -1. 129. Distance between two parallel planes
x – 2 y –1 z + 2 2x + y + 2 = 8 and 4x + 2y +4 + 5 = 0 is [2004]
123. Let the line = = lie in the plane
3 –5 2 9 5
(a) (b)
x + 3y – a + b = 0. Then (a, b) equals [2009] 2 2
(a) (–6, 7) (b) (5, –15)
(c) (–5, 5) (d) (6, –17) 7 3
124. The line passing through the points (5, 1, a) and (3, b, 1) (c) (d)
2 2
æ 17 –13ö 130. Two system of rectangular axes have the same origin. If a
crosses the y -plane at the point çè 0, , ÷ . Then
2 2 ø plane cuts them at distances a, b, c and a' , b' , c ' from the
origin then [2003]
[2008]
(a) a = 2, b = 8 (b) a = 4, b = 6 1 1 1 1 1 1
(a) + + - - - =0
(c) a = 6, b = 4 (d) a = 8, b = 2 a2 b2 c2 a'2 b' 2 c' 2
125. Let L be the line of intersection of the planes 2x + 3y + z =
1 and x + 3y + 2z = 2. If L makes an angle a with the 1 1 1 1 1 1
(b) + + + + + =0
positive x-axis, then cos a equals [2007] a 2
b 2
c 2
a' 2
b' 2
c '2
1
(a) 1 (b) 1 1 1 1 1 1
2 (c) + - + + - =0
2 2 2 2 2
a b c a' b' c'2
1 1
(c) (d) . 1 1 1 1 1 1
3 2 (d) - - + - - =0.
2 2 2 2 2
a b c a' b' c'2
126. The image of the point (–1, 3, 4) in the plane x - 2 y = 0
is 131. The d.r. of normal to the plane through (1, 0, 0), (0, 1, 0)
[2006] which makes an angle p /4 with plane
æ 17 19 ö x + y = 3 are [2002]
(a) ç - , - , 4 ÷ (b) (15,11, 4)
è 3 3 ø
(a) 1, 2 ,1 (b) 1, 1, 2
æ 17 19 ö
(c) ç - , - ,1÷ (d) None of these
è 3 3 ø (c) 1, 1, 2 (d) 2 , 1, 1
127. The distance between the line
ur 132. A plane which passes through the point (3, 2, 0) and the
r = 2i - 2 j + 3k + l(i - j + 4k ) and the plane
ur
r .(i + 5 j + k) = 5 is x-4 y -7 z -4
[2005] line = = is [2002]
10 1 5 4
10
(a) (b)
9 3 3 (a) x – y + z =1 (b) x + y + z = 5
3 10 (c) x + 2y – z = 1 (d) 2x – y + z = 5
(c) (d)
10 3

Downloaded from @Freebooksforjeeneet


Three Dimensional Geometry M-499

136. The intersection of the spheres


Sphere and Miscellaneous Problems
TOPIC Ė on Sphere x 2 + y 2 + z 2 + 7 x - 2 y - z = 13 and

133. If (2, 3, 5) is one end of a diameter of the sphere x2 + y2 + z2 x 2 + y 2 + z 2 - 3x + 3 y + 4 z = 8 is the sa me as the


– 6x – 12y – 2z + 20 = 0, then the cooordinates of the other intersection of one of the sphere and the plane [2004]
end of the diameter are [2007]
(a) (4, 3, 5) (b) (4, 3, – 3) (a) 2 x - y - z = 1 (b) x - 2 y - z = 1
(c) (4, 9, – 3) (d) (4, –3, 3). (c) x - y - 2 z = 1 (d) x - y - z = 1
2 2
134. The plane x + 2y – z = 4 cuts the sphere x + y + z – x + 2 137. The radius of the circle in which the sphere
z – 2 = 0 in a circle of radius [2005] x 2 + y 2 + z 2 + 2 x - 2 y - 4 z - 19 = 0 is cut by the plane
(a) 3 (b) 1 x + 2 y + 2 z + 7 = 0 is [2003]
(c) 2 (d) 2 (a) 4 (b) 1
135. If the plane 2ax – 3ay + 4a + 6 = 0 passes through the (c) 2 (d) 3
midpoint of the line oining the centres of the spheres 138. The shortest distance from the plane 12x + 4 y + 3z = 327
x 2 + y 2 + z 2 + 6 x - 8 y - 2 z = 13 and to the sphere x 2 + y 2 + z 2 + 4 x - 2 y - 6 z = 155 is
x 2 + y 2 + z 2 - 10 x + 4 y - 2 z = 8 then a equals [2005] [2003]
(a) 39 (b) 26
(a) – 1 (b) 1
(c) – 2 (d) 2 4
(c) 11 (d) 13.
13

Downloaded from @Freebooksforjeeneet


EBD_8344
- 522

27
M Mathematics

Probability
2 8
Multiplication Theorem on (a) (b)
3 17
Probability,
TOPIC Ć Independent events, Conditional 4 2
Probability, Baye's Theorem (c) (d)
17 5
5. Let EC denote the complement of an event E. Let E1, E2 and
1. In a game two players A and B take turns in throwing a pair E3 be any pairwise independent events with P(E1 ) > 0
of fair dice starting with player A and total of scores on the
and P(E1 Ç E 2 Ç E 3 ) = 0.
two dice, in each throw is noted. A wins the game if he
throws a total of 6 before B throws a total of 7 and B wins Then P(EC2 Ç E3C / E1 ) is equal to : [Sep. 02, 2020 (II)]
the game if he throws a total of 7 before A throws a total of
six. The game stops as soon as either of the players wins. (a) P(EC2 ) + P(E3 ) (b) P(E3C ) - P(E2C )
The probability of A winning the game is :
(c) P(E3 ) - P(EC2 ) (d) P(EC3 ) - P(E 2 )
[Sep. 04, 2020 (II)]
6. In a box, there are 20 cards, out of which 10 are labelled as
5 31 A and the remaining 10 are labelled as B. Cards are drawn
(a) (b) at random, one after the other and with replacement, till a
31 61
second A-card is obtained. The probability that the second
5 30 A-card appears before the third B-card is :
(c) (d) [Jan. 9, 2020 (I)]
6 61
2. A die is thrown two times and the sum of the scores 9 11
(a) (b)
appearing on the die is observed to be a multiple of 4. 16 16
Then the conditional probability that the score 4 has 13 15
appeared atleast once is : [Sep. 03, 2020 (I)] (c) (d)
16 16
1 1 7. Let A and B be two independent events such that
(a) (b)
4 3 1 1
P(A) = and P(B) = . Then, which of the following is
1 1 3 6
(c) (d) TRUE ? [Jan. 8, 2020 (I)]
8 9
3. The probability that a randomly chosen 5-digit number is 2 1
made from exactly two digits is : [Sep. 03, 2020 (II)] (a) P(A/B) = (b) P(A/B’) =
3 3
135 121 1 1
(a)
104
(b)
104
(c) P(A’/B’) = (d) P(A/(A È B)) =
3 4
150 134
8. An unbiased coin is tossed 5 times. Suppose that a
(c) (d) variable X is assigned the value k when k consecutive
104 104 heads are obtained for k = 3, 4, 5, otherwise X takes the
4. Box I contains 30 cards numbered 1 to 30 and Box II contains value –1. Then the expected value of X, is:
20 cards numbered 31 to 50. A box is selected at random [Jan. 7, 2020 (I)]
and a card is drawn from it. The number on the card is
3 1 3 1
found to be a non-prime number. The probability that the (a) (b) (c) - (d) -
card was drawn from Box I is : [Sep. 02, 2020 (I)] 16 8 16 8

Downloaded from @Freebooksforjeeneet


Probability M - 523
9. In a workshop, there are five machines and the probability 14. Let A and B be two non-null events such that A Ì B .
1 Then, which of the following statements is always correct?
of any one of them to be out of service on a day is .
4 [April 08, 2019 (I)]
If the probability that at most two machines will be out of
3 (a) P(A|B) = P(B) – P(A) (b) P(A|B) ³ P(A)
æ 3ö
service on the same day is ç ÷ k, then k is equal to: (c) P(A|B) £ P(A) (d) P(A|B) = 1
è 4ø
15. The minimum number of times one has to toss a fair coin
[Jan. 7, 2020 (II)]
so that the probability of observing at least one head is at
17 17 least 90% is : [April. 08, 2019 (II)]
(a) (b)
8 4 (a) 5 (b) 3
(c) 4 (d) 2
17 16. In a random experiment, a fair die is rolled until two fours
(c) (d) 4
2 are obtained in succession. The probability that the
10. For an initial screening of an admission test, a candidate experiment will end in the fifth throw of the die is equal to :
[Jan. 12, 2019 (I)]
is given fifty problems to solve. If the probability that the
200 150
4 (a) 5 (b)
candidate can solve any problem is , then the probability 6 65
5
that he is unable to solve less than two problems is : 225 175
(c) 5 (d)
[April 12, 2019 (II)] 6 65
17. In a game, a man wins ` 100 if he gets 5 or 6 on a throw of
201 æ 1 ö
49
316 æ 4 ö
48 a fair die and loses ` 50 for getting any other number on
5 çè 5 ÷ø 25 çè 5 ÷ø
(a) (b) the die. If he decides to throw the die either till he gets a
five or a six or to a maximum of three throws, then his
expected gain/loss (in rupees) is : [Jan. 12, 2019 (II)]
49 48
54 æ 4 ö 164 æ 1 ö 400
5 çè 5 ÷ø 25 çè 5 ÷ø
(c) (d) (a) loss (b) 0
9
11. Assume that each born child is equally likely to be a boy 400 400
or a girl. If two families have two children each, then the (c) gain (d) loss
3 3
conditional probability that all children are girls given that 18. Two integers are selected at random from the set
at least two are girls is: [April 10, 2019 (I)] {1, 2, ...., 11}. Given that the sum of selected numbers is
even, the conditional probability that both the numbers
1 1 are even is :
(a) (b)
11 10 [Jan. 11, 2019 (I)]
1 1 7 1 2 3
(c) (d) (a) (b) (c) (d)
12 17 10 2 5 5
12. Minimum number of times a fair coin must be tossed so 19. An unbiased coin is tossed. If the outcome is a head then
that the probability of getting at least one head is more a pair of unbiased dice is rolled and the sum of the numbers
obtained on them is noted. If the toss of the coin results
than 99% is : [April 10, 2019 (II)] in tail then a card from a well-shuffled pack of nine cards
(a) 5 (b) 6 numbered 1, 2, 3, ..., 9 is randomly picked and the number
(c) 8 (d) 7 on the card is noted. The probability that the noted number
is either 7 or 8 is: [Jan 10, 2019 (I)]
13. Four persons can hit a target correctly with probabilities
13 15 19 19
1 1 1 1 (a) (b) (c) (d)
, , and respectively. If all hit at the target 36 72 72 36
2 3 4 8
20. If the probability of hitting a target by a shooter, in any
independently, then the probability that the target would
1
be hit, is: [April 09, 2019 (I)] shot, is , then the minimum number of independent shots
3
25 7 at the target required by him so that the probability of
(a) (b)
192 32 5
hitting the target at least once is greater than , is:
1 25 6
(c) (d) [Jan. 10, 2019 (II)]
192 32 (a) 3 (b) 6
(c) 5 (d) 4

Downloaded from @Freebooksforjeeneet


EBD_8344
M - 524 Mathematics

21. Two cards are drawn successively with replacement from 27. Let E and F be two independent events. The probability
a well-shuffled deck of 52 cards. Let X denote the random 1
that both E and F happen is and the probability that
variable of number of aces obtained in the two drawn 12
cards. Then P(X = 1) + P(X = 2) equals: [Jan 09, 2019 (I)] 1 P(E)
(a) 49/169 (b) 52/169 neither E nor F happens is , then a value of is :
2 P(F)
(c) 24/169 (d) 25/169 [Online April 9, 2017]
22. An urn contains 5 red and 2 green balls. A ball is drawn at 4 3
random from the urn. If the drawn ball is green, then a red (a) (b)
ball is added to the urn and if the drawn ball is red, then a 3 2
green ball is added to the urn; the original ball is not 1 5
returned to the urn. Now, a second ball is drawn at random (c) (d)
3 12
from it. The probability that the second ball is red is:
[Jan. 09, 2019 (II)] 28. Three persons P, Q and R independently try to hit a target.
21 27 3 1
(a) (b) If the probabilities of their hitting the target are , and
49 49 4 2
26 32 5
(c) (d)
49 49 respectively, then the probability that the target is hit
8
23. A bag contains 4 red and 6 black balls. A ball is drawn at
random from the bag, its colour is observed and this ball by P or Q but not by R is : [Online April 8, 2017]
along with two additional balls of the same colour are 21 9
returned to the bag. If now a ball is drawn at random from (a) (b)
64 64
the bag, then the probability that this drawn ball is red, is :
[2018] 15 39
(c) (d)
64 64
2 1
(a) (b) 29. An unbiased coin is tossed eight times. The probability of
5 5
obtaining at least one head and at least one tail is :
3 3 [Online April 8, 2017]
(c) (d)
4 10 255 127
24. Let A, B and C be three events, which are pair-wise (a) (b)
256 128
independence and E denotes the complement of an event 63 1
(c) (d)
E. If P (A Ç B Ç C) = 0 and P (C) > 0, then P[( A Ç B ) |C] is 64 2
equal to. [Online April 16, 2018] 30. Let two fair six-faced dice A and B be thrown
simultaneously. If E1 is the event that die A shows up
(a) P (A) + P ( B ) (b) P ( A ) – P ( B )
four, E2 is the event that die B shows up two and E3 is the
(c) P ( A ) – P (B) (d) P ( A ) + P ( B ) event that the sum of numbers on both dice is odd, then
25. A player X has a biased coin whose probability of showing
which of the following statements is NOT true ? [2016]
heads is p and a player Y has a fair coin. They start playing
a game with their own coins and play alternately. The player (a) E1 and E3 are independent.
who throws a head first is a winner. If X starts the game, (b) E1, E2 and E3 are independent.
and the probability of winning the game by both the players (c) E1 and E2 are independent.
is equal, then the value of ‘p’ is[Online April 15, 2018] (d) E2 and E3 are independent.
1 1 2
(a)
3
(b)
5 31. If A and B are any two events such that P(A) = and
5
1 2
(c) (d) 3
4 5 P(A Ç B) = , then the conditional probability,,
20
26. If two different numbers are taken from the set (0, 1, 2, 3,
......., 10), then the probability that their sum as well as P(A | A¢ È B¢)) , where A' denotes the complement of A, is
absolute difference are both multiple of 4, is : [2017] equal to : [Online April 9, 2016]
7 6
(a) (b) (a) 11 20 (b) 5 17
55 55
12 14 (c) 8 17 (d) 1 4
(c) (d)
55 55

Downloaded from @Freebooksforjeeneet


Probability M - 525

32. Let X be a set containing 10 elements and P(X) be its k, so that the probability of hitting the target at least once
power set. If A and B are picked up at random from P(X), 7
with replacement, then the probability that A and B have is more than , is : [Online April 9, 2013]
10
equal number elements, is : [Online April 10, 2015] (a) 3 (b) 5

(a)
(2
10
-1 ) (b)
20
C10
38.
(c) 2 (d) 4
Three numbers are chosen at random without replacement
210 210 from {1,2,3,..8}. The probability that their minimum is 3,

(c)
(2
10
-1 ) (d)
20
C10 given that their maximum is 6, is : [2012]
3 1
220 220 (a) (b)
8 5
33. Let A and B be two events such that P A È B = ( ) 1
6
, (c)
1
(d)
2
4 5
39. Let A, B, C, be pairwise independent events with P (C) > 0
(
P AÇ B =
1
4
) ( ) 1
and P A = , where A stands for the
4 c
(
c
)
and P ( A Ç B Ç C ) =0. Then P A Ç B / C . [2011RS]
complement of the event A. Then the events A and B are

(a) independent but not equally likely.


[2014] ( )
(a) P B – P ( B )
c
( ) ( )
(b) P A + P B
c c

(b) independent and equally likely.


(c) mutually exclusive and independent.
( ) ( )
(c) P A - P B
c c c
( )
(d) P A - P ( B )
(d) equally likely but not independent. 40. If C and D are two events such that C Ì D and P(D) ¹ 0,
34. Let A and E be any two events with positive probabilities: then the correct statement among the following is [2011]
Statement - 1: P(E/A) ³ P(A/E) P(E) (a) P (C | D ) ³ P (C ) (b) P (C | D ) < P (C )

Statement - 2: P(A/E) ³ P ( A Ç E ) P( D )
(c) P (C | D) = (d) P (C | D ) = P (C )
[Online April 19, 2014] P (C )
(a) Both the statements are true 41. One ticket is selected at random from 50 tickets numbered
(b) Both the statements are false 00,01,02,...,49. Then the probability that the sum of the
digits on the selected ticket is 8, given that the product of
(c) Statement-1 is true, Statement-2 is false
these digits is ero, equals: [2009]
(d) Statement-1 is false, Statement-2 is true
1 5
35. A, B, C try to hit a target simultaneously but independently. (a) (b)
7 14
Their respective probabilities of hitting the targets are
1 1
3 1 5 (c) (d)
, , . The probability that the target is hit by A or B but 50 14
4 2 8 42. It is given that the events A and B are such that
not by C is : [Online April 23, 2013] 1 1 2
P ( A) = , P( A | B ) = and P ( B | A) = . Then P(B) is
(a) 21/64 (b) 7/8 4 2 3
(c) 7/32 (d) 9/64 [2008]
36. Given two independent events, if the probability that 1 1
26 (a) (b)
exactly one of them occurs is and the probability that 6 3
49
2 1
15 (c) (d)
none of them occurs is , then the probability of more 3 2
49 43. Two aeroplanes I and II bomb a target in succession. The
probable of the two events is : [Online April 22, 2013]
(a) 4/7 (b) 6/7 probabilities of I and II scoring a hit correctly are 0.3 and
(c) 3/7 (d) 5/7 0.2, respectively. The second plane will bomb only if the
2 first misses the target. The probability that the target is hit
37. The probability of a man hitting a target is . He fires at by the second plane is [2007]
5
(a) 0.2 (b) 0.7
the target k times (k, a given number). Then the minimum
(c) 0.06 (d) 0.14.

Downloaded from @Freebooksforjeeneet


EBD_8344
M - 526 Mathematics

44. Three houses are available in a locality. Three persons 49. In a bombing attack, there is 50% chance that a bomb will
apply for the houses. Each applies for one house without hit the target. At least two independent hits are required to
consulting others. The probability that all the three apply destroy the target completely. Then the minimum number
for the same house is [2005] of bombs, that must be dropped to ensure that there is at
least 99% chance of completely destroying the target, is
2 1 _________. [NA Sep. 05, 2020 (II)]
(a) (b)
9 9
1
50. The probability of a man hitting a target is . The least
8 7 10
(c) (d) number of shots required, so that the probability of his
9 9
1
hitting the target at least once is greater than ,
1 4
45. Let A and B be two events such that P ( A È B ) = ,
6 is ___________. [NA Sep. 04, 2020 (I)]

1 51. A random variable X has the following probability


1
P( A Ç B) = and P ( A) = , where A stands for distribution:
4 4
complement of event A. Then events A and B are [2005] X : 1 2 3 4 5
(a) equally likely and mutually exclusive P(X) : K2 2K K 2K 5K2
(b) equally likely but not independent Then, P(X > 2) is equal to: [Jan. 9, 2020 (II)]
(c) independent but not equally likely 7 1
(d) mutually exclusive and independent (a) (b)
12 36
4 1 23
46. The probability that A speaks truth is , while the (c) (d)
5 6 36
52. Let a random variable X have a binomial distribution with
3
probability for B is . The probability that they contradict k
4 mean 8 and variance 4. If P(X d” 2) = 16 , then k is equal
2
each other when asked to speak on a fact is [2004] to:
4 1 [April 12, 2019 (I)]
(a) (b)
5 5 (a) 17 (b) 121
7 3
(c) (d) (c) 1 (d) 137
20 20
47. A problem in mathematics is given to three students A, B, 53. A person throws two fair dice. He wins Rs. 15 for throwing
C and their respective probability of solving the problem a doublet (same numbers on the two dice), wins Rs. 12
when the throw results in the sum of 9, and loses Rs. 6 for
1 1 1
is , and . Probability that the problem is solved is any other outcome on the throw. Then the expected gain/
2 3 4
loss (in Rs.) of the person is : [April 12, 2019 (II)]
[2002]
1 1
3 1 (a) gain (b) loss
(a) (b) 2 4
4 2 1
2 1 (c) loss (d) 2 gain
(c) (d) 2
3 3 54. A bag contains 30 white balls and 10 red balls. 16 balls are
drawn one by one randomly from the bag with replacement.
Random Variables, Probability If X be the number of white balls drawn, then
Distribution, Bernoulli Trails,
TOPIC n Binomial Distribution, Poisson æ mean of X ö
Distribution ç ÷ is equal to:[Jan. 11, 2019 (II)]
è standard deviation of X ø
48. Four fair dice are thrown independently 27 times. Then the
4 3
expected number of times, at least two dice show up a (a) 4 (b) 4 3 (c) 3 2 (d)
3
three or a five, is ______. [NA Sep. 05, 2020 (I)]

Downloaded from @Freebooksforjeeneet


Probability M - 527

55. A box contains 15 green and 10 yellow balls. If 10 balls


1 9
are randomly drawn, one-by-one, with replacement, then (a) log 4 + log 3 (b)
the variance of the number of green balls drawn is: [2017] 10 10 log10 4 – log10 3

6 12 4 1
(a) (b) (c) 6 (d) 4 (c) log 4 – log 3 (d) log 4 – log 3
25 5 10 10 10 10
56. An experiment succeeds twice as often as it fails. The 62. A pair of fair dice is thrown independently three times. The
probability of at least 5 successes in the six trials of this probability of getting a score of exactly 9 twice is [2007]
experiment is : [Online April 10, 2016] (a) 8/729 (b) 8/243 (c) 1/729 (d) 8/9.
496 192 63. At a telephone enquiry system the number of phone calls
(a) (b)
729 729 regarding relevant enquiry follow Poisson distribution with
240 256 an average of 5 phone calls during 10 minute time intervals.
(c) (d) The probability that there is at the most one phone call
729 729
57. If the mean and the variance of a binomial variate X are during a 10-minute time period is [2006]
2 and 1 respectively, then the probability that X takes a 6 5 6 6
value greater than or equal to one is : (a) (b) (c) (d)
e
[Online April 11, 2015] 5 6 55 e5
9 3 1 15 64. A random variable X has Poisson distribution with mean 2.
(a) (b) (c) (d) Then P (X > 1.5) equals [2005]
16 4 16 16
58. If X has a binomial distribution, B(n, p) with parameters n 2 3 3
(a) 2
(b) 0 (c) 1 - (d) 2
and p such that P(X = 2) = P (X = 3), then E(X), the mean of e 2 e
e
variable X, is [Online April 11, 2014] 65. The mean and the variance of a binomial distribution are 4
p p and 2 respectively. Then the probability of 2 successes is
(a) 2 – p (b) 3 – p (c) (d) [2004]
2 3
59. A multiple choice examination has 5 questions. Each 28 219 128 37
(a) (b) (c) (d)
question has three alternative answers of which exactly 256 256 256 256
one is correct. The probability that a student will get 4 or 66. A random variable X has the probability distribution:
more correct answers ust by guessing is: [2013]
X: 1 2 3 4 5 6 7 8
17 13 11 10 p(X): 0.2 0.2 0.1 0.1 0.2 0.1 0.1 0.1
(a) (b) 5 (c) 5 (d) 5
35 3 3 3 For the events E = {X is a prime number } and
60. Consider 5 independent Bernoulli’s trials each with F = { X < 4}, the P( E È F ) is [2004]
probability of success p. If the probability of at least one
(a) 0.50 (b) 0.77 (c) 0.35 (d) 0.87
31
failure is greater than or equal to , then p lies in the 67. The mean and variance of a random variable X having
32 binomial distribution are 4 and 2 respectively, then P
interval [2011]
(X = 1) is [2003]
æ 3 11 ù é 1ù
(a) ç , ú (b) ê0, ú 1 1
è 4 12 û ë 2û (a) (b)
4 32
æ 11 ù æ 1 3ù
(c) ç ,1ú (d) ç , ú
è 12 û è 2 4û 1 1
(c) (d)
æ 1ö 16 8
61. In a binomial distribution B ç n, p = ÷ , if the probability
è 4ø 68. A dice is tossed 5 times. Getting an odd number is
9 considered a success. Then the variance of distribution of
of at least one success is greater than or equal to , then
10 success is [2002]
(a) 8/3 (b) 3/8 (c) 4/5 (d) 5/4
n is greater than: [2009]

Downloaded from @Freebooksforjeeneet


EBD_8344
M-536 Mathematics

28
Properties of
Triangles
b+c c+a a+b
Properties of Triangle, Solutions of 5. Given = = for a DABC with usual
TOPIC Ć Triangles, Inscribed & Enscribed 11 12 13
Circles, Regular Polygons cos A cos B cos C
notation. If = = , then the ordered triad
a b γ
1. Let A(3, 0, –1), B(2, 10, 6) and C (1, 2, 1) be the vertices of (a, b, g) has a value : [Jan. 11, 2019 (II)]
a triangle and M be the midpoint of AC. If G divides BM in (a) (7, 19, 25) (b) (3, 4, 5)
the ratio, 2 : 1, then cos (ÐGOA) (O being the origin) is (c) (5, 12, 13) (d) (19, 7, 25)
6. With the usual notation, in DABC, if ÐA + ÐB = 120 ,
equal to : [April 10, 2019 (I)]
a= 3 + 1 and b = 3 - 1, then the ratio ÐA : ÐB, is:
1 1 [Jan. 10, 2019 (II)]
(a) (b) (a) 7 : 1 (b) 5 : 3
2 15 15
(c) 9 : 7 (d) 3 : 1
1 1 a
(c) (d) 7. In a DABC, = 2 + 3 and ÐC = 60 . Then the ordered
6 10 30 b
2. The angles A, B and C of a triangle ABC are in A.P. and pair (ÐA, ÐB) is equal to : [Online April 10, 2015]
(a) (45 , 75 ) (b) (105 , 15 )
a : b = 1 : 3 . If c = 4 cm, then the area (in sq.cm) of this (c) (15 , 105 ) (d) (75 , 45 )
triangle is : [April 10, 2019 (II)] 8. ABCD is a trape ium such that AB and CD are
2 parallel and BC ^ CD. If ÐADB = q, BC = p and CD = q,
(a) (b) 4 3
3 then AB is equal to : [2013]
4 ( p 2 + q 2 ) sin q p 2 + q 2 cos q
(c) 2 3 (d) (a) (b)
3 p cos q + q sin q p cos q + q sin q
3. If the lengths of the sides of a triangle are in A.P. and the
p2 + q2 ( p 2 + q 2 ) sin q
greatest angle is double the smallest, then a ratio of lengths (c) (d)
p 2 cos q + q 2 sin q ( p cos q + q sin q) 2
of the sides of this triangle is : [April 08, 2019 (II)]
b+c c+a a+b
(a) 5 : 9 : 13 (b) 4 : 5 : 6 9. If in a triangle ABC, = = , then cosA is
11 12 13
(c) 3 : 4 : 5 (d) 5 : 6 : 7 equal to [2012]
4. In a triangle, the sum of lengths of two sides is x and the (a) 5/7 (b) 1/5
product of the lengths of the same two sides is y. if
(c) 35/19 (d) 19/35
x 2 - c 2 = y , where c is the length of the third side of the 10. In a DPQR, If 3 sin P + 4 cos Q = 6 and 4 sin Q + 3 cos
P = 1, then the angle R is equal to : [2012]
triangle, then the circumradius of the triangle is :
[Jan. 11, 2019 (I)] 5p p
(a) (b)
3 c 6 6
(a) y (b)
2 3 p 3p
(c) (d)
c y 4 4
(c) (d)
3 3

Downloaded from @Freebooksforjeeneet


Properties of Triangles M-537

11. For a regular polygon, let r and R be the radii of the inscribed 17. The sum of the radii of inscribed and circumscribed circles
and the circumscribed circles. A false statement among for an n sided regular polygon of side a , is [2003]
the following is [2010]
a æp ö æp ö
r 1 (a) 4 cot ç 2 n ÷ (b) a cot ç n ÷
(a) There is a regular polygon with = è ø è ø
R 2
r 2 a æp ö æp ö
(b) There is a regular polygon with = (c) 2 cot ç 2 n ÷ (d) a cot ç 2 n ÷ .
R 3 è ø è ø

r 3 18. In a triangle with sides a, b, c, r1 > r2 > r3


(c) There is a regular polygon with =
R 2 (which are the ex-radii) then [2002]
r 1 (a) a >b > c (b) a < b < c
(d) There is a regular polygon with =
R 2 (c) a > b and b < c (d) a < b and b > c
12. If in a DABC , the altitudes from the vertices A, B, C on 19. The sides of a triangle are 3x+4y, 4x+3y and 5x+5y where x,
opposite sides are in H.P, then sin A, sin B, sin C are in y > 0 then the triangle is [2002]
[2005] (a) right angled (b) obtuse angled
(a) G. P. (b) A. P.
(c) equilateral (d) none of these
(c) A.P -G..P. (d) H. P
p
13. In a triangle ABC, let ÐC = . If r is the inradius and R is
2
the circumradius of the triangle ABC, then 2 (r + R) equals
[2005]
(a) b + c (b) a + b
(c) a + b + c (d) c + a
14. The sides of a triangle are sin a, cos a and

p
1 + sin a cos a for some 0 < a < . Then the greatest
2
angle of the triangle is [2004]
(a) 150 (b) 90
(c) 120 (d) 60

æCö æ A ö 3b
15. If in a DABC a cos 2 ç ÷ + c cos 2 ç ÷= , then the sides
è2ø è2ø 2

a, b and c [2003]

(a) satisfy a + b = c (b) are in A.P


(c) are in G..P (d) are in H.P
16. In a triangle ABC, medians AD and BE are drawn. If AD = 4,
p p
ÐDAB = and ÐABE = , then the area of the D ABC
6 3
is [2003]
64 8
(a) (b)
3 3
16 32
(c) (d)
3 3 3

Downloaded from @Freebooksforjeeneet

You might also like